Sunteți pe pagina 1din 126

Clerk Mock Test 1

them in the word as in the English alphabet?


Test-I: Reasoning Ability 1) None 2) One 3) Two
4) Three 5) More than three
1. Suresh ranks seventh from the top and thirtyeighth Directions (Q. 11-15): In each question below are
from the bottom in a class. How many students are given two statements followed by two conclusions
there in the class? numbered I and II. You have to take the given statements
1) 40 2) 45 3) 42 4) 44 5) None of these to be true even if they seem to be at variance with
2. Among J, K, L, M and N, each having a different salary, commonly known facts. Read all the conclusions and then
J gets less than M. L gets more than M but less than decide which of the given conclusions logically follows
N. Who among them gets the highest salary? from the given statements, disregarding commonly
1) N 2) Either N or K 3) L known facts. Give answer
4) Can’t be determined 5) None of these 1) if only conclusion I follows.
3. What should come in place of question mark (?) in the 2) if only conclusion II follows.
following series based on the English alphabetical 3) if either conclusion I or II follows.
series? 4) if neither conclusion I nor II follows.
CAT FDW IGZ ? 5) if both conclusions I and II follow.
1) LDH 2) LJC 3) LIJ 4) KTC 5) None of these 11. Statements: Some buses are trucks.
4. Four of the following five are alike in a certain way and No truck is a jeep.
so form a group. Which is the one that does not Conclusions: I. Some jeeps are trucks.
belong to that group? II. Some buses are not jeeps.
1) Tiger 2) Ounce 3) Jackal 12. Statements: All sofas are beds.
4) Leopard 5) Panther Some beds are tables.
5. N is sister of W. I is brother of W. U is father of N and Conclusions: I. At least some sofas being tables
G is mother of I. How is G related to W? is a possibility.
1) Daughter 2) Son II. Some tables are beds.
3) Mother Statements: Some papers are books.
5) None of these All books are pages.
Directions (Q. 6-9): Study the following information Conclusions: I. Some pages are not papers.
carefully and answer the given questions: II. No book is paper.
A man has five luggages V, W, X, Y and Z, each having 14. Statements: No desk is a shelf.
a different weights. V weighs twice as much as W. W Some shelves are stools.
weighs four-and-a-half times as much as X. X weighs half Conclusions: I. No stool is a desk.
as much as Y. Y weighs half as much as Z, and Z weighs II. Some stools are not desks.
less than V but more than X. 15. Statements: Some radios are clocks.
6. Which of the following luggages is the heaviest? No clock is a laptop.
1) Z 2) W 3) V Conclusions: I. Some radios are not laptops.
4) Can’t be determined 5) None of these II. Some clocks are radios.
7. What is the weighs of Z if X weighs 1 kg? Directions (Q. 16-20): In these questions, a
1) 4 kg 2) 2.5 kg 3) 2 kg relationship between different elements is shown in the
4) Can’t be determined 5) None of these statement(s). The statements are followed by two
8. How many luggages are lighter than W? conclusions. Give answer
1) Two 2) Three 3) Four 1) if only conclusion I is true.
4) Can’t be determined 5) None of these 2) if only conclusion II is true.
9. What is the weight of V if X weighs 1 kg? 3) if either conclusion I or II is true.
1) 5 kg 2) 8 kg 3) 7.5 kg 4) if neither conclusion I nor II is true.
4) 9 kg 5) None of these 5) if both conclusions I and II are true.
10. How many such pairs of letters are there in the word 16. Statements: T S=R>Q>P
REFORMS each of which has as many letters between Conclusions: I. P T II. T > Q
17. Statement: P O<N>M L 1) 32 2) 14 3) 26 4) 16 5) None of these
Conclusions: I. O > M II. N L 28. How many Accountants are both Engineers and
18. Statement: A B=C >D E Teachers?
Conclusions: I. E > C II. D < A 1) 29 2) 20 3) 11 4) 18 5) None of these
19. Statements: H > J = K, K > L > T, T V 29. How many Engineers are neither Teachers nor
Conclusions: I. H > L II. J > V Accountants?
20. Statements: U V < W, W = N < L 1) 18 2) 16 3) 11 4) 13 5) None of these
Conclusions: I. L > V II. U N 30. How many Teachers are Engineers?
Directions (Q. 21-25): Study the following information 1) 22 2) 4 3) 18 4) 29 5) None of these
carefully to answer the given questions: Directions (Q. 31-35): Each of the questions below
Eight members of a club are sitting arround a circular consists of a question and two statements numbered I and
table, viz Z, Y, X, W, V, U, T and S, but not necessarily in the II given below it. You have to decide whether the data
same order. Four of them are not facing the centre. W is provided in the statements are sufficient to answer the
facing the centre and is third to the right of Z, who is third question. Read both the statements and give answer
to the right of U, who is facing the centre. X is fourth to the 1) if the data in statement I alone are sufficient to
left of T. S is third to the right of V, who is not a neighbour answer the question, while the data in statement II
of T. V sits second to the right of X. Y is on the immediate alone are not sufficient to answer the question.
right of T. 2) if the data in statement II alone are sufficient to
21. What is S’s position with respect to Y? answer the question, while the data in statement I
1) Third to the left 2) Second to the right alone are not sufficient to answer the question.
3) Second to the left 4) Either 2) or 3) 3) if the data either in statement I alone or in statement
5) None of these II alone are sufficient to answer the question.
22. Who is second to the left of V? 4) if the data in both the statements I and II together
1) W 2) X 3) Y 4) T 5) None of these are not sufficient to answer the question.
23. Which of the following pairs are neighbours of T? 5) if the data in both the statements I and II together
1) W, Y 2) U, S are necessary to answer the question.
3) Z, W 4) Can’t be determined 31. What is the postion of Roshni from the right end of
5) None of these the row?
24. Four of the following five are alike in a certain way and I. There are 18 children between Kamal and Roshni.
so form a group. Which is the one that does not II. Kamal is 10th from the right end of the row.
belong to that group? 32. On which day of the week did Q visit Gujarat?
1) W 2) Y
25. Which of the following statements is/are true? II. Q visited Gujarat after Monday but before Thursday
1) Z is not facing the centre. but not on an odd day of the week.
2) Y is second to the right of W. 33. How is ‘my’ written in a code language?
3) T is facing the centre I. ‘my work is fine’ is written as ‘ma ta ka ha’ in that
4) Both 1) and 2) are true code language.
5) None of these II. ‘how is your work’ is written as ‘ja ma na ka’ in that
Directions (Q. 26-30): The following questions are code language.
based on the diagram given below: 34. On which day of the month was S born?
The circle represents a group of Teachers, the I. S’s mother correctly remembers that S was born
rectangle represents a group of Engineers and the triangle after 15th October but before 19th October.
represents a group of Accountants. II. S’s brother correctly remembers that S was born
after 14th October but before 17th October.
35. How is B related to V?
I. V is mother of B’s son’s only uncle’s son.
II. V is sister-in-law of U, who is sister of W.
Directions (Q . 36-40): Study the following
arrangement carefully and answer the questions given
26. How many Teachers are neither Accountant nor below:
Engineer? 1 $ 6 H J A7  G Q # B 2 M E K % R 5 @ N D I P 4 C T 3 9 F
1) 20 2) 32 3) 22 4) 33 5) None of these 36. If all the symbols in the above arrangement are
27. How many Engineers are Accountants? dropped, which of the following will be 10th from the
right end? He first asked his eldest son. That son replied: “O
1) E 2) N 3) 5 great king, women and servants will mock at me if I were to
4) R 5) None of these take upon myself your old age. I cannot do so. Ask of my
37. Which of the following is fifth to the left of eighteenth younger brothers who are dearer to you than myself.”
from the left end in the above arranement? When the second son was approached, he gently
1) B 2) # 3) 2 refused with the words: “Father, you ask me to take up old
4) M 5) None of these age which destroys not only strength and beauty but also
38. How many such vowels are there in the above – as I see – wisdom. I am not strong enough to do so.”
arrangement each of which is immediately preceded The third son replied: “An old man cannot ride a horse
by a number and immediately followed by a or an elephant. His speech will falter. What can I do in such
consonant? a helpless plight? I cannot agree.”
1) None 2) Two 3) Three The king was angry and disappointed that his three
4) Four 5) None of these sons had declined to do as he wished, but he hoped for
39. What will come in place of question mark (?) in the better from his fourth son, to whom he said: “You should
following series based on the above arrangement? take up my old age. If you exchange your youth with me, I
6AJ #Q 2KE RN@ ? shall give it back to you after some time and take back the
1) NP4 2) @IP 3) IC4 old age with which I have been cursed.”
4) I4C 5) None of these The fourth son begged to be forgiven as this was a
40. How many such consonants are there in the above thing he could by no means consent to. An old man had to
arrangement, each of which is immediately preceded seek the help of others even to keep his body clean, a most
by a number and immediately followed by a letter? pitiful plight. No, much as he loved his father he could not
1) One 2) Three 3) Four do it.
4) None 5) None of these Yayati was struck with sorrow at the refusal of the
four sons. Still, hoping against hope, he supplicated his
Test-II: English Language last son, who had never yet opposed his wishes: “You
must save me. I am afflicted with this old age with its
Directions (Q. 41-50): Read the following passage wrinkles, debility and grey hairs as a result of the curse of
carefully and answer the questions given below it. Certain Sukracharya. It is too hard a trial! If you will take upon
words/phrases in the passage are printed in bold to help yourself these infirmities, I shall enjoy life for just a while
you locate them while answering some of the questions. more and then give you back your youth and resume my
Emperor Yayati was one of the ancestors of the old age and all its sorrows. Pray, do not refuse as your
Pandavas. He had never known defeat. He followed the elder brothers have done.” Puru, the youngest son, moved
dictates of the shastras, adored the gods and venerated by filial love, said: “Father, I gladly give you my youth and
his ancestors with intense devotion. He became famous as relieve you of the sorrows of old age and cares of State. Be
a ruler devoted to the welfare of his subjects. happy.” Hearing these words Yayati embraced him.
But he became prematurely old by the curse of As soon as he touched his son, Yayati became a youth.
Sukracharya for having wronged his wife Devayani. In the Puru, who accepted the old age of his father, ruled the
words of the poet of the Mahabharata: “Yayati attained kingdom and acquired great renown.
that old age which destroys beauty and brings on miseries.” 41. What was/were the reason(s) for Yayati’s premature
It is needless to describe the misery of youth suddenly old age?
blighted into age, where the horrors of loss are accentuated 1) He was addicted to opium.
by pangs of recollection. 2) He was cursed for not devoting his life for the
Yayati, who found himself suddenly an old man, was welfare of his subjects.
still haunted by the desire for sensual enjoyment. He had 3) He was cursed by Sukracharya for having wronged
five beautiful sons, all virtuous and accomplished. Yayati his wife, Devayani.
called them and appealed piteously to their affection: “The 4) He had been punished by the gods for not
curse of your grandfather Sukracharya has made me following the dictates of the shastras.
unexpectedly and prematurely old. I have not had my fill of 5) All the above
the joys of life; for not knowing what was in store for me, I 42. What was the desire which haunted Yayati when he
lived a life of restraint denying myself even lawful turned old suddenly?
pleasures. One of you ought to bear the burden of my old 1) The desire for reputation
age and give his youth in return. He who agrees to this and 2) The desire for wealth
bestows his youth on me will be the ruler of my kingdom. I 3) The desire for youth
desire to enjoy life in the full vigour of youth.” 4) The desire for sensual enjoyment
5) None of these 51. 1) The international law of human rights now comprises
43. What did Yayati ask his five sons to do for him? of / 2) more than eighty universal and regional
(A) He asked them to send him to the forest where he conventions / 3) which have binding force / 4) for
would live an austere life to attain mental poise. state parties. / 5) No error
(B) He asked them to bear the burden of his old age 52. 1) President Barack Obama is getting increasingly / 2)
and give their youth in return. isolation on the issue of / 3) launching a limited / 4)
(C) He asked them to bring him a harlot so as to gratify attack on Syria. / 5) No error
his lust. 53. 1) Some of the long-tailed birds / 2) that emerge early
1) Only (A) 2) Only (B) in the morning, / 3) include the black-headed Jay / 4)
3) Only (C) 4) Both (A) and (C) and the lesser common Eurasian Jay. / 5) No error
5) Both (B) and (C) 54. 1) Few states are grappling with / 2) the issue of finding
44. Why did Puru accept the old age of his father? / 3) the correct legal strategy / 4) to combat the
1) He was moved by filial love for his father. superstition-based social menace called witchcraft. /
2) He wanted to rule the kingdom and deny the share 5) No error
of property to his elder brothers. 55. 1) The State of Karnataka, especially / 2) its coastal
3) He wanted to experience premature old age just districts of Udupi and Dakshina Kannada, / 3) is
like his father. unarguably the cradle / 4) of banking in the country. /
4) He wanted to impress his father so that he could 5) No error
get the throne instead of his brothers. Directions (Q. 56-60): In each of these questions, two
5) None of these sentences I and II are given. Each sentence has a blank in
45. What message does the author want to leave behind it. Five words 1), 2), 3), 4) and 5) are suggested. Out of
through the passage? these only one fits at both the places in the context of each
1) Human desire for everlasting youth is irrelevant. sentence. The number of that word is the answer.
2) Nothing can ever satisfy the desire of a man. 56. I. For the first time in Fiji’s Constitution, we have an
3) One should learn to bear with one’s infirmities. electoral ______ of one person, one vote, one
4) One should live austerely. value.
5) None of these II. He had slept for over fourteen hours, and his
Directions (Q. 46-48): Choose the word/group of ______ seemed to have recuperated admirably.
words which is MOST SIMILAR in meaning to the word/ 1) procedure 2) body 3) process
group of words printed in bold as used in the passage. 4) system 5) method
46. Declined 57. I. The Forest Department is involving local
1) reduced communities in a ______ conservation initiative
4) accepted 5) agreed for the State’s bird population.
47. Restraint II. Protesters found a ______ way of demonstrating
1) curbed 2) reposeful 3) respite against steeply rising oil prices.
4) repression 5) solvent 1) simple 2) novel 3) better
48. Accentuated 4) uniform 5) useful
1) enhanced 2) adopted 3) praised 58. I. The medical industry, once plagued by the ______
4) acquiesced 5) greeted cost of equipment and shortage of specialists, has
Directions (Q. 49-50): Choose the word/group of almost overcome those drawbacks today.
words which is MOST OPPOSITE in meaning of the word/ II. The cost of private treatment can be ______ for
group of words printed in bold as used in the passage. poor.
49. Vigour 1) encouraging 2) affordable 3) rising
1) strength 2) impotency 3) gloom 4) payable 5) prohibitive
4) virility 5) viability 59. I. North-eastern India, at the _______ of the Indo-
50. Infirmities Malayan, Indo-Chinese biogeographic realms, is
1) weaknesses 2) rigidity 3) capabilities famous for its varied and rich biological and
4) skills 5) insanity ecological values.
Directions (Q. 51-55): Read each sentence to find out II. Like most cases of extreme weather, its severity
whether there is any grammatical or idiomatic error in it. was due to an unusual ________ of events.
The error, if any, will be in one part of the sentence. The 1) confluence 2) influence 3) meeting
number of that part is the answer. If there is ‘No error’, 4) happening 5) sequence
the answer is 5). (Ignore errors of punctuation, if any.) 60. I. India is the only country that has _______ the
notion of scientific temper to the level of a account together with auditors’ report and a report
constitutional obligation. by the Central Board on the working and activities
II. They ________ the money to buy the house and of the bank.
two acres of grounds. (C) The State Bank has to close its books and balance
1) called 2) made 3) collected accounts each year as on 31 March or such other
4) propounded 5) raised date as may be specified by the Central Govt.
Directions (Q. 61-65): In the following questions, a (D) The auditors’ report and the report of the Central
sentence has been given with some of its parts in bold. To Board have to be placed before the Parliament.
make the sentence grammatically correct, you have to (E) The audit may be conducted by any person duly
replace the bold part with the correct alternative given qualified to be auditors of companies under
below. If the sentence is correct as it is, give 5) as your Section 226 of the Companies Act.
answer (ie No correction required). (F) The appointment of auditors is done by the
61. The Banking Regulation Act does not contain any Reserve Bank in consultation with the Central
specific provisions for regulation of acceptance of Government.
deposits for banks. 66. Which of the following should be the FIRST sentence
1) for acceptance of deposits for banks after rearrangement?
2) of acceptance of deposits of banks 1) A 2) B 3) D 4) F 5) C
3) of acceptance for deposits of banks 67. Which of the following should be the SECOND
4) for acceptance of deposits in banks sentence after rearrangement?
5) No correction required 1) C 2) A 3) B 4) E 5) F
62. The subsidiary banks of the State Bank of India were 68. Which of the following should be the THIRD sentence
established by different special statues. after rearrangement?
1) special status 2) special statutes 1) E 2) C 3) A 4) B 5) D
3) especial states 4) specialised statutes 69. Which of the following should be the FOURTH
5) No correction required sentence after rearrangement?
63. All banks whose shares are listed with stock 1) B 2) F 3) E 4) A 5) C
exchanges are required to publish their unaudited 70. Which of the following should be the LAST (SIXTH)
quarterly results as per performance prescribed by sentence after rearrangement?
the SEBI. 1) F 2) D 3) C 4) B 5) A
1) as per proforma Directions (Q. 71-80): In the following passage, some
2) as according to proforma of the words have been left out, each of which is indicated
3) as required by proforma by a number. Find the suitable word from the options given
4) according to performance against each number and fill up the blanks with
5) No correction required appropriate words to make the paragraph meaningfully
64. As good as a banker is concerned, the law relating to complete.
indemnities is of great importance. The idea of establishing Regional Rural Banks was
1) As well as 2) As much as 3) As far as (71) in the Twenty Point Economic Programme of July 1975
4) As soon as 5) No correction required to (72) to the credit needs of rural people. The Government
65. Though a bank guarantee is a contingent liability, it is of India appointed the Narasimham Committee in July 1975
always prudent for a banker to secure this contingent to set up the new (73) in order to provide employment to
liability to cover himself in case it is enforced. the rural educated youth and to bring (74) the cost of rural
1) Even though 2) Even if 3) Nonetheless banks by recruiting their staff on the same scale of pay and
4) Nevertheless 5) No correction required allowances as for the employees of State Government or
Directions (Q. 66-70): Rearrange the following six local bodies. The major (75) of the RRBs are to develop the
sentences (A), (B), (C), (D), (E) and (F) in the proper rural economy by providing credit and other (76) for
sequence to form a meaningful paragraph and then answer agriculture, trade, commerce, industry and other productive
the questions given below. activities in the rural areas, (77) to the small and marginal
(A) No Director, member of local board, local farmers, agricultural labourers, artisans and small
committee or an officer of the State Bank shall be entrepreneurs. A Regional Rural Bank is sponsored by a
eligible to be the auditor. commercial bank. For the purpose, the sponsor bank
(B) Within three months of the closing date, it has to requests the Central Government which issues a notification
furnish to the Central Government and the Reserve after consulting the (78) State Government. Normally, an
Bank its balance sheet and profit-and-loss RRB covers one district but it is also permitted to (79) its
branches in other districts. So far the maximum (80) has
been eight districts, as in the case of Manipur Regional
4 3 2
Rural Bank, which covers the entire State of Manipur. 1) 208 2) 210 3) 203
71. 1) rejected 2) conspired 3) mooted 7 7 9
4) meant 5) declined 1 5
72. 1) cater 2) indulge 3) survey 4) 204 5) 206
5 9
4) cause 5) bring
73. 1) unity 2) companion 3) company 90. ? 12 35 1296
4) institution 5) communion 1) 169 2) 196 3) 225 4) 13 5) 14
74. 1) up 2) down 3) about Directions (Q. 91-95): What approximate value should
4) forth 5) into come in place of question mark (?) in the following
75. 1) theme 2) ideas 3) work questions? (Note: You are not expected to calculate the
4) duty 5) objectives exact value.)
76. 1) facilities 2) item 3) goods 91. 37.0231 × 45.4501 – 142.732 + 4.5179 = ?
4) values 5) price 1) 1475 2) 1757 3) 1680 4) 1545 5) 1320
77. 1) clearly 2) certainly 3) particularly 92. 250430.971 + 152730.282 – 98170.195 – 32.70 = ?
4) similarly 5) likely 1) 312188 2) 304958 3) 311715
78. 1) best 2) concerned 3) related 4) 325432 5) 328917
4) nearest 5) willing 93. (171.2)2 – (89.5)2 = ?
79. 1) begin 2) log in 3) start 1) 26433 2) 23877 3) 21299
4) continue 5) open 4) 25122 5) 24155
80. 1) average 2) enclosures 3) beneficiaries 94. 222123.5378 × 34.21 – 22.015 = ?
4) coverage 5) share 1) 7606123 2) 7809465 3) 7552194
4) 7917756 5) 7154189
Test-III: Quantitative Aptitude 95. 9880 ÷ 304 × 3325 ÷ 125 = ?
1) 812 2) 915 3) 865 4) 925 5) 975
Directions (Q. 81-90): What should come in place of Directions (Q. 96-100): What should come in place
question mark (?) in the following questions? of question mark (?) in the following number series?
81. 945.341 – 1042.792 + 875.435 + 31.025 = ? 96. 80 85 ? 87 76 89
1) 408.004 2) 809.009 3) 905.009 1) 78 2) 76 3) 74 4) 75 5) 72
4) 712.009 5) 817.004 97. 5 13 25 ? 61 85
82. 23% of 1045 + 47% of 923 – 21% of 953 = ? 1) 38 2) 39 3) 43 4) 42 5) 41
1) 465.03
4) 474.03 5) 487.03 1) 1 2) 2 3) 3 4) 4 5) 5
83. (7425 ÷ 45 ÷ 5) + (8585 ÷ 85 ÷ 5) = ? 99. 11 121 1331 14641 ? 771561
1) 56.2 2) 63.2 3) 53.2 1) 164031 2) 162021 3) 155021
4) 76.2 5) 65.2 4) 161051 5) 15071
84. 47 × ? = (2931 ÷ 3) + 10 100. 10 ? 34 52 74 100
1) 19 2) 20 3) 23 1) 22 2) 20 3) 23 4) 25 5) 24
4) 22 5) 21 Directions (Q. 101-105): Study the given line graph
85. 8.7 × (105 ÷ 2.5) + 23 – 27 = ? carefully to answer the following questions.
1) 361.4 2) 372.5 3) 388.4
4) 356.5 5) 344.4 8 Jindal Tata SAIL
86. (7 × 7 × 7 × 7 × 7 × 7)4 × (7 × 7 × 7 × 7)8 ÷ (7 × 7)10 = (49)?
7
1) 14 2) 18 3) 16 4) 12 5) 20
87. 10.5 × 4.5 × 12.5 ÷ 4.0 = ? 6
1) 252.71425 2) 167.46525 3) 249.65625 5
4) 147.65625 5) 138.65025 4
34 36 35 3
88. ? 18
27 12 17 2
1) 100 2) 140 3) 120 4) 320 5) 220 1
16 8 4 13 0
89. 78 35 182 89 ? 2008 2009 2010 2011 2012 2013
27 27 27 27 Year
101. What is the difference between the total production 335, 1082, 487, 465, 948, 542 , _____, 664
of SAIL and that of Tata? 1) 768 2) 714 3) 789 4) 782 5) 795
1) 7 lakh tonnes 2) 8 lakh tonnes 3) 9 lakh tonnes 112. What would be the area of a rectangle whose area is
4) 6 lakh tonnes 5) 5 lakh tonnes equal to the area of a circle of radius 7 cm?
102. In which year is the total production the maximum? 1) 77 cm2 2) 154 cm2 3) 184 cm2
2 2
1) 2009 2) 2010 3) 2013 4) 184 cm 5) 150 cm
4) 2011 5) 2012 113. A dishonest vendor professes to sell fruits at the cost
103. What is the ratio of production of Jindal in 2011 to price but he uses a weight of 800 grams in lieu of 1 kg
that of Tata in 2013? weight. Find his percentage gain.
1) 5 : 3 2) 3 : 2 3) 2 : 3 4) 1 : 3 5) 3 : 7 1) 22% 2) 24% 3) 25% 4) 30% 5) 20%
104. The production of SAIL in 2008 is what percentage of 114. P sells a cow to Q at a loss of 10% and Q sells it to R at
the production of Jindal in 2012? a loss of 8%. If R pays `1656 for it, at what price did P
1) 82% 2) 78% 3) 80% buy it?
4) 75% 5) 85% 1) `1533 2) `1803 3) `1217 4) `1672 5) `2000
105. What is the difference between the total production 115. Find the ratio of compound interest to simple interest
of SAIL and that of Jindal? on a sum at 8% per annum for 3 years.
1) 5 lakh tonnes 2) 7 lakh tonnes 3) 6 lakh tonnes 1) 2143 : 3354 2) 1051 : 1529 3) 129 : 427
4) 8 lakh tonnes 5) 12 lakh tonnes 4) 2 : 3 5) 2029 : 1875
106. Three pipes X, Y and Z can fill a tank in 12 minutes, 18 116. Find the compound interest on `12000 for 2 years, the
minutes and 24 minutes respectively. The pipe Y is rate of interest being 3% per annum.
closed 3 minutes before the tank is full. In what time 1) `840.8 2) `935.7 3) `1035.6
will the tank be full? 4) `730.8 5) `473.5
117. A container contains milk and water in the ratio of 4 : 1.
4 6
2) 5 minutes 3) 6 minutes How much mixture should be taken out and replaced
13 13 13 with water so that the container contains milk and
6 water in the ratio of 3 : 2?
5) 8 minutes
13 13 3 1 2 1 2
1) 2) 3) 4) 5)
107. A man can row 24 km upstream and 54 km downstream 9 4 3 5 5
in 6 hours. He can also row 36 km upstream and 48 km 118. A is 2.5 times as good a workman as B and A is able to
downstream in 8 hours. What is the speed of the man finish the work in 50 days less than B. Find the time in
in still water? which they can do it working together.
1) 18.75 kmph 2) 19.25 kmph 3) 17.65 kmph
18 6
4) 15.55 kmph 5) 22.75 kmph 1) 21 days 2) 22 days
108. In how many ways can the letters of the word 19 21 7
IMAGINARY be arranged in such a way that all the 3 1
vowels never come together? 4) 20 days 5) 25 days
4 4
1) 95040 2) 94050 3) 93070
119. Rajan covers a certain distance in 12 minutes if he
4) 86400 5) 98090
runs at a speed of 10 kmph. What should be his speed
109. There are 4 boys and 6 girls. They sit in a row
in order to reduce his running time to 8 minutes?
randomly. What is the probability that all the boys do
1) 15 kmph 2) 14 kmph 3) 12 kmph
not sit together?
4) 8 kmph 5) 18 kmph
11 20 21 11 12 120. If the ratio of the area of two squares is 16 : 25, find the
1) 2) 3) 4) 5)
21 21 34 34 21 ratio of their diagonals.
110. The numerator of a fraction is decreased by 25% and 1) 7 : 2 2) 4 : 9 3) 3 : 2 4) 6 : 7 5) 4 : 5
the denominator is increased by 250%. If the resultant
6
Test-IV: General Awareness
fraction is , what is the original fraction?
5 (with Special Reference to Banking Industry)
22 24 27 28 30 121. With a view to strengthening economic engagement,
1) 2) 3) 4) 5)
5 5 6 5 11 India and the 10-nation ASEAN have decided to ink a
111. Find the missing number, if the average of all eight free trade agreement in services and investments in
numbers is 664. December 2013 in
1) Bali 2) Vientiane 3) Jakarta 3) Nicol David 4) Natalie Grinham
4) Phnom Penh 5) Kuala Lumpur 5) None of these
122. The maiden Global Slavery Index, compiled by the 131. The Banking Ombudsman is a senior official tasked
Perth-based Walk Free Foundation ranking 162 with redressing customer complaints against
countries, has ranked which of the following countries deficiency in certain banking services and is appointed
on top? by the
1) China 2) US 3) South Korea 1) Reserve Bank of India
4) India 5) Brazil 2) Ministry of Finance
123. World Osteoporosis Day is observed on which of the 3) Indian Banks’ Association
following dates every year? 4) High Court of each state
1) 18 Oct 2) 20 Oct 3) 22 Oct 5) None of these
4) 24 Oct 5) None of these 132. India Post has joined hands with which of the following
124. Which of the following financial organisations has exchanges to offer international electronic money
announced to close its retail broking and depository transfer?
businesses in India recently? 1) Wall Street Exchange
1) HSBC 2) Standard Chartered 2) UAE Exchange
3) JP Morgan 4) Barclays 3) Xpress Money Services Ltd
5) None of these 4) Economic Exchange Centre
125. India has launched its ambitious pension scheme, the 5) None of these
Mahatma Gandhi Pravasi Suraksha Yojana, for over 133. The Union Govt has decided to bring which of the
five million overseas workers employed in which of following across India under the core banking solution
the following countries? system to cover more people from rural areas?
1) Saudi Arabia 2) Bahrain 3) Kuwait 1) Microfinance institutions
4) Qatar 5) UAE 2) Post offices
126. The RBI is planning to introduce one billion pieces of 3) Co-operative societies
currency notes in polymer or plastic on a trial basis in 4) NBFCs
the denomination of 5) None of these
1) `5 2) `10 3) `20 4) `50 5) None of these 134. Apollo Tyres of India has lost a bid for dismissal of
127. Which of the following teams won the 30th Indian Oil Cooper Tire’s lawsuit seeking to force a $2.5-bn
Surjit Hockey Tournament Trophy for the record 10th takeover by the former. Cooper Tire is based in
time recently? 1) UK 2) US 3) France
1) Indian Oil, Delhi
2) Punjab and Sind Bank, Delhi 135. The United Nations has elected five new non-
3) ONGC, Delhi permanent members to the Security Council recently.
4) Indian Air Force, Delhi Which of the following countries is not among these
5) Bharat Petroleum, Mumbai five countries?
128. The proposed Bangladesh-China-India-Myanmar 1) Iraq 2) Chile 3) Nigeria
(BCIM) economic corridor aims to connect China’s 4) Chad 5) Saudi Arabia
Kunming city with which of the following Indian cities 136. The ambitious National Food Security Act of India
with a highway running through Bangladesh and aims to provide subsidised foodgrain to what per cent
Myanmar? of the total population of the country?
1) Kolkata 2) Guwahati 3) Dispur 1) 33 per cent 2) 45 per cent 3) 67 per cent
4) Darjeeling 5) None of these 4) 74 per cent 5) None of these
129. India’s ace cueist Aditya Mehta lost to who among 137. Who among the following has been appointed the
the following in the final of the Indian Open snooker new chairman of the Central Electricity Regulatory
tournament after becoming the first Indian ever to Commission (CERC)?
reach the final of a major ranking event? 1) Aftab Alam 2) Rahul Khullar
1) Robbie Williams 2) Ding Junhui 3) Swatanter Kumar 4) Gireesh B Pradhan
3) Alan McManus 4) Shaun Murphy 5) None of these
5) None of these 138. Who among the following has been given the first
130. Who among the following has won the Macau Open Yash Chopra Memorial Award recently?
squash women’s singles title recently? 1) Amitabh Bachchan 2) Shyam Benegal
1) Rachel Grinham 2) Dipika Pallikal 3) Lata Mangeshkar 4) Dilip Kumar
5) None of these
139. As announced by the Union Rural Development 1) Nepal 2) Bangladesh 3) Sri Lanka
Minister Jairam Ramesh, which of the following states 4) Myanmar 5) None of these
would be the biggest beneficiary of the National Food 149. India has signed a border defence cooperation
Security Act (NFSA)? agreement with which of the following countries
1) Odisha 2) Bihar 3) Jharkhand recently?
4) Uttarakhand 5) None of these 1) Pakistan 2) China 3) Bangladesh
140. The newly launched A4AI group aims to drive down 4) Myanmar 5) None of these
the cost of broadband Internet access in developing 150. The Civil Aviation Minister Ajit Singh inaugurated
countries. The term A4AI stands for India’s tallest Air Traffic Control (ATC) tower at which
1) Alliance for Alternative Internet of the following International Airports recently?
2) Alliance for Advance Internet 1) Sardar Vallabhbhai Patel International Airport
3) Alliance for Accredited Internet 2) Indira Gandhi International Airport
4) Alliance for Affordable Internet 3) Chhatrapati Shivaji International Airport
5) None of these 4) Rajiv Gandhi International Airport
141. As per the recently released data by the telecom 5) None of these
regulator TRAI, what is the telecom subscriber base 151. The Cabinet has cleared the proposal for construction
in India at present? of a 15-km tunnel at Zojila pass involving investment
1) 85.11 cr 2) 90.44 cr 3) 102 cr of `9,090 cr to ensure undisrupted round-the-year
4) 104 cr 5) None of these connectivity. The tunnel will be constructed in
142. What is the current Marginal Standing Facility (MSF) 1) Sikkim 2) Assam
rate after the Reserve Bank of India (RBI) reduced it 3) Himachal Pradesh 4) Jammu and Kashmir
by 25 bps recently (29 Oct 2013)? 5) None of these
1) 8.50 per cent 2) 8.75 per cent 3) 9.00 per cent 152. Which of the following Indian women has been ranked
4) 2.25 per cent 5) None of these fourth on the Fortune’s women business leaders list
143. The Maven mission of the US space agency NASA is for 2013?
aimed to explore which of the following planets? 1) Naina Lal Kidwai 2) Shikha Sharma
1) Mercury 2) Jupiter 3) Mars 3) Chitra Ramkrishna 4) Chanda Kochhar
4) Venus 5) None of these 5) None of these
144. After the introduction of a negative list for 153. The Rural Infrastructure Development Fund (RIDF) is
comprehensive taxation of services sector in Jul 2012, maintained by which of the following banks?
the govt has power to tax all services, except the 1) Reserve Bank of India 2) State Bank of India
____services included in the list.
1) 9 2) 11 3) 13 4) 17 5) None of these 5) None of these
145. Which of the following will not be considered a valid 154. According to the latest report on gender equality
proof of identity while opening an account with a issued by the World Economic Forum (WEF) recently,
bank? India stands at which of the following positions among
1) PAN Card 2) Driving Licence 136 nations?
3) Electricity Bill 4) Voter ID 1) 132 2) 121 3) 101 4) 92 5) None of these
5) None of these 155. Malaysian telecom major Maxis Berhad has pumped
146. The Supreme Court has ruled that in the case of in more than `6,000 cr into the Indian firm in which it
bounced cheques, courts cannot impose a fine of more owns a 74 per cent equity stake. Name the wireless
than _______the amount. telephone operator in which it owns the stake.
1) Twice 2) Thrice 3) Four times 1) MTS 2) Aircel 3) Videocon
4) Five times 5) None of these 4) Uninor 5) None of these
147. The Police Commemoration Day is observed on which 156. The BCCI decided to terminate which of the following
of the following dates every year to mark the teams, leaving the IPL an eight-team affair for the time
martyrdom of police personnel killed at Hot Springs in being?
Ladakh in 1959 while thwarting a Chinese attack? 1) Chennai Super Kings 2) Pune Warriors
1) 20 Oct 2) 21 Oct 3) 22 Oct 3) Rajasthan Royals 4) Sunrisers Hyderabad
4) 25 Oct 5) None of these 5) None of these
148. Which of the following countries hosted the 5th South 157. Who among the following has authored the book
Asian Conference on Sanitation (SACOSAN) titled Bankerupt?
recently? 1) Ravi Subramaniam 2) Anees Salime
3) Sumati Bagrath 4) Husain Haqqani
5) None of these 3) in CPU 4) in RAM
158. Who among the following has won the Indian Grand 5) None of these
Prix Formula One championships recently? 168. The portion of CPU that coordinates the activities of
1) Fernando Alonso 2) Lewis Hamilton all other computer components is the
3) Sebastian Vettel 4) Mark Webber 1) motherboard 2) coordination board
5) None of these 3) control unit 4) arithmetic logic unit
159. The St Jude storm, which left thousands of homes 5) None of these
without power and hundreds of flights grounded, hit 169. ________ is a procedure that requires users to enter
which of the following countries recently? an identification code and a matching password.
1) US 2) China 3) Britain 1) Paging 2) Logging in 3) Time sharing
4) Japan 5) None of these 4) Multitasking 5) None of these
160. Rajendra Yadav, who passed away recently, was a 170. Which device is used as the standard pointing device
1) Music director 2) Renowned Hindi writer in a Graphical User Environment?
3) Environmentalist 4) Cartoonist 1) Keyboard 2) Mouse 3) Joystick
5) None of these 4) Trackball 5) None of these
171. What is ISL used for?
Test-V: Computer Knowledge 1) To allow an Ethernet interface to understand frame
tags
161. Computers always use ________ logic . 2) To make two Ethernet interfaces appear as one
1) Decimal 2) Fuzzy 3) Binary 3) To connect an Ethernet switch with a high-speed
4) All the above 5) None of these core switch such as ATM
162. If you press ________, the cell accepts your typing 4) To allow simultaneous routing and switching
as its contents. 5) None of these
1) Enter 2) Ctrl+Enter 3) Tab 172. Which of the following is NOT associated with
4) Insert 5) None of these Computers?
163. ________ is a set of keywords, symbols, and a system l) Bit 2) Binary 3) Pencil
of rules for constructing statements by which humans 4) Mouse 5) Screen
can communicate the instructions to be executed by a 173. A software tool that helps programmers detect errors
computer. in programming logic and display intermediate
1) A computer program calculation results, is
2) A programming language 1) Compiler 2) Assembler 3) Debugger
3) An assembler 4) Interpreter 5) None of these
4) Syntax 174. Which of the following is true for Dialog Boxes?
5) None of these 1) They allow you to talk to someone.
164. The general term “peripheral equipment” is used for 2) They allow the computer to give you access to
1) any device that is attached to a computer system helpful information.
2) large-scale computer systems 3) They display commands options that are available
3) a program collection for that window.
4) other office equipment not associated with a 4) They allow you to enter more information to clarify
desktop computer a menu choice.
5) None of these 5) Any the above
165. What is the purpose of Software Documenting? 175. What is incorrect about software?
1) Use and maintenance of software 1) It can manipulate computer.
2) To sell software at a handsome price 2) It has programs and instructions.
3) To sell software to many customers 3) It cannot be touched.
4) To show its importance 4) It can be touched.
5) None of these 5) None of these
166. The ________ of software contains lists of commands 176. The three sequential functions of CPU operation are
and options. 1) Decode, fetch, execute 2) Execute, decode, fetch
1) menu bar 2) tool bar 3) title bar 3) Fetch, execute, decode 4) Fetch, decode, execute
4) formula bar 5) None of these 5) None of these
167. If a user needs information instantly available to the 177. Which of the following is NOT one of the four major
CPU, it should be stored data processing functions of a computer?
1) on a CD 2) in secondary storage 1) Collecting data
2) Processing data into information 5) None of these
3) Analysing the data or information 188. Antivirus is a(n) ________ software.
4) Storing the data or information 1) system 2) utility
5) None of these 3) application 4) operating system
178. Computers use the ________ language to process 5) None of these
data. 189. The speedometer of a car represents
1) processing 2) kilobyte 1) digital computer 2) hybrid computer
3) binary 4) representational 3) analog computer 4) micro computer
5) None of these 5) mini computer
179. Which of the following is not an advantage of 190. C++ is
multiprogramming? 1) a procedure-oriented language
1) increased throughput 2) a problem-oriented language
2) shorter response time 3) an object-oriented language
3) decreased OS overhead 4) a programme-oriented language
4) ability to assign priorities to jobs 5) None of these
5) None of these 191. A goal of normalisation is to
180. Daisy wheel printer is a type of ________. 1) minimise the number of relationships
1) matrix printer 2) impact printer 2) minimise the number of entities
3) laser printer 4) manual printer 3) minimise the number of tables
5) line printer 4) minimise the number of redundancies
181. A computer that deals with physical quantities is 5) None of these
known as 192. Granting an outside organisation access to Internet
1) Digital Computer 2) Analog Computer web pages is often implemented using a(n)
3) Hybrid Computer 4) Mainframe Computer 1) extranet 2) intranet 3) internet
5) None of these 4) hacker 5) None of these
182. Which of the following is used to measure the speed 193. The capability of the operation system to enable two
of the processor? or more than two instructions to execute
1) Unit 2) Processing speed simultaneously in a single computer system by using
3) Clock speed 4) Memory multiple CPUs is called
5) None of these 1) multitasking 2) multiprogramming
183. First page of any website is called 3) multiprocessing 4) multiexecution
1) home page 2) web page 5) None of these
3) website 4) All the above 194. A collection of related information sorted and dealt
5) None of these with as a unit is called
184. In database, SQL stands for 1) disk 2) data 3) file
1) Simple Query Language 4) floppy 5) None of these
2) Structure Query Language 195. If soft copy is the intangible output, what is hard copy?
3) Structure Question Language 1) The physical parts of the computer
4) All the above 2) The printed parts of the computer
5) None of these 3) The printed output
185. Which hole is used to mark the location of the first 4) The physical output devices
sector in a soft-sectored disk? 5) None of these
1) Address 2) Location 3) Label 196. A ________ is not a form of biometrics.
4) Primary 5) Index 1) fingerprint 2) password 3) retina scan
186. The process of a computer receiving information from 4) breath scan 5) None of these
a server on the Internet is known as 197. Which type of memory is in Pen drive?
1) pulling 2) downloading 3) pushing 1) Flash memory 2) Cache memory
4) uploading 5) transferring 3) EEPROM 4) RAM
187. Which of the following methods will not enter data 5) None of these
into a cell? 198. Switch and bridge networking devices work on OSI
1) Pressing an arrow key ________.
2) Pressing the Esc key 1) Application Layer 2) Physical Layer
3) Pressing the tab key 3) Network Layer 4) Data link Layer
4) Clicking the enter button to the formula bar 5) None of these
199. A self-replicating program similar to a virus which was 200. Unwanted repetitious messages, such as unsolicited
taken from a 1970s science fiction novel by John Bruner bulk e-mail, is known as
entitled Shockwave Rider is 1) Spam 2) Trash 3) Calibri
1) Bug 2) Vice 3) Lice 4) Courier 5) None of these
4) Worm 5) None of these
Clerk Mock Test 2
10. Suraj walks towards north. After a while, he turns to
Test-I: Reasoning Ability his right and a little further to his left. Finally, after
walking a distance of one kilometre, he turns to his left
1. Four of the following five are alike in a certain way and again. In which direction is Suraj moving now?
so form a group, which is the one that does not belong 1) East 2) West 3) North
to that group? 4) South 5) None of these
1) Uncommon 2) Rare 3) Unique Directions (Q. 11-15): Study the following information
4) Beautiful 5) Exceptional carefully and answer the given questions:
2. If the letters of the word VIRTUAL are arranged R, S, T, U, V and W are six family members. All of them
alphabetically, how many letters will not change their have different professions, viz Dentist, Police, Judge,
positions? Shopkeeper, Clerk and Engineer, but not necessarily in the
1) None 2) One 3) Two same order.
4) Three 5) None of these The Clerk is the grandfather of W, who is a Dentist. U,
3. ‘Natural’ is related to ‘Artificial’ in the same way as who is in Police, is married to R. T, the Shopkeeper, is married
‘Public’ is related to _______. to a Judge. S is mother of W and V. There are two married
1) Close 2) Hide 3) Private couples in the family.
4) Personal 5) None of these 11. What is the profession of V?
4. Pointing to a boy, a man said, “I have no brother but 1) Engineer 2) Police 3) Clerk
that boy’s father is my father’s son.” How is the boy 4) Dentist 5) None of these
related to that man? 12. How is R related to V?
1) Father 2) Grand son 3) Nephew 1) Sister 2) Grandfather
4) Son 5) None of these 3) Father 4) Mother
5. Four of the following five are divisible by a certain 5) None of these
number and hence form a group. Which is the one 13. How many male members are there in the family?
that does not belong to that group? 1) Two 2) Three
1) 89 2) 81 3) 117 3) Four 4) Can’t be determined
4) 63 5) 108 5) None of these
6. How many such digits are there in the number 62143879 14. What is the profession of S?
each of which is as far away from the beginning of the 1) Police 2) Shopkeeper
number as when the digits are rearranged in ascending 3) Judge 4) Clerk
order? 5) None of these
1) None 2) One 3) Two 15. Which of the following is one of the couples in the
4) Three 5) None of these family?
7. In a certain code language DENTICLE is written as 1) RS 2) ST 3) UV
TNEDELIC. How is CONFUSED written in that code 4) WV 5) Can’t be determined
language? Directions (Q. 16-20): In each question below is given
1) ENCODEUS 2) FNOCDEUS a group of letters followed by four combinations of digits/
3) OCFNUS 4) USEDCONF symbols numbered (1), (2), (3) and (4). You have to find out
5) None of these which of the combinations correctly represents the group
8. How many meaningful English words can be formed of letters based on the following coding system and mark
with the letters ABER using each letter only once in the number of that combination as the answer. If none of
each word? the four combinations correctly represents the group of
1) None 2) One 3) Two letters, mark 5), ie ‘None of these’, as the answer.
4) Three 5) None of these
9. What should come next in the following letter series?
RQPONM LQPO NMLP ONML??
1) P, O 2) N, M 3) O, P
4) O, N 5) None of these
Conditions: logically follows from the given statements, disregarding
1) If the first letter is a consonant and the last letter is commonly known facts. Give answer
a vowel their codes are to be interchanged. 1) if only conclusion I follows.
2) If both the first and the last letters are consonants 2) if only conclusion II follows.
both are to be coded as the code for the first letter. 3) if either conclusion I or conclusion II follows.
3) If both the first and the last letters are vowels both 4) if neither conclusion I nor conclusion II follows.
are to be coded as the last letter. 5) if both conclusions I and II follow.
16. MARKET 26. Statements: All hotels are rooms.
1) 4©184 2) 4©184 3) 4©18$ All rooms are buildings.
4) 4©148 5) None of these Conclusions: I. All hotels are buildings.
17. DAIMWU II. All buildings are rooms.
1) 2©749@ 2) @©749@ 3) @©7492 (27-28):
4) 2©7492 5) None of these Statements: Some rivers are hills.
18. AMTHDU No hill is a mountain.
1) @$423@ 2) @4$32@ 3) @4$32© 27. Conclusions: I. Some mountains are not rivers.
4) ©4$32@ 5) None of these II. Some rivers are not mountains.
19. MUPRKE 28. Conclusions: I. Some hills are rivers.
1) 8@518 2) 4@514 3) 4@518 II. Some hills are not rivers.
4) 8@514 5) None of these 29. Statements: All stamps are packets.
20. EWMAPH Some packets are tubes.
1) 89©435 2) 894©58 3) 894©53 Conclusions: I. All tubes being stamps is a
4) 89©458 5) None of these possibility.
Directions (Q. 21-25): In the following questions, the II. All packets being tubes is a
symbols , %, ©, @ and are used with the meanings as possibility.
illustrated below: 30. Statements: Some buckets are doors.
A  B means A is greater than B Some doors are windows.
A % B means A is either greater than or equal B Conclusions: I. Some buckets are windows.
A © B means A is smaller than B II. Some buckets are not windows.
A @ B means A is either smaller than or equal B Directions (Q. 31-35): Study the following information
A B means A is equal to B carefully and answer the given questions:
Now, in each of the given questions, assuming the P, N, U, W, R, I, Q and S are sitting around a circle, not
given statements to be true, find which of the two facing the centre. N is third to the right of P, who is second
conclusions I and II given below them is/are definitely to the right of I. R is second to the left of Q, who is an
true. Give answer immediate neighbour of I. U is second to the right of S.
1) if only conclusion I is true. 31. In which of the following combitions is the third
2) if only conclusion II is true. person second to the left of the second person?
3) if either conclusion I or II is true. 1) SWN
4) if neither conclusion I nor II is true. 2) RIU
5) if both conclusions I and II are true. 3) RNU
21. Statements: M @ N, L  N, M P 4) Can’t be determined
Conclusions: I. N P II. P © N 5) None of these
22. Statements: A @ C, M @ F, C  F 32. Who is on the immediate left of R?
Conclusions: I. M A II. C  M 1) I 2) B 3) U
23. Statements: B @ P, C  N, P N 4) N 5) None of these
Conclusions: I. P © C II. C  B 33. Who is third to the right of U?
24. Statements: K © P, Z  K, K % M 1) Q 2) P 3) W
Conclusions: I. Z M II. Z  M 4) S 5) None of these
25. Statements: Z © P, T M, M % Z 34. Which of the following pairs are the neighbours of N?
Conclusions: I. M  Z II. T % P 1) IS 2) US 3) WR
Direction (Q. 26-30): In each question below are given 4) Can’t be determined 5) None of these
two statements followed by two conclusions numbered I 35. Who among the following is second to the left of S?
and II. You have to take the given statements to be true 1) U 2) N 3) W
even if they seem to be at variance with commonly known 4) P 5) None of these
facts and then decide which of the given conclusions
Directions (Q. 36-40): The following questions are mechanism for their spare cash. And although there are
based on the diagram given below: informal ways to save — joining a local savings club, for
example, or simply stuffing money under the mattress —
owning a cow may be a better option.
That is because most people find spending easier than
saving. Immediate pleasures are easier to grasp than future
joys — and so people make spending decisions that they
later regret. Economists refer to this as “myopia”. Cows
force people not to be myopic. Compared with money held
36. Which of the following groups represents the group in savings accounts, cattle are illiquid assets. Taking cash
of Lions and Tigers but not Dogs? from a cow is harder than taking money from an account.
1) H, I 2) I, D, H 3) G, K, H As a result, temptation spending is trickier.
4) L, H, I 5) None of these The paper has implications for poverty-alleviation
37. Which of the following is/are the group of only Lions? strategies and for financial services in developing countries.
1) D, E 2) K, H 3) J, K Aid programmes that try to reduce poverty by distributing
4) J, K, H, I 5) None of these livestock may be ineffective at raising incomes, if the
38. Which of the following represent(s) only Tigers and returns from owning them are so poor. If cows are used as
Dogs? a means of saving, the spread of mobile banking in places
1) G, L, C 2) C 3) A, B, G, L like India will provide another, better option. Even then the
4) A, L, C 5) None of these problem of temptation spending arises. Dean Karlan, one
39. Which of the following is/are the group of all – Lions, of the authors, is interested in the idea of “commitment
Tigers, and Dogs? savings accounts”, whereby people forgo their right to
1) D, E, F 2) H, D, E, I 3) I, H withdraw funds until they reach a specified level.
4) D, E 5) None of these 41. Why do households buy cows despite the fact that
40. Which of the following groups represent(s) the group the return on cattle are not beneficial?
of Lions and Dogs but not Tigers? (A) To get high-quality milk at home
1) F 2) C 3) A, B (B) For spiritual fulfilment from cow ownership
4) J, K 5) None of these (C) Because of ignorance of economic aspects
1) Only (A) 2) Only (B)
Test-II: English Language 3) Only (C) 4) Only (A) and (B)
5) All (A), (B) and (C)
Directions (Q. 41-50): Read the following passage 42. What is economic myopia? Give your answer in the
carefully and answer the questions given below it. Certain context of the given passage.
words/phrases in the passage are printed in bold to help 1) Spending is easier than saving.
you locate them while answering some of the questions. 2) Spending gives pleasure while saving causes
In India there are about 280m cows. They produce hardship.
valuable things — milk, dung and calves. But cattle are 3) Spending decision for immediate pleasure at the
expensive to keep. The biggest outlay is food — the average cost of future repentance
cow consumes fodder worth about 10,000 rupees a year. 4) Inability to see objects clearly placed at a short
Veterinary costs also add up. distance
These expenses are so high that cows are often a poor 5) None of these
investment. According to a new and splendidly titled NBER 43. Which of the following statements is/are contrary to
paper, which looks at cow and buffalo ownership in rural the facts mentioned in the given passage?
areas of northern India, the average return on a cow is 1) Getting money by selling cows is much easier than
-64 per cent once you factor in the cost of labour. withdrawing it from savings accounts.
If returns on cattle are so bad, why do households 2) Cows force people to be economically short-
buy them? People may not be thinking about economics, sighted.
of course. Hindus may derive spiritual fulfilment from cow 3) Aid programmes that try to reduce poverty by
ownership. Households may prefer to produce high-quality distributing livestock may be ineffective.
milk at home, even if doing so costs more. 4) Only 1) and 2)
But the authors suggest that there may also be sound 5) Only 2) and 3)
economic reasoning behind cow ownership. According to 44. What economic reasoning has the author suggested
ICRIER, a think-tank, only 7 per cent of Indian villages behind cow ownership?
have a bank branch. That means people lack a formal savings
1) Rural people prefer to exchange goods for milk. producing alternatives that are cost-efficient and eco-
2) Cow owners are usually illiterate and hence hesitate friendly. / 5) No error
to go to bank branches. 53. 1) The growing cooperation between India and UK /
3) People lack a formal savings mechanism and 2) has been manifested / 3) in all walks of life, / 4) be it
owning a cow is a better alternative for them. history, politics, culture or language. / 5) No error
4) People think it better to have their own cows than 54. 1) The USA is aiming at / 2) double its bilateral trade /
to purchase milk from vendors. 3) with India / 4) by 2015. / 5) No error
5) All the above 55. 1) The CBI court sentenced four people, including a
45. Which of the following is not true in the context of the government servant, to / 2) four-and-a-half years’
given passage? imprisonment / 3) for illegally procuring contacts from
1) The high expenses to keep cows has become a bad Delhi Jal Board / 4) caused a huge loss to the exchequer.
investment. / 5) No error
2) When the labour cost is also added to the total Directions (Q. 56-60): In each of the following
cost the average return on a cow gives a negative sentences there are certain blank spaces. Below each
return. sentence there are five options denoted by the numbers 1),
3) The biggest expense on keeping a cow comes on 2), 3), 4) and 5). Find out which option can be used to fill up
food. the blank in the sentence in the same sequence to make it
4) The NBER paper is concerned with looking at cows meaningfully complete.
and buffalo ownership in the whole of rural India. 56. Judiciary and the Supreme Court are important
5) None of these institutions to see that the government ________ its
Directions (Q. 46-48): Choose the word/group of citizens and works within the powers allocated to it by
words which is MOST SIMILAR in meaning to the word the constitution.
printed in bold as used in the passage. 1) do not tresspass on the rights of
46. Spare 2) did not tresspass on the rights of
1) temporary 2) regular 3) extra 3) have not tresspass on the rights of
4) scanty 5) meager 4) does not tresspass on the rights of
47. Stuffing 5) can not tresspass on the rights of
1) misusing 2) contents 3) making 57. In public perception, CBI is seen as an ________ of
4) sinking 5) depositing political policy of the government of the day and not
48. Alleviation much faith ________ on its investigations.
1) allotment 2) lessen 3) inclination 1) organisation, assumed 2) asset, put
4) implication 5) allowance 3) office, claimed 4) arrangement, imposed
Directions (Q. 49-50): Choose the word/group of 5) instrument, placed
words which is MOST OPPOSITE in meaning of the word 58. Expressing concern in the wake of clashes in
printed in bold as used in the passage. Muzaffarnagar, President Pranab Mukherjee said parts
49. Outlay of the country had ________ a worrying bout of
1) expense 2) plinth 3) outdo communal violence and ________ on strengthening
4) income 5) external local administration for quickly ________ such
50. Forgo situations.
1) accept 2) reject 3) forget 1) observed, expressed, prohibiting
4) remember 5) renounce 2) experienced, stressed, containing
Directions (Q. 51-55): Read each sentence to find out 3) seen, addressed, stopping
whether there is any grammatical or idiomatic error in it. 4) felt, murmured, managing
The error, if any, will be in one part of the sentence. The 5) marked, exclaimed, revoking
number of that part is the answer. If there is ‘No error’, 59. Two days after Diwali, Delhi continued to be ________
the answer is 5). (Ignore errors of punctuation, if any.) in a thick layer of smog.
51. 1) The Centre plans to form an empowerment group / 1) enveloped 2) capsuled 3) filmed
2) under the Oil Minister / 3) to grant operational 4) shelled 5) enclosed
flexibility / 4) in enforcing oil and gas contracts. / 5) 60. His family has filed a ________ for CBI investigation,
No error which is pending in the Allahabad High Court.
52. 1) The rising trend of constructing green buildings / 1) complaint 2) dossier 3) petition
2) and the scarcity of traditional materials have / 3) 4) record 5) list
opened up a sizeable market for young ventures / 4) Directions (Q. 61-65): In each question below is given
a sentence with four words printed in bold type. These are
numbered as 1), 2), 3) and 4). One of these four words corrupted by them.
printed in bold may be either wrongly spelt or inappropriate (E) It is important to be selective about the kind of
in the context of the sentence. Find out the word which is book that one decides to read.
wrongly spelt or inappropriate, if any. The number of that (F) Reading substandard material is simply a waste
word is your answer. If all the words printed in bold are of time.
correctly spelt and also appropriate in the context of the 66. Which of the following will be the FIRST sentence
sentence, mark 5), ie ‘All correct’, as your answer. after rearrangement?
61. ISRO Chairman dismissed arguments that thespending 1) A 2) B 3) C 4) D 5) E
1) 2) 67. Which of the following will be the SECOND sentence
of `460 crore on the Mars orbiter project was a after rearrangement?
3) 1) A 2) D 3) B 4) C 5) F
wastage of money. All correct 68. Which of the following will be the THIRD sentence
4) 5) after rearrangement?
62. A recent article in the international press on the travails 1) F 2) C 3) D 4) A 5) B
1) 69. Which of the following will be the FOURTH sentence
of small restaurants in Delhi shows that there is a lot after rearrangement?
2) 3) 1) B 2) F 3) E 4) D 5) A
in the industry that needs fixing. All correct 70. Which of the following will be the LAST (SIXTH)
4) 5) sentence after rearrangement?
63. The International Civil Aviation Organisation has 1) D 2) F 3) A 4) E 5) C
forecasted that CO2 emissions from international Directions (Q. 71-80): In the following passage, some
1) 2) of the words have been left out, each of which is indicated
aviation will grow from approximately 400 million by a number. Find the suitable word from the options given
3) 4) against each number and fill up the blanks with
tonnes in 2010 to 650 million tonnes by 2020. appropriate words to make the paragraph meaningfully
All correct complete.
5) Illiterate and weak-minded people all over the world
64. Emphasising that the government is taking a holistic often are a (71) to superstitions. Their lives are ruled by
1) 2) them and they base their actions on the various
view on National Spot Exchange Ltd (NSEL) crisis, superstitious (72). Superstitious people can be called
corporate affairs minister said certain entities were backward. They tend to (73) every (74) irrationally. They
being scruitinised to check whether they had violated (75) a happening illogically and (76) their actions on the
3) 4) (77) of these beliefs. Although most people know that
any laws. All correct superstitions are based on (78) and are nowhere close to
5) the truth, but (79) a few people are (80) guided by
65. The linguistic history of Sanskrit reveals that it is the superstitions.
1) 71. 1) plunder 2) prey 3) hunt
fundamental language of the Indo-European family 4) predator 5) suffering
and has a deep-rooted and entrinsic relationship with 72. 1) laws 2) custom 3) beliefs
2) 3) 4) thinking 5) rules
various Indian international languages. All correct 73. 1) interpret 2) interfere 3) interpose
4) 5) 4) mediate 5) meditate
Directions (Q. 66-70): Rearrange the following six 74. 1) ideals 2) imagination 3) invention
sentences (A), (B), (C), (D), (E) and (F) in the proper 4) discovery 5) phenomenon
sequence to form a meaningful paragraph and then answer 75. 1) assist 2) assess 3) ascribe
the questions given below. 4) support 5) manipulate
(A) Man is constantly trying to run after knowledge 76. 1) drive 2) restrict 3) impose
and answers to the various questions that arise 4) guide 5) expose
in his mind. 77. 1) hypothesis 2) fundamentals 3) ideology
(B) To satisfy his thirst for knowledge and information 4) basis 5) conception
he can always turn to books for help. 78. 1) imagination 2) delusion 3) empathy
(C) The human mind is rational and does not take 4) fear 5) logic
things for guaranteed. 79. 1) quit 2) why 3) quite
(D) If we read books that are trash, we are liable to be 4) simply 5) never
80. 1) somehow 2) unusually 3) uptil 4) 1214 5) 2160
4) yet 5) still 96. A trader purchased 200 bags of pulse at `1100 per
bag. He sold 80 bags at 25% profit and 60 bags at 8%
Test-III: Quantitative Aptitude loss. At what rate should the remaining bags be sold
so as to gain 10% on the whole transaction?
Directions (Q. 81-85): What should come in place of 1) `1240 2) `1350 3) `1688
question mark (?) in the following equations? 4) `1188 5) `1488
81. 35962.231 – 4350.972 + 3340.674 – 22387.584 + 33.85 = ? 97. A container contained 100 litres of milk. From this
1) 12598.199 2) 12671.241 3) 12971.714 container 9 litres of milk was replaced by water. This
4) 12868.465 5) 12752.680 process was repeated twice. How much milk is now
82. 23% of 460 × 35% of 270 × ?% of 172 = 429918.3 present in the container?
1) 28 2) 40 3) 25 4) 29 5) 39 1) 84.22 litres 2) 82.81 litres 3) 86.22 litres
83. 3939 ÷ 3 + 6363 ÷ 3 – 9696 ÷ 6 + 123 × 2 = ? 4) 80.14 litres 5) 78.90 litres
1) 2064 2) 1572 3) 1464 4) 1864 5) 1564 98. The area of a rectangle is equal to the area of a circle
84. 34 × 313 + 35 – ? = 2627 + 35 – 23 × 2 whose radius is 28 cm. If the ratio of the length to the
1) 8071 2) 8051 3) 8041 breadth of the rectangle is 14 : 11, what is its perimeter?
4) 8031 5) 8061 1) 250 cm 2) 222 cm 3) 246 cm
85. 2.5 × 4.5 × (28 ÷ 2.5) – (3.5 × 2) + 7.1 – 1.2 = ? 4) 249 cm 5) 200 cm
1) 125.9 2) 126.9 3) 127.9 99. Two dice are thrown together and the sum of the digits
4) 124.9 5) 123.9 observed comes to 10. What is the probability of
Directions (Q. 86-90): What approximate value should showing 4 on one of the dice?
come in place of question mark (?) in the following
1 1 2 1 2
questions? (Note: You are not expected to calculate the 1) 2) 3) 4) 5)
exact value.) 12 18 13 14 15
86. 98.3462 × 125.9251 + 235.0051 – 89.0210 = ? 100. A man invests equal sums at the rate of 10% per annum
1) 12682 2) 12592 3) 12762 compound interest and simple interest respectively
4) 12532 5) 12812 for 2 years. After 2 years, the difference between the
87. 12323 ÷ 23 × 25 + 28.056 – 23.06 = ? compound interest and the simple interest is `1000.
1) 14425.75 2) 13399.5 3) 13541.65 Then the sum is
4) 14465.25 5) 13714.5 1) `100500 2) `110000 3) `120000
88. 9929305.625 – 629610.0021 + 32965.01012 = ? 4) `100000 5) `90000
1) 933335.4 2) 9332770.5 3) 932065.6 101. Satshivam can finish a work in 18 days. Altmash takes
4) 9326606 5) 9332660.6 2.5 times more time than Balasubramanyam and
89. (204.23)2 – (103.15)2 = ? Balasubramanyam takes half as many days as
1) 31068 2) 32068 3) 3100 Satshivam to finish the work alone. They work in pairs
4) 32591 5) 31465 starting with Satshivam and Altmash on the first day,
90. 92687.0051 ÷ 39.0261 = ? Altmash and Balasubramanyam on the 2nd day and
1) 2441.7 2) 2565.6 3) 2376.5 Satshivam and Balasubramanyam on the 3rd day, and
4) 2642.7 5) 2546.5 so on. How many days will be required to finish the
Directions (Q. 91-95): What should come in place of work?
question mark (?) in the following number series? 6 7 5
91. 3 5 13 ? 177 891 5353 1) 6 days 2) 7 days 3) 7 days
13 13 14
1) 46 2) 43 3) 45
4) 48 5) 50 5 6
4) 7 days 5) 6 days
92. 9 11 8 12 ? 13 6 13 11
1) 10 2) 5 3) 8 4) 4 5) 7 102. Three years ago Diwakar’s age was 7 times that of
93. 10 11 26 87 364 ? 11106 Rituraj. Three years hence Diwakar’s age will be 4 times
1) 1845 2) 1946 3) 2050 that of Rituraj. The present age of Rituraj is
4) 2112 5) 1521 1) 11 years 2) 7 years 3) 9 years
94. ? 41 83 167 335 671 1343 4) 21 years 5) 13 years
1) 18 2) 17 3) 20 103. A man swims downstream from one point to another
4) 22 5) 24 which is 6 km apart in 1.5 hours. It covers the same
95. 15120 ? 360 72 18 6 3 distance upstream in 2 hours. Find the speed of the
1) 2240 2) 2540 3) 2050
man in still water. a profit of 8% and 6% in the second and the third year
1) 4 kmph 2) 3.5 kmph 3) 4.2 kmph respectively. Find the net profit for the entire period of
4) 4.4 kmph 5) 4.5 kmph three years.
104. The ratio of the monthly income to the monthly 1) `1619.86 2) `1250.90 3) `1152.53
savings of a family is 8 : 3. If the monthly expense is 4) `1352.71 5) `1560.64
`24000 then find the monthly income. 114. The salary of a person is `15,000. After 6 months it
1)`28500 2)`29600 3)`36700 was increased by 25% and after another 3 months it
4)`34200 5)`38400 was decreased by 15%. Find the salary of the person
105. Two buses 1200 km apart are moving towards each after 9 months.
other. One of them is moving at a speed of 120 kmph 1) `15940.5 2) `15950.5 3) `15937.5
and the other at 80 kmph. After what time will they 4) `15960.5 5) `15714.5
meet each other? 115. In an election there were two candiates – Sushma and
1) 6 hours 2) 8 hours 3) 7 hours Sonia. The total number of votes in their constituency
4) 5 hours 5) 4.5 hours was 30,000, and 70% of the total votes were polled. If
106. Find the missing number if the average of all the eight 40% of the voters cast their votes in favour of Sonia,
numbers is 714. how many votes were received by Sushma?
827 232 465 968 _____ 341 476 599 1) 14500 2) 12600 3) 11900
1) 1392 2) 1591 3) 1602 4) 13700 5) 10500
4) 1804 5) 1050 116. The ratio of the radius of a cylinder to that of a cone is
107. What will come in place of both question marks (?) in 3 : 4, and the ratio of their heights is 4 : 5. What will be
the following equation? the ratio of their volumes?
(?)1.3 48 1) 24 : 31 2) 23 : 25 3) 27 : 20
4) 21 : 26 5) 18 : 23
972 (?)1.7 117. Two pipes Z and Y can fill a cistern in 42 minutes and
1) 42 2) 48 3) 34 4) 36 5) 52
48 minutes respectively. There is also an outlet pipe P.
108. In how many different ways can the letters of the word If all the three pipes are opened together, the tank gets
‘ACHIEVEMENT’ be arranged? filled completely in 30 minutes. How much time will
1) 721800 2) 684600 3) 6652800
pipe P take to empty the full tank?
4) 765800 5) 706800
4 5 6
109. Divide `5204 between Ramesh and Shyam so that 1) 75 min 2) 78 min 3) 80 min
Ramesh’s share at the end of 8 years may be equal to 21 27 23
Shyam’s share at the end of 10 years, compound 7 8
interest being 4% per annum. Find Ramesh’s share. 4) 84 min 5) 88 min
29 19
1)`2504 2)`2604 3)`2600 118. Prashant is twice as old as Mohit and half as old as
4)`2704 5)`2028 Ankur. The sum of Ankur’s and Mohit's age is 85 years.
110. The compound interest on a certain sum in 3 years at What is the age of Prashant (in years)?
the rate of 8% per annum is how much percentage 1) 34 2) 35 3) 33
more than its simple interest? 4) 36 5) 37
1) 9.17% 2) 4.68% 3) 4.65% 119. The ratio of two numbers is 3 : 7. If each number is
4) 7.14% 5) 8.21% increased by 10, the ratio becomes 7 : 13. Find the
111. Two trains moving in opposite directions cross a man difference of the two numbers.
in 27 seconds and 17 seconds respectively, while they 1) 28 2) 26 3) 24 4) 25 5) 27
cross each other in 23 seconds. What is the ratio of 120. Narsimha finishes a work in 15 days by working 10
their speeds? hours a day. Rajshekhar finishes it in 8 days by working
1) 1 : 2 2) 8 : 11 3) 7 : 5 11 hours a day. Find in how many days both can finish
4) 2 : 3 5) 2 : 5 it working together for 12 hours a day.
112. There is a plot 10,000 m2 which is to be sold at the rate
of `2000 per square metre. If a man has `2,50,000 with 1) 3
35
days 2) 4
71
days 3) 3
12
days
him, find the percentage of land that he can purchase 71 149 35
with this amount.
74 77
1) 1.04% 2) 2.1% 3) 1.25% 4) 4 days 5) 4 days
4) 3.1% 5) 4.2% 119 113
113. Dhirubhai started a business with a sum of `18,500
and incurred a loss of 5% in the first year and earned
chairperson and managing director (CMD) of the
Test-IV: General Awareness recently launched Bharatiya Mahila Bank?
1) Naina Lal Kidwai
(with Special Reference to Banking Industry)
2) Shikha Sharma
121. Who among the following became the first member of 3) Usha Ananthasubramanian
the Parliament to lose his seat after the Supreme Court 4) Usha Sangwan
struck down a provision protecting convicted 5) None of these
lawmakers from disqualification on the ground of 129. Which of the following high courts in India ruled
pendency of appeal in higher courts? recently that the Central Bureau of Investigation (CBI)
1) Rashid Masood 2) Lalu Prasad Yadav is unconstitutional?
3) Pappu Yadav 4) Jagdish Sharma 1) Patna High Court 2) Gauhati High Court
5) None of these 3) Allahabad High Court 4) Delhi High Court
122. The cricket legend Sachin Tendulkar and acclaimed 5) None of these
scientist CNR Rao have been chosen for which of the 130. Who among the following has been chosen to be
following prestigious civilian awards of the country awarded the coveted Indira Gandhi Prize for Peace,
recently? Disarmament and Development for 2013?
1) Padma Shree 2) Desh Ratna 1) David Cameron 2) Barack Obama
3) Padma Vibhushan 4) Padma Bhushan 3) Angela Merkel 4) Hillary Rodham Clinton
5) Bharat Ratna 5) None of these
123. Doris Lessing, who died recently, was a Nobel prize- 131. Which of the following is the first foreign company to
winner and a/an own 100 per cent stake in an Indian telecommunication
1) Biologist 2) Economist company as the Foreign Investment Promotion Board
3) Physicist 4) Chemist (FIPB) cleared its proposal to increase its stake in its
5) Author local unit from 74% to 100% recently?
124. Who among the following is slated to be the next 1) Maxis 2) SingTel 3) Sistema
President of the Federation of Indian Chambers of 4) Vodafone 5) None of these
Commerce and Industry (FICCI) in 2014? 132. According to a recent survey by the National Sample
1) Anil Ambani 2) Siddhartha Birla Survey Organisation (NSSO), which of the following
3) Sunil Bharti Mittal 4) Kumar Mangalam Birla cities in India recorded the highest unemployment rate
5) None of these among males in 2009-10 in cities with more than one
125. What is mis-selling? million population?
1) Selling a product which is not suitable for the 1) Bhopal 2) Kolkata 3) Patna
buyer's needs 4) Bhubaneswar 5) None of these
2) Selling a bad product 133. As per the news published in financial newspapers,
3) Selling a new product to an existing customer the govt is planning capital infusion in public sector
4) Preventing buyers from buying products of their banks (PSBs) to help them meet regulatory norms and
choice maintain loan growth to economically important
5) None of these sectors. What is the amount to be infused?
126. In a major relief to the dual-technology (GSM and 1) `10,000 crore 2) `11,000 crore
CDMA both) telecom licence holders, the Department 3) `14,000 crore 4) `19,000 crore
of Telecommunications (DoT) has decided not to scrap 5) None of these
their licences. Which of the following operators hold 134. After the deadly cyclone Phailin, the Prime Minister
a double-technology licence? Manmohan Singh announced an interim assistance
1) Reliance Communications of `1,000 cr for Andhra Pradesh for relief recently. What
2) Tata Teleservices amount did Odisha get for the same?
3) MTS 1) `1,000 cr 2) `1,200 cr 3) `1,500 cr
4) Only 1) and 2) 4) `1,800 cr 5) None of these
5) All the above 135. Narrow money is represented by which of the
127. The women's hockey team of India lost to which of following?
the following teams in the final of the Asian Champions 1) M1 2) M2 3) M3
Trophy hockey tournament in New Delhi recently? 4) M4 5) None of these
1) Malaysia 2) Japan 3) Bangladesh 136. Under which of the following markets, the funds are
4) Indonesia 5) None of these transacted by banks on an overnight basis?
128. Who among the following has been appointed as the 1) Money market 2) Call money market
3) Term money market 4) Capital market `12,000 crore through QIP this fiscal. What does QIP
5) None of these stand for?
137. Reliance Industries Ltd (RIL) became the first private- 1) Qualified Institutional Placement
sector firm to report revenues exceeding `1 lakh crore 2) Quality Institutional Placement
in a quarter. Doing so, it joined which of the following 3) Quantitative Institutional Placement
public sector companies, which is the only other Indian 4) Qualified Insured Placement
company with the distinction so far? 5) None of these
1) ONGC 2) IOC 3) Coal India 145. In a recent circular, the Central Board of Direct Taxes
4) GAIL 5) None of these (CBDT) has exempted senior citizens from paying taxes
138. India signed four agreements including one on defence on income earned through which of the following
cooperation and another on establishment of a Joint means?
Commission with which of the following countries 1) Pension
recently? It is the fastest growing Latin American 2) Interest earned on term deposits
nation at present. 3) Reverse Mortgage Scheme
1) Brazil 2) Argentina 3) Mexico 4) House rent
4) Peru 5) None of these 5) None of these
139. Who among the following has been appointed as the 146. The book Shadow Play has been authored by
first Indian envoy to the 10-nation ASEAN bloc 1) Shashi Deshpande
recently after India decided to set up a separate mission 2) Rajmohan Gandhi
to the group? 3) Nirmalangshu Mukherji
1) TV Rajeswar 2) Suresh Reddy 4) Ramachandra Guha
3) Nirupama Rao 4) Chokila Iyer 5) None of these
5) Ranjan Mathai 147. Which of the following cricketers has been ranked
140. Which of the following films has won the best film numero uno in the Reliance ICC Player Ranking for
award (The Golden Gateway of India award) at the Test all-rounders released recently?
15th Mumbai Film Festival recently? 1) Ravindra Jadeja
1) Qissa 2) Katiyabaaz 2) Ravichandran Ashwin
3) Fandry 4) Ilo Ilo 3) Shakib Al-Hasan
5) La Juala de Oro 4) Jacques Kallis
141. India is in process to develop the fourth generation of 5) None of these
the anti-tank missile Javelins in association with which 148. India's Vice-President Hamid Ansari returned home
of the following countries? recently after an eight-day official visit to three nations,
1) US 2) Israel 3) Russia including Cuba, Peru and
4) France 5) None of these 1) Chile 2) Hungary
142. Who among the following is scheduled to take over 3) United Kingdom 4) Algeria
as the next chief of the Indian Air Force after the current 5) None of these
Air Chief Marshal NAK Browne retires on 31 Dec 149. Who among the following has been crowned Miss
2013? Asia Pacific World 2013 recently?
1) Air Marshal Pradeep Vasant Naik 1) Nehal Bhogaita 2) Srishti Rana
2) Air Marshal Fali Homi Major 3) Navneet Kaur Dhillon 4) Vanya Mishra
3) Air Marshal Arup Raha 5) None of these
4) Air Marshal Pranab Kumar Barbora 150. Who among the following has been sworn in as the
5) None of these new president of the Maldives recently?
143. The overall asset restructuring in the banking system 1) Mohamed Nasheed
has reached an alarming level at present in India. 2) Qasim Ibrahim
Which of the following is not a facility provided to 3) Mohammed Waheed Hassan
stressed borrowers through asset restructuring by 4) Abdulla Yameen Abdul Gayoom
banks? 5) None of these
1) Decreasing repayment period 151. Which of the following organisations has launched a
2) Offering a moratorium nationwide campaign named 'Take the Poo to Loo' in
3) Reducing lending rates India in an effort to raise awareness against open
4) All the above defecation and related health issues?
5) Both 2) and 3) 1) USAIDS 2) WHO 3) Red Cross
144. The state-run IDBI bank is planning to raise up to 4) UNICEF 5) None of these
152. Which of the following companies is preparing to 1) Vietnam 2) China
takeover the Europe's largest mobile carrier Vodafone 3) Japan 4) The Philippines
in 2014? 5) None of these
1) Google 2) AT&T
3) China Mobile 4) Bharti Airtel Test-V: Computer Knowledge
5) None of these
153. Which of the following companies is all set to bring 161. A simple automated program that scans the web for
India's first branded apple called 'Saboro' to the market? specific data and inserts into a designated database.
1) Tata 2) Mahindra & Mahindra 1) Web mining
3) ITC 4) HUL 2) Web database integration
5) None of these 3) Web crawler
154. The Repco Bank, which operates in Tamil Nadu, 4) Web scanning
Andhra Pradesh, Karnataka, Kerala, and the Union 5) Web searching
Territory of Puducherry, is an enterprise under the 162. In web searching a symbol or keyword that instructs
Union Ministry of the search software to make sure that any retrieved
1) Home Affairs records contain a certain word, is called
2) Finance 1) Data operator 2) Increment operator
3) Commerce and Industry 3) Decrement operator 4) Inclusion operator
4) Social Justice and Empowerment 5) Search operator
5) None of these 163. A database that spreads information across different
155. The ever-increasing NPAs of the commercial banks tables while maintaining links between them is referred
have become a serious challenge for the RBI at present. to as
Currently, a loan is considered to be an NPA if its 1) Relational database 2) Rotational database
instalment is not paid for 3) Database connection 4) Tabular database
1) One month 2) Two months 5) None of these
3) Three months 4) Six months 164. All the deleted files go to
5) None of these 1) Recycle Bin 2) Task Bar 3) Tool Bar
156. Which of the following Indian mobile operators has 4) My Computer 5) None of these
started offering Apple's iPhone 5S and iPhone 5C 165. When you are selecting a mouse for a particular
phones at monthly instalments with free unlimited call computer system, what is the most important
and data usages? consideration?
1) Airtel 2) Idea 1) The type of drivers that come with the mouse
3) Aircel 4) Reliance Communications 2) The length of the mouse cord
5) None of these 3) The type of connector the mouse is equipped with
157. India donated 2,500 metric tonnes of wheat, worth 4) The number of buttons the mouse has
USD 2 million, to which of the following countries 5) None of these
recently? 166. Which of the following objects is passed to a Java
1) Ethiopia 2) Chad Bean when one of its properties is set via a JSP action?
3) Kenya 4) Yemen 1) Servlet Request
5) None of these 2) Http Servlet Request
158. William C Lowe, who died recently, is known as 3) Servlet Response
1) the father of IBM personal computer 4) Http Servlet Response
2) the father of Apple personal computer 5) None of these
3) the father of HP personal computer 167. The lifespan of a hard disk drive is
4) the father of Dell personal computer 1) approximately one year
5) None of these 2) approximately two years
159. The International Day of Rural Women is observed 3) approximately five years
on which of the following dates? 4) approximately twenty five years
1) 15 Sep 2) 15 Oct 5) almost unlimited
3) 15 Nov 4) 15 Dec 168. A type of program that is installed on your personal
5) None of these computers to collect information about users is called
160. The super typhoon 'Haiyan', one of the most powerful 1) Adware 2) Malware 3) Spyware
storms ever recorded, hit which of the following 4) Hacker 5) Cracker
countries recently, killing at least 10,000 people? 169. The computers which are made by taking the features
of analog and digital computers are called ________ 5) None of these
computers. 180. A ________ is a non-impact printer that can produce
1) mixed 2) micro 3) hybrid very high-quality, letter-perfect printing.
4) advance 5) None of these 1) daisy wheel printe 2) laser printer
170. Value added network 3) dot-matrix printer 4) thermal printer
1) is used for voice video services 5) drum printer
2) provides exchange of information like invoices, sale- 181. The DBA is
purchase order, etc. 1) a computer device
3) is used for connecting private and public mail 2) a person
system 3) a communication technique
4) is used for fax machine 4) All the above
5) All the above 5) None of these
171. The function of a protocol is 182. A micro processor is
1) data sequencing 2) data routing 1) an IC
3) flow control 4) error control 2) controlled by a program
5) All the above 3) relatively less expensive
172. Data that uses 4 bytes is called 4) All the above
1) data field 2) character 5) None of these
3) word 4) double word 183. Which of the following methods cannot be used to
5) item edit the contents of a cell?
173. The way a particular application views the data from 1) Clicking the formula bar
the database that the application uses is a 2) Pressing the F2 key
1) module 2) schema 3) forms 3) Pressing the Alt key
4) sub-schema 5) relational model 4) Double-clicking the cell
174. The ALU of a computer responds to the commands 5) None of these
coming from 184. What is the primary requisite of a good computer
1) control section 2) cache memory programmer?
3) primary memory 4) external memory 1) Mathematical background
5) None of these 2) Artistic mind
175. A computer understands capital letter by which of the 3) Logical mind
following values? 4) Scientific knowledge
l) A to Z 50 to 100 2) A to Z 65 to 98 5) Clear understanding
3) A to K 65 to 90 4) A to Z 97 to 122 185. Utility Software is used for
5) A to Z 65 to 90 1) Security purpose
176. Which is the first Indian web browser? 2) Special purpose
l) Apple Safari 2) Epic 3) Supporting user and hardwares
3) Google Chrome 4) Netscape 4) All 1), 2) and 3) are correct
5) None of these 5) None of these
177. The ability to convey a message without disclosing 186. A record of columns in DBMS is called
your name or identity is called 1) Tuple 2) Matrix 3) Field
1) privacy 2) security 4) Label 5) None of these
3) encryption 4) anonymity 187. The term ________ designates equipment that might
5) authentication be added to a computer system to enhance its
178. A computer program that scans resumes and is set to functionality.
locate keywords is called a(n) 1) digital device 2) system add-on
1) web interview 2) web discussion 3) disk pack 4) peripheral device
3) web conference 4) information system 5) None of these
5) application tracking system (ATS) 188. The instructions that tell a computer how to carry out
179. POS data entry system is used most extensively by the processing tasks are referred to as computer
the 1) programs 2) processors
1) grocery industry 3) input devices 4) memory modules
2) railroad industry 5) None of these
3) word-processing industry 189. A modem
4) banking industry 1) translates analog signals from a computer into digital
signals that can travel along conventional 2) does not use magnetic media
telephone lines. 3) consists of four main types of devices
2) translates digital signals from a computer into 4) does not store information for later retrieval
analog signals that can travel along conventional 5) None of these
telephone lines. 195. The speed at which the monitor accepts data is called
3) demodulates digital signals from a computer. 1) Bandwidth 2) Interlacing
4) modulates signals from an analog telephone line. 3) Response time 4) Scanning
5) None of these 5) Maximum speed
190. What is the best way to have a data and the slide 196. Computer programs are also known as
number appear on every slide? 1) operating systems 2) documents
1) Choose Tools, Header and Footer, click Slide tab, 3) peripherals 4) applications
select the desired options, click Apply to All 5) None of these
2) Choose Insert, Header and Footer, click Slide tab, 197. The two kinds of main memory are
select the desired options, click Apply to All 1) Primary and Secondary
3) Choose View, Header and Footer, click Slide tab, 2) Random and sequential
select the desired options, click Apply to All 3) ROM and RAM
4) Choose File, Header and Footer, click Slide tab, 4) All the above
select the desired options, click Apply to All 5) None of these
5) None of these 198. An output device that uses words or messages
191. Small application programs that run on a Web page recorded on a magnetic medium to produce audio
and may ensure a form is completed properly or response is
provide animation are known as 1) voice band
1) flash 2) spiders 3) cookies 2) magnetic tape
4) applets 5) sparks 3) voice response unit
192. If you do not want to select any option after opening 4) voice recognition unit
a menu, then click menu title again or press key 5) None of these
________ to close the menu. 199. Which of the following is used to move down a page
1) Shift 2) Tab 3) Escape in a document?
4) F1 5) None of these 1) Jump 2) Fly 3) Wriggle
193. A powerful key that lets you exit a program when 4) Scroll 5) None of these
pushed: 200. The primary device that a computer uses to store
1) arrow keys 2) spacebar information is
3) escape key 4) return key 1) TV 2) storehouse 3) desk
5) None of these 4) hard drive 5) None of these
194. Secondary storage
1) does not require constant power
18. 2; Condition (3) applies 26. 1; All hotels are rooms (A) + All rooms
Answers are buildings (A) = A + A = A = All hotels are
1. 4; Except beautiful, all others are buildings. Hence, conclusion I follows.
synonyms. Again, All rooms are buildings conversion
Some buildings are rooms. Hence,
2. 2; 19. 4; Condition (1) applies
conclusion II does not follow.
27. 2; Some rivers are hills (I) + No hill is a
mountain (E) = I + E = O = Some rivers are
There is only one such letter, I. not mountains. Hence, conclusion II follows.
3. 3; Natural is opposite of Artificial. But I does not follow.
20. 3;
Similarly, Public is opposite of Private. 28. 1; Some rivers are hills conversion
4. 4; Boy’s father = Man’s father’s son Some hills are rivers. Hence, conclusion I
= man follows but II does not follow.
The man is the boy’s father. (21-25): Given, 29. 5; There is no negative statement. Thus
The boy is the man’s son.  >, % ,© <, @ , = the possibilities exist in I and II. Hence, both
5. 1; Rest are divisible by 9. 21. 3; Given statements: M N ... (i) conclusions I and II follow.
L>N ... (ii) 30. 3; Some buckets are doors (I) + Some
6. 4; doors are windows (I) = I + I = No conclusion.
M=P ... (iii)
Combining all these statements, we get But the conclusion are complementary.
Hence, either conclusion I or conclusion II
7. 2; follows.
(31-35):
Check for I. N = P may be true.
Check for II. P < N may be true.
Hence, either conclusion I or conclusion II
Similarly, is true.
22. 2; Given statements:A C ... (i)
M F ... (ii)
C>F ... (iii)
8. 3; The meaningful words that can be Combining all these statements, we get
formed are BEAR and BARE. 31. 2 32. 3 33. 1 34. 2 35. 4
A C>F M
9. 4; RQPONML/QPONML/PONML/ON 36. 1 37. 3 38. 2 39. 4 40. 1
Check for I. We can’t compare M and A.
41. 5 42. 3 43. 4 44. 3 45. 4
Check for II. C > M is true. Hence,
10. 2; 46. 3 47. 5 48. 2 49. 4 50. 1
conclusion II is true.
51. 1; Replace ‘empowerment’ with
23. 5; Given statements: B P ... (i)
‘empowered’
C > N ... (ii)
52. 5
P = N ... (iii)
53. 1; Place ‘the’ before ‘UK’
Combining all these statements, we get
54. 2; Replace ‘double’ with ‘doubling’
Hence, Suraj is moving to the west. 55. 4; Replace ‘caused’ with ‘causing’
(11-15): 56. 4 57. 5 58. 2 59. 1 60. 3
61. 4; Replace ‘wastage’ with ‘waste’
Person Profession Gender
62. 5
R Clerk Male 63. 1; Replace ‘forecasted’ with ‘forecast’
U Police Female Hence, both conclusions I and II are true. 64. 3; Replace ‘scruitinised’ with ‘scrutinised’
V Engineer – 24. 2; Given statements: K < P ... (i) 65. 3; Replace ‘entrinsic’ with ‘intrinsic’
Z > K ... (ii) (66-70): CABEDF
W Dentist –
K M ... (iii) 66. 3 67. 1 68. 5 69. 3 70. 2
S Judge Female Combining all these statements, we get 71. 2 72. 3 73. 1 74. 5 75. 2
T Shopkeeper Male 76. 4 77. 4 78. 1 79. 3 80. 5
81. 1; ? = 35962.231 – 4350.972 + 3340.674
– 22387.584 + 33.85
= 35962.231 + 3340.674.674 + 33.85 –
Check for I. Z M. Hence, conclusion I is
(4350.972 + 22387.584) = 12598.199
not true.
Check for II. Z > M. Hence, conclusion II 23 270 ?
is true. 82. 3; 460 35 172
100 100 100
25. 4; Given statements: Z < P ... (i)
= 429918.3
11. 1 12. 2 13. 4 14. 3 15. 2 T = M ... (ii)
or, 105.8 × 94.5 × ? × 1.72 = 429918.3
16. 1; Condition (2) applies. M Z ... (iii)
or, ? × 17196.732 = 429918.3
Combining all these statements, we get
or, ? = 25
83. 1; 3939 ÷ 3 + 6363 ÷ 3 – 9696 ÷ 6 + 123
× 2 = 1313 + 2121 – 1616 + 246 = 2064
17. 3; Condition (1) applies. 84. 5; ? = 34 × 313 + 35 – ?
= 2627 + 35 – 23 × 2
Hence, conclusion I is not true. Again, we or, 10642 + 35 – ? = 2662 – 46
can’t compare T and P. Hence, conclusion or, 10677 – ? = 2616
II is not true. or, ? = 10677 – 2616 = 8061
85. 4; ? = 2.5 × 4.5 × (28 ÷ 2.5) – (3.5 × 2) 2464 years
+ 7.1 – 1.2 = 2.5 × 4.5 × 11.2 – 7 + 7.1 – 1.2 x2 = = 16 3 years before the age of Rituraj = (x – 3)
14 11
= 126 – 7 + 7.1 – 1.2 = 124.9 x=4 years and the age of Diwakar = 7(x – 3)
86. 4; ? = 98.3462 × 125.9251 + 235.0051 Now, perimeter of rectangle years
– 89.0201 = 98.3 × 126 + 235 – 89 = 2(length + breadth) After 3 years age of Rituraj = (x + 3) years
= 12385.8 + 235 – 89 12386 + 235 – 89 Length of rectangle = 4 × 14 = 56 cm, and age of Diwakar = 4(x + 3) years
12532 breadth = 11 × 4 = 44 cm Now,
12323 Perimeter = 2(56 + 44) cm = 200 cm or, 7(x – 3) + 6 = 4(x + 3)
87. 2; 25 28.056 23.06
23 99. 2; n(S) = 6 × 6 = 36 or, 7x – 21 + 6 = 4x + 12
= 13394.5 – 28 – 23 = 13399.5 Let E be the event that the sum of the num- or, 7x – 4x = 12 + 21 – 6
88. 5; 9929305.625 – 629610.0021 + bers on the two faces is 10. or, 3x = 27
32965.01012 = 9929305.6 – 629610 + E = {(4, 6) (6, 4)} or, x = 9
32965 = 9332660.6 n(E) = 2 Present age of Rituraj = 9 years.
89. 1; (204.23) 2 – (103.15) 2 2 1
103. 2; Speed of man downstream
= (204.2) 2 – (103.1) 2 = 41697.6 – 10629.6 Hence, P(E) = 6
36 18 = kmph = 4 kmph
= 31068
100. 4; Let the sum be `x. 1.5
90. 3; 92687.0051 ÷ 39.0261 = 92687 ÷ 39
According to the question, 6
= 2376.5 2 Speed of man upstream = = 3 kmph
91. 2; The series is 3 × 1 + 2 = 5, 5 × 2 + 3 10 x 2 10 2
x 1 x 1000
= 13, 13 × 3 + 4 = 43, 43 × 4 + 5 = 177, 100 100
177 × 5 + 6 = 891, 891 × 6 + 7 = 5353 1
11 11 x or, Speed of man = (4 +3) kmph
92. 5; The series is 9, 9 + 2 = 11, 11 – 3 = 8, or, x x 1000 2
8 + 4 = 12, 12 – 5 = 7, 7 + 6 = 13, 10 10 5
= 3.5 kmph
13 – 7 = 6 (105 100)x 104. 5; Let the monthly income of the fam-
or, 1000
93. 1; The series is 10, 10 × 1 + 1 2 = 11, 500 ily be `8x and the monthly savings be `3x.
11 × 2 + 2 2 = 26, 26 × 3 + 32 = 87, 1000 500 Monthly expense = `(8x – 3x) = `5x
87 × 4 + 4 2 = 364, 364 × 5 + 5 2 = 1845, or, x = = `100000
5 24000
1845 × 6 + 6 2 = 11106 1 Now, 5x = 24000 = `4800
or, x
94. 3; The series is 20, 20 × 2 + 1 = 41, 101. 3; Satshivam can do work in 1 day.. 5
18
41 × 2 + 1 = 83, 83 × 2 + 1 = 167, Monthly income = 8 × `4800 = `38400
167 × 2 + 1 = 335, 335 × 2 + 1 = 671, 1 105. 1; Time taken by buses to meet each
Balasubramanyam can do work in 1
671 × 2 + 1 = 1343 9 other
15120 day.
95. 5; The series is 15120, = 2160, 1200
7 1 1 2 6 hours
Altmash can do ( = =) (120 80)
2160 360 72 9 2.5 22.5 45
360 , 72, 18 , work in 1 day. 106. 4; Let the missing number be x.
6 5 4
First day Satshivam and Altmash do
18 6 1
6, 3 1 2 1 Then, × (827 + 232 + 465 + 968 + x +
3 2 work. 8
96. 4; Selling price of 80 bags at 25% profit 18 45 10
341 + 476 + 599) = 714
125 Second day Altmash and Balasubramanyam or, x + 3908 = 714 × 8
= ` 80 1100 = `110000 2 1 7
100 or, x + 3908 = 5712
Selling price of 60 bags at 8% loss do work.
45 9 45 or, x = 5712 – 3908 = 1804
92 Third day Balasubramanyam and Satshivam
= ` 1100 60 = `60720 (?)1.3 48
100 1 1 1 107. 4;
Selling price of whole transactions to gain do work. 972 (?)1.7
9 18 6
110 Work done in 3 days or, (?)1.3 × (?)1.7 = 48 × 972
10% = 1100 200 = `242000 or, (?)1.3 + 1.7 = 46656
100 1 7 1 9 14 15 38 19
Selling price of remaining 60 bags = or, (?)3 = 46656 = 36 × 36 × 36 = 36 3
= `(242000 – 60720 – 110000) = `71280 10 45 6 90 90 45 or, ? = 36
19 38 108. 3; The word ‘ACHIEVEMENT’ has
71280 Work done in 6 days = 2
Selling price per bag = = `1188. 45 45 11 letters and E appears 3 times.
60 Required number of arrangements
38 7
97. 2; Required amount of milk Remaining work = 1
45 45 11
9
2
Work remaining after 7 day s = 6652800
100 1 3
= 100 litres
7 1 14 9 5 1
= 109. 4; Ramesh’s share : Shyam’s share
91 91 45 10 90 90 18
= 100 = 82.81 litres 1 1
100 100 1 :
work done by Altmash and 8 10
18 4 4
22 1 1
98. 5; Area of the circle = r2 28 28 100 100
7 45 1 5
= 2464 cm2 Balasubramanyam in = days
7 18 14 1 625
Area of the rectangle = 2464 cm2 1: 1: 676 : 625
2 676
Let the length of the rectangle be 14x and 5 4
Total number of days = 7 days 1
breadth 11x. 14 100
Then, 14x × 11x = 2464 102. 3; Let the present age of Rituraj be x
Dividing `5204 in the ratio 676 : 625. 161. 3 162. 4 163. 1 164. 1 165. 3
1 2 1
= rh (4x) 2 5y m3 166. 5 167. 3 168. 3 169. 3 170. 2
676 3 3
Ramesh’s share = 5204 171. 5 172. 4 173. 4 174. 1 175. 5
676 625 Required ratio 176. 2 177. 4 178. 5 179. 1 180. 2
(3x) 2 4y 32 4 181. 2 182. 4 183. 3 184. 3 185. 4
676 =
= 5204 = 676 × 4 = `2704 1/ 3 (4x) 5y 1/ 3 42 5
2
186. 3 187. 4 188. 1 189. 2 190. 2
1301 191. 1 192. 3 193. 3 194. 1 195. 1
3 3 3
110. 5; Let the sum be `x. = = 27 : 20 196. 4 197. 3 198. 2 199. 4 200. 4
5 4
Compound interest
117. 5; Let the time taken by outlet pipe ‘P’
3 3
8 27 to empty the full tank = x minutes.
= ` x 1 100 x =` x
25
x
Part of tank filled in 1 minute

19683 15625 4058x 1 1 1 90x 2016


=` x =` =
42 48 x 2016x
15625 15625
According to the question,
x 8 3 6x
And simple interest = = ` 90x 2016
100 25 30 1
Required percentage 2016x
4058 6 or, 2700x – 60480 = 2016x
x
15625 25 or, 684x = 60480
= 100% 8.21% more than
6x 60480 8
25 or, x = 88 min.
its simple interest. 684 19
111. 4; Let the speed of the first train be ‘x’ 118. 1; Let the age of Prashant be x years.
m/s and that of the second train be ‘y’ m/s. x
Length of the first train = (x × 17) metres Then, the age of Mohit = years and that
2
and length of the second train = (27 × y)
metres of Ankur = 2x years
17x 27y x
According to the question, 23 2x 85
x y According to the question,
2
or, 17x + 27y = 23x + 23y
or, x = 34 years
x 2
or, 4y = 6x or, 119. 3; Let the numbers be 3x and 7x.
y 3
x 10 7
Ratio of the speed of trains = 2 : 3 Then,
7x 10 13
112. 3; Piece of land purchased by man
or, 13(3x + 10) = 7(7x + 10)
` 2,50, 000
= = 125 m2 or, 39x + 130 = 49x + 70
` 2000 or, 10x = 60 or, x = 6
Percentage of land that he can purchase
Difference of the numbers
with this amount
= (7 × 6 – 6 × 3) = 42 – 18 = 24
125 100 5
= % % 1.25% 120. 4; Work done by Narasimha per day
10, 000 4
1 1
113. 1; Net profit per hour =
15 10 150
95 108 106
= ` 18500 18500 Work done by Rajshekhar per day per hour
100 100 100
1 1
= `(20119.86 – 18500) = `1619.86
114. 3; Salary of the person after 9 months 8 11 88
Work done by both per day per hour to-
125 85
= ` 15000 = `15937.5 1 1
100 100 gether
150 88
115. 2; Total votes polled
Work done by both per day for 12 hours
70
= 30, 000 21000
1 1 238
100
= 12
40 150 88 1100
Votes in favour of Sonia = 21000 800
100 Total work done by both working 12 hours

Votes in favour of Sushma = 21000 – 8400 1100 74


per day is 4 days
= 12600 238 119
116. 3; Let the radius of the cylinder be 3x 121. 1 122. 5 123. 5 124. 2 125. 1
metres and that of the cone be 4x metres. 126. 4 127. 2 128. 3 129. 2 130. 3
Let the height of the cylinder be 4y metres 131. 2 132. 3 133. 3 134. 1 135. 1
and that of the cone be 5y metres. 136. 2 137. 2 138. 4 139. 2 140. 5
Then, the volume of the cylinder = r2 h 141. 1 142. 3 143. 1 144. 1 145. 3
= (3x)2 × 4y m3 146. 1 147. 2 148. 3 149. 2 150. 4
Volume of the cone 151. 4 152. 2 153. 2 154. 1 155. 3
156. 4 157. 4 158. 1 159. 2 160. 4
Clerk Mock Test 4
9. If all the odd digits are deleted from the above
Test-I: Reasoning Ability arrangement, which of the following will be tenth from
the left end of the arrangement?
1. In a certain code language MAINE is written as 1) 6 2) 2 3) 4
GPHCO. How is GRANT written in that code language? 4) 8 5) None of these
1) VPZSH 2) VPZTI 10. How many such 5’s are there in the above arrangement
3) VPBTI 4) PVZTI each of which is immediately preceded by an odd
5) None of these number and followed by an even number?
2. How many such pairs of letters are there in the word 1) None 2) Three 3) One
APPROPRIATE each of which has as many letters 4) Two 5) None of these
between them in the word as in the English alphabet? 11. If one is added to each of the even digits and two is
1) One 2) Two 3) Three added to each of the odd digits given in bold in the
4) Four 5) More than four above arrangement, how many digits will appear twice
3. How many meaningful English words can be formed in the new number thus formed?
using the first, third, seventh and eighth letter of the 1) One 2) Two 3) Three
meaningless word LAMBASTE, using each letter only 4) Four 5) None of these
once in each word? Directions (Q. 12-16): Study the following questions
1) None 2) One 3) Two which are based on the following diagram.
4) Three 5) None of these The diagram shows three figures, each representing
4. Four of the following five are alike in a certain way the people of three countries. The rectangle represents the
and hence form a group. Which is the one that does people of India, the triangle represents the people of
not belong to that group? Russia, the circle represents the people of the USA.

K
1) 89 2) 69 3) 79
4) 49 5) 59
5. If the letters of the word INCREMENT are arranged in
the English alphabetical order, the position of how
many letters will remain unchanged?
1) None 2) One 3) Two
12. How many Russians are also Indians in the above
4) Three 5) None of these
diagram?
6. In a column of twentytwo students, Q is fourth after P
1) 9 2) 8 3) 15 4) 17 5) None of these
from the front and P is 18th from the back. What is the
13. How many Americans are also Russians but not
position of Q from the front?
Indians?
1) Eighth 2) Tenth 3) Ninth

KUNDAN
1) 7 2) 14 3) 12 4) 19 5) None of these
4) Data inadequate 5) None of these
14. How many Indians are neither Russians nor
Directions (Q. 7-11): Study the following arrangement
Americans?
carefully and answer the questions given below:
1) 1 2) 10 3) 7 4) 6 5) None of these
49 4 923 21 57 935 8525 618 59 72 624 2 5831
15. How many Americans are also Indians and Russians?
624
1) 17 2) 7 3) 2 4) 8 5) None of these
7. Which of the following is seventh to the left of the
16. How many Indians are also Americans but not
twentysecond digit from the left end of the above
Russians?
arrangement?
1) 8 2) 2 3) 7 4) 5 5) None of these
1) 9 2) 1 3) 6
Directions (Q. 17-22): Study the following information
4) 5 5) None of these
carefully and answer the given questions:
8. How many such 2’s are there in the above arrangement
Eight persons B1, B2, B3, B4, B5, B6, B7 and B8 are sitting
each of which is immediately preceded by a perfect
in a straight line facing south but not necessarily in the
square?
same order.
1) None 2) One 3) Two
B5 is second to the right of B1 and B7 is third to the left
4) Three 5) None of these
of B1. B5 is not the immediate neighbour of B3 and is not at 3) if either conclusion I or II follows.
the ends of the row. B6 is second to the right of B2, who is 4) if neither conclusion I nor II follows.
not the neighbour of B1. 5) if both conclusions I and II follow.
17. Who is on the immediate right of B1? 28. Statements: Some circles are triangles.
1) B3 2) B4 3) B8 All triangles are squares.
4) Either 2) or 3) 5) None of these Conclusions: I. All squares being circles is a
18. Who among the following pairs is at the ends of the possibility.
row? II. Some triangles are circles.

K
1) B4, B8 2) B2, B4 3) B8, B2 29. Statements: All phones are wires.
4) Can’t be determined 5) None of these No wire is a pole.
19. If B8 is on the immediate right of B5 then who among Conclusions: I. Some poles are phones.
the following is third to the right of B6? II. No phone is a pole.
1) B8 2) B4 3) B1 30. Statements: No pen is a pin.
4) B2 5) None of these Some pins are caps.
20. If B1 is on the immediate left of B4 then who among the Conclusions: I. Some pins are caps.
following is second to the right of B4? II. Some caps are not pens.
1) B5 2) B6 3) B8 (31-32):
4) B3 5) B7 Statements: All discs are sticks.

KUNDAN
21. Who among the following is second to the left of B3? No stick is a drum.
1) B7 2) B2 3) B4 Some drums are covers.
4) B1 5) Can’t be determined 31. Conclusions: I. No disc is a drum.
22. If B5 is on the immediate right of B4 then which of the II. All sticks being discs is a
following pairs is at the extreme ends of the row? possibility.
1) B4, B2 2) B3, B2 3) B8, B4 32. Conclusions: I. All covers being drums is a
4) B2, B8 5) None of these possibility.
Directions (Q. 23-27): In these questions, a II. No stick is a cover.
relationship between different elements is shown in the Directions (Q. 33-37): Study the following information
statements. The statements are followed by two carefully and answer the given questions:
conclusions. Give answer Eight persons I, J, K, L, M, N, O and P are sitting
1) if only conclusion I is true. around a circular table but not necessarily in the same
2) if only conclusion II is true. order. Three of them are facing outward while five are facing
3) if either conclusion I or II is true. towards the centre.
4) if neither conclusion I nor II is true. M is third to the right of K. N is sitting third to the left
5) if both conclusions I and II are true. of M. Three persons are sitting between N and J. O is
(23-24): sitting third to the right of N, who is not facing the centre.
Statement: QP= RN< GE>S L is sitting third to the right of I, who is not facing the
23. Conclusions: I. Q  N II. R > E centre.
24. Conclusions: I. G > P II. S < R 33. Who sits between N and I?
25. Statement: B=YD<W< MN 1) J 2) K 3) M
Conclusions: I. M > Y II. D < N 4) O 5) None of these
(26-27): 34. Who among the following is second to the right of J?
Statement: J< X V >U= W<L< P =T 1) L 2) K 3) M
26. Conclusions: I. P > U II. W < V 4) N 5) None of these
27. Conclusions: I. J < W II. T > U 35. Which of the following statements is/are true with
Directions (Q. 28-32): In each question below are respect to P?
given two/three statements followed by two conclusions 1) P is opposite of O.
numbered I and II. You have to take the given statements 2) P is fourth to the right of O.
to be true even if they seem to be at variance with 3) P is fourth to the left of O.
commonly known facts. Read all the conclusions and then 4) All are true
decide which of the given conclusions logically follows 5) None of these
from the given statements, disregarding commonly 36. Which of the following groups represents the group,
known facts. Give answer of those facing away from the centre?
1) if only conclusion I follows. 1) IKN 2) KNL 3) IJK
2) if only conclusion II follows. 4) LMN 5) None of these
37. What is the position of K with respect to P? a women’s bank. There are two reasons why the idea is not
1) K is third to the right of P. exciting for some of us. First, merely setting up an all-
2) K is second to the left of P. women bank is not likely to address the core issue of
3) K is third to the left of P. attitudinal bias against women, which is so prevalent in
4) K is in front of P. our banking institutions. There is both overt and covert
5) None of these exclusion in the system. Second, there is no guarantee that
Directions (Q. 38-40): Each of the questions below the all-women bank is going to mitigate the problem of
consists of a question and two statements numbered I and financial exclusion of Indian women. My scepticism also
II given below it. You have to decide whether the data stems from the limited success of other previous attempts
provided in the statements are sufficient to answer the at focused banking, such as the setting up of the regional
question. Read both the statements and give answer rural banks.
1) if the data in statement I alone are sufficient to In order to assess the likely impact of the proposed
answer the question, while the data in statement II women-only bank, it would be interesting to draw an
alone are not sufficient to answer the question. analogy with the launching of the regional rural banks in
2) if the data in statement II alone are sufficient to the 1970s. The RRBs were set up in 1976 as special conduits
answer the question, while the data in statement I of credit delivery in rural India. They were supposed to
alone are not sufficient to answer the question. combine the “local feel and familiarity of rural problems
3) if the data either in statement I alone or in statement with the professionalism and large resource base of
II alone are sufficient to answer the question. commercial banks”. Thus, there was an acknowledgment
4) if the data in both the statements I and II together that mainstream commercial banks could not effectively
are not sufficient to answer the question. cater to the needs of the villages, so a new type of locally
5) if the data in both the statements I and II together oriented banks, the RRBs had to be set up. This is very
are necessary to answer the question. similar to the proposed women’s bank’s ambition “to
38. How many sisters does R have? address the gender-related issues of financial inclusion”.
I. N is father of R’s brother L’s father, who has three On the face of it, there is nothing wrong in setting up
children. new institutions that target specific segments of the
II. M is daughter of K, who is daughter-in-law of N. population. However, we have seen in the case of the RRBs
39. In which direction is Rehan’s home with respect to that, less than ten years into operation, their financial

K
Rina’s home? viability became a matter of concern. Starting from 1981,
I. Rina’s home is east of Raju’s home and the home of more than 10 committees were set up to address various
Rehan is to the north of Raju’s home. issues (of financial viability, reconstruction and
II. Rehan’s home is northwest of Nilam’s home and amalgamation, manpower and human resources and
east of Raju’s home. technological upgradation) relating to operation of the
40. Who is the tallest among Shiva, Mira, John and Punit? RRBs. Following this, the RRBs went through the process
I. Mira is not as tall as either Shiva or Punit. of recapitalisation and amalgamation so as to make them
II. John is taller than Shiva. financially sound. Due to amalgamation and mergers, some
RRBs have become large entities that defeat the very
Test-II: English Language concept of “locally oriented” banks.
Ironically the number of urban and metropolitan

KUNDAN
Directions (Q. 41-50): Read the following passage branches of RRBs has increased over the years, while that
carefully and answer the questions given below it. Certain of their rural branches has declined. Between 1992 and
words/phrases in the passage are printed in bold to help 2009, there was a 22-percentage point decline in the
you locate them while answering some of the questions. proportion of rural bank branches of RRBs while there was
That women in India are much more financially a 16-percentage point increase in the share of their non-
excluded than the men is evident from the figures as of rural branches. Thus the creation of “localised rural banks”
March 2011. Only 21 per cent of total bank deposit accounts as a means for tackling the “lack of familiarity of rural
were held by women and these accounted for just about 12 problems on the part of mainstream commercial banks”
per cent of the total volume of deposits. Similarly, women does not seem to have served its purpose. In fact, rural
availed only 18 per cent of the total small credit from banks India is much more financially excluded today when
in 2011. The problem must be understood in the context of compared to the 1990s, both in terms of banking outlets
larger issues arising from the underprivileged status of a and availability of institutional credit.
woman in India. Going by this analogy, it must be asked if the women-
Finance minister P Chidambaram has proposed to deal only bank can promote financial inclusion of India women
with the financial exclusion of Indian women by setting up
unless we address the core issue of exclusion at a more 5) Only (A)
fundamental level. Attitudinal changes in our banking system 45. Which of the following needs to be done to promote
should be an essential and integral part of all our efforts to financial inclusion?
promote financial inclusion. Not too long ago, the Rangarajan 1) General mindset towards women in general and
Committee on financial inclusion had emphasised the poor women in particular needs to be changed.
correction of mindsets of the bank staff. Citing a study 2) Women should be granted financial aids at
conducted in Madhya Pradesh, the committee highlighted comparatively low rate of interest.
that the “majority of the bank branch managers held negative 3) Women at village level should be educated through

K
attitudes towards lending to (the) poor, although (the) poor, meetings, hoardings, banners and posters.
if guided properly, not only succeed as entrepreneurs but 4) It should be made mandatory to bring all rural
also are good repayers”. There is no doubt that if the poor women under the umbrella of financial inclusion.
happens to be a woman, this discrimination gets doubled. 5) None of these
41. Why is the author apprehensive about the success of Directions (Q. 46-48): Choose the word/group of
the women’s bank? words which is MOST SIMILAR in meaning to the word/
1) Because women are not so competent as men. group of words printed in bold as used in the passage.
2) Because previous attempts at focused banking 46. Discrimination
such as the setting up of the RRBs have shown 1) description 2) insight 3) revelation
limited success. 4) distinction 5) contradiction

KUNDAN
3) Because there is no ready infrastructure for the 47. Core
setting up of the all-women’s bank. 1) right 2) temporary 3) central
4) The government itself is under severe financial 4) vast 5) burning
constraints as of now. 48. Stems
5) All the above 1) declines 2) moves 3) extracts
42. Find the incorrect statement regarding Regional Rural 4) trunks 5) originates
Banks. Directions (Q. 49-50): Choose the word/group of
1) The RRBs were set up as a special channel of credit words which is MOST OPPOSITE in meaning of the word/
delivery in rural India. group of words printed in bold as used in the passage.
2) The RRBs were set up to cater to the needs of the 49. Exciting
villages. 1) repulsive 2) provocating 3) interesting
3) The number of urban branches of RRBs has 4) useful 5) elating
increased over the years. 50. Mitigate
4) The number of both rural and metropolitan branches 1) allay 2) enhance 3) relieve
of RRBs has increased over the years. 4) waste 5) vindicate
5) None of these Directions (Q. 51-55): Read each sentence to find out
43. Which of the following statements is/are true about whether there is any grammatical or idiomatic error in it.
the operation of RRBs in the first decade of their The error, if any, will be in one part of the sentence. The
inception? number of that part is the answer. If there is ‘No error’,
1) They did not reflect the desired results as expected the answer is 5). (Ignore errors of punctuation, if any.)
at the time of their launch. 51. 1) Anand did the nation pride by winning the
2) To make the RRBs financially sound the process of international chess championship / 2) five times at a
mergers and amalgamation were adopted. time when / 3) India was faring miserably in / 4)
3) Various committees were formed to address various international sporting events like the Olympics. / 5)
issues related to operations of the RRBs. No error
4) Only 1) and 3) 52. 1) She has a lot of positivity about her / 2) and the
5) All 1), 2) and 3) employees see her as a compassionate woman / 3)
44. What is/are the reason(s) for setting up of the all- who would handle issues from a human perspective /
women’s bank? 4) not as a cold and robotic boss. / 5) No error
(A) Financial inclusion of Indian women 53. 1) Israel is a highly developed agricultural economy, /
(B) To address the core issue of attitudinal bias 2) illustrated by its green fields and expensive
against women infrastructure / 3) as well as by its migrant workforce /
(C) To provide financial assistance and loans only to 4) that is in this sector largely from Thailand. / 5) No
women professionals error
1) All (A), (B) and (C) 2) Only (A) and (B) 54. 1) The transformation of the Indian Constitution over
3) Only (A) and (C) 4) Only (B) and (C) the years / 2) and its flexibility in adapting / 3) to
changing needs constitute an area of / 4) great interest 3) that are lesser active 4) which are very active
to scholars. / 5) No error 5) there are hardly active
55. 1) As she had an education of sorts, she managed to 62. You can ________ on our leaders to stay a step ahead
land a job / 2) that fetch her some money with which / of the law.
3) she paid rent and supported her in-laws / 4) who are 1) buy 2) bank 3) believe
landless agricultural workers. / 5) No error 4) mind 5) stand
Directions (Q. 56-60): In the following questions, a 63. ________ the blanket protests, the ________ of
sentence has been given with some of its parts in bold. To Telangana, India’s 29th state, appears certain.
make the sentence grammatically correct, you have to 1) In spite of, launch 2) Moreover, beginning
replace the bold part with the correct alternative given 3) Despite, birth 4) However, start
below. If the sentence is correct as it is, give 5) as your 5) Unlike, division
answer (ie No correction required). 64. Delhi’s Shastri Bhawan, ________ to the Petroleum
56. The Second World War proved that internal massive and Natural Gas ministry, easily ________ the award
violations of human rights has lead to the break of for the most paranoid corridors of power complex in a
international peace. city that ________ in Machiavellian intrigue.
1) have led the breaking 2) leads to the break up 1) house, received, interests
3) lead to the breach 4) leads to the break 2) building, get, enjoys
5) No correction required 3) palace, achieved, involves
57. The Jindal International Summer School was formally 4) residence, caught, helps
launched on the 1st of March 2013 at the Constitution 5) home, gets, specialises
Club of India, New Delhi. 65. Exide is _______ to present the first ever maintenance-
1) formally launched from the free tubular battery with wheels attached.
2) launched formally since 1) pride 2) proud 3) glorified
3) formally launched on 4) haughty 5) arrogant
4) formally began from the Directions (Q. 66-70): Rearrange the following seven
5) No correction required sentences (A), (B), (C), (D), (E), (F) and (G) in the proper
58. Why did you not returned the money? sequence to form a meaningful paragraph and then answer
1) had you not return 2) you have not repaid the questions given below.

K
3) were you not return 4) did you not return (A) However, these efforts could not produce
5) No correction required significant results on account of policy and
59. I am beginning to realising how difficult it is to be a inadequate budgetary support.
teacher. (B) Keeping in line with the growing demands of
1) have been beginning to realising advanced equipment, infrastructure and scientific
2) have been beginning to realise support, Government of India has taken a lot of
3) am beginning to realise initiatives.
4) I am beginning the realisation of (C) Sports and games have always been seen as an
5) No correction required integral component in the all-round development
60. I have said that in the experimental novel it is best for of human personality.
us to hold to the strictly scientific point of view if we (D) Unlike in the past, modern sports have become

KUNDAN
would wish to base our studies on solid ground. highly competitive.
1) if we wish to (E) Moreover, achievement in sports has always been
2) if we will wish to a source of national pride and prestige.
3) if we could wish to (F) The use of modern equipment, infrastructure and
4) when we should wish to advanced scientific support has changed the
5) No correction required scenario of sports at international level.
Directions (Q. 61-65): In each of the following (G) Apart from being means of entertainment and
sentence there are certain blank spaces. Below each physical fitness, sports have also played a great
sentence there are five options denoted by the number 1), role in generation of spirit of healthy competition
2), 3), 4) and 5). Find out which option can be used to fill up and bonding within the community.
the blank(s) in the sentence in the same sequence to make 66. Which of the following should be the FIRST sentence
it meaningfully complete. after rearrangement?
61. There are numerous NGOs, international and national, 1) G 2) D 3) A
________ in the field of human rights. 4) C 5) B
1) who is very active 2) that is more active
67. Which of the following should be the THIRD sentence
after rearrangement? Test-III: Quantitative Aptitude
1) A 2) E 3) G
4) B 5) F Directions (Q. 81-100): What should come in place
68. Which of the following should be the LAST of question mark (?) in the following equations?
(SEVENTH) sentence after rearrangement? 81. 14973 – ? – 1976 + 3688 = 15344
1) D 2) B 3) C 1) 1431 2) 1341 3) 1236 4) 1680 5) 940
4) F 5) A 82. 777777 ÷ 37 ÷ 3 = ?

K
69. Which of the following should be the SECOND 1) 7601 2) 7061 3) 7007 4) 7337 5) 7610
sentence after rearrangement? 83. 4379 × 777 = ?
1) G 2) A 3) B 1) 3620483 2) 3240683 3) 3562393
4) D 5) E 4) 3402483 5) 3142583
70. Which of the following should be the FOURTH 84. 4 + 0.4 + 0.44 + 0.444 + 0.4444 = ?
sentence after rearrangement? 1) 5.2784 2) 5.3784 3) 5.9884 4) 5.4384 5) 5.7284
1) B 2) C 3) D 85. 106090  14884  ?
4) A 5) G
Directions (Q. 71-80): In the following passage, some 1) 15 2) 17 3) 19 4) 21 5) 23
of the words have been left out, each of which is indicated 86. 22 + 42 + 62 + 72 + 82 + 102 + 122 + 142 = ?

KUNDAN
by a number. Find the suitable word from the options given 1) 6709 2) 609 3) 1006 4) 1240 5) 980
against each number and fill up the blanks with 1 5
appropriate words to make the paragraph meaningfully 87. 7 3  ?
11 13
complete.
1) 24 2) 36 3) 28 4) 27 5) 52
Last April, the Allahabad High Court allowed the
88. 34.5 × 123 × 243.5 ÷ 23.5 × 44.5 = (12)?
farmers’ writ pleas and (71) the project. It (72) that farmers
1) 13 2) 14.5 3) 11 4) 11.5 5) 12
who had received (73) for their land should either return
89. (850 ÷ 68) × (3024 ÷ 36) = ?
the money and take back the land or willingly hand it over
1) 1230 2) 1180 3) 1030 4) 1040 5) 1050
for the project. Around 140 farmers did not accept
compensation. Those who did are in no condition to repay 1 5 9 13
the amount, causing an (74) which the administration is 90. of of of of 3465 = ?
3 7 11 15
(75) to break. 1) 380 2) 585 3) 385 4) 485 5) 715
After the (76) violence, the (77) have been peaceful, 91. 79% of 816 + ? = 700
but the farmers complain they are being (78) by local (79) 1) 46.64 2) 49.86 3) 36.66 4) 55.36 5) 58.36
and officials to give up their land and (80) the protests.
71. 1) enquired 2) stalled 3) interrogated 13824  17576
92. ?
4) inspected 5) checked 576  676 – 624
72. 1) framed 2) prayed 3) rebuked 1) 95 2) 80 3) 70 4) 60 5) 50
4) stipulated 5) questioned
1 6 4
73. 1) fee 2) compensation 3) benefit 93. 2  3 – 2  ?
4) profit 5) premium 9 7 7
74. 1) importance 2) end 3) agreement 4 4 3
4) interest 5) impasse 1) 5 2) 4 3) 5
7 7 7
75. 1) struggling 2) attacking 3) tackling
4) prohibiting 5) labouring 2
76. 1) final 2) initial 3) late 4) 3 5) None of these
7
4) leading 5) starting
77. 1) gossip 2) condition 3) protests
4) riot 5) turmoil 94. 1492  835  43 – 40  81  ?
78. 1) helped 2) rescued 3) intimidated
1) 28 2) 37 3) 39 4) 43 5) 49
4) praised 5) protected
95. 75% of 780 + (?)% of 1662 = 60% of 2360
79. 1) hawkers 2) workers 3) goons
1) 35 2) 20 3) 30 4) 65 5) 50
4) friends 5) person
80. 1) forbid 2) prohibit 3) check 2
4) enforce 5) discontinue 96. 15 ?  1246  of 4465
5
1) 1236 2) 1296 3) 1225 4) 1156 5) 1444 4) 125 5) None of these
97. 45% of 1998 + ? = 65% of 2400 109. Two candidates fought an election. One of them got
1) 660.9 2) 690.8 3) 669.7 4) 780.2 5) 682.4 60% of the total votes and won by 550 votes. If 5% of
votes were declared invalid what was the total number
98. 24  20  ?  120 of votes polled?
1) 36 2) 42 3) 30 4) 40 5) 50 1) 2700 2) 3300 3) 3500
37  37  37  33  33  33 4) 3700 5) None of these
99. ? 110. A number when multiplied by six times of itself gives
37  37  33  33 – 1221
value equal to 1176. What is the number?
1) 4 2) 70 3) 74
1) 14 2) 17 3) 18 4) 16 5) None of these
4) 80 5) None of these
111. The LCM of two numbers is 1540 and their HCF is 20.
4.5  6 – 3.6  5 If one of the numbers is 11 × 20, find the other number.
100. ?
8.8  5 – 5.5  6 1) 160 2) 140 3) 130 4) 180 5) None of these
112. Two sums of money are proportional to 7 : 9. If the first
7 8 9 5 5 one is `140, what is the other?
1) 2) 3) 4) 5)
11 11 11 11 7 1) `180 2) `200 3) `240
101. A train covers the distance from Delhi to Patna at the 4) `220 5) None of these
speed of 42 kmph and from Patna to Delhi at 48 kmph. 113. In how many different ways can the letters of the word
What is the average speed of the train? PRACTICE be arranged?
1) 44.8 kmph 2) 45.5 kmph 3) 46.6 kmph 1) 26100 2) 20160 3) 21060
4) 43.9 kmph 5) None of these 4) 20106 5) None of these
102. The cost of 24 pens and 36 pencils is `720. What is 114. How long does a 110-m-long train running at a speed
the cost of 6 pens and 9 pencils? of 54 kmph take to cross a 205-m-long-bridge?
1) `109 2) `230 3) `180 1) 40 seconds 2) 42 seconds 3) 21 seconds
4) Cannot be determined 5) None of these 4) 20 seconds 5) None of these
103. If the sum of five consecutive even numbers A, B, C, 115. A man swims 21 km downstream and 14 km upstream,
D and E is 110, what is the product of B and D? taking 7 hours each time. What is the speed of the
1) 460 2) 480 3) 520 stream?

K
4) 760 5) None of these 1) 2 kmph 2) 1.5 kmph 3) 1 kmph
104. If 45% of a number is 1291.5, what is 67.5% of that 1
number? 4) kmph 5) None of these
2
1) 1968.35 2) 1925.37 3) 1940.5
4) 1937.25 5) None of these 116. A profit of 25% is earned on goods when a discount
105. Two years ago the ratio of the ages of Manoj and of 20% is allowed on the marked price. What profit
Shyam was 15 : 11. Two years hence the ratio of their percentage will be earned when a discount of 10% is
ages will be 17 : 13. What is the present age of Shyam? allowed on the marked price?
1) 24 years 2) 26 years 3) 28 years 9 3 5
4) 22 years 5) None of these 1) 45 % 2) 42 % 3) 40 %
11 4 8
106. The cost of 19 kg Apples is `1158, that of 17 kg

KUNDAN
Tomatoes is `595, and that of 13 kg Oranges is `949. 2
4) 37 % 5) None of these
What is the total cost of 11 kg Apples, 7 kg Tomatoes 3
and 3 kg Oranges? 117. A 20m long and 16m wide rectangular hall is
1) `1876 2) `1366 3) `1230 surrounded by a 2-metre wide verandah. Find the area
4) `1780 5) None of these of the verandah.
107. Pinky invested an amount of `24500 at the rate of 9% 1) 128m2 2) 130m2 3) 168m2
per annum. After how many years will she get a simple 4) 160m 2
5) None of these
interest of `37485? 118. The difference between the compound interest and
1) 13 years 2) 15 years 3) 17 years the simple interest on a certain sum of money at 10%
4) 19 years 5) None of these per annum for 2 years is `15. Find the sum.
108. 50 persons can complete a piece of work in 50 days. 1) `2000 2) `1600 3) `1500
How many persons with twice the efficiency will 4) `2500 5) None of these
complete the same piece of work in 10 days? 119. Two horses are sold for `1955 each. The first one is
1) 110 2) 115 3) 120 sold at 15% profit and the other one at 15% loss. What
is the total profit or loss?
1) `110 profit 2) `90 loss 3) `90 profit 128. The RBI has undertaken Cheque Truncation System
4) `110 loss 5) None of these (CTS) as a new method for clearing of cheques. Which
120. A number when divided by 527 gives a remainder 31. of the following is NOT correct about the CTS system?
What remainder will be obtained by dividing the same 1) Physical movement of cheques takes place in this
number by 17? system.
1) 14 2) 12 3) 13 2) Electronic image of the cheque is transmitted.
4) 8 5) None of these 3) Clearing-related frauds and cost of collection are
reduced.

K
Test-IV: General Awareness 4) Only 1) and 3)
5) Only 2) and 3)
(with Special Reference to Banking Industry)
129. Which of the following banks has the highest number
121. Which of the following countries has launched an of branches outside India?
unmanned lunar probe Chang’e-3 comprising a lander 1) HDFC Bank 2) ICICI Bank
and a Moon rover recently? 3) Bank of Baroda 4) Indian Overseas Bank
1) Japan 2) US 3) Russia 5) Punjab National Bank
4) China 5) India 130. The book Scandal Point has been authored by who
122. Which of the following countries has offered six oil among the following?
and gas blocks to India’s ONGC Videsh Ltd without 1) Fahad Samar 2) FS Aijazuddin

KUNDAN
bidding in South China Sea recently? 3) Talmiz Ahmad 4) Sudhir Kakar
1) Philippines 2) Vietnam 3) Thailand 5) None of these
4) Myanmar 5) None of these 131. Who among the following has won the Formula One’s
123. Which of the following is NOT correct with respect to season-ending Brazilian Grand Prix recently?
the National Electronic Fund Transfer (NEFT)? 1) Sebastian Vettel 2) Kimi Raikkonen
1) It is a nationwide payment system facilitating one- 3) Lewis Hamilton 4) Mark Webber
to-one funds transfer. 5) None of these
2) No physical transfer of funds takes place, hence 132. Justice Ajay Manikrao Khanwilkar has been sworn in
very low risk. as the 22nd Chief Justice of which of the following
3) Even having a bank account is not at all necessary High Courts recently?
to avail this facility. 1) Gauhati High Court
4) All the above are correct 2) Madhya Pradesh High Court
5) Only 1) and 2) 3) Himachal Pradesh High Court
124. Expand the term MFI as used in banking/financial 4) Jammu and Kashmir High Court
parlance. 5) None of these
1) Market Finance Institution 133. Which of the following cannot be categorised as a
2) Medium Finance Institution debt instrument?
3) Micro Finance Institution 1) Loan 2) Certificate of deposit
4) Mega Finance Institution 3) Commercial paper 4) Bond
5) None of these 5) Stock
125. The two-day third edition of the biennial BRICS 134. Banks in India normally follow a practice of providing
International Competition Conference was organised additional interest rate on retail domestic term deposits
in which of the following countries recently? of _________.
1) South Africa 2) China 3) India 1) Handicaps 2) Senior citizens 3) Rural people
4) Brazil 5) None of these 4) Married women 5) None of these
126. Who among the following has dethroned India’s five- 135. Mexico has announced to award its 2013 International
time title holder Viswanathan Anand as the World Prize for Equality and Non-Discrimination to who
Champion recently? among the following?
1) Veselin Topalov 2) Garry Kasparov 1) Barack Obama 2) Malala Yousafzai
3) Vladimir Kramnik 4) Magnus Carlsen 3) Hillary Clinton 4) Sachin Tendulkar
5) None of these 5) None of these
127. The National Education Day is observed every year 136. India and which of the following countries signed a
on which of the following dates? Memorandum of Understanding (MoU) to strengthen
1) 11 Nov 2) 12 Nov 3) 15 Nov co-operation in public administration and governance
4) 19 Nov 5) None of these as part of the efforts to enhance their strategic
partnership recently?
1) Indonesia 2) Thailand 3) Malaysia 3) Serena Williams
4) Singapore 5) None of these 4) Victoria Azarenka
137. Name the inventor of the world's most popular assault 5) None of these
rifle, the AK-47, a simple and durable weapon of war 146. Who among the following has been appointed the
used by tens of millions in about 100 countries, who new army chief of Pakistan?
died recently. 1) Haroon Aslam 2) Ashfaq Parvez Kayani
1) Hugo Schmeisser 2) Samuel Colt 3) Rashid Mehmood 4) Raheel Sharif
3) Helge Palmcrantz 4) Mikhail Kalashnikov 5) None of these
5) None of these 147. In the event of a bank failure, Deposit Insurance and
138. Who among the following has been appointed the Credit Guarantee Corporation (DICGC) protects bank
Chairman of the National Bank for Agriculture and deposits that are payable in India. Which of the
Rural Development (NABARD) recently? following is not covered by DICGC?
1) Prakash Bakshi 2) Harsh Kumar Bhanwala 1) Deposits of Central/State Govts
3) Santosh Nayar 4) Sushil Muhnot 2) Current
5) None of these 3) Recurring
139. President Pranab Mukherjee inaugurated the 10-day 4) Fixed
Hornbill festival and took part in the 50th anniversary 5) Savings
celebration of the statehood of ________recently. 148. Who among the following won the women's title in
1) Assam 2) Manipur 3) Tripura the $15,000 Tata Open India international Challenge
4) Nagaland 5) Meghalaya recently?
140. The country’s first fully privately-financed developed 1) Della Destiara Haris 2) Shendy Puspa Irawati
metro system has been launched in the public-private 3) Sayali Gokhale 4) PV Sindhu
partnership mode in which of the following cities 5) None of these
recently? 149. Assessees paying excise duty and service tax of what
1) Gurgaon 2) Mumbai 3) Bangalore amount will have to mandatorily pay the levies
4) Jaipur 5) None of these electronically with effect from Jan 2014?
141. Who among the following has won the World Cup of 1) `1 lakh 2) `2 lakh 3) `5 lakh
golf recently? 4) `10 lakh 5) None of these

K
1) Anirban Lahiri 2) SSP Chowrasia 150. The United Nations Day is observed on
3) Mohammad Siddikur 4) Jason Day 1) 24 Oct 2) 26 Oct 3) 28 Oct
5) None of these 4) 30 Oct 5) None of these
142. Which of the following is/are eligible for priority sector 151. Under the Liberalised Remittance Scheme started by
lending by banks in India? the RBI in 2004, all resident individuals, including
1) Micro and Small Enterprises minors, are allowed to freely remit up to what amount
2) Economically weaker sections per financial year (Apr–Mar)?
3) Agriculture 1) USD 25,000 2) USD 50,000 3) USD 75,000
4) Only 1) and 3) 4) USD 1,00,000 5) None of these
5) All 1), 2) and 3) 152. What does the term red herring refer to in the context
143. Banks have now been advised by the Reserve Bank of an IPO?

KUNDAN
of India (RBI) to offer a ‘Basic Savings Bank Deposit 1) Minimum offer per share
Account (BSBDA)’ to all their customers. A customer 2) Number of shares offered
can open what number of BSBDAs in one bank? 3) Preliminary prospectus
1) Only one 2) Only two 3) Only three 4) Size of IPO
4) Only four 5) Any number 5) None of these
144. India successfully test-fired its Dhanush ballistic 153. Who among the following has been appointed the
missile recently. The nuclear-capable surface-to- new chief justice of Pakistan?
surface missile is a naval variant of which of the 1) Tassaduq Hussain Jillani
following India’s indigenously developed missiles? 2) Iftikhar Chaudhary
1) Agni 2) Trishul 3) Aakash 3) Abdul Hameed Dogar
4) Prithvi 5) None of these 4) Rana Bhagwandas
145. Who among the following has been named the WTA 5) None of these
Player of the Year 2013? 154. Who among the following has been crowned Miss
1) Li Na Universe 2013 at the 62nd edition of the beauty
2) Maria Sharapova pageant recently?
1) Manasi Moghe 164. The capability of a microcomputer can be enhanced
2) Constanza Baez with the help of
3) Patricia Yurena Rodriguez 1) secondary memory 2) additional processor
4) Gabriela Isler 3) expansion card 4) buffer memory
5) None of these 5) None of these
155. Which of the following countries has announced to 165. The lowest form of computer language is called
abolish the decades-old “gentan” production 1) BASIC 2) FORTRAN
adjustment system for rice by the year ending Mar 3) Machine Language 4) COBOL

K
2019 to bring reforms in agriculture sector? 5) None of these
1) China 2) South Korea 3) Japan 166. The linking of computers with a communication system
4) Malaysia 5) None of these is called
156. Which of the following is NOT a financial/banking 1) Networking 2) Pairing 3) Interlocking
term? 4) Assembling 5) Sharing
1) El Nino Effect 2) SWIFT 3) CAMELS 167. Which of the following is/are a scanning device?
4) CBS 5) ECS 1) OMR 2) MICR 3) BCR
157. The Reserve Bank of India (RBI) recently directed the 4) All the above 5) None of these
banks to charge customers for transaction SMS alerts 168. Which of the following is/are not a Web browser(s)?
on _____basis. 1) Mosaic 2) Cello 3) Lynx

KUNDAN
1) Fixed fee 2) Usage 3) Free 4) All of these 5) None of these
4) Double the actual fee 5) None of these 169. Which term identifies a specific computer on the web
158. Satyananda Mishra has been appointed the Chairman and the main page of the entire site?
of which of the following exchanges in the country 1) URL 2) Website address
recently? 3) Hyperlink 4) Domain name
1) National Stock Exchange (NSE) 5) None of these
2) Multi Commodity Exchange (MCX) 170. The code that relational database management systems
3) Bombay Stock Exchange (BSE) use to perform their database task is referred to as
4) MCX Stock Exchange (MCX-SX) 1) QBE 2) SQL 3) OLAP
5) None of these 4) Sequel Server 5) None of these
159. The Union Petroleum Minister Veerappa Moily 171. Personal computers can be connected together to form
inaugurated the first “Synthetic Rubber Plant” of the a
country at which of the following places recently? 1) server 2) supercomputer 3) enterprise
1) Chennai 2) Hyderabad 3) Panipat 4) network 5) None of these
4) Mangalore 5) None of these 172. A button that makes character either upper or lower
160. Who among the following has lifted the women’s case and numbers the symbols is
singles title of the Macau Open Grand Prix Gold 1) monitor 2) shift key 3) icon
badminton tournament recently? 4) mouse 5) None of these
1) Saina Nehwal 2) Michelle Li 3) PV Sindhu 173. Which of the following computers is not considered
4) Wang Shixian 5) Li Xuerui as a portable computer?
1) Notebook computer 2) Laptop
Test-V: Computer Knowledge 3) Mini computer 4) PDA
5) Tablet
161. Information in a computer is stored as 174. Preparing a magnetic disk for data storage is called
1) analog data 2) digital data 3) modem data 1) debugging 2) formatting 3) booting
4) watts data 5) None of these 4) commissioning 5) compiling
162. What is the full form of USB as used in computer- 175. A register that contains the result of an operation is
related activities? called a(n)
1) Universal Security Block 2) Ultra Serial Block 1) action register 2) program counter
3) United Service Block 4) Universal Serial Bus 3) accumulator 4) stack pointer
5) None of these 5) data register
163. ________ determines the appearance of your desktop 176. Third-generation language is
and controls the performance of your hardware. 1) machine language 2) assembly language
1) Windows Explorer 2) Internet Explorer 3) high-level language 4) middle-level language
3) Program file 4) Control Panel 5) low-level language
5) System file 177. ATM cells comprise how many bytes?
1) 53 bytes 2) 57 bytes 3) 45 bytes 5) None of these
4) 48 bytes 5) None of these 189. A good practice, before installing software on the
178. The device used to carry digital data on analog lines Windows XP operating system, is to establish a(n)
is called ________ so that if anything goes wrong during
1) modem 2) multiplexer 3) modulator installation you can restore your system back to how
4) demodulator 5) None it was before you started.
179. Dumb terminals have terminals and 1) backup 2) image 3) restore point
1) mouse 2) speakers 3) keyboard 4) upgrade 5) None of these
4) mouse or speakers 5) None of these 190. To resolve problems with Microsoft products, check
180. The portion of the CPU that coordinates the activities the ________, an online collection of more than
of all other computer components is the 250,000 articles written by Microsoft support
1) motherboard 2) coordination board professionals.
3) control unit 4) arithmetic logic unit 1) FAQs 2) Knowledge Base
5) None of these 3) integrated help 4) manual
181. Which of the following is to be used when you want 5) None of these
to make all letters capital without using the shift key 191. In a Batch Processing Operating System, interaction
for each character? between the user and the processor is
1) Shifter 2) Upper case 3) Caps lock key 1) limited
4) Icon 5) None of these 2) not possible
182. Which of the following is a device that reads the 3) unlimited
information contained on a disk and transfers it to the 4) depends upon user only
computer’s memory? 5) depends upon amount of work
1) Monitor 2) Screen 3) Keyboard 192. In case of computer, “JUMPER” is
4) Disk drive 5) None of these 1) used to configure the hard disk
183. Which view in PowerPoint can be used to enter 2) used to configure the primary memory
speaker’s comments? 3) used to play computer game
1) Normal 2) Slide show 4) a software language
3) Slide sorter 4) Notes page view 5) a machine language

K
5) None of these 193. A small or intelligent device is so called because it
184. The boxes that are displayed to indicate that the text contains within it a
pictures or objects are placed in it is called 1) Computer 2) Microcomputer
1) Placeholder 2) Auto text 3) Text box 3) Programmable data 4) Sensor
4) Word art 5) None of these 5) None of these
185. When you select a mouse for a particular computer 194. Which technology is used in compact disks?
system, what is the most important consideration? 1) Mechanical 2) Electrical
1) The type of drivers that come with the mouse 3) Electromagnetic 4) Laser
2) The length of the mouse cord 5) None of these
3) The type of connector the mouse is equipped with 195. The process that rearranges file fragments into
4) The number of buttons the mouse has contiguous files is called

KUNDAN
5) None of these 1) file filing 2) folder folding 3) unfragging
186. Which of the following is an acronym for Electronic 4) defragmentation 5) file folding
Delay Storage Automatic Calculator? 196. ________ virus describes the awful consequences
1) UNIVAC 2) EDSAC 3) EDVAC of not acting immediately.
4) Abacus 5) None of these 1) Android 2) Spoofing 3) Phishing
187. When data changes in multiple lists and all lists are 4) Misleading e-mail 5) Sabotage
not updated, this causes 197. ________ operating systems are found on ROM chips
1) data redundancy 2) information overload in portable or dedicated devices.
3) duplicate data 4) data inconsistency 1) Server 2) Network 3) Embedded
5) None of these 4) Stand-alone 5) Windows
188. ________ are words that a programming language 198. Identify the odd term among the following groups.
has set aside for its own use. 1) Optical fibre 2) Co-axial cable
1) Control words 2) Reserved words 3) Twisted pair wire 4) Micro waves
3) Control structures 4) Reserved keys 5) None of these
199. POP (Post Office Protocol) is used for
1) accepting e-mail 2) accessing any e-mail
3) creating e-mail 4) transfering e-mail
5) None of these
200. The full form of COBOL is
1) Common Business Oriented Language
2) Common Bright Oriented Language
3) Co-axis Business Oriented Language
4) Common Logic Oriented Language

K
5) None of these

KUNDAN
Answers
1. 2;
to each odd digit we get 5 and 3 are the digits
that appear in the number twice. 9 5 3 7 3 5
12. 4; 9 + 8 = 17
26. 5; Given statement:

13. 1
14. 3; 6 + 1 = 7
15. 4
16. 2 Thus, V > W or W < V is true. Again, U < P
(17-22): or P > U is true.
Hence, both conclusion I and II are true.
2. 5;
27. 2; Given statement:
3. 2; First letter  L
Third letter  M
Seventh letter  T
Eighth letter  E
The meaningful English word is MELT.
4. 4; Only 49 is a perfect square. 17. 4 18. 4 19. 2 20. 3 21. 1 Thus, U < T or T > U is true. But we can’t
22. 4 compare J and W.
5. 2; 23. 1; Hence, conclusion II is true but I is not true.
Given statement: Q  P = R  N < G  E > S 28. 5; There is no negative statement. Thus,
Thus, Q  N is true. Hence, conclusion I is the possibility in I exists. Hence, conclusion
6. 3; P is 18th from the back.
true. Again, we can’t compare R and E. I follows. Again, Some circles are triangles
The position of Q from the back is 18 – 4
Hence, conclusion II does not hold true.  conversion  Some triangles are circles.
= 14th
24. 4; Given statement: Hence, conclusion II follows.
Hence, the position of Q from the front is
(22 – 14 + 1 =) 9th QP=RN<GE>S 29. 2; All phones are wires (A) + No wire is a
7. 4; 7th to the left of the 22nd digit from Thus, we can’t compare G and P, or S and R. pole (E) = A + E = E = No phone is a pole.
the left is (22 – 7 =) 15th from the left end, Hence, conclusion I and II both are not true. Hence, conclusion II follows but I does not
ie 5. 25. 2; Given statement: follow.
8. 3; 92, 42 30. 2; No pen is a pin (E) + Some pins are
9. 1; The new series is 4 4 2 2 8 2 6 8 2 6 2 caps (I) = E + I = O* = Some caps are not
4 2 8 6 2 4. So, the tenth digit from the left pens. Hence, conclusion II follows. But
end is 6. conclusion I does not follow because it is a
10. 3; 3 5 8 restatement of statement I.
11. 2; 8 3 1 6 2 4 31. 5; All discs are sticks (A) + No stick is a
Hence, conclusion II is true but I is not true. drum (E) = A + E = E = No disc is a drum.
After adding one to each even digit and two
Hence, conclusion I follows. 831  100
777777 or, ? =  50
Again, the possibility in II exists from first 82. 3; ? = 777777 ÷ 37 ÷ 3 = 1662
statement. Hence, conclusion II follows. 37  3
32. 1; Some drums are covers  conversion = 7007 2
96. 2; 15 × ? + 1246 = × 4465 = 1786
 Some covers are drums. Thus, the 83. 4; ? = 4379 × 777 = (4300 + 79) × 777 5
possibility in I exists. Hence, conclusion I = 334100 + 61383 = 3402483
follows. 84. 5; ? = 4 + 0.4 + 0.44 + 0.444 + 0.4444 or, 15 × ? = 1786 – 1246 = 540
Again, No stick is a drum (E) + Some drums = 5.7284
are covers (I) = E + I = O* = Some covers are 540
or, ? = = 36
not sticks. Hence, conclusion II does not 85. 1; ? = 10609  14884  103  122 15
follow. or, ? = (36)2 = 1296
(33-37): = 225 = 15 97. 1; ? = (65 × 24 – 45 × 19.98)
= 1560 – 899.1 = 660.9
86. 2; ? = 22 + 42 + 62 + 72 + 82 + 102 + 122 +
98. 3; 24 × 20 × ? = (120)2
142 = 4 + 16 + 36 + 49 + 64 + 100 + 144 +
196 = 609 120 120
?= = 30
78 44 24  20
87. 1; ? =   24
11 13
a 3  b3
88. 3; (12)? = 34.5 × 123 × 243.5 ÷ 23.5 × 44.5 99. 2; a + b =
a 2  b 2 – ab
= (3 × 4)4.5 × 123 × 123.5
33. 5; P 34. 2 35. 4 36. 1 37. 3 where a = 37, b = 33
= (12)4.5 × 123 × (12)3.5 = (12)4.5 + 3 + 3.5
38. 4; From I. = (12)11  a + b = 70
 ? = 11 27.0 – 18.0 9
89. 5; (850 ÷ 68) × (3024 ÷ 36) 100. 3; ? = 
44 – 33 11
= 12.5 × 84 = 1050
101. 1; Let the distance between Delhi and
1 5 9 13 Patna be x km.
90. 2; ? =     3465 = 585
3 7 11 15 x x
From II. t1 = ,t =
79 42 2 48
91. 4; ? = 700 –  816 = 700 – 644.64
100 x x 15x
Now, t1 + t2 =  =
= 55.36 42 48 336
13824  17576 31400  Average speed
Hence, both are not sufficient to answer the 92. 5; ? = =  50

K
question. 576  676 – 624 628 2x 336
=  2x   44.8 kmph
39. 1;  15x  15x
19 27 18 19  27 – 162  
93. 1; ? =  –   336 
From I. 9 7 7 63
102. 3; Let the cost of one pen be `x and
351 36 4 the cost of one pencil be `y.
= 5 5
63 63 7 Then, 24x + 36y = 720 ... (i)

Hence, Rehan’s home is in southwest 1


Multiplying eqn (i) by , we get
direction with respect to Rina’s home. 94. 3; ? = 1492 835  43 – 40  81 4
Hence, statement I alone is sufficient to 6x + 9y = `180
answer the question. Hence cost of 6 pens and 9 pencils = `180.
40. 4; 103. 2; Let A be x.
= 1492  835  43 – 40  9
From I and II. There may be several Then, B = x + 2, C = x + 4, D = x + 6 and E

KUNDAN
possibilities. =x+8
Both are not sufficient to answer the Now, x + x + 2 + x + 4 + x + 6 + x + 8 = 110
= 1492  835  43 – 7
question. or, 5x = 110 – 20 = 90
41. 2 42. 4 43. 5 44. 2 45. 1  x = 18
46. 4 47. 3 48. 5 49. 1 50. 2 = 1492  835  6  B = 20 and D = 24
51. 1; Replace ‘pride’ with ‘proud’ Hence, B × D = 20 × 24 = 480
52. 3; Replace ‘human’ with ‘humane’ = 104. 4; Let the number be x.
1492  841 = 1492  29
53. 5
45
54. 4; Replace ‘to’ with ‘for’ Now, x  = 1291.5
= 1521 = 39 100
55. 2; Replace ‘fetch’ with ‘fetched’
56. 3 57. 5 58. 4 59. 3 60. 5 or, x = 2870
75  780 ? 1662 60 2360
61. 4 62. 2 63. 3 64. 5 65. 2 95. 5;  =  67.5% of 2870
100 100 100
(66-70): CGEDFBA 67.5  2870
66. 4 67. 2 68. 5 69. 1 70. 3 ?1662 = = 1937.25
71. 2 72. 4 73. 2 74. 5 75. 1 or, 585  = 1416 100
100 105. 1; Let the present age of Manoj and
76. 2 77. 3 78. 3 79. 3 80. 5
81. 2; 14973 – ? – 1976 + 3688 = 15344 Shyam be x years and y years respectively.
1662  ?
or, ? = 14973 + 3688 – 15344 – 1976 or,  1416 – 585  831
100
= 1341
114. 3; Total length = 110 + 205 = 315m
x – 2 15 115
Then, two years ago, y – 2  11 5 Then, x × = `1955
100
 Speed = 54 × = 15 ms
or, 11x – 22 = 15y – 30 18
1955 100
or, 11x – 15y = –8 ... (i) 315 or, x = = `1700
115
Again, after two years,  Time = = 21 seconds
15
x  2 17 85
 115. 4; Let the speed of the man be x kmph Again, y × = `1955
y  2 13 and that of the stream be y kmph. 100

K
or, 13x + 26 = 17y + 34 21 1955 100
or, 13x – 17y = 8 ... (ii) Then, x + y = = 3 ... (i) or, y = = `2300
7 85
Multiplying eqn (i) by 13 and eqn (ii) by 11,
 Total cost = 1700 + 2300 = `4000
we get 14
And, x – y = =2 ... (ii) Selling price = 1955 × 2 = `3910
143x – 195y  –104 7  Loss = 4000 – 3910 = `90
– 143  187 y  –88 Solving eqn (i) and (ii), we get 120. 1; Number = 527 × Q + 31
= 17 × 31 × Q + 17 + 14
– 8 y  –192 5 1
x= kmph and y = kmph = 17{31 × Q + 1} + 14
 y = 24 years 2 2 So, when divided by 17, the remainder will be
116. 3; Let the marked price be `x. 14 .
1558
106. 2; Rate of Apple per kg =  82 121. 4 122. 2 123. 4 124. 3 125. 3
19 x
After 20% discount, selling price = x – 126. 4 127. 1 128. 1 129. 3 130. 1
5 131. 1 132. 2 133. 5 134. 2 135. 2

KUNDAN
595
Tomato = = `35 per kg 4x 136. 3 137. 4 138. 2 139. 4 140. 1
17 =` 141. 4 142. 5 143. 1 144. 4 145. 3
5
949 146. 4 147. 1 148. 2 149. 1 150. 1
Orange = = `73 per kg 4x 100 16 x 151. 3 152. 3 153. 1 154. 4 155. 3
13  Cost = × =` 156. 1 157. 2 158. 2 159. 3 160. 3
11 kg Apple + 7 kg Tomato + 3 kg Orange 5 125 25
161. 2 162. 4 163. 4 164. 3 165. 3
11 × 82 + 7 × 35 + 3 × 73 Now, after allowing 10% discount, 166. 1 167. 4 168. 5 169. 1 170. 2
= 902 + 245 + 219 = `1366 x 9x 171. 4 172. 2 173. 3 174. 2 175. 3
SP = x – = 176. 3 177. 1 178. 1 179. 3 180. 3
100  SI 100  37485 10 10
107. 3; T = = 181. 3 182. 4 183. 4 184. 1 185. 3
PR 24500  9
9 x 16 x 13x 186. 2 187. 4 188. 2 189. 3 190. 2
= 17 years  Profit = – = 191. 1; In a batch processing operating
108. 4; For 50 days, the number of persons 10 25 50
system, interaction between the user and the
required = 50 13x 25 325 processor is limited or there is no interaction
For 10 days, the number of persons required Now, % profit =   100 =
50 16x 8 at all during the execution of work. Data and
50 50 programs that need to be processed are
= = 250 5 bundled and collected as a ‘batch’ and executed
10 = 40 %
8 together.
As efficiency is double, so persons required 192. 1; Several different connectors and
= 125 117. 4; jumpers are used to configure the hard disk
109. 5; Let the total votes polled be x. and connect it to the rest of the system. The
60 35 number and types of connectors on the hard
Then, x × –x× = 550 disk depend on the data interface it uses to
100 100
connect to the system, the manufacturer of
25x the drive and any special features that the
or,  550 or, x = 2200 drive may possess.
100
The area of hall PQSR = 20 × 16 193. 4 194. 4 195. 4
110. 1; Let the number be x.
= 320m2 196. 4 197. 3 198. 4
Then, x × 6x = 1176
The length of the rectangular region ABDC 199. 2 200. 1
1176 = 20 + 2 + 2 = 24m
 x2 = = 196 And breadth of the rectangular region ABDC
6
= 16 + 2 + 2 = 20m
x= 196 = 14 Now, area of the rectangle ABDC
111. 2; Product of two numbers = 24 × 20
= LCM × HCF = 480 m2
 Area of the verandah = 480 – 320
1540  20 = 160 m2
 Reqd number =  140
11 20
2
112. 1; Let the sums of money be 7x and 9x.  10 
118. 3; Difference = Sum 
 7x = 140,  x = 20,  9x = `180  100 
113. 2; Here the number of letters in the
 Sum = 15 × 100 = `1500
word is 8 and C occurs twice.
119. 2; Let the cost of the first horse be `x
8! and that of the second horse be `y.
 Number of ways = = 20160
2!
Clerk Mock Test 5
1) B is an immediate neighbour of C.
Test-I: Reasoning Ability 2) B is second to the left of @.

K
3) B is on the immediate right of A.
1. How many meaningful English words can be formed 4) B is at one end of the row.
with the letters DNEO, using each letter only once in 5) None of these
each word? 9. Which of the following pairs are at the extreme ends
1) None 2) One 3) Two of the row?
4) Three 5) None of these 1) D, A 2) @, B 3) A, 
2. How many such pairs of letters are there in the word 4) Can’t be determined 5) None of these
BALANCE each of which has as many letters between 10. Which of the following letters is between two symbols?
them in the word as in the English alphabetical series? 1) D 2) A 3) C
1) None 2) One 3) Two 4) B 5) None of these
4) Three 5) More than three Directions (Q. 11-15): In these questions, a

KUNDAN
3. In a certain code language FOAM is written as 5%9$ relationship between different elements is shown in the
and FINE is written as 5@#7. How is NAME written in statement(s). The statement(s) is/are followed by two
that code language? conclusions. Give answer
1) $#9% 2) #9$7 3) #9%5 1) if only conclusion I is true.
4) #7@$ 5) None of these 2) if only conclusion II is true.
4. In a certain code language GARDEN is written as 3) if either conclusion I or II is true.
PBELFB. How is PUBLIC written in that code 4) if neither conclusion I nor II is true.
language? 5) if both conclusions I and II are true.
1) ZVNAJJ 2) AJJZVN 3) ZNVAJJ 11. Statements: K < N = O, P > W < K
4) ZNVJJA 5) None of these Conclusions: I. P > N II. O > W
5. Which of the following will complete the given series? 12. Statements: C = P, G < C, N > G
_YC_ D_ E_ _U_T H_ I Conclusions: I. N > C II. P = N
1) GVFXWBZ 2) BXWVFGS 3) WZBGVFG 13. Statements: I > N  T, E < R = T
4) BZWXFVZ 5) None of these Conclusions: I. I > R II. E  N
Directions (Q. 6-10): Study the following information 14. Statement: V< Q =A> W= Z > P
carefully and answer the given questions. Conclusions: I. Q  Z II. V < P
Eight members of a club are sitting in a straight line 15. Statement: ZX<Y>M>N
and their names are coded in English alphabets and Conclusions: I. N < Y II. Z  M
symbols – A, $, B, , @, C, D and ©, but not necessarily in Directions (Q. 16-20): In each question below are
the same order. All the members are facing north. given two/three statements followed by two conclusions
C is sitting fourth from the left end but is not an numbered I and II. You have to take the given statements
immediate neighbour of  and is not the neighbour of @. © to be true even if they seem to be at variance with
is sitting between B and C. A is not an immediate neighbour commonly known facts. Read all the conclusions and then
of any symbol. D is not an immediate neighbour of $. @ is decide which of the given conclusions logically follows
sitting second from the right end but is not an immediate from the given statements, disregarding commonly
neighbour of B.  is second to the right of C. known facts. Give answer
6. How many persons are sitting between C and D? 1) if only conclusion I follows.
1) None 2) Two 3) Three 2) if only conclusion II follows.
4) One 5) None of these 3) if either conclusion I or II follows.
7. Which of the following pairs are immediate neighbours 4) if neither conclusion I nor II follows.
of ? 5) if both conclusions I and II follow.
1) $, D 2) @, D 3) C, @ 16. Statements: All coins are papers.
4) C, D 5) None of these Some papers are books.
8. Which of the following statements is true with respect Conclusions: I. Some books are coins.
to B? II. At least some books are papers.
17. Statements: Some men are old. 27. How many such numbers are there in the above
No old is a boy. arrangement each of which is immediately preceded
Conclusions: I. Some boys are not men. by a vowel?
II. All boys being men is a 1) None 2) One 3) Two
possibility. 4) Three 5) None of these
18. Statements: Some pens are books. 28. How many such consonants are there in the above
All pages are books. arrangement each of which is immediately followed
Conclusions: I. At least some pens are pages. by a consonant?
II. All books being pages is a 1) None 2) One 3) Two
possibility. 4) Three 5) None of these
(19-20): 29. ‘O’ is related to ‘A’ in the same way as ‘9’ is related to
Statements: Some chocolates are sweets. ‘Y’. Following the same pattern, ‘8’ is related to which
No sweet is a candy. of the following?
No fruit is a candy. 1) T 2) @ 3) A
19. Conclusions: I. No sweet is fruit. 4) U 5) None of these
II. Some chocolates are not candies. 30. Which of the following is the sixth to the right of the
20. Conclusions: I. No candy is sweet. thirteenth from the right end?
II. Some chocolates are candies. 1) @ 2) Y 3) E
Directions (Q. 21-25): Study the following information 4) A 5) None of these
carefully and answer the given questions. Directions (Q. 31-35): The following questions are
J, H, N, P and F are five scientists involved in an based on the diagram given below:
international research project and hold a meeting in New The circle represents the group of LLBs.
York. There are certain limitations on their language skills. The Triangle represents the group of the members of
Scientists J knows only Italian and European. H is Red Cross Society.
good at Italian and English. N likes to speak English and The rectangle represents the group of UGC members.
European languages. P knows Chinese and Italian. F likes
to speak the three languages other than Italian.
21. Who among the following cannot converse without
an interpreter?
1) J, H 2) N, P 3) F, N

K
4) P, F 5) None of these
22. Which of the following languages was used the least
in the meeting? 31. Who among the following represent(s) only the
1) Chinese 2) Italian 3) English members of Red Cross Society and LLBs?
4) European 5) Can’t be determined 1) G, C 2) C, F, G 3) C
23. Besides F, who among the following can converse 4) F 5) None of these
with P without an interpreter? 32. How many LLBs are the members of UGC?
1) Only H and N 2) Only N 3) Only J and H 1) None 2) One 3) Two
4) Only H 5) None of these 4) Three 5) None of these
24. Who among the following speak Italian? 33. Who among the following are the members of only

KUNDAN
1) J, F, N 2) H, N, F 3) J, P, F Red Cross Society?
4) P, H, J 5) None of these 1) G, H, F 2) H, G 3) F, H
25. How many scientists know Chinese? 4) G, F 5) None of these
1) Two 2) Three 3) Four 34. Which of the following LLBs are the members of both
4) Can’t be determined 5) None of these Red Cross Society and UGC?
Directions (Q . 26-30): Study the following 1) H, D, J 2) D, J 3) A, B, J, D
arrangement carefully and answer the questions given 4) D, J, C 5) None of these
below: 35. Who among the following is/are the members of both
# O P5 Q 6 A S E 9 % X Z 1Y8 L H3 T @ AU 4 7 8 W Red Cross Society and UGC but not the members of
26. If all the symbols are dropped from the given LLB?
arrangement which element will be eighth from the left 1) H 2) C 3) D, J
end? 4) A, B, D, J 5) None of these
1) X 2) 9 3) Z Directions (Q. 36-40): Study the following information
4) E 5) None of these carefully and answer the given questions.
Eight friends L, M, N, O, P, Q, R and S are sitting calls “megatrends” – things that are highly likely to occur
around a circular table facing the centre, but not necessarily – and “game-changers”, which are far less certain. Among
in the same order. the megatrends is growing prosperity across the globe.
R is sitting third to the left of O, who is between P and “The growth of the global middle class constitutes a
N. M is sitting between L and N. Q is not opposite O. tectonic shift: for the first time, a majority of the world’s
36. Who among the following sits third to the left of P? population will not be impoverished, and the middle classes
1) L 2) S 3) Q will be the most important social and economic sector in
4) M 5) None of these the vast majority of countries around the world,” the report

K
37. How many persons sit between P and L? says.
1) Two 2) One 3) Three With prosperity spreading across the globe will come
4) None 5) None of these shifts in influence and power. “The diffusion of power
38. Who among the following sits on the immediate right among countries will have a dramatic impact by 2030. Asia
of O? will have surpassed North America and Europe combined
1) N 2) P 3) M in terms of global power, based upon GDP, population size,
4) Q 5) Can’t be determined military spending, and technological investment. China
39. Which of the following pairs are the neighbours of R? alone will probably have the largest economy, surpassing
1) P, S 2) O, M 3) L, Q that of the United States a few years before 2030,” the
4) Q, S 5) None of these report says. In addition to China, India and Brazil, regional
40. Four of the following five are alike in a certain way players such as Colombia, Indonesia, Nigeria, South Africa

KUNDAN
based on their positions in the above arrangement and Turkey will become especially important to the global
and hence form a group. Find out the one that does economy.
not belong to that group. “Meanwhile, the economies of Europe, Japan, and
1) S, O 2) P, L 3) Q, M 4) R, N 5) M, P Russia are likely to continue their slow relative declines,”
the report says. “In a tectonic shift, individuals and small
Test-II: English Language groups will have greater access to lethal and disruptive
technologies (particularly precision-strike capabilities,
Directions (Q. 41-50): Read the following passage cyber instruments, and bio-terror weaponry), enabling them
carefully and answer the questions given below it. Certain to perpetrate large-scale violence – a capability formerly
words/phrases in the passage are printed in bold to help the monopoly of states,” it says.
you locate them while answering some of the questions. The megatrends also point to increased instability
A US intelligence portrait of the world in 2030 predicts because of rising demand for water, food and energy
that China will be the largest economic power, climate compounded by climate change.
change will create instability by contributing to water and 41. As per a US intelligence portrait of the world in 2030,
food shortages, and there will be a “tectonic shift” with what would be the impact of the diffusion of power
the rise of a global middle class. among countries?
The National Intelligence Council’s (NIC) Global 1) Regional players such as Indonesia, Nigeria,
Trends Report, published every five years, paints a future Colombia and South Africa will not play any
in which US power will greatly diminish but no other significant role in the global economy.
individual state rises to supplant it. 2) Asia will have surpassed North America and Europe
“There will not be any hegemonic power. Power will combined in terms of global power, population size,
shift to networks and coalitions in a multipolar world,” it military spending and technological investment.
says. The report offers a series of potential scenarios for 3) Europe, Japan and Russia will register a significant
2030. It says the best outcome would be the one in which economic growth.
“China and the US collaborate on a range of issues, leading 4) Individuals and small groups will have no access
to broader global co-operation.” It says the worst is a world to disruptive technologies.
in which “the US draws inward and globalisation stalls.” 5) All the above
“A collapse or sudden retreat of US power probably would 42. Which of the following statements is not true
result in an extended period of global anarchy; no leading according to the given passage?
power would be likely to replace the United States as (A) According to NIC’s Global Trends Report, US
guarantor of the international order,” it says, working on power will greatly reduce but no other individual
the assumption that the US is a force for stability – a state will rise to replace it.
premise open to challenge in Iraq and elsewhere in the (B) Power will shift to a multipolar world.
Middle East and beyond. (C) There is no chance of global anarchy even during
The NIC report draws a distinction between what it collapse of US power.
1) Only (A) 2) Only (B) 50. Retreat
3) Only (C) 4) Only (A) and (B) 1) withdraw 2) refuge 3) reminiscence
5) Only (B) and (C) 4) reverse 5) advance
43. Which of the following is not among the predictions Directions (Q. 51-55): Pick out the most effective
made about the world in 2030 by a US intelligence word from the given words to fill in the blank to make the
report? sentence meaningfully complete.
1) There will be a tectonic shift with the rise of a global 51. Having played the piano since the age of eight, he
middle class. was sure he was ________ to pursue a career in the
2) Due to climate change there will be shortages of music industry.
food and water. 1) assured 2) meant 3) dreamt
3) China will emerge as the largest economic power. 4) reduced 5) declined
4) India will be a technologically advanced country. 52. If honesty, humility, hope and power ________ Sachin
5) None of these Tendulkar, then his era is something we should learn
44. What could be the worst outcome of the US intelligence from.
report about potential scenarios for 2030? 1) marks 2) reflects 3) figures
1) India would emerge as the most powerful nuclear 4) proposes 5) symbolise
country in the world. 53. The promise of a new Nepal seems to be on the retreat,
2) China and the US would collaborate on a range of with the Maoists losing people’s goodwill and
issues leading to a broader global cooperation. ________ an opportunity to create a more democratic
3) The United States would shrink inward and the nation under a new constitution.
process of globalisation would stall. 1) reeling 2) enduring 3) squandering
4) Japan would merge as the technology giant among 4) crushing 5) extorting
the Asian countries. 54. The author fairly records the ________ realities which
5) None of these faced the man.
45. What would be the impact of megatrends? Give your 1) grim 2) stern 3) trim
answer in the context of the given passage. 4) baseless 5) welcome
1) There will be a shift of power from upper class to 55. The deal signed in November could have been
middle class. clinched easily when Iran met in Geneva earlier in the
2) A majority of the world population will not be same month, but was undermined at the ________
impoverished. hour by France.

K
3) The middle classes will be the most important social 1) beginning 2) end 3) twelfth
and economic sector in the vast majority of 4) eleventh 5) first
countries around the world. Directions (Q. 56-60): In each of these questions, two
4) Only 2) and 3) sentences I and II are given. Each sentence has a blank in
5) All 1), 2) and 3) it. Five words 1), 2), 3), 4) and 5) are suggested. Out of
Directions (Q. 46-48): Choose the word/group of these only one fits at both the places in the context of each
words which is MOST SIMILAR in meaning to the word/ sentence. The number of that word is the answer.
group of words printed in bold as used in the passage. 56. I. Laloo’s conviction in the fodder scam has brought
46. Access the curtains down on a politician ________ with
1) assent 2) mishap 3) approach remarkable politician sense, rustic wit and uncanny

KUNDAN
4) applaud 5) enough ability to connect with people.
47. Supplant II. For centuries, a whole range of foods has been
1) beseech 2) replace 3) suspect ________ with subtle flavours through the use of
4) support 5) endure herbs.
48. Distinction 1) filled 2) devoid 3) endowed
1) importance 2) definite 3) discard 4) enriched 5) occupied
4) difference 5) confusion 57. I. Gandhi is remembered in South Africa now as an
Directions (Q. 49-50): Choose the word/group of important leader of the second rank who ________
words which is MOST OPPOSITE in meaning of the word/ the African movement.
group of words printed in bold as used in the passage. II. The car’s performance was effortless and its
49. Diminish handling was precise and quickly ________
1) surge 2) wonder confidence.
3) increase 4) wane 1) instigated 2) inflamed 3) instilled
5) decline 4) started 5) inspired
58. I. In London late-night taxis are ________ and far (D) It its broadest sense education may be defined as
from cheap. the process whereby the social heritage of a group
II. The most dramatic breakthrough that occurred in is passed on from one generation to another and
the recent past in physics is, undoubtedly, the whereby the child learns the rules of social
experimental discovery of the ________ particle behaviour.
sought by physicists for over four decades, known (E) It is a process of transmission of social heritage.
as the Higgs-boson. (F) It is synonymous with socialisation.
1) unavailable 2) deceptive 3) unseen 66. Which of the following should be the THIRD sentence

K
4) elusive 5) costly after rearrangement?
59. I. Often, opposition parties ________ a motion 1) A 2) B 3) C 4) D 5) E
criticising the government for doing nothing about 67. Which of the following should be the LAST (SIXTH)
the problem. sentence after rearrangement?
II. The only question is whether the President can 1) B 2) F 3) A 4) C 5) D
use his extraordinary political skills to turn the 68. Which of the following should be the FOURTH
________ on his opponents. sentence after rearrangement?
1) table 2) bring 3) claim 1) C 2) E 3) D 4) B 5) A
4) hold 5) face 69. Which of the following should be the FIRST sentence
60. I. He had realised the mistake he had made and was after rearrangement?
now ________ himself. 1) F 2) C 3) B 4) E 5) D

KUNDAN
II. Does this institution have any ________ features? 70. Which of the following should be the SECOND
1) redressing 2) redeeming 3) recovering sentence after rearrangement?
4) compensating 5) changing 1) D 2) B 3) E 4) F 5) C
Directions (Q. 61-65): Read each sentence to find out Directions (Q. 71-80): In the following passage, some
whether there is any grammatical or idiomatic error in it. of the words have been left out, each of which is indicated
The error, if any, will be in one part of the sentence. The by a number. Find the suitable word from the options given
number of that part is the answer. If there is ‘No error’, against each number and fill up the blanks with
the answer is 5). (Ignore errors of punctuation, if any.) appropriate words to make the paragraph meaningfully
61. 1) The number of patients have / 2) risen this year by complete.
/ 3) as much as forty per cent / 4) in comparison to ten RBI governor Raghuram Rajan demonstrated that he
per cent last year. / 5) No error meant to go beyond platitudes on financial inclusion by
62. 1) Police sources said senior officers of the rank of / 2) setting up a committee on financial services for small
Deputy Commissioner of Police and Joint businesses and low-income households. The committee,
Commissioner of Police / 3) were ordered patrolling / headed by Nachiket Mor, has lots on its (71), including a
4) various roads till 2 : 30 am. / 5) No error proposal to give official (72) to a new segment of non-
63. 1) The Education Minister has welcomed the new / 2) banking finance companies (NBFCs) that specifically (73)
nursery guidelines, saying these / 3) will help to make to small production units and low-income households. The
the / 4) current education system transparent. / 5) No proposal, made by Chennai-based Shriram Capital Ltd, (74)
error serious consideration. Particularly when it echoes the views
64. 1) One should keep / 2) one’s promise / 3) if one wants of a government-appointed Key Advisory Group on the
respect / 4) from both friends and foes. / 5) No error subject.
65. 1) A small aircraft made an emergency landing / 2) on The RBI’s long-term regulatory thrust has been to (75)
National Highway 69 and take off / 3) from the same NBFCs to extinction, seeing them as vermin that gnaw away
place / 4) a couple of hours later. / 5) No error at probity and prudence in the world of finance. The RBI
Directions (Q. 66-70): Rearrange the following six wants banks to take over all the functions that are (76) by
sentences (A), (B), (C), (D), (E) and (F) in the proper NBFCs. Over the years, the RBI has (77) down the number
sequence to form a meaningful paragraph and then answer of NBFCs that are allowed to take deposits from 1,420 in
the questions given below. 1998 to 311 in 2010. The regulations that govern them have
(A) It is a conscious training of the young for the (78) been tightened. At the same time, banks have failed to
later adoption of adult roles. (79) the responsibility of meeting the credit requirements
(B) It is an attempt to shape the development of the of India’s teeming, diverse, dynamic and under-funded
coming generation in accordance with the social unorganised sector. The sector (80) 78% of its credit needs
ideals of life. from NBFCs, moneylenders and other informal sources.
(C) Education is one of the basic activities in all And at an exorbitant cost.
human societies.
71. 1) programme 2) plan 3) platter
92. ? + 676 = 74
4) utensils 5) platform
72. 1) recognition 2) order 3) consensus 1) 1936 2) 2116 3) 2304 4) 2704 5) 3136
4) value 5) perception 93. 73804 + 81819 – 68527 – 45366 = ?
73. 1) take 2) call 3) send 1) 41730 2) 42130 3) 42830 4) 43670 5) 43730
4) accommodate 5) lend 94. (3.6)–2.8 × (46.656)1.2 ÷ 12.96 = (3.6)?
74. 1) preserve 2) deserves 3) entitles 1) 1.2 2) –1.2 3) 4.4
4) wants 5) attracted 4) –4.4 5) None of these
75. 1) drown 2) make 3) throw 4 5
95. of 1316 – of 1152 = ?
4) drive 5) strive 7 9
76. 1) performed 2) worked 3) finished 1) 104 2) 108 3) 112 4) 116 5) 120
4) served 5) contrived 96. 84 + ? = 14641 + 11
77. 1) turned 2) noted 3) brought
4) pull 5) came 1) 36 2) 40 3) 44 4) 48 5) 52
78. 1) grisly 2) largely 3) naturally 97. 11304 ÷ 157 + 65 = ?
4) formally 5) gradually 1) 131 2) 137 3) 143 4) 149 5) 153
79. 1) overtake 2) shoulder 3) head 98. 18.4 ÷ 0.32 + 17.5 = ?
4) eye 5) handle 1) 75 2) 84 3) 90 4) 96 5) 105
80. 1) affords 2) completes 3) filled 3 5
4) meets 5) requisite 99. of of 5928 = ?
19 13
1) 320 2) 360 3) 380 4) 420 5) 450
Test-III: Quantitative Aptitude 3
100. 2744  3 5832  ?
Directions (Q. 81-100): What value should come in 1) 784 2) 900 3) 1024 4) 1156 5) 1296
place of question mark (?) in the following equations? 101. An item is sold on 20% discount and the dealer still
67.7 – 35.4 gets 20% profit. If the tag price of the item is `1125
81. ?
1. 9 then what is its cost price?
1) 15 2) 17 3) 19 4) 21 5) 24 1) `600 2) `750 3) `800
82. 18% of 56% of ? = 559.44 4) `850 5) `900

K
1) 4550 2) 5050 3) 5500 4) 5550 5) 5750 102. An amount is given at an iterest of 8% pa. What is the
83. 6.5 × 7.5 × 8.5 × ? = 16575 amount if the difference of compound and simple
1) 20 2) 40 3) 604) 80 5) 120 interest for 2 years is `352?
84. ?% of 848 = 678.4 1) `55000 2) `50000 3) `45000
1) 70 2) 75 3) 80 4) 85 5) 90 4) `40000 5) `35000
85. 4327.83 + 1068.54 – 3136.26 – 1119.81 = ? 103. A bag contains 6 red, 5 black and 3 blue marbles. If 2
1) 964.3 2) 1084.3 3) 1410.3 marbles are drawn at random, what is the probability
4) 1140.3 5) 1440.3 that both are of the same colour?
86. 27 + 28 × 3.5 – 14 × 6.5 = ?
4 11 27 64 5
1) 48 2) 42 3) 38 4) 34 5) 28 1) 2) 3) 4) 5)
13 13 91 91 12
4

KUNDAN
87. of 1058.4 = ?% of 448 104. How many different words can be formed with the
7
1) 175 2) 165 3) 155 4) 145 5) 135 letters of the word ‘CALCULATOR’?
1) 441800 2) 453600 3) 457400
? 16 4) 464100 5) None of these
88. 
441 ? 105. The top of a 24-metre-high tower makes an angle of
1) 72 2) 84 3) 96 4) 108 5) 124 elevation of 60º with the bottom of a pole and an angle
of elevation of 30º with the top of the pole. What is the
89. 1369  72.25  ? height of the pole?
1) 302.5 2) 306.5 3) 308.5 4) 312.5 5) 314.5 1) 8m 2) 12m 3) 16m
90. 45% of 660 – 32% of 555 = ? 4) 20m 5) None of these
1) 113.4 2) 116.4 3) 119.4 4) 123.4 5) 127.4 106. How long will an 80-m-long train travelling at 30 kmph
91. 854.4 ÷ 4.8 = ? take to cross a bridge whose length is 100m?
1) 164 2) 172 3) 175 4) 178 5) 184 1) 17.8 seconds 2) 18.4 seconds 3) 19.2 seconds
4) 20.5 seconds 5) 21.6 seconds 120. A person spent 40% of his monthly salary on house
107. A car completes a journey in 24.5 hours. It travelled rent and 25% of the remaining salary on food and he
the first half at 15 kmph and the second half at 20 saved the remaining amount. If he saves `48600
kmph. What is the distance? annually then what is his monthly salary?
1) 320 km 2) 260 km 3) 420 km 4) 450 km 5) 480 km 1) `8000 2) `9000 3) `10000
108. A bag contains coins of 5 rupees, 2 rupees and 1 rupee, 4) `12000 5) `15000
whose numbers are in the ratio of 9 : 7 : 4. If the total
amount in the bag is `378 then what is the total number Test-IV: General Awareness

K
of coins of 5-rupee denomination?
1) 24 2) 42 3) 48 4) 54 5) 72 (with Special Reference to Banking Industry)
109. An item when sold at a profit of 20% yields `260 more
than when sold at a loss of 20%. What is the cost 121. The Union Cabinet gave its nod for conferring
price of the item? constitutional status on the proposed Judicial
1) `550 2) `650 3) `750 4) `850 5) `950 Appointments Commission (JAC) recently. The
110. 5 men and 4 women together earn `517 and 3 men and Commission will be responsible for
6 women together earn `483. Then what is the wage 1) appointment and transfer of judges of lower
judiciary
of 8 men and 8 women?
2) appointment and transfer of judges of higher
1) `864 2) `884 3) `904
judiciary
4) `924 5) `942

KUNDAN
3) appointment of judges to non-judicial activities
111. When 40 is added to 40% of a number, the result thus
4) appointment of judges after their superannuation
obtained is obtained 40% of 325. What is the number?
5) None of these
1) 200 2) 225 3) 250 4) 275 5) 300
122. Which of the following countries has developed its
112. If (84)2 is added to the square of a number, the answer
own satellite navigation system Beidou and is offering
so obtained is 10900. What is the number?
it free of cost to its neighbouring countries?
1) 62 2) 64 3) 66 4) 68 5) 70
1) South Korea 2) Japan 3) Singapore
113. What is the average of the following set of scores?
4) China 5) Malaysia
189, 276, 312, 447, 581, 613, 774
123. The government has decided to form an Exchange
1) 452 2) 454 3) 456 4) 458 5) 462
Traded Fund (ETF) comprising scrips of 11 blue-chip
114. When 36 is added to a number, the number becomes
state-owned firms with a view to garnering what
7 amount from the market in the next fiscal year?
its . What is the number?
4 1) `2500 cr 2) `3000 cr 3) `3500 cr
1) 42 2) 44 3) 48 4) 52 5) 56 4) `2550 cr 5) `4000 cr
124. The country’s biggest lender, State Bank of India
1088 ?
115.  (SBI), appointed who among the following as its
? 833 Managing Director and group executive in charge of
1) 952 2) 916 3) 884 4) 842 5) 804 corporate banking recently? The seat was vacated by
116. Which of the following is the largest fraction? Arundhati Bhattarcharya, who became the Chairman
3 4 5 10 11 of the bank in Oct 2013.
1) 2) 3) 4) 5) 1) Amar Lal Daultani 2) Rakesh Sethi
7 9 11 17 19
3) P Pradeep Kumar 4) CV Rajendran
117. What should come in place of question mark (?) in the 5) Rajeev Rishi
following number series? 125. A debit card holder cannot perform which of the
4 5 19 82 377 ? following functions?
1) 1904 2) 1928 3) 1942 4) 1956 5) 1966 1) Make Point of Sale (PoS) purchase
3 2) Recharge pre-paid mobile phones
118. of a number exceeds its one-fourth by 10. What is 3) Pay life insurance premium
7
4) Make donation
the number?
5) None of the above
1) 42 2) 56 3) 70 4) 84 5) 28
126. The 11-day long world famous Dhanu Yatra of Bargarh
119. The price of 8 pens and 20 copies is `416. What is the
kicked off in Bargarh district in which of the following
price of 10 pens and 25 copies?
states recently?
1) `520 2) `540 3) `560
1) Rajasthan 2) Tamil Nadu
4) `580 5) `640
3) Madhya Pradesh 4) Odisha
5) Gujarat 5) None of these
127. Which of the following countries successfully tested 135. Which of the following is the best option to park
its advanced missile defense system ‘Arrow 3’ money for a long period of time to earn a high rate of
recently? The system to be operational by 2016 can interest?
intercept missiles closer to their launch sites. 1) Savings account 2) Current account
1) US 2) Israel 3) China 3) Fixed Deposit account 4) Recurring account
4) Japan 5) South Korea 5) None of these
128. Who among the following has taken over as the new 136. The Reserve Bank of India (RBI) granted QCCP status
president of NASSCOM, the premier organisation that to Clearing Corporation of India Ltd (CCIL) recently.
represents Indian software industry? QCCP stands for
1) Phaneesh Murthy 2) Kris Gopalakrishnan 1) Queried Central Counterparty
3) Lakshmi Narayanan 4) R Chandrasekhar 2) Qualifying Central Counterparty
5) None of these 3) Qualitative Central Counterparty
129. What is take-out financing? 4) Quantitative Central Counterparty
1) A method of providing finance for longer-duration 5) None of these
projects 137. Name the first aircraft carrier of China which completed
2) A method of providing finance for short-duration its trial recently.
projects 1) Minsk 2) Kiev 3) Varyag
3) A method of providing finance without any 4) Liaoning 5) None of these
paperwork 138. Who among the following has been appointed the
4) A method of providing finance on very low interest new president of the South Asian Federation of
rates Accountants (SAFA) recently?
5) None of these 1) Vinod Rai 2) Subodh Kumar Agrawal
130. Senior Citizen Savings Scheme (SCSS) is a high-return 3) VN Kaul 4) Shashi Kant Sharma
scheme launched in 2004 by the govt with a time period 5) None of these
of five years. Who among the following, 60 yrs (55 yrs 139. The World Bank agreed to extend its support to which
in some cases) and above, is eligible to open an of the following state govts with an assistance
account under this scheme? programme of about `2,254 cr ($490 mn) for road and
1) Non-Resident Indians (NRIs) other projects over the coming two years?
2) Persons of Indian Origin (PIOs) 1) Rajasthan 2) Madhya Pradesh

K
3) Hindu Undivided Family (HUF) 3) Odisha 4) Tamil Nadu
4) A resident Indian 5) Kerala
5) All the above 140. The Govt is mulling introducing provisions for removal
131. Who among the following has lifted the Chennai Open of CBI Director, who gets a fixed tenure of ______(min)
tennis trophy recently? after a 1997 Supreme Court order.
1) Stanislas Wawrinka 2) Edouard Roger-Vasselin 1) One year 2) Two years 3) Three years
3) Tomas Berdych 4) Richard Gasquet 4) Five years 5) Not fixed
5) None of these 141. China eased its stringent one-child policy, imposed
132. One of the best footballers of all time, Eusebio da Silva more than three decades ago, recently. Now, which of
Ferreira, died recently. He belonged to which of the the following couples will be allowed to have two

KUNDAN
following countries? children?
1) France 2) England 3) Portugal 1) Couples having first child with disability
4) Spain 5) None of these 2) Couples where either parent has no siblings
133. The book titled The Hundred Names of Darkness has 3) Couples having a baby girl as first child
been authored by 4) Couples having combined income of at least 1 mn
1) Nilanjana Roy 2) Srinath Perur yuan per annum
3) Frank Dikotter 4) Amish Tripathi 5) None of these
5) None of these 142. Which of the following countries is all set to host first
134. The process through which an investor’s physical Wimbledon tennis event outside UK?
share certificate gets converted to electronic format 1) Australia 2) India 3) Japan
which is maintained in an account with the Depository 4) China 5) None of these
Participant, is termed as 143. Which of the following abbreviations and their
1) Materialisation 2) Dematerialisation expanded forms is NOT matched correctly?
3) Conversion 4) Prioritisation 1) BIFR: Board for Industrial and Financial
Reconstruction following dates to be observed as the World Wildlife
2) CBS: Central Banking Solution Day recently?
3) FPO: Follow-on Public Offer 1) 3 Jan 2) 3 Feb 3) 3 Mar
4) NCD: Non-Convertible Debentures 4) 3 Apr 5) None of these
5) None of these 153. When the RBI opts to reduce liquidity in the banking
144. We often read in financial newspapers about the RBI system, it increases which of the following?
reducing or increasing certain rates by a few basis 1) CRR 2) SLR 3) Repo rate
points. Which of the following represents one basis 4) Reverse repo rate 5) Any of the above

K
point? 154. What is the term used for the debt instrument, secured
1) One per cent by the collateral of an immovable property, in which
2) One-tenth of one per cent the borrower is obliged to pay back the debt on terms
3) One-hundredth of one per cent agreed upon?
4) One-thousandth of one per cent 1) Lien 2) Assignment 3) Mortgage
5) None of the above 4) Hypothecation 5) Pledge
145. International Civil Aviation Organisation (ICAO) has 155. The Foreign Investment Promotion Board (FIPB)
set which of the following dates in 2015 as deadline to approved the proposal to acquire shares for over
phase out non-machine readable passports? `10,000 cr by which of the following telecom majors
1) 24 Mar 2) 24 Sep 3) 24 Nov operating in India, making it one of the largest foreign
4) 24 Dec 5) None of these investors in India?

KUNDAN
146. BCIM is a strategic economic corridor proposed to be 1) Systema 2) AT&T 3) Vodafone
set up covering which of the following countries? 4) Maxis 5) None of these
1) Brazil, China, India and Myanmar 156. Jayanthi Natarajan resigned from the Union Council
2) Brazil, China, India and Malaysia of Ministers recently. She was Minister of State with
3) Bangladesh, China, India and Myanmar independent charge of
4) Bangladesh, China, India and Malaysia 1) Environment and Forests
5) None of these 2) Communications and Information Technology
147. As per the Year End Review, 2013 of the Govt of India, 3) Textiles
what is India’s position in the production of crude 4) Planning
steel in the world? 5) None of these
1) Second 2) Third 3) Fourth 157. Mary Barra has been named as the chief executive
4) Fifth 5) None of these officer (CEO) by which of the following auto majors,
148. What does the letter ‘A’ represent in the abbreviation making her the first female CEO in the global
CRAR? automotive industry?
1) Allocation 2) Advance 3) Adequacy 1) Ford 2) General Motors
4) Assets 5) None of these 3) Renault 4) Suzuki
149. Revolving credit is made available by the issuer in the 5) None of these
case of which of the following instruments? 158. A Banking Ombudsman can reject a complaint at any
1) Debit card 2) Credit card stage on which of the following grounds?
3) Cashless claim card 4) ATM card 1) There is no loss or damage or inconvenience
5) None of these caused to the complainant in the opinion of the
150. Who among the following received the Polly Umrigar Banking Ombudsman.
Award for being India’s best international cricketer of 2) The compensation sought from the Banking
2012-13, at the BCCI’s annual awards ceremony? Ombudsman is beyond `10 lakh.
1) Abhishek Nayar 2) Ravichandran Ashwin 3) The complaint is not pursued by the complainant
3) MS Dhoni 4) Cheteshwar Pujara with reasonable diligence.
5) None of these 4) Any of the above
151. As MSP stands for Minimum Support Price, what does 5) Only 1) and 3)
MEP stand for? 159. The Supreme Court of which of the following
1) Minimum Exchange Price countries, in a recent landmark judgment, has given
2) Minimum Exemption Price voters the right to cast negative votes during the
3) Minimum Entry Price parliamentary or local elections?
4) Minimum Export Price 1) Bangladesh 2) Egypt 3) Myanmar
5) None of these 4) Nepal 5) Brazil
152. The United Nations (UN) has declared which of the 160. The United Nations (UN) has pegged India’s economic
growth forecast for 2013 to what per cent recently? 168. The secret code that restricts entry to some programs
1) 3.8 per cent 2) 4.2 per cent 3) 4.8 per cent is
4) 5.6 per cent 5) None of these 1) entry code 2) passport 3) password
4) access code 5) None of these
Test-V: Computer Knowledge 169. The blinking symbol which indicates the next position
of the character is called
161. In multitasking 1) Delete key 2) Control key 3) Cursor
1) we can use only one CPU. 4) Return key 5) None of these
2) we follow single-user concept. 170. A small figure which depicts some application on the
3) we follow multi-user concept. screen is called
4) Both (1) and (2) 1) Menu 2) Photo 3) Modem
5) Both (1) and (3) 4) Icon 5) None of these
162. In OSI model, the job of dialogue control and token 171. On an Excel sheet the active cell is indicated by
management is performed by ________ layer. ________.
1) Presentation 2) Network 3) Session 1) a dark wide border 2) a dotted border
4) Transport 5) Datalink 3) a blinking border 4) All the above
163. Programs from the same developer, sold bundled 5) None of these
together, that provide better integration and share 172. An Excel workbook is a collection of ________.
common features, toolbars, and menus are known as 1) workbooks 2) worksheets
1) Software suites 3) charts 4) worksheets and charts
2) Integrated software packages 5) None of these
3) Software processing packages 173. Pick the one that is used for logical operations or
4) Personal information managers comparisons such as less than, equal to or greater
5) None of these than.
164. Which of the following is correct about a data 1) Arithmetic and Logic Unit
warehouse? 2) Control Unit
1) It can be updated by end users. 3) Both 1) and 2)
2) It contains numerous naming conventions and 4) 2) only and that too rarely
formats. 5) None of these
3) It is organised around important subject areas. 174. Analog computer works on the supply of

K
4) It contains only current data. 1) continuous electrical pulses
5) None of these 2) electrical pulses but not continuous
165. On a school computer, Lucy learned how to copy 3) magnetic strength
programs. A classmate asked her to copy a program 4) None of these
for his home use. Her most ethical response would be 5) All of these
which of the following? 175. A hard copy of a document is
1) I’ll copy it, but you have to promise not to tell 1) stored in the hard disk 2) stored on a floppy
anyone. 3) stored on a CD 4) printed on the printer
2) I don’t know how to copy disks. 5) None of these
3) I can’t copy it because it will break copyright laws. 176. The capability of the operating system to enable two

KUNDAN
4) I’ll copy it for you, but you can’t copy it for anyone or more than two programs to execute simultaneously
else. in a single computer system by using a single
5) None of these processor is called
166. Which of the following is not true? 1) Multiprocessing 2) Multitasking
1) LOGO stands for Live On Google’s Orkut. 3) Multiprogramming 4) Multiexecution
2) BIOS is a system software. 5) None of these
3) GRUB is a Linux boot loader. 177. ENIAC stands for
4) Ruby is a web scripting language. 1) Electrical Numerical Integrator and Calculator
5) LILO is a Linux boot loader. 2) Electronic Numerical Integrator and Computer
167. Which of the following are computers that can be 3) Electronic Numerical Integrator and Calculator
carried around easily? 4) Electronic Number Integrator and Calculator
1) Laptops 2) Supercomputers 5) None of these
3) PCs 4) Minicomputers 178. The first generation of computers was based on which
5) None of these of the following technologies?
1) Transistor 2) Vacuum Tube 3) LSI 5) None of these
4) VLSI 5) None of these 188. A ________ is approximately a million bytes.
179. Before a disk drive can access any sector record, a 1) gigabyte 2) kilobyte 3) megabyte
computer program has to provide the record’s disk 4) terabyte 5) None of these
address. What information does this address specify? 189. The________enables your computer to connect to
1) Track number 2) Sector number other computers.
3) Surface number 4) All the above 1) video card 2) sound card
5) None of these 3) network interface card 4) controller card

K
180. As compared to diskettes, the hard disks are 5) None of these
1) more expensive 2) more portable 190. What menu is selected to save or save as?
3) less rigid 4) slowly accessed 1) Tools 2) File 3) Format
5) None of these 4) Edit 5) None of these
181. To access a Mainframe Computer, users normally use 191. Soft copy refers to
1) terminal 2) PC 3) Laptop 1) printed output 2) music sounds
4) iPad 5) None of these 3) screen output 4) digitising
182. The device used to carry digital data on analog lines 5) None of these
is called 192. The method of file organisation in which data records
1) Modem 2) Multiplexer 3) Modulator in a file are arranged in a specified order according to
4) Demodulator 5) None of these a key field, is known as the

KUNDAN
183. Microsoft Works is an example of a(n) 1) Direct access method
1) software suite 2) Queuing method
2) personal information manager 3) Predetermined method
3) integrated software application package 4) Sequential access method
4) limited software edition 5) None of these
5) None of these 193. Which of the following was the first widely used OS
184. Microsoft Office, Corel WordPerfect Office, and Lotus with graphical interface?
SmartSuite are examples of 1) OS12 2) MVS
1) software suites 3) Mac OS 4) DOS
2) open-source products 5) VMS
3) freeware products 194. Lynx is a type of
4) personal information management applications 1) search engine 2) web index
5) None of these 3) meta-search engine 4) web browser
185. ________ is a software that is available for use at no 5) newsgroup
cost or for an optional fee, but usually with one or 195. In mark sensing, data is read
more restricted usage rights. 1) optically 2) electrically 3) magnetically
1) Firmware 2) Teachware 3) Testware 4) mechanically 5) None of these
4) Freeware 5) Groupware 196. ________ layer of OSI model is also called as end-to-
186. ________ involves checking or logging the data in; and layer.
checking the data for accuracy; entering the data into 1) Presentation 2) Network 3) Session
the computer; transforming the data and developing 4) Transport 5) Application
and documenting a database structure that integrates 197. To move to the beginning of a line of text, press the
the various measures. ________ key.
1) Data mining 1) Home 2) Alt 3) Pageup
2) Database 4) Enter 5) None of these
3) Data preparation 198. A ________ is a computer attached to the Internet
4) Data structuring that runs a special Web server software and can send
5) Data validation Web pages out to other computers over the Internet.
187. ‘MICR’ technology used for clearance of cheques by 1) web client 2) web system 3) web page
banks refers to 4) web server 5) None of these
1) Magnetic Ink Character Recognition 199. Which of the following best explains e-governance?
2) Magnetic Intelligence Character Recognition 1) Improving the functioning of Government
3) Magnetic Information Cable Recognition 2) Teaching Government employees the basics of
4) Magnetic Insurance Cases Recognition computing
3) Delivery of public services through internet
4) Framing of cyber-laws of chatting on internet
5) Convergence of e-mail and video-conferencing
200. To send another station a message, the main thing a
user must know is
1) how the network works.
2) the other station’s address.
3) whether the network is packet-switched or circuit-
switched.
4) whether this is a voice or data network.
5) None of these

Thus, we can’t compare P and N. Hence, I is Thus, we can’t compare Z and M. Hence,
Answers not true. (Z  M) is not true. Hence conclusion II is
1. 3; Given letters are DENO. Again, W < O or O > W is true. Hence, not true.
DONE and NODE are the only two words conclusion II is true. But Y > N or N < Y is true. Hence, conclusion
formed. 12. 4; Given statements: C = P ... (i) I is true.
G<C ... (ii) 16. 2; All coins are papers (A) + Some papers

K
2. 4;
N>G ... (iii) are books (I) = A + I = No conclusion. Hence,
Combining these statements, we have conclusion I does not follow.
3. 2; Again, Some papers are books  conversion
F O A M N E I  Some books are papers. Hence, conclusion
5 % 9 $ # 7 @ II follows.
Thus, we can’t compare N and C, or P and 17. 2; Some men are old (I) + No old is a boy
So, N. (E) = I + E = O = Some men are not boys.
Hence, neither conclusion I nor II is true. Hence, conclusion I does not follow. But the
13. 1; Given statements: I > N  T ... (i) possibility in II exists. Thus, conclusion II
E < R = T ... (ii) follows.
4. 1; Combining these statements, we have 18. 2; Some pens are books  conversion
 Some books are pens. Now, All pages are
books (A) + Some books are pens (I) = A + I

KUNDAN
Similarly, = No conclusion. Hence, conclusion I does
not follow. There is no negative statement.
Hence, the possibility in II exists. Thus,
Thus, R < I or I > R is true. Hence, conclusion conclusion II follows.
5. 2; B Y C X D W E V F U G T H S I I is true. 19. 2; No fruit is a candy  conversion 
(6-10): Again, E < N. Thus, (E  N) is not true. No candy is a fruit. Now, No sweet is a candy
Hence, conclusion II is not true. (E) + No candy is a fruit (E) = E + E = No
14. 4; Given statements: conclusion. Hence, conclusion I does not
6. 3 follow.
7. 5; $, @ Thus we can’t compare V and P. Hence, Again, Some chocolates are sweets (I) + No
8. 3 9. 1 10. 3 conclusion II is not true. sweet is a candy (E) = I + E = O = Some
11. 2; Given statements: K < N = O ... (i) Again, Q > Z is true. Hence, conclusion I (Q chocolates are not candies. Hence, conclusion
P > W < K ... (ii)  Z) is not true. II follows.
Combining these statements, we get 15. 1; Given statement: 20. 1; No sweet is a candy  conversion 
No candy is sweet. Hence, conclusion I
follows. But conclusion II does not follow.
(21-25): 88. 2; (?)2 = 441 × 16 = (21)2 × (4)2 104. 2; Total number of letters in the word.
Language Persons = (21 × 4)2 CALCULATER is 10, and A, C and L occur
 ? = 21 × 4 = 84 twice.
Italian J, H, P
89. 5; ? = 1369  72.25 10!
English H, N, F  Number of arrangements = = 453600
= 37 × 8.5 = 314.5 2!2!2!
European J, N, F 105. 3; Let the height of the pole be CE.
660 555
90. 3; ? =  45 –  32
Chinese P, F 100 100
21. 2 22. 1 23. 3 24. 4 25. 1 = 297 – 177.6 = 119.4

K
26. 2; New arrangement 854.4
91. 4; ? = = 178
4.8
92. 3; ? = 74 – 676
27. 3; E 9, U 4
28. 3; X Z, L H = 74 – 26 = 48
 ? = (48)2 = 2304
29. 2; Similarly, 93. 1; ? = 73804 + 81819 – 68527 – 45366 Then, in ADE, we get
= 41730
24
94. 2; (3.6)? = (3.6)–2.8 × (3.6)3 × 1.2 ÷ (3.6)2 tan 60º =
So, = (3.6)–2.8 + 3.6 – 2 = (3.6)–1.2 d
? = –1.2 24
4  1316 5 1152 or, DE =
30. 1; Sixth to the right of thirteenth from 95. 3; ? = – 3
7 9

KUNDAN
the right end is (13 – 6 =) 7th from the right Again, in ABC, we have
= 752 – 640 = 112
end, ie @.
31. 3 32. 3 33. 4 34. 2 35. 1 96. 4; ? = AB
14641  11 – 84 tan 30º =
BC
= 121 + 11 – 84 = 48
97. 2; ? = 11304 ÷ 157 + 65 24 1
= 72 + 65 = 137  AB = × = 8m
3 3
98. 1; ? = 18.4 ÷ 0.32 + 17.5
= 57.5 + 17.5 = 75 (  BC = DE)
3  5  5928  CE = BD = AD – AB = 24 – 8 = 16m
36. 2 37. 3 38. 1 39. 4 40. 5 99. 2; ? = = 360
19  13 5 25
41. 2 42. 3 43. 4 44. 3 45. 4 106. 5; Speed of train = 30 × = m/sec
100. 3; ? = 3 2744 + 3 5832 18 3
46. 3 47. 2 48. 4 49. 3 50. 5
51. 2 52. 5 53. 3 54. 1 55. 4 = 14 + 18 = 32  Length = 80 + 100 = 180m
56. 3 57. 5 58. 4 59. 1 60. 2  ? = (32)2 = 1024 3
61. 1; Replace ‘have’ with ‘has’ 80  Time = 180 = 21.6 seconds
25
62. 3; Replace ‘patrolling’ with ‘to patrol’ 101. 2; SP = 1125 × = `900
100 107. 3; Let the distance be x km.
63. 3; Delete ‘to’ Let the cost price be `x.
64. 5 x x
65. 2; Replace ‘take’ with ‘took’ 120 Then,  = 24.5
Then, x 
= 900 2  15 2  20
(66-70): CEDAFB 100
66. 4 67. 1 68. 5 69. 2 70. 3  x = `750 x x
or,   49
71. 3 72. 1 73. 5 74. 2 75. 4 Quicker Method: 15 20
76. 1 77. 3 78. 5 79. 2 80. 4
80 100 4 x  3x
67.7 – 35.4 32.3 Cost price = 1125 ×  = `750 or, = 49
81. 2; ? =   17 100 120 60
1.9 1.9
Difference  1002 60 49
559.44  100  100 102. 1; Amount = x= = 420 km
82. 4; ? = = 5550 r 2 7
56 18
108. 4; Let the number of coins be 9x, 7x
16575 352 10000 and 4x respectively.
83. 2; ? = = 40 = = `55000
65  7.5  8.5 64 Then, 9x × 5 + 7x × 2 + 4x = 378
103. 1; Total number of marbles in the bag or, 45x + 14x + 4x = 378
678.4  100 = 6 + 5 + 3 = 14
84. 3; ? =  80 378
848 x= =6
14! 14  13 63
85. 4; ? = 4327.83 + 1068.54 – 3136.26 –  n(S) = 14 C 2 = = Hence, no. of five-rupee coins = 9x = 9 × 6
1119.81 = 5396.37 – 4256.07 = 1140.3 12!2! 2
= 91 = 54
86. 4; ? = 27 + 28 × 3.5 – 14 × 6.5
109. 2; Let the cost price be `x.
= 27 + 98 – 91 = 34 And, n(E) = 6 C 2  5 C2 3 C 2
120x 80x
4  1058.4 448  ? = 15 + 10 + 3 = 28 Then, –  260
87. 5;  100 100
7 100
28 4 or, 40x = 260 × 100
 P(E) = =
105840  4 91 13
? =  135
7  448
119. 1; Let the price of a pen and that of a storage units. A megabyte is often referred
260100
x= = `650 copy be `x and `y respectively. to as a ‘Meg’ in Computer Vernacular.
40 Then, 8x + 20y = 416 ... (i) 189. 3 190. 2 191. 5 192. 4 193. 3
110. 3; Let the wage of a man be `x and that 194. 4; Lynx is a text-based web browser.
5
of a woman be `y. Now, eqn (i) × gives 195. 2 196. 4 197. 1 198. 4 199. 3
Then, 5x + 4y = 517 ... (i) 4 200. 2
And 3x + 6y = 483
5 5 5
or, x + 2y = 161 ... (ii) 8x  + 20y × = 416
Solving eqn (i) and (ii) 4 4 4
x = 65, y = 48 or, 10x + 25y = 520
 x + y = 113 Hence the cost of 10 pens and 25 copies is
 8x + 8y = 904 `52 0.
Hence, the earning of 8 men and 8 women 120. 2; Let the person’s monthly salary be
together will be `904. `x.
111. 2; Let the number be x.
40 2x
40x 40  Rent = x  =`
100 5
Then, 40 + = 325 × = 130
100 100
 2x  3x
40x Remaining amount =  x –  =
 5  5
or, = 130 – 40 = 90
100
Amount spent on food
90100 3x 25 3x
x= = 225 =  =`
40 5 100 20
112. 1; Let the number be x.
Then, x2 = 10900 – (84)2 3x 3x
Now, the remaining amount = –
= 10900 – 7056 5 20
= 3844
12 x – 3x 9x
x= 3844 = 62 62 = 62 = =`
20 20
113. 3; Average
9x
189  276  312  447  581  613  774 Thus, he saves .
= 20
7
Again, according to the question.
3192 9 x 48600
= = 456  = 4050
7 20 12
114. 3; Let the number be x.

K
4050 20
7x x= = `9000
Then, x + 36 = 9
4
121. 2 122. 4 123. 2 124. 3 125. 5
or, 4x + 144 = 7x 126. 4 127. 2 128. 4 129. 1 130. 4
or, 3x = 144 131. 1 132. 3 133. 1 134. 2 135. 3
 x = 48 136. 2 137. 4 138. 2 139. 1 140. 2
115. 1; (?)2 = 1088 × 833 141. 2 142. 2
= (17 × 64) × (17 × 49) 143. 2; Core Banking Solutions
= (7 × 8 × 17)2 144. 3 145. 3
 ? = 7 × 8 × 17 = 952 146. 3 147. 3 148. 4 149. 2 150. 2
3 4 5 151. 4 152. 3 153. 5 154. 3 155. 3
116. 4; = 0.428, = 0.44, = 0.45 156. 1 157. 2 158. 4 159. 4 160. 3
7 9 11

KUNDAN
161. 4 162. 3 163. 1 164. 3 165. 3
10 11 166. 1 167. 1 168. 1 169. 3 170. 4
= 0.58, = 0.57 171. 1 172. 4 173. 1 174. 1 175. 4
17 19
176. 2 177. 2 178. 2 179. 4 180. 1
10 181. 1 182. 1 183. 1 184. 1
Hence, the largest fraction is . 185. 4; Freeware (from “free” and “soft-
17
ware”)
117. 5; The series is ×1 + 1 , ×2 + 32, ×3 + 52,
2
186. 3
×4 + 72 ...
187. 1
118. 2; Let the number be x.
188. 3; A Megabyte (MB) is a unit of mea-
3x x surement used to quantify computer memory
Then, – = 10
7 4 and storage. One megabyte is the equivalent
of approximately one million bytes and one
12 x – 7 x thousand kilobytes (the exact conversion is
or, = 10 1 megabyte = 10,48,576 bytes and 1,024 ki-
28
or, 5x = 280 lobytes in binary measurement). Megabytes
 x = `56 are also used to describe the size of digital
files and images as well as capacity of digital
Clerk Mock Test 6
9. Four of the following five are divisible by a certain
Test-I: Reasoning Ability number and hence form a group. Which is the one
that does not belong to that group?
1. O, P, Q, R, S and T are sitting in a row facing north. T 1) 98 2) 85 3) 21 4) 49 5) 63
sits between S and Q. P is the neighbour of R and O. R 10. What will come next in the given series?
is the neighbour of Q. How many persons are there A1 B2 C3 D4 B2 C3 D4 A1 C3 D4 A1 ? ?
between T and P? 1) D4, A1 2) B2, CB 3) B2, D4
1) None 2) One 3) Two 4) C3, D4 5) None of these
4) Three 5) Can’t be determined Directions (Q. 11-15): In these questions, a
2. How many such B’s are there in the following relationship between different elements is shown in the
alphabetical series each of which is immediately statement(s). The statements are followed by two
followed by D but not immediately preceded by C? conclusions. Give answer

K
N D B D BA Z BAC D B D C B DAE D B D C B D 1) if only conclusion I is true.
1) One 2) Two 3) Three 2) if only conclusion II is true.
4) More than three 5) None of these 3) if either conclusion I or II is true.
3. How many pairs of letters are there in the word 4) if neither conclusion I nor II is true.
CAPITULATE each of which has as many letters 5) if both conclusions I and II are true.
between them in the word as in the English alphabetical 11. Statement: J K>P= R<N=S
series? Conclusions: I. S P II. J > R
1) None 2) One 3) Two 4) Three 5) None of these 12. Statement: Q < R < S T, U = R
4. In a certain code language NOVEMBER is written as Conclusions: I. T = U II. S U
ERNYEVNO. How is DECEMBER written in that code 13. Statement: L M N, J = I L
language? Conclusions: I. J > N II. N = J

KUNDAN
1) XVDEERNY 2) ERNYXVDE 3) EFDFNCED 14. Statement: A<C<E=F>H J
4) CEDEXVDE 5) None of these Conclusions: I. A < F II. J < E
5. Mr Kumar left his home for his office 15 minutes earlier 15. Statements: V = W > X = Y, T > Z = X
than his usual time. It takes 10 minutes to reach the Conclusions: I. V > Z II. T > W
office. He reached the office at 9 : 30 am. At what time Directions (Q. 16-20): Study the following information
does he usually leave home for the office? carefully and answer the given questions:
1) 9 : 35 am 2) 9 : 15 am 3) 9 : 30 am In a certain code language ‘milk is very tasty’ is written
4) Can’t be determined 5) None of these as ‘ta la ja sa’, ‘tea is black’ is written as ‘ha ja ka’ and
6. Rahul is twentyfirst from the left end in a row of 40 ‘sweet milk and tea’ is written as ‘ha pa sa ra’.
boys, and Prakash is twentyfirst from the right end in 16. What is the code for ‘milk’?
the same row. How many boys are there between them 1) ja 2) la 3) sa
in the row? 4) pa 5) None of these
1) One 2) Two 3) Three 17. ‘sweet black tea’ can be coded as
4) Data inadequate 5) None of these 1) ha ja sa 2) ka ha ra 3) pa ha ra
7. The mother of Aanchal’s brother is the only daughter 4) Can’t be determined 5) None of these
of Meena’s father. How is Aanchal related to Meena? 18. Which of the following is coded as ‘ta’?
1) Sister 2) Daughter 3) Mother 1) very 2) sweet
4) Granddaughter 5) None of these 3) Either tasty or very 4) black
8. A starts from his home and goes 2 km straight, then he 5) None of these
turns to his right and goes 1 km. Again, he turns to his 19. Which of the following is the code for ‘black’?
right and goes 2 km. Finally he turns to his left in 1) ra 2) pa 3) Either sa or ka
south direction and goes 1 km. The first 2 km travelled 4) ka 5) None of these
by A was towards which direction? 20. ‘ja’ is the code for
1) East 2) South 3) North 1) milk 2) and 3) is
4) Data inadequate 5) None of these 4) tasty 5) None of these
Directions (Q . 21-25): Study the following 30. For how many days does the youngest brother work
arrangement carefully and answer the given questions: in the office in a week?
K 2 8 % P# B 3 H $ G T I AYE £ A 4 9 L U @ 7 C N 1) One 2) Two 3) Three
21. If all the symbols are dropped from the above 4) Can’t be determined 5) None of these
arrangement, which of the following will be tenth from Directions (Q. 31-35): In each question below are
the right end? given two/three statements followed by two conclusions
1) A 2) Y 3) 9 4) G 5) None of these numbered I and II. You have to take the given statements
22. How many such consonants are there in the above to be true even if they seem to be at variance with
arrangement each of which is not immediately preceded commonly known facts. Read all the conclusions and then
by a symbol? decide which of the given conclusions logically follows
1) None 2) One 3) Two from the given statements, disregarding commonly
4) Three 5) More than three known facts. Give answer
23. How many such letters are there in the above 1) if only conclusion I follows.
arrangement each of which is immediately followed 2) if only conclusion II follows.
by a number? 3) if either conclusion I or II follows.
1) One 2) Two 3) Three 4) if neither conclusion I nor II follows.
4) None 5) None of these 5) if both conclusions I and II follow.
24. How many such symbols are there in the above 31. Statements: Some logics are customs.

K
arrangement each of which is not immediately preceded No custom is duty.
by a consonant but followed by a vowel? Conclusions: I. Some logics are not duty.
1) Two 2) Three 3) Four II. All logics being duty is a
4) None 5) None of these possibility.
25. If all the alphabets in the above arrangement are (32-33):
arranged in English alphabetical order within the series, Statements: All numbers are letters.
which of the following will be sixteenth from the left? Some letters are symbols.
1) N 2) A 3) L 4) K 5) 4 No symbol is a triangle.
Directions (Q. 26-30): Study the following information 32. Conclusions: I. Some letters are triangles.
carefully and answer the given questions. II. All symbols being numbers is a
There are three brothers X, Y and Z. All of them work possibility.

KUNDAN
in the same company. They work in the office at different 33. Conclusions: I. All symbols are numbers.
times on different days. X works in the office from 12 noon II. Some letters are not triangles.
to 4 pm on Tuesday, Thursday and Sunday. His younger 34. Statements: No page is a book.
brother Y works in the office on Monday, Thursday, Friday Some books are copies.
and Sunday from 10 am to 2 pm. The eldest brother works Conclusions: I. Some copies are not pages.
in the office from 9 am to 12 noon on Monday, Wednesday II. At least some copies are books.
and Thursday and between 2 pm and 4 pm on Friday, 35. Statements: All marbles are stones.
Saturday and Sunday. Some stones are gems.
26. Who among the following works for the maximum Conclusions: I. All gems being marbles is a
number of days in the office? possibility.
1) X 2) Y 3) Z II. All stones being marbles is a
4) Either Z or Y 5) None of these possibility.
27. On which of the following days do the youngest and Directions (Q. 36-40): Study the following information
the eldest brother work together in the office at the carefully and answer the given questions.
same time? Eight friends J, K, L, M, N, O, P and Q are sitting in a
1) Friday 2) Monday 3) Thursday circle facing the centre. K is sitting fourth to the left of J
4) Monday and Thursday 5) Friday and Saturday and is neighbour of P and O. O is sitting fourth to the right
28. Who among the following work(s) for the minimum of N and third to the left of L. M is the immediate neighbour
number of days in a week? of Q, but not of J.
1) X 2) Z 3) Y 36. Who among the following is opposite M?
4) Both X and Y 5) Can’t be determined 1) P 2) L 3) K 4) N 5) None of these
29. On which of the following days do all three brothers 37. What is the position of J with respect to Q?
work together in the office at the same time? 1) Third to the left 2) Second to the right
1) None 2) Monday 3) Thursday 3) Fourth to the left 4) Can’t be determined
4) Sunday 5) Can’t be determined 5) None of these
38. Who among the following is sitting second to the Dr Tomar.
right of K? In this technique, the bottom portion of the bottle is
1) N 2) L 3) M 4) O 5) None of these removed using a sharp knife and the bottle filled with water.
39. Which of the following pairs are immediate neighbours It is hung upside-down from a stake next to the sapling in
of P? the field. The plastic tube with the nozzle is made to touch
1) K, L 2) M, K 3) O, L 4) M, O 5) None of these the ground near the plant root.
40. Who among the following is sitting on the immediate Instead of allowing water to be irrigated through the
left of M? channels in the field or pouring it on the crop with a bucket,
1) J 2) O 3) Q water starts dripping on the soil surface making the root
4) Can’t be determined 5) None of these zone wet, thereby supplying moisture continuously to the
crop.
Test-II: English Language The plastic control knob in the middle of the tube can
be adjusted to control the drip. The farmer purchased about
Directions (Q. 41-50): Read the following passage 350 bottles from a waste paper mart and started using them
carefully and answer the questions given below it. Certain for his cultivation. His entire family used to help him in
words/phrases in the passage are printed in bold to help filling the bottles with water.
you locate them while answering some of the questions. “I have been able to get a net profit of `15,200 from
Irrigation poses the greatest challenge for a farmer less than a hectare till date by using this method for my

K
since water availability needs to be there almost throughout vegetables,” says the farmer.
the year. Monsoon failure means fall in yield and farmers The state agriculture department conferred an
are the direct sufferers when the rains are inadquate. achievement award on him for adopting this simple yet
“Especially in a country like India, the unseasonal and effective method to overcome water shortage.
unpredictable weather, especially when it turns into Many others in the surrounding areas have also started
droughts, forces agriculture scientists and farmers to think adopting this method for their crops.
of alternative simple and effective solutions to overcome According to Dr Tomar, this type of irrigation is quite
the crises,” says Dr IS Tomar, Programme Coordinator, popular in African countries like Kenya. Many American
Krishi Vigyan Kendra, Indian Council of Agriculture farmers are also using this system to grow their kitchen
Research (ICAR) near Rajgarh Naka, Jhabua. gardens.
One case study that has been quite popular is use of But in India it is not popular. The reason could be that

KUNDAN
discarded plastic saline bottles for irrigation by farmers in it is time-consuming and labour-intensive.
Jhabua district in Madhya Pradesh. Basically a tribal- Though today there are several government schemes
dominated dry region, agriculture is the main source of and subsidies available to instal a full-fledged drip irrigation
livelihood for the people there. Maize, blackgram, soyabean, system, the paperwork involved is quite laborious and
wheat, and cotton are normally grown. delays cultivation work.
To enhance income of the farmers of the area, an 41. Which of the following is the characteristic feature of
attempt was made to introduce improved vegetable Indian agriculture? Give your answer in the context of
cultivation under National Agricultural Innovation Sub the passage.
Project called Integrated Farming System for Sustainable 1) Agriculture is amongst the fastest-growing
Rural Livelihood in undulating and rainfed areas. industries in India.
Mr Ramash Bariya, a small farmer from the village, 2) India is amongst the top three global producers of
started growing vegetables under this project guidance in many crops, including wheat, rice, pulses and
a small area. He got a good profit initially and this vegetables.
encouraged him to start growing some gourd varieties like 3) Farmers in India largely depend on monsoon.
bitter and sponge gourds. He prepared a small nursery for 4) Scientific research and development has reduced
raising the seedlings but faced acute water shortage due farmers’ dependence on monsoon.
to delayed monsoon. 5) The average size of land holding is very small.
Worried, he discussed with NAIP project scientists, 42. Why do agriculture scientists in India carry out
who advised him to adopt an innovative irrigation scientific research?
technique using waste saline bottle. 1) To compete with agricultural productivity across
“We wanted to try out this method in the farmer’s field the world
since it is quite cheap and effective and farmers who cannot 2) To face unseasonal and unpredictable weather in
financially afford big drip systems can try this. But farmers India
should realise that it requires labour and patience,” says 3) To satisfy an urge to help farmers and encourage
them for more production number of that part is the answer. If there is ‘No error’,
4) Due to pressure created by the Central government the answer is 5). (Ignore errors of punctuation, if any.)
5) All the above 51. 1) Elections in India is similar to the apocryphal story
43. According to the given passage, which of the / 2) of the elephant whose reality / 3) is variously
following is true about Jhabua district in Madhya misinterpreted / 4) by blind men feeling different parts
Pradesh? of it. / 5) No error
1) It is a tribal-dominated region. 52. 1) If you are caught driving above the speed limit / 2)
2) Agriculture is the main source of livelihood of the you might end up spending six months / 3) in jail rather
people living over there. than get off / 4) by just paying a small fine. / 5) No
3) It is basically a humid region. error
4) Only 1) and 2) 53. 1) Known for her immaculate turnout in public, / 2)
5) All 1), 2) and 3) Thatcher’s appointments diary reveals, she had / 3)
44. Which of the following is not true about waste saline her hair style 120 times in 1984, / 4) an average of once
water irrigation technique? in three days. / 5) No error
1) It is an effective irrigation technique. 54. 1) Snow, high winds and a glacial chill / 2) hit north-
2) It is helpful for such farmers as are not financially eastern United States as a blast of / 3) brutal wintry
strong enough to afford big drip system. weather bore down / 4) on many states and major cities,
3) It requires a lot of labour and patience. snarling air traffic. / 5) No error

K
4) This technique is quite popular in some of the 55. 1) The successful launch of Mangalyaan is crucial for
African countries. India / 2) as it will be the first step / 3) towards building
5) None of these rockets / 4) that can carry heavier payloads. / 5) No
45. What, according to the given passage, is/are the main error
hindrance in the way of adopting waste saline water Directions (Q. 56-60): In the following questions, a
irrigation technique in India? sentence has been given with some of its parts in bold. To
1) It is a time-consuming and labour-intensive make the sentence grammatically correct, you have to
technique. replace the bold part with the correct alternative given
2) Waste saline water bottles are not readily available. below. If the sentence is correct as it is, give 5) as your
3) This technique is not effective in dry regions. answer (ie No correction required).
4) Waste saline water bottles are fragile and brittle. 56. The banks provide safe deposit lockers to the

KUNDAN
5) All the above customers which hire them on a lease basis.
Directions (Q. 46-48): Choose the word/group of 1) that hire them 2) who hire them 3) who hire it
words which is MOST SIMILAR in meaning to the word/ 4) which hire it 5) No correction required
group of words printed in bold as used in the passage. 57. In order to ensuring that banks are not used by the
46. Overcome money launderers, the RBI has issued KYC guidelines.
1) vanquish 2) exceed 3) solve 1) ensure 2) assure 3) insure
4) break 5) rebut 4) be ensured 5) No correction required
47. Undulating 58. Payments for purchases can be made by cash, cheques
1) uprooting 2) bouncing 3) twisted or cards.
4) apathetic 5) wavy 1) done by cash 2) made through cash
48. Discarded 3) made in cash 4) performed with cash
1) dismissed 2) costly 3) cheap 5) No correction required
4) empty 5) rejected 59. The vacancy was filled by Shushma who the chairman
Directions (Q. 49-50): Choose the word/group of said ought to be promoted.
words which is MOST OPPOSITE in meaning of the word/ 1) filled with Shushma whom
group of words printed in bold as used in the passage. 2) filled from Shushma whom
49. Sharp 3) filled up with Shushma who
1) dull 2) blunt 3) flat 4) filled by Shushma to whom
4) slow 5) fine 5) No correction required
50. Enhance 60. The year 2013 has seen women coming out of their
1) amplify 2) exalt 3) augment shell to protest against the injustice meted at them or
4) diminish 5) enlarge fellow women.
Directions (Q. 51-55): Read each sentence to find out 1) meted at her 2) meted by them
whether there is any grammatical or idiomatic error in it. 3) meted over them 4) meted out to them
The error, if any, will be in one part of the sentence. The
5) No correction required (A) This led to customer-oriented mergers and
Directions (Q. 61-65): In each question below is given acquisitions, corporate agencies, strategic
a sentence with four words printed in bold type. These are alliances, business partnerships etc.
numbered as 1), 2), 3) and 4). One of these four words (B) The firms are now attempting to cross-sell the
printed in bold may be either wrongly spelt or inappropriate products to fulfil the customer needs and create a
in the context of the sentence. Find out the word which is win-win situation.
wrongly spelt or inappropriate, if any. The number of that (C) Marketing has seen great changes in its journey
word is your answer. If all the words printed in bold are to today’s times.
correctly spelt and also appropriate in the context of the (D) Thus, in modern times the marketing concepts
sentence, mark 5), ie ‘All correct’, as your answer. have undergone great change and become more
61. In the 9th Ministerial Conference of World Trade complex.
Organization convened in Bali from 3-6 December 2013, (E) In modern marketing, the focus has shifted from
1) existing products to products the customer may
India took a principled position for ensuring that expect from the firm.
2) (F) It has also redefined the corporate goals, mission
livelihoods of subsistence farmers are safeguard and and vision and the total attitude of the firms.
3) 4) 66. Which of the following will be the FIRST sentence
the concern of food security of millions of poor across after rearrangement?

K
the developing world protected. All correct 1) A 2) B 3) C 4) D 5) E
5) 67. Which of the following will be the SECOND sentence
62. To understand the power of intellectual globalism in after rearrangement?
1) 2) 1) E 2) C 3) B 4) F 5) A
India, we have to pay little attention to a diagnosis of 68. Which of the following will be the THIRD sentence
3) 4) after rearrangement?
Tagore. All correct 1) B 2) E 3) D 4) A 5) C
5) 69. Which of the following will be the FOURTH sentence
63. The reason may be India’s remarkable heterogeneity, after rearrangement?
1) 1) C 2) F 3) A 4) E 5) B
or its persistent give-and-take across the borders – 70. Which of the following will be the LAST (SIXTH)

KUNDAN
2) sentence after rearrangement?
stretching over many millennia. All correct 1) F 2) B 3) E 4) C 5) D
3) 4) 5) Directions (Q. 71-80): In the following passage, some
64. Despite the enormity of her win – virtually destroying of the words have been left out, each of which is indicated
1) by a number. Find the suitable word from the options given
the Congress in Rajasthan by noching up 162 out of against each number and fill up the blanks with
2) appropriate words to make the paragraph meaningfully
199 seats for BJP – Raje declines to discuss why her complete.
vanquished rival and former chief minister Ashok Gehlot Rapidly growing cities in India have increased the
3) demand for natural resources and (71) public services,
fell from grace so spectacularly. All correct including water supply. But (72) existing infrastructure is
4) 5) outdated and in desperate need of repair, where, according
65. Petronet LNG limited has signed a long-term Time to the World Bank, about half the water supplied for
Charter Agreement with a consortium led by NYK of domestic consumption is lost (73) internal leakages or theft.
1) Other issues such as intermittent water supply (74)
Japan to hire a fourth LNG sheep which will haul the lack of water metering and, more importantly, loss of water
2) 3) (75) from unmetered supply, often (76) as free supply, are
fuel from Gorgon in Australia to Petronets terminal at increasing.
4) There is thus a growing need to (77) safe water on a
Kochi in Kerala for a period of 19 years. All correct continuous basis for all as a human right. (78) over 90 per
5) cent of urban households in India have (79) to water supply
Directions (Q. 66-70): Rearrange the following six services, this access is often not equal, not (80) and not
sentences (A), (B), (C), (D), (E) and (F) in the proper safe.
sequence to form a meaningful paragraph and then answer 71. 1) primary 2) basic 3) useful
the questions given below.
4) natural 5) fundamental 2 5
72. 1) again 2) when 3) non- 94. of of 6916 = ? × 5
7 13
4) an 5) the 1) 146 2) 148 3) 152 4) 154 5) 156
73. 1) after 2) through 3) in
4) with 5) by 95. 40% of 175 = ? + 36
74. 1) without 2) against 3) and 1) 1936 2) 1764 3) 1444 4) 1296 5) 1156
4) with 5) for 96. 3 3
2197 6859 ?
75. 1) arising 2) coming 3) generating
4) causing 5) created 1) 247 2) 234 3) 221 4) 208 5) None of these
76. 1) taken 2) declared 3) given 97. 38.44 4225 = ?
4) known 5) camouflaged
1) 465 2) 372 3) 403 4) 434 5) 527
77. 1) channelise 2) avail 3) bring
4) provide 5) manage 3
98. 13 5780 – 13406 ?
78. 1) But 2) Why 3) Whilst 5
4) When 5) Then 1) 64304 2) 65202 3) 66408 4) 67312 5) 68404
79. 1) knowledge 2) problem 3) access 99. 703 ÷ 1.9 =?% of 2960
4) reach 5) excess 1) 10 2) 12.5 3) 15 4) 17.5 5) 20
80. 1) sustainable 2) confirmed 3) supported

K
4) assisted 5) granted 2 1 5
100. of of of 12096 = ?% of 1600
7 8 9
Test-III: Quantitative Aptitude 1) 10 2) 12.5 3) 15 4) 17.5 5) 20
101. What is the difference between a discount of 39% on
Directions (Q. 81-100): What should come in place `15400 and two successive discounts of 24% and 15%
of question mark (?) in the following questions? on the same amount?
81. 73340 ÷ 1632 + 163 = ? 1) `542.8 2) `548.6 3) `552.2
1) 202 2) 204 3) 206 4) 208 5) 210 4) `554.4 5) None of these
82. 78432 – 49007 + 31628 – 12545 = ? 102. What value should come in place of question mark (?)
1) 48124 2) 48508 3) 47318 4) 47906 5) 45448 in the following number series?
52, 58, 83, 145, 268, 482, ?

KUNDAN
83. ? + 1365 = 78% of 1850
1) 804 2) 812 3) 817 4) 823 5) 827
1) 6084 2) 5776 3) 5476 4) 5184 5) 4624 103. A shopkeeper marks his items 48% more than the real
84. 0.045 ÷ 1.2 × 84 = ? price and allows 25% discount to customers. What is
1) 2.4 2) 3.0 3) 3.15 4) 3.45 5) 3.75 his profit percentage?
85. 943.28 – 193.16 + 216.08 – 84.75 = ? 1) 11% 2) 15% 3) 17% 4) 21% 5) 23%
1) 878.15 2) 881.45 3) 882.25 4) 883.75 5) 884.05 104. What sum of money will yield `309.6 as interest at the
86. 1596 × 16 ÷ 38 = ? rate of 8% pa in 6 years?
1) 576 2) 608 3) 672 4) 704 5) 736 1) `625 2) `635 3) `645
87. 7.56 ÷ 0.28 = ?% of 135 4) `655 5) `675
1) 16 2) 20 3) 24 4) 30 5) 36 105. If (58)2 is added to the square of a number, the answer
88. 46% of 765 + 34% of 345 = ? thus obtained is 12200. What is the number?
1) 443.2 2) 454.6 3) 461.8 4) 469.2 5) 472.5 1) 92 2) 94 3) 96 4) 98 5) 102
89. 65% of 1350 + 72% of ? = 1417.5 106. What value should come in place of question mark (?)
1) 540 2) 650 3) 750 4) 850 5) 775 in the given question?
90. 35% of 1300 – ? = 45% of 980
72 ?
1) 14 2) 196 3) 169 4) 144 5) 296
? 162
91. 83752 + 31705 – ? = 49015
1) 66442 2) 64462 3) 66244 4) 66224 5) 64642 1) 102 2) 104 3) 106 4) 108 5) 112

92. 43 + ? + 86 = 158 3
107. The breadth of a rectangular park is of its length. If
7
1) 441 2) 529 3) 625 4) 729 5) 841
93. 75 × 4.8 ÷ 3.2 = ? the perimeter of the park is 140 metres, what is its
1) 110.25 2) 112.5 3) 114.75 4) 116.25 5) 117.5 area?
1) 861 sq m 2) 1127 sq m 3) 1029 sq m
4) 1081 sq m 5) 987 sq m
108. A person bought 15 kg of rice at the rate of `34 per kg
and 30 kg of rice at the rate of `27 per kg. He mixed the
two types of rice and sold the mixture at the rate of `32 Test-IV: General Awareness
per kg. What is his gain per cent? (Answer in
(with Special Reference to Banking Industry)
approximate value)
1) 7% 2) 9% 3) 11% 4) 13% 5) 15% 121. India is considering buying as well as co-producing
109. In an examination, a candidate must get 65% marks to 15 ShinMaywa US-2 amphibious patrol aircraft from
pass. If a candidate scores 303 marks and is declared which of the following countries?
failed by 35 marks, then what is the maximum marks? 1) US 2) Russia 3) Israel
1) 450 2) 480 3) 520 4) 550 5) 580 4) Japan 5) Sweden
110. A dealer bought an item at 25% discount on its original 122. The world’s largest 4,000-MW ultra mega solar power
price and sold it at 20% increase on the original price. project will be set up in which of the following states?
What is his percentage profit? Six public sector undertakings under three central
1) 45% 2) 50% 3) 60% ministries have signed MoU for it recently.
4) 75% 5) None of these 1) Madhya Pradesh 2) Rajasthan
111. The difference between the present age of A and of B 3) Uttarakhand 4) Himachal Pradesh
is 32 years. If 6 years ago their ages were in the ratio of 5) None of these
3 : 1, then what is the present age of B? 123. Which of the following state govts has launched a
1) 22 years 2) 24 years 3) 28 years new pension scheme dubbed as ‘Samajwadi Pension

K
4) 30 years 5) 32 years Yojna’, which will cover about 40 lakh poorest of the
112. A certain number of men can do a piece of work in 40 poor in the state? Each beneficiary under the scheme
days. If there had been 8 men more it could have been will receive `500 per month.
finished in 5 days less. Find the number of men. 1) Uttar Pradesh 2) Bihar 3) Haryana
1) 48 men 2) 56 men 3) 60 men 4) 64 men 5) 72 men 4) Punjab 5) Tamil Nadu
113. The compound interest on a certain sum at the rate of 124. Which of the following terms is not related to banking/
5% pa for 2 years is `287. What is the sum? finance?
1) `2100 2) `2400 3) `2800 1) Debenture 2) Black box 3) Zero coupons
4) `3000 5) `3200 4) Face value 5) Actuaries
114. A tank is normally filled in 9 hours, but takes 3 hours 125. The RBI increased the MSF rate to 8 per cent in its
longer to fill it because of a leak. If the tank is full, then

KUNDAN
recent quarterly policy review. What does the letter
how long will the leak take to empty the tank? ‘S’ denote in the term MSF?
1) 15 hours 2) 21 hours 3) 27 hours 1) Standing 2) Statutory 3) Subsidy
4) 36 hours 5) None of these 4) Subscription 5) Standard
115. What is the average of the following set of scores? 126. Name the author of the book titled Vanity Bagh. He
347, 481, 516, 669, 723, 814, 923 won The Hindu Prize for best fiction of 2013 for this
1) 579 2) 597 3) 613 4) 639 5) 647 novel.
116. In how many different ways can the letters of the word 1) Amish Tripathi 2) Anees Salim 3) Supriya Dravid
POLICE be arranged? 4) Fahad Samar 5) Talmiz Ahmad
1) 6 2) 36 3) 720 4) 120 5) None of these 127. The govt notified which of the following communities
117. When 32 is added to 64% of a number, the result is as the national minority recently?
25% of 576. What is the number? 1) Jain 2) Buddhist 3) Parsi
1) 175 2) 225 3) 250 4) 275 5) None of these 4) Wahabi 5) None of these
118. The cost of a cricket bat after 16% discount is `1134. 128. Who among the following won the women’s singles
What is the cost of the bat before discount? title of the Australia Open tennis tournament recently?
1) `1250 2) `1300 3) `1350 4) `1400 5) `1450 1) Li Na 2) Serena Williams
119. An amount was distributed among A, B and C in the 3) Ana Ivanovic 4) Dominika Cibulkova
ratio of 7 : 11 : 16. If A received `4986 less than C, then 5) None of these
what was the share of B? 129. The permanent account number (PAN), which is an
1) `3878 2) `6094 3) `7212 4) `8416 5) `8864 alphanumeric number allotted by the income tax
120. A person spent 40% of his monthly salary on house department to taxpayers, consists of how many digits?
rent and 30% of the remaining amount on food. If the 1) Seven digits 2) Eight digits 3) Nine digits
person is left with `3591 then what is his monthly 4) Ten digits 5) Eleven digits
salary? 130. The service tax exemption has been extended to which
1) `8250 2) `8350 3) `8450 4) `8550 5) `8650 of the following services in the interim budget for 2014-
15 presented in the Lok Sabha recently? 4) 74 per cent 5) None of these
1) Storage and warehousing of rice 140. What amount has been allocated by the govt in the
2) Blood banks interim budget for 2014-15 for interest subsidy for all
3) Insurance broking agencies education loans availed up to 31-03-2009?
4) All the above 1) `2,000 cr 2) `2,200 cr 3) `2,600 cr
5) Only 1) and 2) 4) `2,900 cr 5) `3,200 cr
131. The Govt has appointed who among the following as 141. India is ranked at which of the following places out of
the new chief of the Hyderabad-based National Police 178 countries in the 2014 Environmental Performance
Academy? It is the alma mater for IPS officers in the Index (EPI) issued by Yale University recently?
country. Switzerland tops the 2014 EPI.
1) Letika Saran 2) K Vijay Kumar 1) 72nd 2) 73rd 3) 77th 4) 118th 5) 155th
3) Subhas Goswami 4) Aruna Bahuguna 142. Who among the following has become the first woman
5) None of these to be inducted in the International Cricket Council’s
132. Laimdota Straujuma has become the first woman Prime (ICC’s) umpires’ panel recently?
Minister of which of the following countries recently? 1) Katrina Keenan 2) Haidee Tiffen
1) Latvia 2) Eretria 3) Slovakia 3) Kathy Cross 4) Emily Drum
4) Slovenia 5) Serbia 5) None of these
133. Who among the following has taken over as the new 143. The country’s first monorail service commenced its

K
Chairman of the National Dairy Development Board operations in which of the following cities recently?
(NDDB) recently? 1) Jaipur 2) Bangalore 3) Bhopal
1) Amrita Patel 2) T Nanda Kumar 4) Mumbai 5) None of these
3) Vipul Chaudhary 4) MGVK Bhanu 144. Which of the following regulators regulates credit
5) None of these rating agencies in India?
134. As announced by the finance minister in the interim 1) Reserve Bank of India (RBI)
budget for 2014-15, the Govt will provide what amount 2) Securities and Exchange Board of India (SEBI)
for capital infusion in state run banks in 2014-15? 3) Insurance Regulatory and Development Authority
1) `80 bn 2) `92 bn 3) `108 bn (IRDA)
4) `112 bn 5) `125 bn 4) Forward Markets Commission (FMC)
135. In the interim budget for 2014-15, presented in the 5) None of these

KUNDAN
Parliament recently, the fiscal deficit has been pegged 145. Finance minister unveiled a 10-point agenda that
at what per cent of the GDP for the financial year 2013- would help make India the third largest economy by
14? 2043. Currently, the size of Indian economy is the
1) 4.6 per cent 2) 4.9 per cent 3) 5.6 per cent ______ largest in the world in nominal GDP terms.
4) 5.9 per cent 5) 6.6 per cent 1) 5th 2) 7th 3) 9th 4) 11th 5) 12th
136. Which of the following cities hosted the annual two- 146. Which of the following social networking sites
day conference of central and state statistical completed 10 years in operation on 4 Feb 2014?
organisations recently? 1) Twitter 2) Myspace 3) Facebook
1) Panjim 2) Kolkata 3) Chennai 4) Orkut 5) None of these
4) Hyderabad 5) New Delhi 147. The Union govt launched ‘Meghraj’ project recently.
137. The National Voters’ Day is celebrated every year on This project is related to which of the following?
which of the following dates in India? 1) Cloud computing
1) 21 Jan 2) 22 Jan 3) 23 Jan 4) 24 Jan 5) 25 Jan 2) Mother and child tracking
138. Akhilesh Das Gupta was unanimously re-elected as 3) Weather forecasting
President of which of the following sports bodies in 4) Artificial rain
the country recently? 5) None of these
1) Badminton Association of India (BAI) 148. The govt has approved increase in the fair and
2) Indian Olympic Association (IOA) remunerative price (FRP) for sugarcane by 10 rupees
3) All India Football Federation (AIFF) per quintal for 2014-15. What is the new FRP per quintal
4) Archery Association of India (AAI) for sugarcane for the current sugar year?
5) None of these 1) `220 2) `225 3) `210 4) `270 5) `282
139. Currently, the aggregate foreign investment (FDI, FII 149. Star series banknotes issued by the RBI refer to which
and NRI) limit for public sector banks (PSBs) is of the following?
1) 20 per cent 2) 24 per cent 3) 49 per cent 1) Banknotes issued to track black money
2) Banknotes used to replace those with defective
printing the country to achieve 100 per cent sanitation in rural
3) Banknotes issued for a short period and urban households, schools, sanitary complexes
4) Plastic notes and Aanganwadi centres?
5) None of these 1) Uttarakhand 2) Manipur 3) Kerala
150. Which of the following instruments is NOT a 4) Goa 5) Sikkim
negotiable instrument? 159. 2014 Winter Olympics were held in Sochi, Russia
1) Banknote 2) Promissory note recently. 2018 Winter Olympics are scheduled to take
3) Bill of exchange 4) Cheque place in which of the following countries?
5) Demand draft 1) Pyeongchang, South Korea
151. The govt has announced to extend the visa-on-arrival 2) Turin, Italy
facility to all countries except eight with effect from 3) Salt Lake City, United States
Sep 2014. Which of the following countries has/have 4) Chamonix, France
not been covered under this facility? 5) None of these
1) Iraq 2) Sri Lanka 3) Afghanistan 160. India’s indigenous aircraft Sitara, whose development
4) Pakistan 5) All the above is in advanced stages of certification, is a/an
152. Which of the following automobile manufacturers 1) Intermediate Jet Trainer
launched a new car Celerio, a hatchback with 2) Light Combat Aircraft
revolutionary ‘automatic manual transmission’ facility 3) Transport Aircraft

K
at the 12th India Auto Expo recently? 4) Advanced Jet Trainer
1) Tata Motors 2) Maruti Suzuki 3) Honda 5) None of these
4) Renault 5) None of these
153. As per the recent estimates of the Planning Test-V: Computer Knowledge
Commission, which of the following states had the
highest number of poor people (5.98 crore) living below 161. Microsoft Office is
poverty line in 2011-12? 1) Shareware
1) West Bengal 2) Maharashtra 2) Public-domain software
3) Madhya Pradesh 4) Bihar 3) Open-source software
5) Uttar Pradesh 4) A vertical market application
154. The Reserve Bank of India has removed the interest 5) An application suite

KUNDAN
rate cap on loans given by microfinance institutions 162. How many options does a binary choice offer?
(MFIs) effective from 1 Apr and such lenders will now 1) None
be able to charge interest rate based on their borrowing 2) One
cost. What was the earlier cap for interest rate? 3) Two
1) 18 per cent 2) 22 per cent 3) 24 per cent 4) It depends on the amount of memory on the
4) 26 per cent 5) 28 per cent computer.
155. Through which of the following do banks ensure that 5) It depends on the speed of the computer ’s
money from illegal sources do not come to them? processor.
1) CIBIL record 2) Know Your Customer (KYC) 163. Which of the following is used for broadband
3) Core banking 4) NEFT communications?
5) None of these 1) Cable modem 2) DSL modem
156. The Reserve Bank of India (RBI) has permitted some 3) Satellite modem 4) All the above
non-bank entities to set up white-label ATMs in India. 5) None of these
Which of the following is/are among them? 164. ________ a document means the file is transferred
1) Tata Communications Payment Solutions from your computer to another computer.
2) Vakrangee 1) Downloading
3) Muthoot Finance 2) Really simple syndication
4) Prizm Payment Services 3) Accessing
5) All the above 4) Upgrading
157. Who among the following has been elected as the 5) Uploading
new Prime Minister of Nepal recently? 165. The desktop contains small graphics called
1) KP Sharma Oli 2) Ram Chandra Poudel 1) windows 2) logos 3) icons
3) Jhala Nath Khanal 4) Sushil Koirala 4) pictures 5) None of these
5) None of these 166. A specialised network device that determines the next
158. Which of the following states has become the first in network point to which it can forward a data packet
towards the destination of the packet is commonly 5) storage units
known as 178. The process of a computer transferring information
1) bridge 2) router 3) switch into the server on the Internet is called
4) hub 5) repeater 1) inputting 2) outputting 3) downloading
167. Personal logs or journal entries posted on the web are 4) uploading 5) storing
known as 179. Communication between a computer and a CPU uses
1) web casts 2) blogs ________ transmission mode.
3) list serv 4) subject directories 1) automatic 2) simplex 3) half-duplex
5) registry 4) full-duplex 5) None of these
168. In an OS ________ feature allows a single computer 180. ISDN is an acronym for
to process the task of several users at different stations 1) Integrated Standard Digital Network
in round-robin fashion. 2) Intelligent Services Digital Network
1) multiprocessing 2) multitasking 3) Integrated Services Digital Network
3) time-sharing 4) multiprogramming 4) Integrated Services Data Network
5) time processing 5) None of these
169. When computer is booted ________ are loaded into 181. To modify the structure of a created table, which SQL
main memory. command is used?
1) utility programs 1) change 2) alter 3) update

K
2) word processing instructions 4) modify 5) None of these
3) software instructions 182. ________ can be used to send an e-mail to a large
4) internal command instructions group at one time.
5) external command instructions 1) Group 2) Alias 3) Mail server
170. Which key is used in combination with another key to 4) Listserv 5) Remote server
perform a specific task? 183. UTP and STP are the types of which cable?
1) Function 2) Control 3) Arrow 1) Fibre-optic cable 2) Co-axial cable
4) Space bar 5) None of these 3) Twisted pair cable 4) Wired medium
171. ________ provides platform for other softwares to 5) None of these
run on. 184. What is Outlook Express?
1) System software 2) Operating system 1) Scheduler 2) E-mail Client 3) Address Book

KUNDAN
3) Application software 4) Software package 4) All the above 5) None of these
5) None of these 185. Off-line operation is the operation of devices without
172. ________ layer exists between session layer and the control of
network layer in OSI model. 1) ALU 2) control unit 3) processor
1) Presentation 2) Data link 3) Transport 4) memory 5) CPU
4) Application 5) Physical 186. A device that prints one line at a time is known as
173. Two or more computers connected to each other for 1) line printer 2) chain printer
sharing information form a 3) drum printer 4) Any of the above
1) network 2) router 3) server 5) None of these
4) tunnel 5) pipeline 187. The basic goal of a computer process is to convert
174. Errors in a computer program are referred to as data into
1) bugs 2) mistakes 1) information 2) tables 3) files
3) item overlooked 4) blunders 4) graphs 5) None of these
5) None of these 188. A disk on which you store information is
175. Extension of a Word file in MS-Office 2007 is 1) plate 2) data disk 3) paper disk
1) .doc 2) .docx 3) .docxs 4) TV disk 5) None of these
4) All the above 5) None of these 189. Internal memory in a CPU is nothing but
176. The most common input devices include 1) a set of registers 2) a set of ALU
1) monitor and keyboard 2) monitor and mouse 3) a microprocessor 4) a BUS
3) mouse and keyboard 4) printer and mouse 5) None of these
5) None of these 190. What type of port sends data one bit at a time?
177. Plotter, Speakers and Monitor are examples of 1) Serial 2) Parallel 3) USB
1) input devices 2) processing units 4) FireWire 5) None of these
3) output devices 4) utilities 191. The connection between a multimedia presentation
and a file containing a song to be played is called a 196. ________ is a method of storing data on two or more
1) link 2) chain 3) pointer hard drives that work in combination to do the job of
4) tie 5) None of these a larger drive.
192. A ________ is a design tool that shows the overall 1) DAID 2) RAID 3) Shadowing
flow of a multimedia presentation. 4) Casting 5) MiniSD
1) link 2) graphical map 3) storyboard 197. Algorithm and Flow Chart help us
4) Gantt chart 5) None of these 1) know the memory capacity.
193. When we purchase a product on a mobile phone, the 2) identify the base of a number system.
transaction is called 3) direct the output to a printer.
1) Web Commerce 2) E-Commerce 4) specify the problem completely and clearly.
3) M-Commerce 4) Mobile Purchases 5) None of these
5) None of these 198. Which of the following shortcut keys is used to check
194. What is the name given to the standard set of rules spelling ?
that two computers use to communicate with each 1) F1 2) F2 3) F7
other? 4) F9 5) None of these
1) Access Control 2) Domain 199. When did arch rivals IBM and Apple Computers Inc
3) Link 4) Protocol decide to join hands?
5) TCP/IP 1) 1978 2) 1991 3) 1984

K
195. Computer-aided design (CAD) software can do all of 4) 1985 5) None of these
the following except 200. Word processing, spreadsheet and photo-editing are
1) designing and creating a Web page. examples of
2) allowing engineers to build virtual models. 1) application software
3) allowing engineers to create automated designs and 2) system software
model visualisations. 3) operating system software
4) work in conjunction with global positioning 4) platform software
systems (GPS). 5) None of these
5) None of these

KUNDAN
K
KUNDAN
Answers
1. 3; There are two possibilities:
8. 1;

5. 1; Mr Kumar left his home at 9 : 30 9. 2; Except 85 all are divisible by 7.


2. 3; DBD, DBD, DBD – 0 : 10 = 9 : 20 am. Hence, 9 : 20 am is 15 10. 3; A1 B2 C3 D4 / B2 C3 D4 A1 / C3 D4
3. 4; minutes earlier than his usual time. So, 9 : 20 A1 B2 / D4
+ 15 = 9 : 35 am. Hence, Mr Kumar leaves 11. 2;
his home daily at 9 : 35 am.
6. 5; Rahul is 21st from left end, hence from
4. 2; right end he is (40 – 21 + 1) = 20th
Prakash is 21st from right. Hence there is
no boy between them in the row. Thus, P < S is true. Hence, conclusion (S P)
7. 2; does not follow.
Again, J > R is true. Hence, conclusion II is
true.
12. 4;
Given statements: Q < R < S T ... (i)
U=R ... (ii)
Hence, Aanchal is daughter of Meena.
Combining both statements, we get 32. 2; Some letters are symbols (I) + No 27 100
Q<U=R<S T symbol is a triangle (E) = I + E = O = Some ?= = 20
135
Again, U < S or S > U is true. But conclusion letters are not triangles. Hence, conclusion I
46 765 34 345
(S U) does not follow. does not follow. First and second statements 88. 4; ? =
Thus, we can’t compare U and T. Hence, are positive statements. Hence, the 100 100
neither conclusion I nor II is true. possibility in II exists. Thus conclusion II = 351.9 + 117.3 = 469.2
13. 4; Statements: L M N ... (i) follows. 89. 3; 65% of 1350 + 72% of ? = 1417.5
J=I L ... (ii) 33. 2; All numbers are letters (A) + Some 65 1350 72 ?
or, 1417.5
Combining both statements, we get letters are symbols (I) = A + I = No 100 100
J=I L M N conclusion. Hence, conclusion I does not 72 ?
or, 1417.5 – 877.5 540
Thus, we can’t compare J and N. Hence, follow. 100
conclusion I and II are not true. Again, Some letters are symbols (I) + No 54 100
14. 5; Statement: symbol is a triangle (E) = I + E = O = Some or, ? = = 750
72
letters are not triangles. Hence, conclusion
90. 2; 35% of 1300 – ? = 45% of 980
II follows.
34. 5; No page is a book (E) + Some books 35 1300 45 980
or, – = ?
are copies (I) = E + I = O* = Some copies are 100 100
not pages. Hence, conclusion I follows. or, 455 – 441 = ?
Thus, A < F is true. Again, E > J or J < E is
Again, Some books are copies conversion
true. or, ? = 14
Some copies are books. Hence, conclusion
Hence, both conclusions I and II are true.
II follows. ? = 14 × 14 = 196
15. 1; Statements: V = W > X = Y ... (i)
35. 5; There are no negative statements. 91. 1; ? = 83752 + 31705 – 49015 = 66442
T>Z=X ... (ii)
Hence, both conclusions I and II follow.

K
Combining both statements, we get 92. 5; ? = 158 – 86 – 43 = 29
(36-40): or, ? = (29)2 = 841
Now, we can’t compare W and T. But, V > Z 75 4.8
is true. Hence, conclusion I is true. 93. 2; ? = = 112.5
3.2
(16-20): 2 5 6916
milk is very tasty ta la ja sa ... (i) 94. 3; ? × 5 = = 760
7 13
tea is black ha ja ka ... (ii)
sweet milk and tea ha pa sa ra ... (iii) 760
36. 2 37. 5; Immediate left or, ? = = 152
From (i) and (ii) is ja ... (iv) 5
38. 2 39. 1 40. 3 40 175
From (i) and (iii) milk sa ... (v)
41. 3 42. 2 43. 4 44. 5 45. 1 95. 5; ? = – 36 = 70 – 36 = 34
From (i), (iv) and (v) very/tasty 100
46. 3 47. 5 48. 5 49. 2 50. 4
ta/la ... (vi) 2
or, ? = (34) = 1156
51. 1; Replace ‘is’ with ‘are’
From (ii) and (iii), 96. 1; ? = 3 2197 3 6859 = 13 × 19 = 247
52. 3; Replace ‘get’ with ‘getting’

KUNDAN
tea ha ... (vii)
53. 3; Replace ‘style’ with ‘styled’ 97. 3; ? = 38.44 4425 = 6.2 × 65 = 403
From (ii), (iv) and (vii)
54. 2; Insert ‘the’ after ‘hit’
black ka ... (viii) 68
55. 5 56. 2 57. 1 58. 3 59. 5 98. 2; ? = 5780 – 13406
From (iii), (v) and (vii) 5
60. 4
sweet/and pa/ra ... (ix) = 78608 – 13406 = 65202
61. 4; safeguarded
16. 3 17. 2 18. 3 19. 4 20. 3
62. 3; a little ? 2960
21. 1; New arrangement 99. 2; = 703 ÷ 1.9 = 370
63. 5 100
64. 2; notching 370 100
65. 2; ship or, ? = = 12.5
2960
22. 5; 3H, GT, AY, 9L, 7C, and CN (66-70): CEAFBD
? 1600 2 5 12096
23. 2; K2 and B3 66. 3 67. 1 68. 4 69. 2 70. 5 100. 3; = 240
71. 2 72. 5 73. 2 74. 4 75. 1 100 7 8 9
24. 4
76. 5 77. 4 78. 3 79. 3 80. 1 240
25. 1; New arrangement: or, ? = = 15
A28%B#C3E$GHIKLN£A 49P 73440 16
81. 4; ? = 163 = 45 + 163 = 208 101. 4; 39% of 15400 = `6006
T@7UY 1632
Hence, sixteenth from the left is N. 82. 2; ? = 78432 – 49007 + 31628 – 12545 Successive discounts are 24% and 15%.
(26-30): = 110060 – 61552 = 48508 24
24% of 14500 = 15400 × = `3696
Day 78 1850 100
83. 1; ? = – 1365 Remaining amount = 15400 – 3696
Time Mon Tue Wed Thu Fri Sat Sun 100
= 1443 – 1365 = 78 = 11704
9 to 10 Z Z Z
? = (78)2 = 6084 15
10 to 12 YZ Z YZ Y Y Again, 15% of 11704 = 11704 ×
100
12 to 2 Y X XY Y XY 0.045
84. 3; ? = 84 = 0.0375 × 84 = 3.15 = `1755.6
2 to 4 X X Z Z XZ 1. 2 Total discount = 3696 + 1755.6 = 5451.6
26. 3 27. 4 28. 1 29. 1 85. 2; ? = 943.28 – 193.16 + 216.08 – 84.75 Difference = 6006 – 5451.6 = `554.4
30. 5; Four days = 1159.36 – 277.91 = 881.45 102. 4; The series is +23 – 2, +33 – 2, +43 – 2,
31. 1; Some logics are customs (I) + No 1596 16 +53 – 2, ...
custom is duty (E) = I + E = O = Some logics 86. 3; ? = = 672 103. 1; Let the cost price be `x
38
are not duty. Hence conclusion I follows, but ? 135 148x
II does not follow. 87. 2; = 7.56 ÷ 0.28 = 27 Marked price = `
100 100
148x 75 111 Now, 35(x + 8) = 40x 30% of the remaining salary is spent on food,
And SP = =` x or, 35x + 280 = 40x
100 100 100 3x 30 9x
280 Then, = `
5 100 50
111 x= = 56
x–x 5 3x 9 x 21x
100 100 113. 3; Let the sum be `x. Remaining amount = – = `
Profit % = 5 50 50
x 2
105 21x 3591 50
11x 1 Then, 287 = x × – x Now, 3591 x= = `8550
= 100 11% 100 50 21
100 x
104. 3; According to the question, r 121. 4 122. 2 123. 1 124. 2 125. 1
 CI P 1 –P 126. 2 127. 1 128. 1 129. 4 130. 5
P 8 6 100
309.6 = 131. 4 132. 1 133. 2 134. 4 135. 1
100 441x 41x
or, 287 = –x 136. 1 137. 5 138. 1 139. 1 140. 3
30960 400 400
P= = `645 141. 5 142. 3 143. 4 144. 2 145. 4
48 287 146. 3 147. 1 148. 1 149. 2 150. 1
105. 2; Let the number be x. x= 400 = `2800
41 151. 5 152. 2 153. 5 154. 4 155. 2
Then, (58) 2 + (x) 2 = 12200 114. 4; Let the leak empty the tank in x 156. 5 157. 4 158. 5 159. 1 160. 1
or, x2 = 12200 – 3364 = 8836 hours. 161. 5 162. 3 163. 4 164. 5 165. 3
x = 8836 = 94 So, in one hour the tank will fill 166. 2 167. 2 168. 3 169. 4 170. 2
106. 4; (?)2 = 72 × 162 = 9 × 8 × 81 × 2 1 1 x –9 171. 2 172. 3 173. 1 174. 1 175. 2
= 81 × 16 × 9 – = part. 176. 3 177. 3 178. 4 179. 2 180. 3
9 x 9x
or, ? = 9 × 4 × 3 = 108 9x 181. 2 182. 4 183. 3 184. 4 185. 5
107. 3; Let its length be x m. So, the tank will be filled in hours. 186. 4 187. 1 188. 2 189. 1 190. 1
x –9

K
3x 9x 191. 1 192. 3 193. 3 194. 4 195. 1
Then, breadth = m Again, = 9 + 3 = 12 196. 2; RAID (Redundant Arrays of Inde-
7 x –9 pendent Disks) is a method of storing data
Perimeter of the rectangular park = 2(l + b) or, 12x – 108 = 9x or, 3x = 108
on two or more hard drives that work in
3x x = 36 hours
10 x combination to do the job of a larger drive.
= 2 x 140 or, = 70 115. 4;
Although RAID can be used to increase per-
7 7 Average
formance, it is most often used to protect
3x 347 481 516 669 723 814 923 critical data on a storage server.
x = 49m and breadth = = 21m =
7 7 197. 4 198. 3 199. 2 200. 1
Area = 49 × 21 = 1029 sq m 4473
108. 2; Cost price = 15 × 34 + 30 × 27 = = 639
7
= 510 + 810 = `1320
116. 3; Number of ways = 6! = 720
SP = 45 × 32 = `1440

KUNDAN
117. 1; Let the number be x.
(1440 – 1320) 1200
Profit % = 100 = 64 25
1320 132 Then, x × + 32 = 576 = 144
= 9.090 9% 100 100
109. 3; Let the maximum marks be x. 64 x
65 or, = 144 – 32 = 112
100
Then, x × = 303 + 35 = 338
100 112 100
or, x = = 175
338 100 64
or, x = = 520 118. 3; Let the cost of the bat be `x.
65
110. 3; Let the original price be `100. x 16
So the cost price = `75 and the selling price Then, x – = 1134
100
= `120. 84x
or, 1134
(120 – 75) 4500 100
Profit % = 100 = = 60
75 75 1134 100
111. 1; Let the present age of A and B be x x= = `1350
84
years and y years respectively. 119. 2; Let the share of A, B and C be 7x,
Then, x – y = 32 ... (i) 11x and 16x respectively.
Now, 6 years ago, Then, 16x – 7x = 4986
x –6 3 or, 9x = 4986 or, x = 554
y –6 1 So, the share of B = 11x
or, x – 6 = 3y – 18 = 11 × 554 = `6094
or, x – 3y = –12 ... (ii) 120. 4; Let his monthly salary be `x.
Solving eqn (i) and (ii), we get He spent 40% on rent.
x = 54 years, y = 22 years 40 4x
112. 2; Let the total number of men be x. So x × =`
100 10
1 4x 6x
Then one man’s 1 day’s work = So the remaining amount = x – =
40x 10 10
If there were 8 more men 3x
1 = `
5
then, one man’s one day’s work = 35( x 8)
Clerk Mock Test 7
7. Who among the following are young, graduate and
Test-I: Reasoning Ability MBBS?
1) D and A 2) A and B 3) A and E
Directions (Q. 1-6): Study the following information 4) I and B 5) None of these
carefully and answer the given questions: 8. Which of the following groups represents graduates?
A, B, C, D, E, F, G and H are eight persons sitting in a 1) FEALI 2) LGBIE 3) AIEBF
circle facing the centre but not necessarily in the same 4) EIGHB 5) None of these
order. 9. Which of the following groups represents only MBBS
B is second to the right of D. G is third to the right of and only clerks respectively?
E. C is second to the left of A, who is second to the left of 1) DK, HJ 2) GHJ, LKD 3) CL, HJ
F, who is not an immediate neighbour of B and D. 4) FC, DK 5) None of these
1. Who among the following sits between F and G? 10. Which of the following groups represents young
1) E 2) H 3) A

K
graduates and clerks but not MBBS?
4) C 5) None of these 1) B, I 2) A, B 3) B 4) I 5) None of these
2. Who among the following sits third to the right of C? 11. Which of the following statements is true?
1) E 2) F 3) A 1) All young are MBBS and clerk.
4) H 5) None of these 2) Some graduates are MBBS.
3. Who sits on the immediate right of H? 3) All clerks are MBBS but not young.
1) B 2) A 3) G 4) All MBBS are clerks.
4) F 5) Can’t be determined 5) None of these
4. In which of the following combinations is the first Directions (Q. 12-16): In each question below is given
person sitting between the second and the third a group of letters followed by four combinations of digits/
person? symbols numbered (1), (2), (3) and (4). You have to find out

KUNDAN
1) ABC 2) EFH 3) FGH which of the combinations correctly represents the group
4) BCA 5) None of these of letters based on the following coding system and mark
5. Which of the following statements is true? the number of that combination as the answer. If none of
1) C is opposite E. the four combinations correctly represents the group of
2) E is second to the left of F. letters, mark 5), ie ‘None of these’, as the answer.
3) B is opposite H.
4) G is on the immediate right of D.
5) None of these
6. Who sits on the immediate right of A?
1) B 2) F 3) E Conditions:
4) C 5) None of these 1) If the first and the last letters are consonants, both
Directions (Q. 7-11): The following questions are are to be coded as the code for the first letter.
based on the diagram given below: 2) If the first letter is a consonant and the last letter is
as vowel their codes are to be interchanged.
3) If both the first and the last letters are vowels, both
are to be coded as the last letter.
12. TABDRE
1) 93#2@7 2) 73#@29 3) 93#@27
4) 73#@27 5) None of these
13. QHFKAN
1) 4©$134 2) 4©$138 3) £©$134
(i) The rectangle represents MBBS. 4) 4©1$34 5) None of these
(ii) The triangle represents young people. 14. IAJEPI
(iii) The circle represents clerks. 1) A%367$ 2) %367%A 3) %367A%
(iv) The square represents graduates. 4) 6%367A 5) None of these
15. NRJTBP 2) if the data in statement II alone are sufficient to
1) £26#9@ 2) £%269£ 3) £962#© answer the question, while the data in statement I
4) £269#£ 5) None of these alone are not sufficient to answer the question.
16. JATHQE 3) if the data either in statement I alone or in statement
1) 739©46 2) 639©47 3) 639©46 II alone are sufficient to answer the question.
4) 739©47 5) None of these 4) if the data in both the statements I and II together
Directions (Q. 17-18): Four of the following five are are not sufficient to answer the question.
alike in a certain way and hence form a group. Which is 5) if the data in both the statements I and II together
the one that does not belong to that group? are necessary to answer the question.
17. 1) Kinship 2) Diversity 3) Sameness 27. Among five persons J, K, L, M and N sitting in a row,
4) Conformity 5) Similarity who is in the middle of the row?
18. 1) Refute 2) Confute 3) Crunch I. K is between L and M, neither of whom is at the
4) Concur 5) Quash extreme ends.
19. How many such pairs of letters are there in the word II. J and N are at the extreme ends and L is to the right
SURRENDER, each of which has as many letters of J.
between them in the word as in the English alphabet? 28. Is R greater than T among P, Q, R, S and T?
1) None 2) One 3) Two I. P is greater than or equal to R. Q is greater than S,
4) Three 5) None of these who is greater than or equal to T. Q is equal to R.

K
20. In a certain code HCKVJ is the code for FAITH and II. T is greater than P, who is equal to S. R is greater
TKIJV is the code for RIGHT. In the same code than Q, who is smaller than S.
language ETQYP is the code for which of the 29. How is A related to C?
following? I. A is daughter of B.
1) BROWN 2) CREAM 3) CROWN II. A’s father’s only sibling X is C’s paternal aunt.
4) AMONG 5) None of these 30. Among A, B, C and D, who is the shortest?
21. What will come next in the given alphabetical series? I. B and D are shorter than C but not shorter than A.
XVU WUT VTS USR ? II. A is as tall as B but shorter than C.
1) TRQ 2) RST 3) SRQ 4) TUV 5) None of these Directions (Q. 31-35): In each question below are
Directions (Q. 22-26): In these questions, a given two/three statements followed by two conclusions

KUNDAN
relationship between different elements is shown in the numbered I and II. You have to take the given statements
statements. The statements are followed by two to be true even if they seem to be at variance with
conclusions. Give answer commonly known facts. Read all the conclusions and then
1) if only conclusion I is true. decide which of the given conclusions logically follows
2) if only conclusion II is true. from the given statements, disregarding commonly
3) if either conclusion I or II is true. known facts. Give answer
4) if neither conclusion I nor II is true. 1) if only conclusion I follows.
5) if both conclusions I and II are true. 2) if only conclusion II follows.
22. Statements: F  N, R  N, H > R 3) if either conclusion I or II follows.
Conclusions: I. H > F II. R  F 4) if neither conclusion I nor II follows.
23. Statements: M < T, K  T, N < K 5) if both conclusions I and II follow.
Conclusions: I. N > M II. T > N (31-32):
24. Statements: T > H, W > H, M  W Statements: Some mice are keyboards.
Conclusions: I. T > W II. H  M No keyboard is a key.
25. Statements: D = M, L  P, M > P All keys are rings.
Conclusions: I. P < D II. M > P 31. Conclusions: I. Some rings are not keyboards.
26. Statements: J  B, R = F > T, B < R II. All mice being keys is a
Conclusions: I. J > T II. T  B possibility.
Directions (Q. 27-30): Each of the questions below 32. Conclusions: I. Some mice are rings.
consists of a question and two statements numbered I and II. All rings being mice is a
II given below it. You have to decide whether the data possibility.
provided in the statements are sufficient to answer the 33. Statements: All bags are books.
question. Read both the statements and give answer Some books are boxes.
1) if the data in statement I alone are sufficient to Conclusions: I. All boxes being bags is a
answer the question, while the data in statement II possibility.
alone are not sufficient to answer the question.
II. Some bags are boxes. well?"
34. Statements: No kite is an aeroplane. Vidura acquainted him with his mission: "Everyone in
Some aeroplanes are birds. Hastinapura is well. How fares it with you all? I have come
Conclusions: I. Some birds are definitely not kites. to invite you on behalf of King Dhritarashtra to come and
II. No aeroplane is a kite. see the newly erected hall of games. A beautiful hall has
35. Statements: Some pots are belts. been erected there even like yours. The king would like
No belt is a shirt. you to come with your brothers, see everything, have a
Conclusions: I. Some shirts are not pots. game of dice and return to your capital."
II. All shirts are definitely not belts. Yudhishthira seemed to ask counsel of Vidura:
Directions (Q. 36-40): Study the following information "Wagering games create quarrels among Kshatriyas. A wise
carefully and answer the given questions: man will avoid them if he can. We are ever abiding by your
Ten persons are sitting in two parallel rows containing advice. What would you have us do?"
5 persons each in such a way that there is an equal distance Vidura replied: "Everyone is aware that the playing of
between adjacent persons. In the 1st row E, F, G, H and I are dice is the root of many evils. I did my best to oppose this
sitting and all of them are facing south. In the 2nd row T, U, idea. Still the king has commanded me to invite you and I
V, W and Z are sitting, but not necessarily in the same have come. You may do as you like." Despite this warning,
order. In the given seating arrangement each member sitting Yudhishthira went to Hastinapura with his brothers and
in a row faces another member of the other row. retinue.

K
The one who is an immediate neighbour of H is facing It may be asked why the wise Yudhishthira responded
W. F does not sit at the extreme ends of the row. V and Z to the invitation. Three reasons may be given. Men rush
are immediate neighbours. Only one person sits between I consciously on their ruin impelled by lust, gambling and
and G. Z sits second to the left of the person who faces I. E drink. Yudhishthira was fond of gambling. The Kshatriya
faces the person who is on the immediate left of Z. T sits at tradition made it a matter of etiquette and honour not to
the left end of the row. E and H are not immediate refuse an invitation to a game of dice. There is a third
neighbours. reason too. True to the vow he took at the time, Vyasa had
36. Who among the following sits between G and I? warned him of the quarrels that would arise leading to
1) H 2) F 3) E destruction of the race. Yudhishthira would not give any
4) T 5) None of these occasion for displeasure or complaint by refusing the
37. Who among the following faces G? invitation of Dhritarashtra. These causes conspired with

KUNDAN
1) U 2) V 3) H his natural inclination to make Yudhishthira accept the
4) Z 5) None of these invitation and go to Hastinapura. The Pandavas and their
38. Who among the following sits second to the right of retinue stopped in the magnificent palace reserved for them.
the person who faces U? Yudhishthira rested on the day of arrival and, after the
1) F 2) I 3) W daily routine of duties, went to the hall of games the next
4) V 5) None of these morning.
39. How many persons sit between E and H? After the exchange of customary greetings, Sakuni
1) One 2) Two 3) Three announced to Yudhishthira that the cloth for playing the
4) Can’t be determined 5) None of these game had been spread and invited him to it.
40. Four of the following five are alike in a certain way Yudhishthira at first said: "O king, gambling is bad. It
based on the given seating arrangement and hence is not through heroism or merit that one succeeds in a
form a group. Which is the one that does not belong game of chance. Asita, Devala and other wise rishis who
to that group? were well-versed in worldly affairs have declared that
1) GZ 2) FV 3) WI gambling should be avoided since it offers scope for deceit.
4) UH 5) EV They have also said that conquest in battle is the proper
path for the Kshatriyas. You are not unaware of it."
Test-II: English Language But a part of himself, weakened by addiction to
gambling, was at war with his judgment and in his heart of
Directions (Q. 41-50): Read the following passage hearts Yudhishthira desired to play. In his discussion with
carefully and answer the questions given below it. Certain Sakuni, we see this inner conflict. The keen-witted Sakuni
words/phrases in the passage are printed in bold to help spotted this weakness at once and said: "What is wrong
you locate them while answering some of the questions. with the game? What, in fact, is a battle? What is even a
At the sight of Vidura, Yudhishthira anxiously discussion between Vedic scholars? The learned man wins
inquired: "Why are you so cheerless? Is it well with all our victory over the ignorant. The better man wins in every
relations in Hastinapura? Are the king and the princes case. It is just a test of strength or skill, that is all, and there
is nothing wrong in it. As for the result, in every field of (A) Because it is not through merit that one succeeds
activity, the expert defeats the beginner, and that is what in a game of gambling.
happens in a game of dice also. But if you are afraid, you (B) Because it involves dishonesty.
need not play. But do not come out with this worn excuse (C) Because it allows even slaves to be staked at.
of right and wrong." 1) Only (A) 2) Only (B) and (C)
Yudhishthira replied: "Well, who is to play with me?" 3) Only (B) 4) Only (A) and (C)
Duryodhana said: "Mine is the responsibility for finding 5) Only (A) and (B)
the stakes in the form of wealth and gems to play the game. 45. According to the passage, which of the following
My uncle Sakuni will actually cast the dice in my stead." statements is not true?
Yudhishthira had thought himself secure of defeating 1) The learned man wins over the ignorant.
Duryodhana in play but Sakuni was a different matter, for 2) Gambling brings in a lot of money and hence should
Sakuni was a recognised expert. So he hesitated and said: be revered.
"It is not, I think, customary for one man to play on behalf 3) The better man wins in every case.
of another." 4) Gambling is a test of skill and strategy.
Sakuni retorted tauntingly: "I see that you are forging 5) None of these
another excuse." Directions (Q. 46-48): Choose the word/group of
Yudhishthira flushed and, casting caution to the winds, words which is MOST SIMILAR in meaning to the word/
replied: "Well, I shall play." group of words printed in bold as used in the passage.

K
The hall was fully crowded. Drona, Kripa, Bhishma, 46. Conspired
Vidura, and Dhritarashtra were seated there. They knew 1) disagreed 2) neglected 3) colluded
that the game would end viciously and sat unhappily 4) promoted 5) advised
witnessing what they could not prevent. The assembled 47. Viciously
princes watched the game with great interest and 1) peacefully 2) kindly 3) cordially
enthusiasm. 4) horribly 5) roughly
41. Why did Duryodhana ask Sakuni to cast the dice in 48. Hesitated
his place? 1) continued 2) stumbled 3) stayed
1) Because Sakuni was interested in playing with 4) attacked 5) persisted
Yudhishthira. Directions (Q. 49-50): Choose the word/group of

KUNDAN
2) Because Sakuni was honest in each of his throws. words which is MOST OPPOSITE in meaning of the word/
3) Because Sakuni was a recognised expert in group of words printed in bold as used in the passage.
gambling. 49. Conquest
4) Because Sakuni had no mind to defeat the 1) surrender 2) annex 3) invade
Pandavas. 4) takeover 5) overthrow
5) None of these 50. Impelled
42. Despite the warning given by Vidura, why did 1) compelled 2) influenced
Yudhishthira go to Hastinapur to play the dice match? 3) obliged 4) moved
(A) Because Yudhishthira was fond of gambling. 5) discouraged
(B) The Kshatriya tradition made it a matter of Directions (Q. 51-55): Read each sentence to find out
etiquette not to refuse an invitation to a game of whether there is any grammatical or idiomatic error in it.
dice. The error, if any, will be in one part of the sentence. The
(C) Yudhishthira did not want to give any occasion number of that part is the answer. If there is ‘No error’,
for displeasure among his relatives. the answer is 5). (Ignore errors of punctuation, if any.)
1) Only (A) 2) Only (B) 51. 1) All of the defendants are accused of involved / 2) in
3) Only (C) 4) All (A), (B) and (C) the killings of intellectuals, political opponents and
5) Only (A) and (C) secularists / 3) in the brutal war / 4) of independence
43. Why have rishis declared that gambling should be in 1971. / 5) No error
avoided? 52. 1) The essence of Jhumpa Lahiri’s work lies / 2) in the
1) Because it offers scope for drinking wine conflicted inner lives of her characters, / 3) their trials
2) Because it offers scope for deceit and tribulations around difficult lives choices, and
3) Because it is a game which involves misbehaviour their perpetual struggles / 4) to negotiate their
4) Because it is a game which can lead to fight emotions vis-a-vis a hostile universe. / 5) No error
5) None of these 53. 1) Despite so much global concern and international
44. Why has gambling been described as bad and a game support, / 2) satisfactory recovery of tiger population
of chance by Yudhishthira? / 3) still elude us / 4) in all the tiger-range countries. / 5)
No error 2) had a key role played
54. 1) If one wants to use the film medium for social and 3) has a key role to play
political causes, / 2) the film-maker has to be adapt in 4) had a key role being played
the language of cinema, / 3) like a soldier who has to 5) No correction required
be a master / 4) of his weapons to survive on the 62. Raman Singh is a man of many a sobriquets.
battlefront. / 5) No error 1) people of many 2) man of many
55. 1) A new compound developed at the Tokyo Institute 3) person of much 4) human of most of
of Technology / 2) shows highly unusual conducting 5) No correction required
properties / 3) this could be used / 4) in future 63. Delhi’s three-time chief minister was known for her
electronic components. / 5) No error ability to read the pulse of voters.
Directions (Q. 56-60): Pick out the most effective 1) has been known for 2) has known for
word from the given words to fill in the blank(s) to make 3) has been knowing for 4) had knew about
the sentence meaningfully complete. 5) No correction required
56. Two political scions who _______ as young Members 64. Proteins are the primary building brick of the human
of Parliament ________ a decade ago are now body.
ministers with key infrastructure portfolios like 1) building fabric 2) construction slabs
telecom, shipping and corporate affairs. 3) building blocks 4) framework entity
1) launched, scarcely 2) opened, merely 5) No correction required

K
3) took, hardly 4) began, around 65. Doing something for the first time can be difficult and
5) debuted, nearly frustrated.
57. Tarun Tejpal publicly shouted from the rooftop against 1) a difficulty and frustration
sexual harassment and privately ________ in morally 2) difficult and frustrating
suspect behaviour ________ the same roof. 3) difficult to frustrate
1) entertain, in 2) indulged, under 3) gratify, at 4) difficult with frustration
4) oblige, beneath5) cater, below 5) No correction required
58. The Ghaziabad court may have given the ________ Directions (Q. 66-70): Rearrange the following six
in the Aarushi Talwar case, but a lot many loose ends sentences (A), (B), (C), (D), (E) and (F) in the proper
still remain. sequence to form a meaningful paragraph and then answer
1) award 2) decree 3) sentence the questions given below.

KUNDAN
4) verdict 5) conclusion (A) In economic terms, any product being capable of
59. There comes a moment in the life of a nation when the giving customer satisfaction needs to have ‘place’
usurper rises from the ________ of disenchantment and ‘time’ utility.
and ________ the comforting ________ of the time. (B) For an effective conclusion of marketing, the
1) recesses, shatters, certainties functioning of other elements in the chain is as
2) break, crack, surety important as that of the producer.
3) interval, impair, lock (C) This is provided by the “Distribution” function
4) intermission, smash, stock of marketing mix.
5) nook, crush, store (D) One of the four Ps of marketing mix is ‘place’ or
60. Set against the Congress’s ________ headquarters ‘Distribution’.
and ________ state branches, BJP has the advantage (E) In the normal goods distribution process, entities
of a galvanising general at the top and able warriors in other than the producers are also associated and
the provinces. there is a chain between the producer and the
1) declined, shabby 2) slackened, damaged consumer.
3) diminished, dilapidated 4) reduced, shaky (F) All endeavours of production will come to nought,
5) subsidised, battered if the products do not reach the consumers for
Directions (Q. 61-65): In the following questions, a consumption.
sentence has been given with some of its parts in bold. To 66. Which of the following should be the FIRST sentence
make the sentence grammatically correct, you have to after rearrangement?
replace the bold part with the correct alternative given 1) F 2) A 3) B 4) C 5) D
below. If the sentence is correct as it is, give 5) as your 67. Which of the following should be the SECOND
answer (ie No correction required). sentence after rearrangement?
61. Energy have a key role to play in the uninterrupted 1) B 2) A 3) D 4) F 5) E
growth of Indian economy. 68. Which of the following should be the THIRD sentence
1) has a key role to be played
after rearrangement? 4) approve 5) let
1) E 2) B 3) F 4) C 5) A 78. 1) manipulate 2) mutate 3) transfer
69. Which of the following should be the LAST (SIXTH) 4) turn 5) convert
sentence after rearrangement? 79. 1) relevant 2) compatible 3) suited
1) B 2) C 3) E 4) D 5) F 4) admissible 5) proper
70. Which of the following should be the FIFTH sentence 80. 1) field 2) work 3) business
after rearrangement? 4) function 5) occupation
1) A 2) F 3) E 4) B 5) C
Directions (Q. 71-80): In the following passage, some Test-III: Quantitative Aptitude
of the words have been left out, each of which is indicated
by a number. Find the suitable word from the options given Directions (Q. 81-100): What should come in place
against each number and fill up the blanks with of question mark (?) in the following questions?
appropriate words to make the paragraph meaningfully 81. 7.84 10.24 = ?
complete.
The framework released by the RBI for entry and 1) 12.48 2) 10.12 3) 9.64 4) 8.96 5) 8.24
expansion of foreign banks in India makes sense. If foreign 82. 73174 – 29617 + 43156 – 31619 = ?
banks do enter India, it would drive local banks to become 1) 55094 2) 54684 3) 53874
(71) efficient and encourage mergers and acquisitions. 4) 52124 5) None of these

K
These banks will also bring (72) technology and expertise 83. 2209 11.5 = ?
to foster financial inclusion. The RBI’s final rules provide a 1) 544.5 2) 542.5 3) 540.5 4) 538.5 5) 536.5
foreign bank nearly the (73) freedom as a private sector
bank in opening branches if it takes the form of a local 192  267  357
84. ?
subsidiary. New or existing (74) will have to set up wholly- 2  3 6  7
owned arms if they are systemically important foreign 1) 70184 2) 72624 3) 74484
banks, or banks that have a complex (75) or those that do 4) 76364 5) None of these
not have adequate disclosure requirement in their home 85. 319.24 – 106.89 + 431.65 – 240.33 = ?
jurisdictions or offer home residents preferential protection 1) 407.67 2) 421.27 3) 403.67
in case of a problem. 4) 424.57 5) None of these

KUNDAN
However, those present in India before August 2010 – 86. 48% of 1735 + 36% of 1545 = ?
Citi, HSBC and Standard Chartered – have the (76) to 1) 1184 2) 1216 3) 1278 4) 1324 5) 1389
continue as branches. Rightly, the regulator must encourage
their conversion into local subsidiaries, following Indian 87. 1.96  23.04  ?
laws. It will ring-fence the capital and assets of the bank 1) 3.8 2) 3.4 3) 2.8
within India and (77) effective risk control. Incentives by 4) 2.6 5) None of these
way of operational freedom could persuade some banks to 88. 0.748 ÷ 0.044 = 20% of ?
(78) into wholly-owned subsidiaries. India is a growing 1) 125 2) 105 3) 85 4) 65 5) 45
market and more branches would do no harm. However, 89. 658 × ? × 8 = 94752
technology has made physical branch networks much less 1) 14 2) 16 3) 18 4) 22 5) 24
(79) today. An example is the credit card (80) of foreign 90. 172 + 232 = ?
banks that has far surpassed their branch-network size.
1) 1600 2) 274576 3) 374544
71. 1) less 2) added 3) further
4) 669124 5) None of these
4) more 5) also
72. 1) down 2) in 3) up 1
4) around 5) forward 91. (2)–3.7 ÷ (32)–1 × = (2)?
16
73. 1) equivalent 2) likely 3) duplicate 1) –12.7 2) –2.7 3) 5.3 4) 3.7 5) None of these
4) different 5) same
74. 1) contestants 2) aspirant 3) entrants 92. 295.84  ?
4) candidates 5) petitioners 1) 16.8 2) 17.2 3) 17.4 4) 17.6 5) 16.6
75. 1) structure 2) construction 3) architecture 93. 56 × 21.8 ÷ 8 + 18.6 = ?
4) texture 5) configuration 1) 165 2) 168.4 3) 171.2 4) 174 5) 176.8
76. 1) discretion 2) free will 3) prerogative 94. 16.4 × 7.75 × 12 = ?
4) option 5) claim 1) 1122.6 2) 1426.8 3) 1628.4 4) 1525.2 5) 1320
77. 1) endow 2) enable 3) qualify 3
95. 68921  3 17576  ?
1) 61 2) 63 3) 65 4) 67 5) 69 110. The average age of 12 persons is 32 years. If the age
of one more person is added, the average decreases
1
96. (16807)–2 × (49)7 × = (7)? by one year. What is the age of the new person?
343 1) 19 years 2) 23 years 3) 25 years
1) 1 2) –3 3) 7 4) 2 5) –5 4) 21 years 5) 27 years
97. 24.8% of 1550 – 13.2% of 1125 = ? 111. The rate of growth of population of a city is 10% per
1) 231.1 2) 235.9 3) 239.4 4) 243 5) 246.5 annum. If in the year 2010 the population of the city
4 5 8 was 22 lakh then what will be its population in the year
98. of of of 7623 = ? 2013?
7 9 11
1) 27.142 lakh 2) 28.436 lakh 3) 29.282 lakh
1) 1320 2) 1540 3) 1760 4) 1980 5) 2120 4) 30.012 lakh 5) 31.64 lakh
99. 48% of 325 + 987 ÷ 47 = 125% of ? 112. A 175-metre-long train is running at a speed of 58 kmph.
1) 1296 2) 1416 3) 1524 4) 1684 5) 1704 In what time will it cross a man who is walking in the
100. 6084  4356  888  ? opposite direction at a speed of 5 kmph?
1) 9 seconds 2) 8 seconds 3) 20 seconds
1) 66 2) 68 3) 72 4) 74 5) 76 4) 10 seconds 5) 12 seconds
101. At what rate of simple interest per annum does a person 113. Sandeep incurred a loss of 30% on selling an article
get an interest of `4706.1 on the principal amount of for `5670. What is the cost price of the article?
`11205 after 7 years?

K
1) `9600 2) `9200 3) `8700
1) 4% pa 2) 6% pa 3) 8% pa 4) 12% pa 5) 14% pa 4) `8900 5) `8100
102. The difference of a two-digit number and the number 114. What is the area of a circle whose diameter is 33.6 cm?
obtained after interchanging its two digits is 36. If the 1) 814.02 sq cm 2) 834.8 sq cm 3) 864.84 sq cm
sum of the two digits of the number is 12, then what is 4) 887.04 sq cm 5) 896.6 sq cm
the number? 115. There is a rectangular room whose length, breadth
1) 57 2) 75 3) 84 4) 93 5) None of these and height are 15m, 9m and 4m respectively. What will
103. In how many different ways can the letters of the word be the cost of painting the four walls of the room if the
BOTTLE be arranged? rate of painting is `8.40 per square metre?
1) 60 2) 360 3) 120 4) 720 5) 36 1) `1484.6 2) `1548.2 3) `1612.8
104. The ratio of the length to the breadth of a rectangular 4) `1712.4 5) `1804.6

KUNDAN
field is 16 : 9. If the area of the field is 11664 sq m, what 116. A man can row 42 km upstream and 68 km downstream
is its perimeter? in 10 hours. Also, he can row 56 km upstream and 85
1) 420m 2) 450m 3) 480m 4) 520m 5) 560m km downstream in 13 hours. What is the speed of the
105. What will be the next number in the following number man in still water?
series? 1) 18 kmph 2) 16 kmph 3) 14 kmph
9155, 6961, 5627, 4901, 4555, ? 4) 12 kmph 5) 8 kmph
1) 4343 2) 4337 3) 4433 4) 4413 5) 4403 117. Out of the given fractions, which is the largest?
106. An amount of `110000 is invested at compound
interest payable annually. If the rate of interest is 1 2 8 5 4
1) 2) 3) 4) 5)
11% pa, what will be the total interest after two years? 3 7 19 14 11
1) `23481 2) `25531 3) `24200 118. The area of a circle is equal to the area of a rectangle
4) `26416 5) None of these whose perimeter is equal to 69 m and whose breadth is
107. An item is sold for `1652 at a profit of 18%. What equal to 13.5m. What is the area of the circle?
would have been the gain or loss per cent had it been 1) 278.5 sq m 2) 281.5 sq m 3) 283.5 sq m
sold for `1512? 4) 287.5 sq m 5) 296.5 sq m
1) 6% 2) 8% 3) 10% 4) 12% 5) 14% 119. There is sufficient food for 200 men for 45 days. After
7 24 days 95 men left. For how many days will the
108. The difference between 72% of a number and of remaining food last for the remaining men?
5
1) 28 days 2) 30 days 3) 32 days
the same number is 125.8. What is 40% of that number? 4) 36 days 5) 40 days
1) 64 2) 68 3) 72 4) 74 5) 78 120. A can do a piece of work in 36 days, while B alone can
109. The cost of 16 pens and 20 pencils is `124. What will do the same piece of work in 30 days. With the help of
be the cost of 28 pens and 35 pencils? C they finish the work in 9 days. In how many days
1) `284 2) `251 3) `227 can C alone do the same piece of work?
4) `217 5) `208 1) 20 days 2) 24 days 3) 25 days
4) 36 days 5) 40 days 4) M-Cap 5) Actuaries
130. The RBI has repeatedly stressed that investors and
Test-IV: General Awareness users should exercise caution before using Bitcoins
as a consistent means of exchange. Bitcoin is a
(with Special Reference to Banking Industry) 1) Digital currency 2) Regional currency
3) Fictitious money 4) Symbolic currency
121. Joachim Gauck, who visited India recently, is the 5) None of these
President of which of the following countries? 131. India has emerged as the fourth-largest country to
1) Spain 2) Germany 3) New Zealand grow biotech or genetically modified (GM) crops in
4) Italy 5) None of these 2013. Which of the following countries is the largest
122. The Finance Minister P Chidambaram presented grower of GM crops as of now?
Interim Budget for 2014-15 in the Lok Sabha recently. 1) Brazil 2) Canada 3) US
This was the ________budget of the Union Govt. 4) Spain 5) Portugal
1) 81st 2) 82nd 3) 83rd 4) 84th 5) 85th 132. Which of the following cellular operators has acquired
123. Finance Minister P Chidambaram allocated an Mumbai based Loop Mobile for about `700 cr
additional `1,000 cr to the Nirbhaya Fund while recently?
presenting the Interim Budget recently. Nirbhaya Fund
1) Vodafone 2) Maxis 3) Telenor
is aimed at which of the following?
4) Idea 5) Bharti Airtel

K
1) Checking female foeticide
133. Om Prakash Singh Karhana gave the lone gold to India
2) Employment generation for women
in the 6th Asian Indoor Athletics Championships event
3) Safety and empowerment of women
held at Hangzhou (China) recently. He plays which of
4) Providing loan to women entrepreneurs
the following games?
5) None of these
1) Billiards 2) Pole vault 3) Triple jump
124. What does the letter ‘P’ denote in the abbreviation
4) Shot put 5) None of these
FIPB, a term we often read in the financial newspapers?
134. Tier I capital of banks, also termed as core capital,
1) Preference 2) Promotion 3) Protection
consists of which of the following?
4) Prohibition 5) Proposal
1) Share capital
125. Different stakeholders have raised concerns over
2) Disclosed reserves
RBI’s Nachiket Mor panel’s recommendation to create

KUNDAN
3) Certain reserves and subordinate debts
dedicated banks for rural inclusion. These banks will
4) All the above
be known as which of the following?
5) Only 1) and 2) minus goodwill, if any
1) Development banks 2) Payments banks
135. Capital to risk-weighted assets ratio (CRAR) is arrived
3) Permanent banks 4) Priority banks
at by dividing the capital of the bank with aggregated
5) None of these
risk-weighted assets. These risk-weighted assets
126. Defence spending has been raised by 10 per cent from
belong to
the previous year in the recent Interim Budget. What
1) Credit risk 2) Market risk
is size of the fund allocated for defence spending for 3) Operational risk 4) Only 1) and 2)
2014-15? 5) All 1), 2) and 3)
1) `2.11 tn 2) `2.24 tn 3) `2.44 tn 136. Hamad bin Isa Al Khalifa, who visited India on a three
4) `3.25 tn 5) `3.85 tn day official visit recently, is the present king of which
127. Jnanpith Awardee Amar Kant, who passed away of the following countries?
recently, was a noted writer of which of the following 1) Oman 2) Jordan 3) Yemen
languages? 4) Bahrain 5) None of these
1) Hindi 2) Urdu 3) Bengali 137. Which of the following states in India has been
4) Telugu 5) Tamil declared as the best state in the country for
128. Brendon McCullum and BJ Watling made 352 runs implementing Rural Tourism Project and has been
recently for the sixth wicket, which is the highest by conferred the National Tourism Award 2012-13
any pair in Test cricket so far. Both the players play for recently?
which of the following countries? 1) Sikkim 2) Kerala
1) Australia 2) West Indies 3) South Africa 3) Andhra Pradesh 4) Maharashtra
4) New Zealand 5) None of these 5) Rajasthan
129. Which of the following is NOT a money/finance- 138. Which of the following countries ordered the
related term? immediate recall of its Ambassador in India, Daniele
1) Reti opening 2) Dividend 3) Divestment Mancini, for consultations recently?
1) Spain 2) Italy 3) Norway now been declared dead after attempts to revive it
4) Sweden 5) None of these failed recently?
139. Forward Contract, Options and Swaps are the examples 1) Japan 2) Russia 3) France
of which of the following? 4) India 5) China
1) Derivative 2) Gilt-edged securities 147. Who among the following took over as the new
3) Preferential shares 4) IPOs chairperson of Broadcasting Content Complaints
5) None of these Council (BCCC), a self-regulatory body for non-news
140. Every bank is required to maintain, at the close of channels in India?
business every day, a minimum proportion of their net 1) Indira Jaising
demand and time liabilities as liquid assets in the form 2) Justice Mukul Mudgal
of cash (book value), gold (current market value) and 3) Justice Asok Kumar Ganguly
balances in unencumbered approved securities. It is 4) Justice Swatanter Kumar
termed as 5) Justice SH Kapadia
1) Cash Reserve Ratio (CRR) 148. Which of the following has become the first rail system
2) Statutory Liquidity Ratio (SLR) in the world to be awarded the Gold Standard
3) Marginal Standing Facility (MSF) foundation — a globally accepted certification
4) Excess reserve standard for carbon mitigation projects recently?
5) None of these 1) Delhi Metro 2) New York Metro

K
141. The 22nd session of the World Economic Forum 3) London Metro 4) Kolkata Metro
(WEF) was held in the Swiss city of Davos recently. 5) None of these
Which of the following is correct about WEF? It is a/ 149. We often read about decreasing or increasing different
an rates and ratios by the RBI by certain basis points.
1) UN body One basis point is equal to which of the following?
2) US agency 1) 1% 2) 0.1% 3) 0.01%
3) Independent international organisation 4) 0.001% 5) None of these
4) World Bank institution 150. An Individual having a ‘Basic Savings Bank Deposit
5) None of these Account’ in a bank cannot have which of the following
142. The issue of the pepper spray incident in the Lok types of accounts in that bank?
Sabha, which saw widespread outrage and 1) Savings account 2) Term/Fixed Deposit

KUNDAN
condemnation, was referred by the Lok Sabha Speaker 3) Recurring Deposit 4) All the above
Meira Kumar to the Privileges Committee with punitive 5) None of the above
powers. Who is heading the Committee? 151. Who among the following has authored the recently
1) PC Chacko 2) P Chidambaram published book titled Catch Up: Developing
3) Kamal Nath 4) Mallikarjun Kharge Countries in the World Economy?
5) None of these 1) Ratnesh Dwivedi 2) Deepak Nayyar
143. India has been ranked at which of the following 3) Pankaj Mishra 4) Jaswant Singh
positions in the recently released World Press 5) None of these
Freedom Index published by the media watchdog 152. Yulia Vyacheslavovna Lipnitskaya became the
Reporters Without Borders (RSF)? youngest athlete (15 years old) to win Gold at Winter
1) 140th 2) 158th 3) 172nd 4) 173rd 5) 175th Olympics held in Sochi (Russia) recently. She belongs
144. Which of the following is a leading electronic-payment to which of the following countries?
technology firm? 1) Ukraine 2) Russia
1) Nifty 2) Visa 3) Max 3) Czech Republic 4) Kazakhstan
4) APEC 5) None of these 5) None of these
145. LIBOR, which is acronym for London Inter Bank 153. Harish Rawat was sworn in as the new chief minister
Offered Rate, is the interest rate at which banks offer of which of the following states recently?
to lend funds to who among the following? 1) Chhattisgarh 2) Karnataka
1) Govts 3) Himachal Pradesh 4) Uttarakhand
2) Interbank market 5) None of these
3) Major infrastructure projects 154. KYC is an acronym for “Know Your Customer”, a term
4) Acquisitions by firms used for customer identification process. It does not
5) None of these help banks in which of the following?
146. Which of the following countries had launched its 1) Determining true identity of account holder
first lunar rover Yutu or the Jade Rabbit, which has 2) Source of funds
3) Nature of customer’s business 3) the sheet tab name 4) the worksheet name
4) Increase profitability 5) None of these
5) All the above 165. ________ contains location of file on the disk.
155. The banking ombudsman scheme provides an 1) Boot sector 2) File allocation table
expeditious and inexpensive forum to bank customers 3) Data area 4) Root folder
for resolution of complaints. Introduced by the RBI 5) Memory unit
with effect from 1995, the scheme covers which of the 166. Hub is applicable in ________ layer.
following types of banks? 1) physical 2) datalink 3) network
1) Public Sector Banks 4) transport 5) application
2) Private Banks 167. The ‘QWERTY’ keyboard ________.
3) Regional Rural Banks 1) is the most popular keyboard.
4) Scheduled Primary Co-Operative Banks 2) is the fastest keyboard.
5) All the above 3) is the keyboard that is rarely used.
156. Under the Liberalised Remittance Scheme introduced 4) is the keyboard having complex key layout.
by the RBI, all resident individuals, including minors, 5) None of these
are allowed to freely remit up to USD 75,000 per financial 168. What type of computer can support hundreds of
year. Remittances directly or indirectly to which of the people at a time?
following countries is not permitted under the scheme? 1) Desktop 2) Super Computer

K
1) Pakistan 2) Mauritius 3) Nepal 3) Micro Computer 4) Mini Computer
4) Bhutan 5) All the above 5) Mainframe Computer
157. Which of the following teams won the 2013-14 season 169. ________ uses viruses to destroy computer hardware
of the Ranji Trophy cricket tournament recently? and software.
1) Karnataka 2) Maharashtra 3) Punjab 1) Spoofing 2) Tunneling 3) Sabotage
4) Delhi 5) None of these 4) Phishing 5) Flooding
158. Moulaye Ould Mohamed Laghdaf resigned along with 170. How many types of data can be entered in a worksheet?
his entire cabinet recently as the prime minister of 1) Three 2) Four 3) Two
which of the following countries? He had been holding 4) Five 5) Six
the post since 2008. 171. Which type of protocol is used for accessing internet/

KUNDAN
1) Mauritania 2) Fiji web on mobile phones?
3) Trinidad and Tobago 4) Nigeria 1) PPP 2) HTTP 3) FTP 4) WAP 5) SMTP
5) Surinam 172. It is easier to change the name of file using ________
159. Who among the following will head the 7th Central process.
Pay Commission set up by the Union Govt? 1) Transforming 2) Christening 3) Renaming
1) Meena Agarwal 2) Rathin Roy 4) Retagging 5) None of these
3) Vivek Rae 4) Ashok Kumar Mathur 173. A series 100 POST error code indicates a problem with
5) None of these which of the following?
160. World Cancer Day is observed on which of the 1) Hard drive 2) RAM or ROM 3) System board
following dates annually? 4) Video adapter 5) None of these
1) 1 Feb 2) 2 Feb 3) 3 Feb 4) 4 Feb 5) 5 Feb 174. To access a mainframe or supercomputer, users often
use a
Test-V: Computer Knowledge 1) terminal 2) node 3) desktop
4) handheld 5) None of these
161. Memory in a PC is normally addressable through 175. Deleted data remains on a disk until
1) bits 2) bytes 3) nibble 1) the data is overwritten.
4) All of these 5) None of these 2) the recycle bin is emptied.
162. The command that is used to format a drive through 3) a file compression utility is used.
command prompt is 4) the disk is scanned.
1) Scandisk 2) X Format 3) Chkdisk 5) None of these
4) Format 5) None of these 176. Which of the following displays the contents of an
163. Information on a computer is stored as active cell?
1) analog data 2) digital data 3) watts data 1) Toolbar 2) Menu bar 3) Namebox
4) modem data 5) None of these 4) Formula bar 5) None of these
164. The default header for a worksheet is 177. The ________ tells the computer how to use its
1) your name 2) the date and time components.
1) utility 2) network is the very first page that we see on opening a website
3) operating system 4) application program 1) Home page, Web page 2) Website, Home page
5) None of these 3) Web page, Home page 4) Web page, Website
178. The ability of an OS to run more than one application 5) None of these
at a time is called 191. Windows Explorer is a ________.
1) multitasking 1) PC 2) drive 3) browser
2) object-oriented programming 4) network 5) file manager
3) multi-user computing 192. To open search option for computer press________
4) time-sharing 1) window key + E 2) window key + F
5) None of these 3) window key + E + CTRL 4) window key + F + CTRL
179. Name the component which is required to process 5) window key + M
data into information and consists of integrated 193. Binary numbers need more places for counting because
circuits. 1) They are always big numbers
1) Hard disk 2) RAM 3) CPU 2) Any number of 0’s can be added in front of them
4) ROM 5) None of these 3) Binary base is small
180. Which of the following parts of the computer displays 4) 0’s and 1’s have to be properly spaced apart
the work done? 5) None of these
1) RAM 2) Printer 3) Monitor 194. ________ operating systems are used in robotics,

K
4) ROM 5) None of these multimedia and animation.
181. You can print 1) MAC 2) Window 3) Real-Time
1) a selected range of cells 4) UNIX 5) None of these
2) a single worksheet 195. Which of the following types of data can’t be entered
3) an entire workbook into a worksheet cell?
4) All the above 1) Value 2) Formula 3) Date and time
5) None of these 4) Label 5) None of these
182. 10 Base 2 type of ethernet uses ________ topology. 196. What is the most common type of compatibility that a
1) ring 2) bus 3) star 4) mesh 5) tree software manufacturer will provide for new versions
183. The local antenna for satellite connections is called a of their software?
1) VSAT 2) modem 3) terminal 1) Both backward and forward compatibility

KUNDAN
4) DTA 5) MSAT 2) Neither backward nor forward compatibility
184. In relational model, a tuple is referred as a 3) Backward compatibility but not forward
1) field 2) record 3) file compatibility
4) data item 5) data base 4) Forward compatibility but not backward
185. Uniform resource locator is another name of ________ compatibility
1) web bar 2) web address 3) title bar 5) None of these
4) address 5) formula bar 197. In Baudot code, how many numbers of bits per symbol
186. Which of the following commands is used to create a are used?
directory? 1) seven 2) five 3) six 4) eight 5) two
1) Dir 2) Mkdir 3) Md 198. The name of a file in MS Word cannot contain
4) Both 2) and 3) 5) Both 1) and 2) 1) Letter 2) Space 3) Number
187. What is the default font size of a cell content in Excel? 4) Underscore 5) None of these
1) 8 2) 9 3) 10 199. A storage device used to compensate the differences
4) 11 5) 12 in rates of flow of data from one device to another is
188. When you want to move some text from one page to 1) Buffer 2) Channel 3) Call
another, the best method is 4) Chip 5) None of these
1) delete and replace 2) find and replace 200. A device that generates the periodic signals used to
3) cut and paste 4) drag and drop control the timing of all CPU operations is called
5) move and paste 1) COM 2) Collate 3) Chip
189. The output of an AND gate with three inputs, A, B, 4) Clock 5) None of these
and C, is HIGH when
1) A = 1, B = 1, C = 0 2) A = 0, B = 0, C = 0
3) A = 1, B = 1, C = 1 4) A = 1, B = 0, C = 1
5) None of these
190. ________ is a collection of web pages and a ________
K
Answers
(1-6):
20. 3;

Similarly,

So,
Thus, D > P or P < D is true. Hence
conclusion I is true.
But, M > P is a restatement. Hence,
conclusion II does not follow.
26. 4; Statements: J  B
R=F>T
... (i)
... (ii)
1. 2 2. 1 3. 3 4. 4 5. 3 B<R ... (iii)

KUNDAN
6. 3 Combining all these statements, we get
7. 2 8. 3 9. 1 10. 4 11. 2 21. 1; Each corresponding letter moves one
JB<R=F>T
12. 2; Condition (2) applies letter backward in the alphabet.
We can’t compare J and T or B and T.
22. 1; Statements: F  N ... (i)
Hence, neither conclusion I nor II is true.
RN ... (ii) 27. 1; From I.
H>R ... (iii)
Combining all these statements, we get
K is between L and M. Hence, I alone is
13. 1; Condition (1) applies FNR<H sufficient to answer the question.
Thus, F < H or H > F is true. Hence 28. 1; From I.
conclusion I is true.
Again, F  R or R  F is true. Hence conclusion
II (R  F) is not true. Hence, R > T.
14. 3; Condition (3) applies
23. 2; Statements: M < T ... (i)
KT ... (ii) From II.
N<K ... (iii)
Combining all these statements, we get Hence, I alone is sufficient to answer the
15. 4; Condition (1) applies M<TK>N question.
We can’t compare M and N. Hence I is not 29. 5; From I.
true. But T > N. Hence II is true.
24. 4; Statements: T > H ... (i)
W>H ... (ii)
16. 1; Condition (2) applies From II.
MW ... (iii)
Combining all these statements, we get
T>H<WM
From both I and II.
We can’t compare T and W or H and M.
A being female is sister of C. Hence, both I
17. 2; Except ‘diversity’ all are synonyms. Hence neither conclusion I nor II is true. and II are sufficient to answer the question.
18. 4; Except ‘concur’ all are synonyms. 25. 1; Statements: D = M ... (i) 30. 1; From I. C > B, D > A
19. 3; LP ... (ii) Thus A is the shortest and statement I alone
M> P ... (iii) is sufficient.
Combining all these statements, we get From II. C > A = B
Hence II alone is not sufficient to answer
87. 4; ? = 4706.1 100
the question. 1.96  23.04 r= = 6% pa
31. 1; No keyboard is a key (E) + All keys 11205  7
are rings (A) = E + A = O* = Some rings are = 1.96  4.8 = 6.76 = 2.6 102. 3; Let the number be 10x + y.
not keyboards. Hence, conclusion I follows. Then, according to the questions,
Again, Some mice are keyboards (I) + No 0.748
88. 3; 20% of ? = = 17 x + y = 12 ... (i)
keyboard is a key (I) = I + E = O = Some mice 0.044 Again, according to the question,
are not keys. Thus, the possibility in II does 10x + y – (10y + x) = 36
not exist. Hence, conclusion II does not 17 100
or, ? = = 85 or, 9(x – y) = 36
follow. 20
x–y=4 ... (ii)
32. 4; I + E = O = Some mice are not keys + 89. 3; 658 × ? × 8 = 94752
All keys are rings (A) = O + A = No conclusion. Solving eqn (i) and (ii), we get
94752 x = 8, y = 4
Hence, conclusion I does not follow. ? = = 18
There is a negative statement. Therefore, 658  8  Number = 10 × 8 + 4 = 84
the possibility may not exist. Hence, 103. 2; In the word BOTTLE there are 6
90. 4; ? = 289 + 529 = 818 letters and T occurs twice.
conclusion II does not follow.
33. 1; There is no negative statement.  ? = (818)2 = 669124
6!
Hence, conclusion I follows. 1  Number of ways of arrangement =
Again, All bags are books (A) + Some books ? –3.7 5 –1 2!
91. 2; 2 = (2) ÷ (2 ) ×
are boxes (I) = A + I = No conclusion. Hence, 24 = 360
conclusion II does not follow. = (2)–3.7 ÷ (2)–5 × (2)–4 104. 2; Let its length and breadth be x m and
34. 5; No kite is an aeroplane (E) + Some = (2)–3.7 + 5 – 4 = (2)–2.7 y m respectively.
aeroplanes are birds (I) = E + I = O* = Some  ? = –2.7

K
According to the question,
birds are not kites. Hence, conclusion I
92. 2; ? = 295.84  17.2  17.2 = 17.2 x 16
follows.
Again, No kite is an aeroplane  conversion y = 9
56 21.8
 No aeroplane is a kite. Hence, conclusion 93. 3; ? = + 18.6 or, 9x = 16y ... (i)
8
II follows.
= 152.6 + 18.6 = 171.2 And the area of the rectangle = l × b
35. 2; Some pots are belts (I) + No belt is a
94. 4; ? = 16.4 × 7.75 × 12 = 127.1 × 12 Then, xy = 11664 ... (ii)
shirt (E) = I + E = O = Some pots are not
shirts. Hence, conclusion I does not follow. = 1525.2 From eqn (i) and (ii), we get
Again, No belt is a shirt  conversion  No x = 144, y = 81
95. 4; ? = 3 68921  3 17576  41  26  67
shirt is a belt. Hence, conclusion II follows.  Perimeter of the rectangle = 2(144 + 81)
(36-40): 96. 1; 7? = (75)–2 × (72)7 × (7)–3 = 450m
= (7)–10 + 14 – 3 = (7)1 105. 3; The series is –133 + 3, –113 – 3,
?=1 –93 + 3, –73 – 3, ...

KUNDAN
n
1550 1125  r 
97. 2; ? = 24.8 × – 13.2  106. 2; CI = P1   –P
36. 2 37. 4 38. 1 39. 3 40. 5 100 100  100 
41. 3 42. 4 43. 2 44. 5 45. 2
= 384.4 – 148.5 = 235.9
46. 3 47. 4 48. 2 49. 1 50. 5 2
51. 1; Replace ‘involved’ with ‘involvement’ 4  5  8  7623  111 
98. 3; ? =  1760 Now, 110000    – 110000
or ‘being involved’ 7  9  11  100 
52. 3; Replace ‘lives’ with ‘life’
48  325 987 12.5  ? = 135531 – 110000 = `25531
53. 3; Replace ‘elude’ with ‘eludes’
99. 2;  =
54. 2; Replace ‘adapt’ with ‘adept’ 100 47 100 100
55. 3; Replace ‘this’ with ‘that’ 107. 2; Cost price = 1652 × = `1400
118
56. 5 57. 2 58. 4 59. 1 60. 3 12.5  ?
or,  156  121  177 If SP becomes `1512,
61. 3 62. 2 63. 5 64. 3 65. 2 100
(66-70): DACFEB then % profit
66. 5 67. 2 68. 4 69. 1 70. 3 17700
?=  1416 1512 – 1400 112
71. 4 72. 2 73. 5 74. 3 75. 1 12.5 = × 100 = = 8%
1400 14
76. 4 77. 2 78. 5 79. 1 80. 3
888 108. 4; Let the number be x.
81. 4; 100. 4;  6084  4356
7.84  10.24 = 2.8 × 3.2 = 8.96 ? 72 x 7 x
82. 1; 73174 + 43156 – 29617 – 31619 Then, – = –125.8
= 78  66 = 144 = 12 100 5
= 116330 – 61236 = 55094
83. 3; ? = 888 18x 7 x
2209 × 11.5 = 47 × 11.5 = 540.5 or, – = –125.8
?= = 74 25 5
12
192  267  357 or, –17x = –125.8 × 25
84. 2; ? = = 72624 p r t
23 6 7  x = 185
101. 2; SI =
85. 3; ? = 319.24 + 431.65 – 106.89 – 100 40
240.33 = 750.89 – 347.22 = 403.67 Now, Now, 40% of x = 185 × = 74
100
48 1735 36  1545
86. 5; ? =  11205  r  7 109. 4; Let the cost of a pen and a pencil be
100 100 4706.1 = `x and `y respectively.
100
= 832.8 + 556.2 = 1389
Then, 16x + 20y = 124 ... (i) 131. 3 132. 5 133. 4 134. 5 135. 5
24
7 = = 12 kmph 136. 4 137. 1 138. 2 139. 1 140. 2
2
On multipling eqn (i) by , we get 141. 3 142. 1 143. 1 144. 2 145. 2
4
1 2 8 146. 5 147. 2 148. 1 149. 3 150. 1
7 7 7 117. 3; = 0.333, = 0.285, = 0.421 151. 2 152. 2 153. 4 154. 4 155. 5
16x × + 20y × = 124 × 3 7 19
4 4 4 156. 5 157. 1 158. 1 159. 4 160. 4
or, 28x + 35y = `217 5 4 161. 2
= 0.357, = 0.363 162. 4; Format drive name is used to format
110. 1; Total age of 12 persons = 12 × 32 14 11
= 384 years 118. 3; Let the length of the rectangle be a drive through command prompt.
Total age of (12 + 1) persons = 13 × 31 x m. 163. 2 164. 5 165. 2 166. 1 167. 1
= 403 years 168. 5 169. 3
Now, age of new person = 403 – 384 69 170. 1; Three types of data, ie number/char-
Then, x = – 13.5 = 34.5 – 13.5 = 21m
= 19 years 2 acter, text and formulas can be entered in a
111. 3; Population of the city in 2013  Area of the rectangle = 21 × 13.5 worksheet.
= 283.5 sq m = Area of the circle. 171. 4; Wireless Application Protocol (WAP)
110 110 110
= 22 × × × = 29.282 lakh Hence, area of the circle = 283.5 sq m is used in mobile phones. A mobile phone has
100 100 100 a built-in screen for wireless access to email
112. 4; Length of the train = 175m 119. 5; For 200 men food left for 45 – 24
and web.
Relative speed = 58 + 5 = 63 kmph = 21 days
172. 3 173. 3 174. 1 175. 2 176. 4
For 1 man food left will last for
5 177. 3 178. 1 179. 3 180. 3 181. 4
= 63 m/s 21 × 200 days.
18 182. 2 183. 1 184. 2 185. 2 186. 4

K
21 200 187. 4 188. 3 189. 3 190. 2 191. 5
Time taken by the train the cross the man For 105 men food left will last for
105 192. 4 193. 3 194. 3 195. 5 196. 3
175 175  18 = 40 days 197. 2 198. 5 199. 1 200. 4
= 5 = 63  5 = 5 × 2 = 10 seconds
63  1
18
120. 1; A’s 1 day’s work =
36
100
113. 5; Cost price = 5670 × = `8100 1
70
B’s 1 day’s work =
114. 4; Diameter of the circle = d 30
= 2r = 33.6 cm
1
 r = 16.8 cm Now, (A + B + C)’s 1 day’s work =
 Area of the circle = r 2 9

KUNDAN
22 1  1
= × 16.8 × 16.8 = 887.04 C’s 1 day’s work = – –
7 9 36 30
115. 3; Area of four walls
= 2 × (length + breadth) × height 60 – 15 – 18 27 1
= = =
= 2 × (15 + 9) × 4 540 540 20
= 2 × 24 × 4 = 192 sq m  C can complete the work in 20 days
 Cost of painting = 8.40 × 192 = `1612.8 121. 2 122. 3 123. 3 124. 2 125. 2
116. 4; Let the speed upstream be x kmph 126. 2 127. 1 128. 4 129. 1 130. 1
and downstream be y kmph.
42 68
Then, x  y = 10

or, 21u + 34v = 5 ... (i)


1 1
(where = u, y = v)
x

56 85
Again, x  y  13

or, 56u + 85v = 13 ... (ii)


1 1
(where  u, v)
x y
Solving eqn (i) and (ii),
1 1
u= and v =
7 17
 x = 7 kmph and y = 17 kmph
7  17
 Speed of person in still water =
2
Clerk Mock Test 8
9. How many such consonants are there in the above
Test-I: Reasoning Ability

K
arrangement each of which is not immediately preceded
by a symbol but followed by a vowel?
Directions (Q. 1-5): Study the following information 1) One 2) Two 3) Three
carefully and answer the given questions: 4) Four 5) None of these
Eight friends A, B, C, D, E, F, G and H are sitting around 10. How many such vowels are there in the above
a circle facing the centre, but not necessarily in the same arrangement each of which is immediately preceded
order. There are two female members in the group. by a number?
C sits third to the left of A and second to the right of E, 1) One 2) Two 3) Three
who is not a male member. B is second to the right of C. D 4) Four 5) None of these
is second to the right of G, who is second to the right of A. Directions (Q. 11-15): In these questions, a
There are three members between two female members. F relationship between different elements is shown in the

KUNDAN
is not on the immediate right of C. statements. The statements are followed by two
1. Who among the following sits on the immediate right conclusions. Give answer
of C? 1) if only conclusion I is true.
1) G 2) H 3) B 2) if only conclusion II is true.
4) D 5) None of these 3) if either conclusion I or II is true.
2. Which of the following pairs are immediate neighbours 4) if neither conclusion I nor II is true.
of H? 5) if both conclusions I and II are true.
1) C, B 2) E, C 3) A, G 11. Statements: K  L > M  N < P, Q = K
4) F, B 5) None of these Conclusions: I. Q > M II. P > L
3. Which of the following pairs represents female 12. Statements: R  S = T  E, L = J > E
members? Conclusions: I. L < T II. J > R
1) A, E 2) D, B 3) F, C 13. Statements: W > H = A < T, E > B  W
4) B, E 5) None of these Conclusions: I. T > W II. H  E
4. Who among the following is sitting opposite A? 14. Statements: M  Q = R, P > S  N < R
1) C 2) E 3) D Conclusions: I. Q > S II. M  N
4) F 5) None of these 15. Statement: D> EF =G H>I
5. Who among the following is third to the right of B? Conclusions: I. E  G II. H  F
1) D 2) E 3) G 16. How many such pairs of letters are there in the word
4) F 5) None of these DEVISED each of which has as many letters between
Directions (Q. 6-10): Study the following arrangement them in the word as in the English alphabetical series?
carefully and answer the questions given below: 1) None 2) One 3) Two
A% DFKE 8JQ 1A VT U2$W #6BG @ IL7 3H 4) Three 5) None of these
6. If all the symbols are dropped from the above 17. Among D, E, F, G and H, each having a different height,
arrangement, which of the following will be eleventh F is taller than only D and G is shorter than E and taller
from the right end? than H. Who among the following is the tallest?
1) W 2) 2 3) U 4) T 5) None of these 1) E 2) H 3) G 4) F 5) None of these
7. How many such letters are there in the above 18. In a certain code PIPE is written as ‘5954’ and REST is
arrangement each of which is immediately followed written as ‘8426’. How is SITE written in that code?
by a number? 1) 2468 2) 9526 3) 2964
1) None 2) One 3) Two 4) 2694 5) None of these
4) Three 5) None of these 19. In a certain code language FEMALES is written as
8. How many such symbols are there in the above OFHBUFN. How is ORDINAL written in that code
arrangement each of which is immediately preceded language?
by a consonant and followed by a vowel? 1) NBPFSQJ 2) PSEJOBM 3) FSQJPBN
1) None 2) One 3) Two 4) FSQJNBP 5) None of these
4) Three 5) None of these 20. If red means orange, orange means blue, blue means
green, green means black, black means white, and 1) Colgate 2) Vicco 3) Pepsodent
white means pink, then what is the colour of a crow? 4) Closeup 5) None of these
1) Orange 2) White 3) Pink 27. Which of the following toothpastes is displayed on
4) Blue 5) None of these the immediate left of Pepsodent?
Directions (Q. 21-25): In each question below are 1) Closeup
given two statements followed by two conclusions 2) Either Closup or Oral-B
numbered I and II. You have to take the given statements 3) Oral-B
to be true even if they seem to be at variance with 4) Babul
commonly known facts. Read all the conclusions and then 5) None of these
decide which of the given conclusions logically follows 28. At which of the following screen numbers is
from the given statements, disregarding commonly Pepsodent displayed?
known facts. Give answer 1) 2 2) 4 3) 3
1) if only conclusion I follows. 4) 5 5) None of these
2) if only conclusion II follows. 29. If the positions of Closeup and Oral-B are interchanged
3) if either conclusion I or II follows. then which of the following toothpastes is displayed
4) if neither conclusion I nor II follows. at screen number 2?
5) if both conclusions I and II follow. 1) Oral-B 2) Pepsodent 3) Babul
21. Statements: All walls are floors. 4) Colgate 5) None of these
All floors are windows. 30. Which of the following pairs of toothpastes are
Conclusions: I. All windows being walls is a displayed on screen number 4 and 5 respectively?
possibility. 1) Babul, Vicco 2) Pepsodent, Oral-B
II. Some floors are walls. 3) Colgate, Closeup 4) Oral-B, Vicco
22. Statements: All tables are desks. 5) None of these
Some desks are benches. Directions (Q. 31-35): The following questions are
Conclusions: I. Some tables are benches. based on the diagram give below:
II. Some benches are desks.
23. Statements: All purses are rings.
No ring is a hand.

K
Conclusions: I. Some hands being purses is a
possibility.
II. No purse is a hand.
24. Statements: No scooter is a bike. 31. Which of the following groups represents the group
No truck is a bike. of reporters of Outlook and Frontline but not that of
Conclusions: I. No scooter is truck. India Today?
II. Some bikes are truck. 1) F, D 2) A, K, D, F 3) J, A, D, C
25. Statements: Some watches are needles. 4) C, F, D, K 5) None of these
All needles are walls. 32. Which of the following represents the group of
Conclusions: I. Some watches are walls. reporters of India Today, Frontline and Outlook?

KUNDAN
II. Some walls are needles. 1) H 2) E 3) F
Directions (Q. 26-30): Study the following information 4) I 5) None of these
carefully to answer the given questions: 33. Which of the following groups represents reporters
There are six types of toothpaste – Pepsodent, Babul, of only Frontline?
Colgate, Closeup, Oral-B and Vicco – displayed on six 1) B, I, G 2) C, F, J 3) B, C, I
window screens numbered 1 to 6 at a shop from left to 4) J, C, B 5) None of these
right for advertisement as per company requirements. 34. Which of the following statements is true?
Closeup and Colgate are displayed next to each other. There 1) G is the group of only Outlook.
are three types of toothpaste displayed between Colgate 2) E is the group of only India Today and Frontline.
and Vicco. Peposodent is displayed between Oral-B and 3) H is the group of only Outlook and India Today.
Closeup but three windows away from Babul, which cannot 4) C and J make the group of India Today.
be displayed next to Closeup. Neither Babul nor Vicco can 5) None of these
be displayed at window 1. 35. Which of the following represents the group of
26. Which of the following toothpastes is between Oral- reporters of only India Today and Frontline?
B and Babul? 1) G, B 2) I, C 3) A 4) F, D 5) I
Directions (Q. 36-38): Study the following information ICICI and Larsen & Toubro, for example. The problem here is
carefully to answer the given questions: that there are so few of them and the CEO tenure in these firms
All the letters of English alphabet are numbered in is so long that it creates few opportunities for new leaders to
reverse order, viz 26 for A, 25 for B, and so on. These rise.
numbers are used to denote the corresponding letters in Our many public sector companies emasculate their
the questions. leaders so much that no competent professional would
36. Which of the following numbers denotes the code for seriously consider leading these important behemoths.

K
a meaningful word? Finally, there are multinationals like HUL, Suzuki and
1) 15 – 26 – 14 – 25 2) 21 – 17 – 18 – 9 Samsung. In these firms, most important decisions are made
3) 13 – 7 – 26 – 5 4) 5 – 12 – 17 – 23 outside India and so, a promising leader has to leave India
5) None of these and get back to headquarters to rise. So, it is indeed true
37. Which of the following numbers denotes the sum of that India is still a small pond for an ambitious and talented
the codes of letters of VOTE? professional manager. Hopefully, as Indian firms globalise
1) 50 2) 62 3) 46 4) 48 5) None of these and professionalise and more entrepreneurial firms achieve
38. What will be the sum of the numbers denoted by all scale, this will change. But in the short term, the best
the vowels in the alphabet? opportunities for the very best talent are still outside India.
1) 51 2) 56 3) 80 4) 84 5) None of these Another question worth asking is whether a non-
Directions (Q. 39-40): Four of the following five are Indian immigrant could have risen to the top of one of our

KUNDAN
alike in a certain way and hence from a group. Which is iconic firms. The strength of the US is that it is able to
the one that does not belong to that group? attract and assimilate immigrants of incredible ability. Intel,
39. 1) Habitual 2) Chronic 3) Routine Google and Yahoo! were all started by immigrants.
4) Seldom 5) Natural Indian immigrants run important firms such as
40. 1) Departure 2) Reaching 3) Gate Microsoft, Master Card and Pepsi. But how attractive and
4) Approach 5) Entrance open is India to global talent?
Would, and could, a brilliant Bangladeshi, Nepali or
Test-II: English Language Sri Lankan make it to the top in India? Could an American
or European be a future CEO of Mahindra, Airtel or Infosys?
Directions (Q. 41-50): Read the following passage Globally, business success is increasingly driven by
carefully and answer the questions given below it. Certain innovation and entrepreneurship and skilled talent do
words/phrases in the passage are printed in bold to help disproportionately well.
you locate them while answering some of the questions. As Indian companies try to succeed globally, they
When we Indians are starved of things to feel proud must become more open to talent carrying different
about, the appointment of Satya Nadella as the CEO of the passports. India will need to examine immigration policies
iconic Microsoft has given us a reason to take pride in the to welcome skilled professionals.
success of a fellow Indian. For all our complaints about the US’ restrictions on
Not only is Satya Indian by birth, he went to ordinary immigration of skilled workers, we ourselves remain quite
schools and colleges, got to the top on his own merit and, closed. If we could make India a less challenging place to
most of all, remained a nice, normal and humble guy. We do business and if we could become more welcoming of
can relate to Satya and his journey in a way that we can’t high-end talent regardless of nationality we would reverse
relate to, say, Steve Jobs or Bill Gates, and that’s what is so the brain drain and become a magnet for innovators and
inspiring. In his success, we see the possibility of our own entrepreneurs who would revitalise our economy in
success. At a time where young people are looking for role unimaginable ways.
models to emulate, Satya is certainly a wonderful one. 41. In what way can Satya Nadella be a help for Indian
However, at least one commentator has stirred a guys?
hornet’s nest by asking if Satya’s success is, in fact, a slap 1) Satya being a fellow of Indian origin can help Indian
in our face. Could Satya have become the CEO of a major students out of way.
Indian company? Or did he have to leave the country to 2) Satya can be a role model and inspiration for young
succeed? Corporate India is dominated by family Indians.
businesses. The right genes are still an important requisite 3) He can help Indian IT industry by imparting special
for ultimate success. But this is changing slowly training to Indian technocrats.
Even promoters are slowly ceding the CEO job to loyal 4) He can help set up more Microsoft offices in India.
professionals. There are a handful of important companies 5) None of these
where shareholding is diversified and that have had 42. What prompted one of the commentators to ask if
professional CEOs for a long time: the Tata group, HDFC, Satya’s success was, in fact, a slap in our face?
1) India is a developing country and the people 1) neglect 2) mock 3) mimic
belonging to lower-middle class do not get 4) formulate 5) follow
appropriate opportunity to develop their skills. 47. Emasculate
2) There is no scope of promotion for a poor fellow, 1) strengthen 2) debilitate 3) assist
however intelligent he may be. 4) help 5) aid
3) Corporate India is dominated by family business and 48. Behemoths
no person other than one from the family can easily 1) brothers 2) guys 3) giants
reach the top position of that corporate office. 4) leaders 5) scholars
4) In India blood relation is given more importance Directions (Q. 49-50): Choose the word/group of
than intelligence. words which is MOST OPPOSITE in meaning of the word/
5) All the above group of words printed in bold as used in the passage.
43. Which of the following is/are not true in the context of 49. Ceding
the given passage? 1) conceding 2) capitulating 3) waiving
(A) A handful of important companies such as Tata 4) holding 5) conveying
group, HDFC, ICICI and L & T have had 50. Assimilate
professional CEOs for a long time. 1) exclude 2) comprehend 3) grasp
(B) Now, promoters are rapidly allowing loyal 4) incorporate 5) take in
professionals to reach the top position of CEOs. Directions (Q. 51-55): Read each sentence to find out
(C) The tenure of the CEO in companies like HDFC, whether there is any grammatical or idiomatic error in it.
ICICI and Tata group are so long that it creates The error, if any, will be in one part of the sentence. The
few opportunities for new leaders to rise. number of that part is the answer. If there is ‘No error’,
1) Only (A) and (B) 2) Only (B) and (C) the answer is 5). (Ignore errors of punctuation, if any.)
3) Only (B) 4) Only (A) 51. 1) Hafiz Mohammad Saeed, the leader of internationally
5) Only (C) recognised terror outfit / 2) Laskar-e-Taiba, which
44. Why has it been said that India is still a small pond for carried on horrific attack / 3) in Mumbai, was placed /
an ambitious and talented manager? 4) under house arrest in Lahore. / 5) No error
1) Because most of the multinational companies take 52. 1) The group of four emerging countries in terms of
most important decisions in the country of their economy / 2) has set June 2015 as the next date / 3) by

K
origin and the promising leader has to go there and which a consensus / 4) could be arrived. / 5) No error
come back to India if they want to rise. 53. 1) With the Supreme Court declining to hear his plea /
2) Because most of the organisations do not want to 2) to recall its non-bailable arrest warrant, / 3) Sahara
give chance even to a talented Indian manager. Chief Subrata Roy surrendered to the Lucknow police
3) Because Indian companies are not so advanced / 4) and arrested in connection with the contempt case.
and hence there is very little opportunity of growth / 5) No error
here even for talented managers. 54. 1) Law Minister Kapil Sibal along with ministers Jairam
4) Because the mindset of Indian corporate business Ramesh and V Narayan Swamy / 2) were called to the
houses is such that they prefer foreign talents to meeting / 3) chaired by Congress president Sonia
Indian talent. Gandhi / 4) at Prime Minister Manmohan Singh’s

KUNDAN
5) None of these residence. / 5) No error
45. What suggestions has/have been made by the author 55. 1) Obama administration wants / 2) its own preferred
for Indian companies trying to succeed globally? candidate / 3) Arseniy Yatsenyuk, the leader of the
1) Indian companies should open their branches Fatherland party, / 4) to lead Ukraine. / 5) No error
abroad. Directions (Q. 56-60): In each of these questions, two
2) Indian companies should urge the Govt of India sentences I and II are given. Each sentence has a blank in
for relaxing its immigration policy. it. Five words 1), 2), 3), 4) and 5) are suggested. Out of
3) Special provisions should be made for foreign these only one fits at both the places in the context of each
companies. sentence. The number of that word is the answer.
4) Indian companies must be more open to talented 56. I. Odisha Police claimed to have arrested a senior
professionals. Maoist leader close to a central committee member
5) All the above who is allegedly involved in several attacks,
Directions (Q. 46-48): Choose the word/group of including last year’s ________ on a BSF convoy.
words which is MOST SIMILAR in meaning to the word/ II. The gunmen, lying in ________, opened fire, killing
group of words printed in bold as used in the passage. the driver.
46. Emulate 1) invasion 2) ambush 3) hiding
4) camouflage 5) pitfall 64. After a relationship ________ over one-and-a-half
57. I. All that a tourist has to do is to submit an application centuries, on July 14, Kolkata bid an emotional
online and, within two to three days, a travel ________ to a mode of communication called the
authorisation will be mailed ________. telegram.
II. I predict that the council will ________ water on 1) traversing, farewell 2) extending, relation
the tax issue. 3) spanning, adieu 4) ranging, goodbye
1) again 2) then 3) these upon 5) covering, hello

K
4) back 5) throw 65. _______ the defeat of Tipu Sultan in the Fourth Anglo
58. I. During his interview with Times Now, Rahul Gandhi Mysore War (1798-99) in Srirangpatnam, the British
did not look ________ into the camera. army ________ itself in the vicinity of the old city
II. All her efforts to keep him to the ________ and area of Bangalore.
narrow have been rewarded. 1) By, opened 2) With, garrisoned
1) straight 2) forward 3) direct 4) front 5) good 3) At, guarded 4) On, armoured
59. I. After living under a military regime for more than 5) For, left
seven years, the Fijians are ________ visible Directions (Q. 66-70): Rearrange the following seven
enthusiasm for elections. sentences (A), (B), (C), (D), (E), (F) and (G) in the proper
II. Among the protesters and war veterans proudly sequence to form a meaningful paragraph and then answer
________ their medals was Arvind.

KUNDAN
the questions given below.
1) disparaging 2) expressing 3) displaying (A) The procuring of fruits, berries, greens and seeds
4) floundering 5) exclaiming was supplemented by some hunting.
60. I. At a time when the global economy is still (B) As an aid in this search for food, primitive people
________ under the impact of the economic invented weapons and tools.
slowdown, the financial sector in India has kept (C) In places where animals were abundant, the
the economy afloat. technique of hunting became highly developed.
II. I stood up and almost fell, ________ against the (D) Man moved forward again and he learned to
wall. domesticate animals, particularly the big animals
1) rushing 2) facing 3) suffering such as cattle.
4) almost 5) reeling (E) In the primitive days man satisfied his hunger by
Directions (Q. 61-65): In each of the following searching for food and living upon what he could
sentences there are certain blank spaces. Below each raise.
sentence there are five options denoted by the numbers 1), (F) Most of the hunting people had already
2), 3), 4) and 5). Find out which option can be used to fill up domesticated the dog, which aided in the hunt
the blank(s) in the sentence in the same sequence to make and also helped somewhat in transportation.
it meaningfully complete. (G) The hunting people did not go separately but
61. Sikkim was ________ seriously by the September 2011 rather lived in small groups and moved in a group
earthquake, which measured 6.9 on the Richter for hunt.
________. 66. Which of the following should be the FIRST sentence
1) effected, range 2) damaged, metre after rearrangement?
3) haunted, calibration 4) affected, scale 1) A 2) C 3) D 4) B 5) E
5) blemished, system 67. Which of the following should be the SECOND
62. India has a vibrant publishing scene, thanks to not sentence after rearrangement?
only the big publishers but also a lot of quality- 1) F 2) B 3) A 4) C 5) G
________ little publishing houses that ________ it 68. Which of the following should be the LAST
going. (SEVENTH) sentence after rearrangement?
1) conscious, keep 2) responsive, put 1) C 2) A 3) G 4) F 5) B
3) apprised, retain 4) mindful, retain 69. Which of the following should be the FIFTH sentence
5) cognisant, enjoy after rearrangement?
63. The staunch proponents of economic liberalisation 1) G 2) D 3) B 4) A 5) C
have often ________ that the relaxation of state 70. Which of the following should be the THIRD sentence
________ will lead to phenomenal growth of after rearrangement?
industries spurred by FDI. 1) E 2) F 3) A 4) G 5) D
1) contended, neglect 2) jumped, inability Directions (Q. 71-80): In the following passage, some
3) hassled, domination 4) crossed, dominion of the words have been left out, each of which is indicated
5) argued, control by a number. Find the suitable word from the options given
against each number and fill up the blanks with 84. 84368 + 65466 – 72009 – 13964 = ?
appropriate words to make the passage meaningfully 1) 61481 2) 62921 3) 63861
complete. 4) 64241 5) None of these
Where do new ideas come from? How do they get put 85. 12% of 555 + 15% of 666 = ?
into action? How can we create social structures that (71) 1) 166.5 2) 167.5 3) 168.5 4) 169.5 5) None of these
cooperative, productive and creative? These are perhaps
86. 142 + ? + 271 = 490
the most critical (72) for any society and they are especially
important (73) because of global competition, 1) 5041 2) 5329 3) 5929 4) 6241 5) None of these
environmental challenges and the (74) of rot from within. 87. 782 ÷ 1.7 – 372 = ?
In the last few years, our lives have been (75) by 1) 94 2) 88 3) 82 4) 76 5) None of these
networks combining people and computers, allowing 88. 2209  3969  12.5% of ?
greater (76) and faster change... Today, virtual crowds can
1) 1040 2) 960 3) 880 4) 720 5) None of these
form in minutes and consist of millions of people from all
over the world. And with (77) new day, it may be a different 3
set of millions of people contributing and commenting. 89. 8 of 1365  ?  3
5
To (78) our new world, we must extend familiar 1) 3163 2) 3683 3) 3353 4) 3913 5) None of these
economic and political ideas to (79) the effects of these
millions of people learning from one another and (80) one 3 4 6
90. of of of 9240 = ?
another’s opinions. 5 7 11
71. 1) is 2) was 3) are 1) 1728 2) 1536 3) 1872 4) 2016 5) None of these
4) has 5) had 91. 1.2% of 1225 + 2.4% of 975 = ?
72. 1) apprehensions 2) doubts 3) wishes 1) 36.3 2) 37.4 3) 38.1 4) 39.2 5) None of these
4) quest 5) questions 92. ? ÷ 3.6 = 780
73. 1) now 2) then 3) still 1) 2712 2) 2808 3) 2848 4) 2936 5) None of these
4) yet 5) present 93. (1.3 × 3.3 × 7.3) ÷ 0.3 = ?
74. 1) blackmail 2) scent 3) thunder 1) 102.81 2) 103.63 3) 104.39
4) threat 5) hazard 4) 105.54 5) None of these
75. 1) altered 2) transformed 3) mutated

K
94. 14.44  380.25  ?
4) restructured 5) reconstructed
76. 1) blockage 2) concurrence 1) 73.4 2) 74.1 3) 75.8 4) 76.2 5) 77.5
3) encouragement 4) attendance 95. 164% of 735 – 18.4 = ?
5) participation 1) 1173 2) 1181 3) 1187 4) 1193 5) 1197
77. 1) each 2) any 3) particular 96. A shopkeeper buys a cycle for ` 815 and he spends
4) individual 5) personal ` 145 on its repair. If he sells the cycle for ` 1296, then
78. 1) get 2) sense 3) apprehend what is his percentage profit?
4) understand 5) grasp 1) 15% 2) 20% 3) 25% 4) 30% 5) 35%
79. 1) bear 2) embrace 3) include 97. A person purchased an item and sold it at a loss of
4) admit 5) hold 15%. Had he bought it for 25% less and sold it for

KUNDAN
80. 1) creating 2) influencing 3) impacting ` 117 more, he would have earned a profit of 35%. What
4) fluctuating 5) accessing is the cost price of the item?
1) ` 640 2) ` 720 3) ` 750 4) ` 800 5) ` 840
Test-III: Quantitative Aptitude 98. ` 1450 amounts to ` 1740 in 5 years at simple interest.
If the interest rate is increased by 4%, how much would
Directions (Q. 81-95): What value should come in it amount to?
place of question mark (?) in the following equations? 1) ` 1840 2) ` 1960 3) ` 2030
81. 28% of 85 + 13.2 = ? 4) ` 2120 5) ` 2150
1) 35 2) 37 3) 39 4) 41 5) None of these 99. The present population of a city is 24 lakh. If the annual
birth rate and death rate are 6.8% and 1.8%
7 3 respectively, then what will be its population after 3
82. of 58 + of 139.2 = ?
5 8 years?
1) 133.4 2) 137.2 3) 127.8 4) 131.6 5) None of these 1) 26.436 lakh 2) 26.814 lakh 3) 27.342 lakh
3 4) 27.783 lakh 5) 28.014 lakh
83. 17576  3 4096  ?
100. What is the principal, if the compound interest
1) 42 2) 44 3) 46 4) 48 5) None of these compounded annually at the rate of 10% pa for three
years is ` 3641? 112. In how many different ways can the letters of the word
1) ` 10000 2) ` 10500 3) ` 11000 LIGHT be rearranged?
4) ` 11500 5) ` 12000 1) 36 2) 120 3) 35 4) 119 5) None of these
101. Three-sevenths of five-elevenths of four-thirteenths 113. When 134 is substracted from a number it is reduced
of a number is 960. Then what is the number?
7
1) 16016 2) 16412 3) 16640 4) 16750 5) 16910 to of its value. What is the number?
102. A and B together can do a piece of work in 16 days 9

K
and A alone can do it in 80 days. In how many days 1) 621 2) 603 3) 585 4) 567 5) 549
can B alone do it? 114. If the length of a rectangle is increased by 24% and its
1) 20 days 2) 24 days 3) 32 days breadth is decreased by 10%, then by what per cent
4) 40 days 5) None of these will the area of the rectangle be increased/decreased?
103. Three pipes A, B and C can fill a tank in 4 hours. After 1) 9.8% 2) 11.6% 3) 12.4% 4) 14% 5) 16%
working together for 2 hours, C is closed, and A and B 115. A brick measures 25 cm by 15 cm by 5 cm. How many
fill the tank in 10 hours. Then how much time will C bricks will be required to make 40m long, 1.5m high
alone take to fill the tank? and 1m thick wall?
1) 4 hours 2) 5 hours 3) 8 hours 1) 32000 2) 36000 3) 40000
4) 10 hours 5) 12 hours 4) 42000 5) 48000
104. Rajeev alone can do a work in 45 days but with the 116. How many spheres of diameter 2 cm can be made by

KUNDAN
help of Kisan he can do the same work in 20 days. If melting a metallic cylinder whose height is 35 cm and
they get ` 2925 for the work, then what will be the radius 8 cm?
share of Kisan? 1) 1248 2) 1420 3) 1680 4) 1792 5) 1812
1) ` 1225 2) ` 1300 3) ` 1475 117. In the following number series, one number is wrong.
4) ` 1625 5) ` 1750 Find out that number.
14, 15, 34, 116, 460, 2325
2548 ? 1) 15 2) 34 3) 116 4) 460 5) 2325
105. 
? 1872 118. The average age of six students of a class was 17.5
1) 2048 2) 2184 3) 2224 4) 2316 5) 2496 years. When one student left the class the average
106. A student goes to school at a speed of 6 kmph and age became 16 years. What is the age of the student
returns home at a speed of 4.5 kmph. If he takes 10.5 who left the class?
hours in all, then how far is the school from his home? 1) 15 years 2) 20 years 3) 24 years
1) 18 km 2) 21 km 3) 24 km 4) 27 km 5) 36 km 4) 25 years 5) None of these
107. A table is sold at a profit of 12%. Had it been sold for 119. ` 41517 is to be distributed amongst A, B and C in the
` 50 more, then there would have been a profit of 20%. ratio of 3 : 7 : 11. What is the share of B?
What is the cost price of the table? 1) ` 5931 2) ` 9885 3) ` 13839
1) ` 625 2) ` 650 3) ` 675 4) ` 700 5) ` 725 4) ` 17793 5) None of these
108. Two trains were running in opposite directions at the 120. A dealer sold a DVD player at a loss of 7%. Had he
speed of 48 kmph and 24 kmph respectively. If the sold it for ` 234 more, he would have gained 11%. For
faster train passed a man sitting in the slower train in what value should he sell it in order to gain 16%?
9.5 seconds, what is the length of the faster train? 1) ` 1484 2) ` 1508 3) ` 1248
1) 180m 2) 190m 3) 210m 4) ` 1300 5) ` 1624
4) 240m 5) None of these
109. What value should come in the place of (?) in the Test-IV: General Awareness
following series?
(with Special Reference to Banking Industry)
8, 16, 38, 86, 178, 338, ?
1) 512 2) 536 3) 548 4) 572 5) 596 121. Name the Rajya Sabha MP who has been indicted by
110. What is the difference between a discount of 35% on a US jury in an alleged international conspiracy to
` 1800 and two successive discounts of 20% and 15% bribe state and central govt officials in India to allow
on the same amount? mining of titanium minerals.
1) ` 50 2) ` 32 3) ` 54 4) ` 78 5) None of these 1) KVP Ramachandra Rao 2) GV Harsha Kumar
111. A bag contains 7 red, 5 blue and 3 black balls. If two 3) SPY Reddy 4) K Keshava Rao
balls are drawn at random, what is the probability that 5) T Subbarami Reddy
both of them are blue? 122. Karim Massimov has been appointed the new Prime
2 2 1 1 5 Minister of which of the following countries after Serik
1) 2) 3) 4) 5)
105 21 12 21 21
Akhmetov resigned recently? Chambers of Commerce and Industry (FICCI), India’s
1) Kazakhstan 2) Kyrgyzstan economy is likely to grow at what per cent in 2014-15?
3) Tajikistan 4) Afghanistan 1) 3.5 per cent 2) 4.5 per cent 3) 5 per cent
5) None of these 4) 5.5 per cent 5) 6 per cent
123. The Sports Ministry has derecognised which of the 133. Who among the following won the men’s singles title
following sports associations with immediate effect of the 2014 Miami Masters?
after it refused to comply with ministry’s directives? 1) Rafael Nadal 2) Roger Federer
1) Indian Amateur Boxing Federation 3) Novak Djokovic 4) Stanislas Wawrinka
2) Badminton Association of India 5) Juan Martin del Potro
3) All India Tennis Association 134. Which of the following taglines is NOT associated
4) Archery Association of India with the State Bank of India (SBI)?
5) None of these 1) Pure banking, nothing else
124. Which of the following risks is associated with failure 2) The name you can bank upon
of internal process of a bank or business organisation? 3) A bank of the common man
1) Credit risk 2) Operational risk 4) The banker to every Indian
3) Procedural risk 4) Settlement risk 5) With you all the way
5) None of these 135. STCI Finance Limited was promoted in May 1994 with
125. Doha Bank, which got permission from the Reserve the objective of fostering an active secondary market
Bank of India (RBI) to open the branch in Dec 2013, is in Govt of India securities and public sector bonds by
all set to open its first branch in which of the following which of the following bodies?
cities? 1) IRDA 2) SEBI 3) RBI
1) Mumbai 2) New Delhi 4) IBA 5) None of these
3) Hyderabad 4) Thiruvananthapuram 136. The Supreme Court, in its recent verdict, has held that
5) Chennai the Comptroller and Auditor General (CAG) is
126. Amnesty International released the Global Death empowered to audit which of the following types of
Penalty Report 2013 in London recently. Which of the companies as they held national wealth?
following countries tops the list of leading executioners 1) Telecommunication Companies
during the period of 2007-2013? 2) Infrastructure companies

K
1) USA 2) Saudi Arabia 3) Iraq 3) Cement manufacturers
4) Iran 5) China 4) Airlines
127. Which of the following teams won the women’s title 5) None of these
of the 64th National Basketball Championships 137. Jens Stoltenberg will take over as new Secretary
recently? General of the 28-nation military alliance NATO in Oct
1) Chhattisgarh 2) Railways 3) Tamil Nadu this year. He is the former premier of which of the
4) Punjab 5) None of these following countries?
128. In a revamp of the cricket establishment BCCI, the 1) Denmark 2) Norway 3) Croatia
Supreme Court handed it over to who among the 4) Bulgaria 5) Belgium
following during the upcoming IPL7? 138. Which of the following teams has won the 2014

KUNDAN
1) Kapil Dev 2) Sunil Gavaskar 3) Ravi Shastri Deodhar Trophy recently?
4) Rahul Dravid 5) None of these 1) North Zone 2) West Zone 3) Central Zone
129. As per the data available with Securities and Exchange 4) South Zone 5) None of these
Board of India (Sebi), Foreign Institutional Investors 139. Which of the following bodies releases Exchange
(FIIs) have made net inflows of nearly ________ in Control Manual from time to time in the context of
the Indian stocks during fiscal year 2013-14. foreign exchange in India?
1) ` 60,000 cr 2) ` 70,000 cr 3) ` 80,000 cr 1) FIPP 2) CCI 3) SEBI
4) ` 90,000 cr 5) ` 1,00,000 cr 4) RBI 5) None of these
130. Which of the following is NOT a banking-related term? 140. When relatively high-cost debt is replaced with that
1) SWIFT 2) CAMELS 3) TRIPS of lower-cost borrowing to take advantage of falling
4) CAR 5) STRIPS interest rates, it is called
131. Who has authored the book titled Urban Villager? 1) amortisation 2) credit management
1) Pallavi Aiyar 2) Vandana Vasudevan 3) debt replacement 4) debt swap
3) Mukul Kesavan 4) Sidharth Bhatia 5) None of these
5) Malvika Singh 141. What is India’s ranking in the ‘Women in Politics Map
132. According to the survey of Federation of Indian 2014’ released by the Inter-Parliamentary Union (IPU)
and UN Women recently? 2) Initial Public Offering
1) 24th 2) 42nd 3) 73rd 4) 92nd 5) None of these 3) Rights issue
142. Which of the following countries has been suspended 4) Follow-on Public Offering
from the powerful G8 grouping recently? 5) None of these
1) Italy 2) Germany 3) Russia 151. Which of the following states has been ranked on top
4) Canada 5) None of these among the country’s 20 largest states in “Economic
143. Who among the following emerged winner of the Freedom of the States of India” report 2013? The

K
inaugural Tata T1 Prima Truck Racing Championships annually published report estimated economic freedom
held at the Buddh International Circuit recently? based on data for 2013.
1) David Jenkins 2) Mathew Summerfield 1) Madhya Pradesh 2) Haryana
3) Stuart Oliver 4) Matt Allison 3) Kerala 4) Gujarat
5) None of these 5) Tamil Nadu
144. Which of the following is NOT a part of the prudent 152. What is the expanded form of the term CAGR?
guidelines issued by the Reserve Bank of India (RBI)? 1) Calculated Annual Growth Rate
1) Loan recovery rules 2) Compounded Annual Growth Rate
2) Income recognition 3) Corresponding Annual Growth Rate
3) Provisioning requirements 4) Corrected Annual Growth Rate
4) Capital Adequacy 5) Comparative Annual Growth Rate

KUNDAN
5) None of these 153. Dada Saheb Phalke awardee VK Murthy passed away
145. In the context of banking sector, DRT is associated recently. He was a renowned
with 1) cinematographer 2) story writer
1) Recurring deposits 3) composer 4) lyrist
2) Loan recovery 5) actor
3) Deposit insurance 154. What does letter ‘A’ denote in the term LAF?
4) Demand and time liabilities of banks 1) Adequacy 2) Addition 3) Adjustment
5) None of these 4) Advance 5) Accreditation
146. Soma Mondal has been appointed the new Director 155. Which of the following is NOT considered a part of
(Commercial) of which of the following Navratna PSUs time and demand liabilities of the banks for the purpose
recently? She is the first woman employee of the of cash reserve ratio (CRR)?
company to be elevated to the post of director. 1) Paid-up capital 2) Fixed deposit
1) Bharat Electronics Limited 3) Savings deposit 4) Recurring deposit
2) National Aluminium Company Limited 5) None of these
3) Shipping Corporation of India Limited 156. Sonia Gandhi, Narendra Modi and Rahul Gandhi have
4) Rashtriya Ispat Nigam Limited been named among top 5 in the list of 100 Most
5) Oil India Limited Influential Asian People of 2014 published by Asian
147. The READ (Read-Engage-Achieve-Dream) initiative, Awards Ltd recently. Who among the following tops
aimed at improving the reading skills of primary school the list?
children in the country, is launched by agencies of 1) Park Guen-Hye 2) Pranab Mukherjee
India and which of the following countries recently? 3) Xi Jinping 4) Ban ki moon
1) South Korea 2) China 3) Japan 5) Shinzo Abe
4) US 5) Singapore 157. Which of the following pairs of players and sports
148. Who is the winner of the 2014 edition of the Templeton they are associated with is NOT matched correctly?
Prize? 1) Sanam Singh – Archery
1) Jan Sokol 2) Dominik Duka 3) Tomas Halik 2) Sharath Kamal – Table tennis
4) Miloslav Vlk 5) None of these 3) Narain Karthikeyan – Motor racing
149. In which of the following types of banking, the banks 4) Ashwini Ponnappa – Badminton
offer all types of financial services to expand their 5) None of the above
business? 158. For the Rio Olympics 2016, the Brazilian authorities
1) Narrow banking 2) Social banking have unveiled an infrastructure budget of around USD
3) Universal banking 4) Wholesale banking 10.76 bn which in its local currency will be 24.1 billion.
5) Retail banking Identify the Brazilian currency?
150. The issue of fresh securities by an unlisted company 1) Brazilian dollar 2) Brazilian Real 3) Brazilian Peso
for the first time is called 4) Brazilian Nuevo 5) None of these
1) Exchange Traded Fund 159. Who among the following has been appointed the
Secretary of the Department of Industrial Policy and 2) Digital Signature
Promotion (DIPP) recently? 3) Asymmetric Cryptosystem
1) Saurabh Chandra 2) Ruma Pal 4) Block Cypher
3) Amitabh Kant 4) Vivek Rae 5) None of these
5) None of these 169. ________ hardware technology was used by fifth-
160. Following Russia’s annexation of Ukraine’s Crimea generation computers.
region, NATO suspended all practical civilian and 1) Transistors 2) Valves 3) ICS
military co-operation with Russia recently. Which of 4) ULSI 5) VLSI
the following nations is NOT a member of the 28-nation 170. For each instruction in program memory the CPU goes
military alliance headquartered at Brussels? through a –
1) Slovenia 2) Ukraine 3) Iceland 1) decode - fetch - execute sequence
4) Estonia 5) Albania 2) fetch - decode - execute sequence
3) fetch - execute - decode sequence
Test-V: Computer Knowledge 4) execute - store - decode sequence
5) fetch - store - decode sequence
161. The right click option of mouse is NOT used in which 171. A proxy server is used for which of the following?
of the following? 1) To provide security against unauthorised users
1) To copy selected text into the clipboard 2) To process client requests for web pages
2) To rename file 3) To process client requests for database access
3) To cut the selected text 4) To provide TCP/IP
4) To access properties of an object 5) None of these
5) None of these 172. A repair for a known software bug, usually available
162. ________ is the process of dividing the disk into at no charge on the Internet, is called a(n)
tracks and sectors. 1) Version 2) Patch 3) Tutorial
1) Tracking 2) Formatting 3) Crashing 4) FAQ 5) None of these
4) Allotting 5) None of these 173. A modem performs
163. A place in the computer system where data and 1) Modulation 2) Demodulation
programs are temporarily stored is called 3) Data compression 4) All of these

K
1) paste 2) open 3) memory 5) None of these
4) pocket 5) None of these 174. Which access method is used for obtaining a record
164. Correcting errors in a program is referred to as from a cassette tape?
1) debugging 2) bugging 3) rectifying 1) Direct 2) Sequential 3) Random
4) modifying 5) None of these 4) All of the above 5) None of these
165. Graphical diagrams are used to represent multiple 175. A failure that occurs in an Oracle program when there
perspectives of a system which include is a logical failure in handling of a statement is called
1) use-case, class and state diagrams. 1) Instance failure 2) Media failure
2) state, interaction and derivative diagrams. 3) Process failure 4) Statement failure
3) interaction, relationship and class diagrams. 5) User error

KUNDAN
4) deployment, relationship and use-case diagrams. 176. Which of the following represents the size of a floppy?
5) None of these 1) 3.5 inches 2) 5.25 inches 3) 8.0 inches
166. Which is the best view for representing your thoughts 4) All the above 5) None of these
as a presentation on a computer? 177. To format a number in date format, press
1) Outline view 2) Notes page view 1) Ctrl + Shift + @ 2) Ctrl + Shift + #
3) Slide sorter view 4) Slide view 3) Ctrl + Shift + % 4) Ctrl + Shift + $
5) None of these 5) Ctrl + Shift + *
167. What does SMPS stand for? 178. ________ uses an embossed steel band to form the
1) Switched Mode Power Supply letters printed on the paper.
2) Single Mode Power Supply 1) Laser printer 2) Dot-matrix printer
3) Simple Mode Power Supply 3) Daisy wheel printer 4) Band printer
4) Synchronous Mode Power Supply 5) Ink-jet printer
5) None of these 179. OR, XOR and AND functions can be performed by
168. RSA is ________ of a computer in the CPU.
1) Symmetric Cryptosystem 1) Memory 2) CU 3) Register
4) ALU 5) Processor
180. Internet provides 1) sub-class 2) superclass 3) parent-class
1) Online comunication 4) deprived class 5) None of these
2) Software sharing 190. The capacity of program counter is
3) Customer support service 1) 8 bits 2) 14 bits 3) 16 bits
4) Online journals and magazines 4) 12 bits 5) 32 bits
5) All of the above 191. In ________-generation computer the speed was
181. System software is the set of programs that enables measured in nanoseconds.

K
your computer’s hardware devices and ________ 1) first 2) second 3) third
software to work together. 4) fourth 5) fifth
1) Management 2) Processing 3) Utility 192. The process of transferring of files from your computer
4) Application 5) None of these to the computer on the internet is called
182. ________ are specially designed computer chips that 1) downloading 2) uploading 3) FTP
reside inside other devices, such as your car or your 4) JPEG 5) downsizing
electronic thermostat. 193. Timing and control unit is a part of a(n)
1) Servers 1) ALU 2) Processor 3) Memory
2) Embedded computers 4) CMOS 5) BIOS
3) Robotic computers 194. Chip is the common name for a(n)
4) Mainframes 1) transistor 2) resistor

KUNDAN
5) None of these 3) integrated circuit 4) semiconductor
183. SMPT, FTP and DNS are applications of the ________ 5) None of these
layer. 195. ________ is developed specifically for a distinct
1) data link 2) network 3) transport industry.
4) application 4) None of these 1) Personal productivity software
184. According to which of the following, in a database, 2) Application software
should information be organised and accessed? 3) Decision support system
1) Physical position 2) Logical structure 4) Vertical market software
3) Data dictionary 4) Physical structure 5) System software
5) None of these 196. Which of the following services is not provided by
185. WORM stands for Internet service provider?
1) Wanted One Read Memory 1) Internet transit
2) Write Once Read Many 2) Domain name registration
3) Write Original Read Many 3) Hosting
4) All the above 4) Dial-up access
5) None of these 5) None of these
186. Which of the following is a separate software layer 197. A source program is written in a ________ language.
that acts as ‘glue’ between the client and the server 1) high-level 2) low-level 3) binary
parts of an application and provides a programming 4) All the above 5) None of these
abstraction? 198. The transfer rate of a standard USB 2.0 Device is
1) Freeware 2) Firmware 1) 100 M bit/s 2) 250 M bit/s 3) 480 M bit/s
3) Shareware 4) User-supported software 4) 500 M bit/s 5) 1G bit/s
5) Middleware 199. The control part of a disk is known as
187. Which of the following is a must for a computer? 1) surface 2) hub 3) cylinder
1) Chip 2) Data 3) Mouse 4) cluster 5) None of these
4) Processor 5) None of these 200. Which of the following languages is used for process
188. Disk can be used to store control?
1) Random files 2) Sequential files 3) Both 1) and 2) 1) ADA 2) COBOL 3) BASIC
4) Horizontal files 5) None of these 4) PASCAL 5) LISP
189. A derived class may also be called as
K
Combining both statements, we get
Answers RS=TE<J=L 16. 3;
(1-5): Thus, T < L or L > T is true. Hence I (L < T)
17. 1; E > G > H > F > D. Hence, E is the
is not true.
tallest among them.
Again, R < J or J > R is true.
Hence, only conclusion II is true. 18. 3; P I E R S T
13. 4; Given statements: 5 9 4 8 2 6
W> H =A<T ... (i)
So,
E>BW ... (ii)
Combining both statements, we get
1. 2 2. 1 3. 4 4. 3 5. 3 19. 4;
6. 3; New arrangement: A D F K E 8 J Q 1 V

KUNDAN
T U 2 W 6 B G I L 7 3 H.
Hence, eleventh from right end is U.
7. 5; E8, Q1, U2, L7. Hence there are four Hence, neither I nor II is true. Similarly,
such letters. 14. 1; Given statements:
8. 2; G @ I MQ=R ... (i)
9. 2; FKE, VTU P>SN<R ... (ii)
20. 5; The colour of crow is black, and green
10. 5 Combining both statements, we get
means black. Hence, the colour of the crow
11. 1; Given statements: is green.
KL>MN<P ... (i) 21. 5; There is no negative statement. Thus,
Q=K ... (ii) the possibility in I exist. Hence conclusion I
Combining both statements, we get follows.
Hence, conclusion I is true. But conclusion All walls are floors  conversion  Some
II (M  N) is not true. floors are walls. Hence, conclusion II follows.
15. 5; Given statement: 22. 2; All tables are desks (A) + Some desks
are benches (I) = A + I = No conclusion.
Hence, only conclusion I is true. Hence conclusion I does not follow.
12. 2; Given statements: Thus, E  G is true. Hence, conclusion I is Again, Some desks are benches  conversion
RS=TE ... (i) true. Again, F  H or H  F is true  Some benches are desks. Hence conclusion
L=J>E ... (ii) Hence, conclusion II is true. II follows.
23. 2; All purses are rings (A) + No ring is a or, ? = (77)2 = 5929
105 105 105
hand (E) = A + E = E = No purse is a hand. 782 = 24 × × × = 27.783 lakh
Hence conclusion II follows. But conclusion 87. 2; ? = – 372 = 460 – 372 = 88 100 100 100
1.7
I does not follow. 100. 3; Let the principal be 100.
12.5  ?
24. 4; No scooter is a bike (E) + No truck is 88. 3;  2209  3969 Then, the amount after 3 years
a bike (E)  conversion  No bike is a 100
= 47 + 63 = 110 110 110 110
truck (E) = E + E = No conclusion. Hence = 100 × × × = 133.1
conclusion I does not follow. 110 100 100 100 100
?= = 880  CI = 133.1 – 100 = ` 33.10
No truck is a bike  conversion  No bike 12.5

K
is a truck (E). Hence conclusion II does not 43 When CI = ` 33.10 then principal = 100
89. 4; 3 × ? = 1365 = 11739 Similarly, when CI = 3641 then principal
follow. 5
25. 5; Some watches are needles (I) + All 11739 100
needles are walls (A) = I + A = I = Some ?= = 3913 =  3641 = ` 11000
3 33.1
watches are walls. Hence conclusion I follows.
All needles are walls  conversion  Some 9240  3  4  6
90. 1; ? = = 1728 3 5 4
walls are needles. Hence conclusion II follows. 5  7  11 101. 1; x × × × = 960
7 11 13
(26-30): 1.2  1225 2.4  975
91. 3; ? =  960  7  1113
Window 100 100 x= = 16016
1 2 3 4 5 6 3 5  4
Screen = 14.7 + 23.4 = 38.1 102. 1; B’s 1 day’s work
Tooth Close- Pepso- Oral-- 92. 2; ? = 780 × 3.6 = 2808
Colgate Vico Babul 1 1 5 –1 4 1
paste up dent B 1.3  3.3  7.3 = – = = =

KUNDAN
26. 2 27. 1 28. 3 29. 1 30. 4 93. 3; ? = = 104.39 16 80 80 80 20
0.3
31. 1 32. 2 33. 4 34. 3 35. 5  B will take 20 days to complete it.
94. 2; ? = 14.44  380.25 103. 2; (A + B + C) can fill in 1 hour
36. 1;
= 3.8 × 19.5 = 74.1 1
= part of tank
164 735 4
37. 3; 95. 3; ? = – 18.4 (A + B + C) can fill in 2 hours
100
2 1
= 1205.4 – 18.4 = 1187 = = part of tank
96. 5; CP = 815 + 145 = ` 960 4 2
38. 4; So, (A + B) can fill the tank in 10 × 2 = 20
And SP = ` 1296
 Percentage profit hours
 C 1 hour’s work
1296 – 960 336 100
=  100 = = 35% 1 1 5 –1 4 1
39. 4; Except seldom all other words are 960 960 = – = = =
synonyms. 4 20 20 20 5
97. 2; Let the cost price be ` x.
40. 1; Except departure all other are C’s alone will fill it in 5 hours.
synonyms. 85x 17 x 45 20
41. 2 42. 3 43. 3 44. 1 45. 4 Then, SP = =` 104. 4; Kisan can do the work in
100 20 25
46. 5 47. 2 48. 3 49. 4 50. 1
= 36 days
51. 2; Replace ‘on’ with ‘out’ 3x 17 x
Now, new CP = ` and new SP =  117 Rajeev share : Kisan’s share = 36 : 45 = 4 : 5
52. 3; Replace ‘which’ with ‘when’ 4 20
5
53. 4; Insert ‘was’ before ‘arrested’ According to the question,  Kisan’s share = 2925 × = ` 1625
54. 2; Replace ‘were’ with ‘was’ 9
17 x 3x 105. 2; (?)2 = 2548 × 1872 = 196 × 13 × 13
55. 1; Add ‘the’ before ‘Obama’  117 –
20 4  100  35 × 144 = 144 × 169 × 196
56. 2 57. 4 58. 1 59. 3 60. 5
3x ? = 12 × 13 × 14 = 2184
61. 4 62. 1 63. 5 64. 3 65. 2
(66-70): EBACGDF 4 106. 4; Let the school be x km far from his
66. 5 67. 2 68. 4 69. 1 70. 3 17 x – 15x  2340 4 home.
or,  100 = 35 x x
71. 3 72. 5 73. 1 74. 4 75. 2 20 3x Then,   10.5
76. 5 77. 1 78. 4 79. 3 80. 2 6 4.5
28  85 2 x  2340 4
81. 2; ? =  13.2 = 23.8 + 13.2 = 37 or,   100  35 x 2x 21
100 20 3x or,  
6 9 2
or, 40x + 2340 × 20 = 105x
7 3 3x  4 x 21
82. 1; ? =  58   139.2 or, 65x = 2340 × 20 
5 8 or,
18 2
= 81.2 + 52.2 = 133.4 2340 20
x= = ` 720 7 x 21
65 or, 
83. 1; ? = 3 17576  3 4096 = 26 + 16 = 42 18 2
84. 3; ? = 84368 + 65466 – 72009 – 13964 290 100
98. 3;  r = = 4% 21 18
= 63861 1450  5 x= = 27 km
12  555 15  666 2 7
85. 1; ? =  1450  8  5 107. 1; Let the cost price be ` x.
100 100 Now, SI = = ` 580
100
= 66.6 + 99.9 = 166.5 120 x  112
 Amount = 1450 + 580 = ` 2030 Now, x  –  50
86. 3; ? = 490 – 142 – 271 = 77 99. 4; Reqd population 100 100
 Area = 5580 146. 2 147. 4 148. 3 149. 3 150. 2
50100
x= = ` 625 5580 – 5000 151. 4 152. 2 153. 1 154. 3 155. 1
8 % increase =  100 156. 3 157. 1 158. 2 159. 3 160. 2
5000
108. 2; Relative speed = 48 + 24 = 72 kmph 161. 5 162. 2 163. 3 164. 1 165. 1
580  100 166. 1 167. 1 168. 2 169. 4 170. 2
5 =  11.6% increase
= 72 = 20 ms–1 5000 171. 1 172. 2 173. 4 174. 2 175. 4
18 176. 4 177. 2 178. 4 179. 4 180. 5
115. 1; Volume of brick
 Length of the faster train = 20 × 9.5 181. 4 182. 2 183. 4 184. 2 185. 2
= 190m 25 15 5 3 186. 5 187. 4
=    cu m
109. 5; The series is +13 + 7, +23 + 14, +33 + 100 100 100 1600 188. 3; Both Sequential and Random files
21, +43 + 28, ... Volume of wall = 40 × 1.5 × 1 cu m can be stored on a disk.
110. 3; CP = 11800, discount = 35%, 189. 1 190. 3 191. 3 192. 2 193. 2
Again SP = 1170 3 1600 194. 3 195. 4 196. 5 197. 1 198. 3
 Number of bricks = 40 × ×
CP = ` 1800, discount = 20%, 2 3 199. 2 200. 1
then SP = 32000
= ` 1440 116. 3; Volume of the cylinder =  × 82 × 35
Again, discount = 15% =  × 65 × 35
 SP = ` 1224 4 4
 Difference = 1224 – 1170 = 54 Volume of the sphere = (1)3 
3 3
111. 2; Number of elements in the sample
 Number of spheres
spaces
3
15 15! =   64  35   1680
n(S) = C2  4
13!2!
117. 3; The series is ×1 + 12, ×2 + 22, ×3 + 32,
15 14 ×4 + 42, ...
= = 105 These should be 111 in place of 116.
2
118. 4; Total age of six students = 17.5 × 6
5 5! = 105 years
Number of events = n(E) = C2   10
3!2! Total age of remaining 5 students = 16 × 5
= 80 years
10 2 Age of student who left the class
 P(E) = =
105 21 = 105 – 80 = 25 years
112. 4; Total number of arrangements = 5! 7
= 120 119. 3; B’s share = 41517 = ` 13839

K
21
But one word is LIGHT itself.
So the number of rearrangements = 120 – 1 120. 2; Let the cost price be ` x.
= 119 111x 93x
Then, –  234
113. 2; Let the number be x. 100 100
7x 234100
Then, x – = 134 or, x = = ` 1300
9 18
116
134 9 For 16% gain, SP = 1300 = ` 1508
100
or, x = = 603
2 121. 1 122. 1 123. 1 124. 2 125. 1
114. 2; Let the length of the rectangle be 126. 5 127. 1 128. 2 129. 3 130. 3
100m and breadth be 50m. 131. 2 132. 4 133. 3 134. 2 135. 3
 Area = 5000 sq m 136. 1 137. 2 138. 2 139. 4 140. 4

KUNDAN
Now, length = 100 + 24 = 124m, breadth 141. 3 142. 3 143. 3 144. 1 145. 2
= 50 – 5 = 45m
Clerk Mock Test 9
7. Who among the following is sitting third to the right
Test-I: Reasoning Ability of E?
1) F 2) G 3) D 4) A 5) None of these
Directions (Q. 1-5): In each question below are given 8. Which of the following statements is false?
two statements followed by two conclusions numbered I 1) D is exactly between H and C.
and II. You have to take the given statements to be true 2) E is opposite F.
even if they seem to be at variance with commonly known 3) C is second to the left of A.
facts. Read all the conclusions and then decide which of 4) All are false
the given conclusions logically follows from the given 5) None of these
statements, disregarding commonly known facts. Give 9. Who among the following is third to the left of A?
answer 1) E 2) H 3) D 4) G 5) None of these
1) if only conclusion I follows. 10. Who among the following is opposite D?
2) if only conclusion II follows. 1) A 2) E 3) B 4) H 5) None of these
3) if either conclusion I or II follows. Directions (Q. 11-15): Study the following information
4) if neither conclusion I nor II follows. carefully to answer the questions given below. Following
5) if both conclusions I and II follow. are the conditions for selecting Chief Manager in an
1. Statements: Some cloths are shirts. organisation:
All shirts are stalls. The candidate must
Conclusions: I. All stalls being cloths is a (i) be a graduate in any discipline with at least 60 per
possibility. cent marks.
II. Some shirts are not cloths. (ii) have secured at least 55% marks in the interview.
2. Statements: Some hours are minutes. (iii) be at least 28 years and not more than 40 years as
Some minutes are not seconds. on 01.03.2014.
Conclusions: I. Some seconds are hours.

K
(iv) be a postgraduate degree/diploma holder in
II. All hours are seconds. Marketing/Sales Management.
3. Statements: No pin is a staple. (v) have post-qualification work experience of at least
Some pens are staples. five years in sales of any organisation.
Conclusions: I. Some staples are not pins. In the case of a candidate who satisfies all the
II. Some pens are not pins. conditions except
4. Statements: All letters are numbers. (a) (ii) above, but has secured more than 70% marks in
Some numbers are not squares. graduation, the case is to be referred to GM-Sales.
Conclusions: I. Some squares are not letters. (b) (v) above, but has post-qualification work
II. Some numbers are letters. experience of at least three years as Manager Sales
5. Statements: All hankies are towels. in an organisation, the case is to be referred to

KUNDAN
No towel is cloth. President.
Conclusions: I. No hanky is cloth. In each question below, details of one candidate are
II. Some towels are hankies. provided. You have to take one of the following courses of
Directions (Q. 6-10): Study the following information action based on the information and the conditions and sub-
carefully to answer the given questions: conditions given above and mark the number of that course
There are eight friends A, B, C, D, E, F, G and H sitting of action as the answer. You are not to assume anything other
arround the circle facing outward from the centre. G is third than the information provided in each case. All these cases
to the right of A and is not the neighbour of F or H. D and are given to you as on 01.03.2014. Give answer
C are immediate neighbours of G. F is second to the right of 1) if the candidate is to be selected.
B and second to the left of G. D is not the neighbour of E or 2) if the candidate is not to be selected.
F. E is the immediate neighbour of B and H. 3) if the data provided are inadequate to take a
6. Who among the following are immediate neighbours decision.
of F? 4) if the case is to be referred to GM-Sales.
1) B, C 2) E, C 3) C, A 4) H, A 5) None of these 5) if the case is to be referred to President.
11. Payal was born on 25th March 1978. She has secured 18. APHWME
62% marks in graduation and 58% marks in the 1) 7A%4©1 2) 1A%4©A 3) $4%4©$
interview. She has been working for the past six years 4) 1A%4©1 5) None of these
as Manager Sales in an organisation after completing 19. TWUIPD
her Postgraduate Diploma in Sales Management. 1) @469A3 2) @649A@ 3) @469A@
12. Chandan Pal has secured 61% marks in graduation 4) 3@469A 5) None of these
and 56% marks in the interview. He has been working 20. AWIUMH
in the marketing division of a company for the past 1) 7496©% 2) %7496© 3) ©7496©

K
seven years after completing his Postgraduate Diploma 4) %496©7 5) None of these
with 50% marks. Directions (Q. 21-25): Study the following information
13. Seema Kumari was born on 13th April 1983. She has carefully and answer the questions that follow:
secured 50% marks in the interview and 72% marks in A ÷ B means A is son of B
graduation. She has been working in the sales division A × B means A is sister of B
of an organisation for the past five years after A + B means A is brother of B
completing her Postgraduate degree in Sales A – B means A is mother of B
Management. 21. How is G related to H in the expression
14. Pranav Sharma was born on 17th May 1975. He has ‘G × R + V ÷ H’?
been working in the sales division of an organisation 1) Sister 2) Daughter 3) Son

KUNDAN
for the past ten years after completing his Post- 4) Mother 5) None of these
graduate degree in Sales Management with 50% marks. 22. Which of the following expressions represents
He has secured 68% marks in graduation and 51% ‘B is the husband of A’?
marks in the interview. 1) A × I – E + B 2) A – I + E ÷ B 3) A + I ÷ E × B
15. Vinod Malhotra was born on 5th March 1979. He has 4) A ÷ I × E + B 5) None of these
been working as manager in the sales division of a 23. How is V related to T in the expression
company for the past three years after completing his ‘T ÷ R + V × N’?
Postgraduate Diploma in Marketing Management in 1) Niece 2) Father 3) Uncle
2010. He has secured 65% marks in graduation and 4) Aunt 5) None of these
56% in the interview. 24. How is P related to J in the expression ‘J × K ÷ M – P’?
Directions (Q. 16-20): In each question below is given 1) Sister 2) Brother 3) Father
a group of letters followed by four combinations of digits/ 4) Can’t be determined 5) None of these
symbols numbered (1), (2), (3) and (4). You have to find out 25. Which of the following expressions represents ‘J is
which of the combinations correctly represents the group wife of E’?
of letters based on the following coding system and mark 1) E ÷ F × G + H – J 2) E × G ÷ H + F – J
the number of that combination as the answer. If none of 3) J – H × G ÷ E + F 4) Both 1) and 2)
the four combinations correctly represents the group of 5) None of these
letters, mark 5), ie ‘None of these’, as the answer. Directions (Q. 26-30): Study the following information
carefully and answer the given questions:
There are six cars P, Q, R, S, T and U parked in two
rows. Each car has a different colour, viz Red, Yellow, Blue,
Green, Orange and White. All the cars are of different prices.
Conditions: T is the most expensive car and is parked exactly opposite
1) If the first letter is a consonant and the last letter is the Red car. The lowest priced car is parked exactly opposite
a vowel, their codes are to be interchanged. the Green car. U, the Orange car, is parked between P and S.
2) If both the first and the last letters are consonants R, a Yellow car, is parked opposite P. Q is of Green colour
both are to be coded as the code for the first letter. and is parked opposite U. P is not of Blue colour and is
3) If both the first and the last letters are vowels, both more expensive than R, but not more expensive than S or
are to be coded as the code for the last letter. Q.
16. JTIHAD 26. What is the colour of the most expensive car?
1) 5@9%75 2) 5@9%73 3) 3@9%75 1) White 2) Blue 3) Red
4) 59@%75 5) None of these 4) Yellow 5) None of these
17. PUMKRE 27. What is the colour of the car which is diagonally
1) A6©$21 2) 16©$2A 3) 16©$21 opposite the White car?
4) A©6$2A 5) None of these 1) Blue 2) Yellow 3) Green
4) Orange 5) None of these
28. Which of the following cars is the second most
expensive car? Test-II: English Language
1) S 2) T 3) Q
4) P 5) Can’t be determined Directions (Q. 41-45): Read the passage carefully
29. Which of the following cars is parked opposite S? and answer the questions given below it. Certain words/
1) P 2) Q 3) T 4) R 5) None of these phrases are given in bold to help you locate them while
30. What is the colour of the car which is parked exactly answering some of the questions.
opposite the Yellow car? Scientists have found irrefutable evidence that Mars
1) Green 2) Orange 3) Red once had the ingredients for life. This confirmation comes
4) White 5) None of these seven months after NASA’s rover Curiosity landed on
Directions (Q. 31-35): In the questions given below, Mars.
the relationship between different elements is shown in Analysis of powdered samples drilled out from inside
the statements. These statements are followed by two an ancient and once water-soaked rock at the rover’s Gale
conclusions. Mark answer Crater landing site shows clays, sulphates and other
1) if only conclusion I follows. minerals that are all key to life, scientists told reporters at
2) if only conclusion II follows. NASA headquarters in Washington on March 12.
3) if either conclusion I or II follows. The water that once flowed through the area known
4) if neither conclusion I nor II follows. as Yellowknife Bay, was likely drinkable, said Curiosity’s
5) if both conclusions I and II follow. lead scientist John Grotzinger, who is with the California
31. Statements: M < J, J  R, R = K, K < B Institute of Technology.
Conclusions: I. K = J II. B > M The analysis stopped short of a confirmation of
32. Statements: P = W, V  U, W < C, C > V organics, which are key to most Earth-like life. But with 17
Conclusions: I. C > P II. U < C months left in the rover’s primary mission, scientists said
33. Statements: Q  R, N > M, N  R they expect to delve further into that question. Science
Conclusions: I. Q  N II. R  M operations currently are suspended because of a computer
(34-35): glitch, which is expected to be resolved shortly.
Statements: U > X, Y  Z, C = D, Y  X, D > Z Whether or not Mars has or ever had life, it should
34. Conclusions: I. U > Y II. Z  X have at one time at least had organic compounds delivered
35. Conclusions: I. C > Z II. Y < C to its surface by organic-rich comets and asteroids.

K
36. Mahesh walks 20m north from his house. Then, he Finding places where the organics could have been
turns to his west and covers 8m. Again, he turns south preserved, however, is a much trickier prospect than finding
and covers 6m. Finally, turning to east, he covers 8m. the environmental niches and chemistry needed to support
How far is he from his house and in which direction? life, scientists said.
1) 12 metres, East 2) 14 metres, North In May, following a one-month interruption of radio
3) 20 metres, South 4) 10 metres, West communication caused by the positions of Earth and Mars,
5) None of these scientists plan to drill a second hole into the Gale Crater
37. In a certain code language LATEST is written as rock to look for organic compounds.
NXVBUQ. How is STRONG written in that code “If there was organic material there, it could have been
language? preserved,” said David Blake, principal investigator for
1) TUSPOH 2) VRUQQI 3) UQTLPD Curiosity’s Chemistry and Mineralogy, or CheMin,

KUNDAN
4) TUQLPD 5) None of these experiment.
38. How many such pairs of letters are there in the word A lack of organics, however, would not rule out the
VENGEANCE each of which has as many letters Yellowknife Bay site as suitable for life, scientists added.
betwen them in the word as in the English alphabet? “You don’t have to have carbon present in a geological
1) None 2) One 3) Two 4) Three 5) Four environment that’s habitable in order to have microbial
39. How many meaningful English words can be formed metabolism occur,” Grotzinger said. Some micro-organisms
with the letters BAGR using each letter only once in on Earth, for example, can feed on inorganic compounds,
each word? such as what are found inside rocks.
1) None 2) One 3) Three “There does need to be sources of carbon somewhere,
4) Two 5) None of these but if it’s just CO2, you can have chemoautotrophic organic
40. What should come next in the following number compounds based on that carbon,” Grozinger said.
sequence? Analysis shows the Gale Crater rock contains carbon
132465132465132465132?? dioxide, in addition to hydrogen, oxygen, phosphorus,
1) 6, 5 2) 2, 4 3) 5, 1 4) 4, 6 5) None of these sulphur and nitrogen.
Carbon dioxide provides a key ingredient in the 5) Mars is made of only geological materials and there
building blocks for life, all of which have now been found is no possibility of life there.
in the Mars rock sample, Grotzinger said. 44. Which of the following conclusions can be drawn on
The $2.5-billion, nuclear-powered Curiosity rover the basis of facts mentioned in the given passage?
landed inside the giant Gale Crater impact basin, located 1) Chemoautotrophic organisms with the help of CO2
near the Martian equator, on August 6 for a two-year generate organic compounds.
mission. Scientists were drawn to the area because of a 2) Life is not possible without CO2.
five-km mountain of sediment, called Mount Sharp, rising 3) Some of the micro-organisms on Earth can feed on

K
from the crater floor. inorganic compounds.
But shortly after the rover’s landing, the team decided 4) Gale Crater contains not only hydrogen, oxygen
to first explore the Yellowknife Bay area, located in the and phosphorus but also carbon dioxide.
opposite direction from Mount Sharp. 5) All the above
Observations from Mars orbiters showed three 45. What is the meaning of the word ‘irrefutable’ as used
different types of terrain coming together in Yellowknife in the given passage?
Bay, plus a low elevation, all hints that water could have 1) indefinite 2) dubious 3) questionable
once flowed and pooled on the surface. 4) indisputable 5) inaccurate
41. Which of the following is not necessarily correct in Directions (Q. 46-50): Read each sentence to find out
the context of the passage? whether there is any grammatical or idiomatic error in it.

KUNDAN
1) On the basis of evidence found on Mars scientists The error, if any, will be in one part of the sentence. The
have confirmed that ingredients for life once existed number of that part is the answer. If there is ‘No error’,
on Mars. the answer is 5). (Ignore errors of punctuation, if any.)
2) NASA’s rover Curiosity landed on Mars near Gale 46. 1) Harsh Vardhan said he was looking forward / 2) to
Crater. serve his roots, as he had spent / 3) the first 15 years
3) Drinkable water flowed through Yellowknife Bay of his life / 4) in the streets of old Delhi. / 5) No error
once. 47. 1) A family office has to/ 2) execute all tasks related to
4) Scientists are not sure if organic compounds existed / 3) manage the wealth of / 4) the super-rich family. / 5)
on Mars. No error
5) None of these 48. 1) Service providers in the financial industry, specially
42. What do clays, sulphates and other minerals found private banks / 2) and wealth management firms, are
near the rover’s landing site suggest? increasingly / 3) forced to standardise their / 4) products
(A) Clays, sulphates and other minerals found near to optimise profits. / 5) No error
the rover’s landing site suggest that huge volcanic 49. 1) Social or impact funds are private equity-like funds
eruptions might have taken place in the remote / 2) that pool money from investors / 3) and put it to
past. work in a portfolio of ventures / 4) that meet the funds’
(B) Clays, sulphates and other minerals found at the objective. / 5) No error
rover’s landing site revealed that Mars once had 50. 1) While residential land purchases / 2) have benefited
favourable conditions for life. many investors / 3) buying land can be risky, / 4)
(C) The evidence of clay found near the rover’s especially for a NRI. / 5) No error
landing site suggests that once water flowed Directions (Q. 51-55): Rearrange the following five
through the area. sentences (A), (B), (C), (D) and (E) in the proper sequence
1) All (A), (B) and (C) 2) Only (B) to form a meaningful paragraph and then answer the
3) Only (C) 4) Only (A) and (B) questions given below.
5) Only (B) and (C) (A) The working group had examined fair-practice
43. Find the correct statement according to the passage. codes adopted by bankers in other domains like
1) Though there is no evidence of organics at UK, Canada, Hong Kong, Singapore and
Yellowknife Bay site yet the possibility of life over Australia and prepared a draft Banker’s Fair
there cannot be ruled out. Practice Code, duly incorporating some of the finer
2) There can be no life without carbon or organic points from those documents.
compound. (B) The RBI, while announcing the formation of the
3) Even micro-organisms cannot exist in geological Banking Codes and Standards Board of India
environment where there is complete lack of (BCSBI) in the annual Policy Statement, had
organics. requested the IBA to set up a working group to
4) The presence of water confirms the existence of draft a comprehensive fair practice code, covering
life on Mars.
all the areas of customer service for uniform ________ to acknowledge the existence of the AAP
adoption by banks. leader.
(C) BCSBI made further refinements to the code and 1) released, acquiesced 2) disclosed, accorded
the Code of Bank’s Commitment to Customers 3) issued, accommodated 4) publicised, descended
was brought out. 5) announced, condescended
(D) Accordingly, the IBA had set up a working group 60. The re-incarnation and revival of Uttarakhand in the
to study the international practices and review ________ of one of the worst carnages in the history
the existing codes. of humanity is one marked by perseverance, patience,
(E) It further refined the draft code, incorporating the _________ courage, and a never-say-die spirit.
suggestions from the member banks and 1) repercussion, efficient 2) last, foolproof
submitted it to the BCSBI. 3) aftermath, infallible 4) consequence, faulty
51. Which of the following should be the FIRST sentence 5) end, acceptable
after rearrangement? Directions (Q. 61-65): In each question below is given
1) A 2) E 3) C 4) B 5) D a sentence with four words printed in bold type. These are
52. Which of the following should be the SECOND numbered as 1), 2), 3) and 4). One of these four words
sentence after rearrangement? printed in bold may be either wrongly spelt or inappropriate
1) D 2) A 3) B 4) E 5) C in the context of the sentence. Find out the word which is
53. Which of the following should be the THIRD sentence wrongly spelt or inappropriate, if any. The number of that
after rearrangement? word is your answer. If all the words printed in bold are
1) C 2) D 3) B 4) E 5) A correctly spelt and also appropriate in the context of the
54. Which of the following should be the LAST (FIFTH) sentence, mark 5), ie ‘All correct’, as your answer.
sentence after rearrangement? 61. Online food and resturants listings startup Zomato is
1) B 2) C 3) A 4) D 5) E 1)
55. Which of the following should be the FOURTH moving beyond traditional hiring techniques as it looks
sentence after rearrangement? 2)
1) A 2) B 3) E 4) C 5) D to double its employee strength in the coming year
Directions (Q. 56-60): Each question below has two and expand its global footprint across the Americas,
blanks, each blank indicating that something has been 3) 4)
omitted. Choose the set of words for each blank that best Europe, South-East Asia and Australia. All correct

K
fits the meaning of the sentence as a whole. 5)
56. Manmohan Singh’s inability to visit Nagaland “even 62. In India too, The Revelations is the book that has
once in 10 years” has ________ three-time state CM 1)
Neiphieu Rio, leader of the Naga Reo, leader of the caught the attention of critics, authorities and market
Naga People’s Front, to jump into the ________ for 2)
the state’s lone Lok Sabha seat. players, but that is yet to spark a huge and cry as in
1) promoted, field 2) dissuaded, battle 3) 4)
3) stopped, fight 4) prompted, fray the US. All correct
5) prevented, contest 5)
57. In Indian politics, ________ one’s cake and eating it 63. Penalty for banks that harrass customers by delaying
too ________ a completely different meaning. 1)

KUNDAN
1) making, sends 2) having, takes transfer of loans, doing away with processing fee for
3) preparing, has 4) cooking, takes shifting within the same bank from one type to another
5) cutting, had and an industry benchmark base rate for borrowers
58. The ________ went so far that in 1993, within two 2)
years after the ________ of the Soviet Union, their are among the recommendations of a central bank
very Parliament had to be bombed by Russian tanks 3)
to get the MPs to come out and surrender. panel on pricing of loans to make the process smoother
1) anarchy, dissolution 2) turmoil, beginning 4)
3) unrest, construction 4) disorder, solution for retail borrowers. All correct
5) confusion, division 5)
59. The day after Arvind Kejriwal took a holy dip into the 64. Kejriwal’s conduct offers sharp contrast to a paradigm
Ganges and ________ his intention of contesting 1)
against Narendra Modi here, the PM candidate finally of insult and revenge championed by others, even if
2) Directions (Q. 71-80): In the following passage, some
no one imagined that Kejriwal would be heralding of the words have been left out, each of which is indicated
3) by a number. Find the suitable word from the options given
such a redefinition of a politician’s touchy-feely against each number and fill up the blanks with
4) appropriate words to make the paragraph meaningfully
relationship with the people. All correct complete.
5) Bank customers may no (71) have to maintain a
65. Since February, as a crisis erupted in Ukraine and laid minimum balance in their savings account as the Reserve

K
1) 2) Bank of India has (72) banks to do away (73) the practice
to the annexation of Crimea in March by Russia, of levying penalty on account holders who don’t do (74).
3) While this spells good news for account holders, some
western nations have recoiled from it. All correct bankers said this would increase their (75) and they might
4) 5) start charging for some of the services they were offering
Directions (Q. 66-70): In the following questions, a free.
sentence has been given with some of its parts in bold. To “I think what they are saying is, if you don’t charge
make the sentence grammatically correct, you have to on (76) minimum balance then you are authorised to charge
replace the bold part with the correct alternative given on the transaction,” said HDFC Bank’s managing director
below. If the sentence is correct as it is, give 5) as you Aditya Puri.

KUNDAN
answer (ie No correction required). For instance, if a customer has a balance of less than
66. Human life and human dignity have been disregarded ` 10000 in the savings account, the bank may not charge
through history and continue to be disregarded today. for non-maintenance of minimum balance, but could (77)
1) through history and continues charge for more than two ATM transactions a month.
2) throughout history and continue RBI has urged banks to (78) the liability of customers
3) throughout the history and continues in cases like non-maintenance of balance or electronic
4) althrough the history and continued transactions (79) banks are not able to (80) customer
5) No correction required negligence.
67. The history of mankind is efforts marked by respect 71. 1) where 2) one 3) more
to ensure the dignity of human beings. 4) body 5) longer
1) marked by respect to ensure 72. 1) directed 2) called 3) said
2) respect marked by efforts to ensure 4) offered 5) dictated
3) marked by efforts to ensure 73. 1) by 2) from 3) with
4) respect to ensure efforts 4) off 5) without
5) No correction required 74. 1) it 2) so 3) therefore
68. What broke him was the hopelessness that breaks so 4) same 5) even
many farmers in this hard scrabble region called 75. 1) prices 2) assets 3) incomes
Vidarbha. 4) costs 5) rates
1) What broken him was 76. 1) maintaining 2) putting 3) placing
2) What should broke him was 4) transacting 5) ascertaining
3) What was broken were 77. 1) in spite of 2) in lieu of 3) instead
4) What had broke him was 4) despite 5) not
5) No correction required 78. 1) estimate 2) guess 3) exempt
69. The Parrikar regime get a ban imposed on bovine 4) fix 5) limit
slaughter, forcing Goans to go without affordable beef 79. 1) why 2) where 3) however
for months. 4) moreover 5) otherwise
1) Parrikar regiment get 80. 1) accuse 2) show 3) condone
2) The Parrikar regiment have got 4) prove 5) counter
3) The Parrikar got
4) The Parrikar regime got Test-III: Quantitative Aptitude
5) No correction required
70. Football stars in India does not enjoy same status and Directions (Q. 81-95): What value should come in
adulation reserved for their counterparts in cricket. place of question mark (?) in the following equations?
1) do not enjoy the same 2) does enjoy some of the 81. 18.5% of 124= ?
3) enjoys the similar 4) do not enjoys the same 1) 21.32 2) 22.94 3) 23.64 4) 24.48 5) 25.64
5) No correction required
82. ( 1444 ÷ 5) × 3.25 = ? 99. A shopkeeper marks his goods 32% above the cost
price but allows 25% discount for cash payment. If he
1) 24.7 2) 25.4 3) 26.6 4) 27.2 5) 28.5 sells the article for ` 891, then what is its cost price?
3 1) ` 900 2) ` 850 3) ` 800
83. 17.8 + of 89.6 = ? 4) ` 750 5) None of these
7
1) 52.8 2) 54.4 3) 56.2 4) 58.6 5) 60.4 100. A money lender lends ` 11500 to a person A for 2 years
84. 373.816 + 274.102 – 199.573 – 108.108 = ? and ` 8500 to another person B for 3 years. If he gets
1) 338.147 2) 339.787 3) 340.237 4) 341.347 5) 342.107 ` 3395 as interest, then what is the rate of interest per
85. 14.8 × 7.25 = 20% of ? annum?
1) 524.5 2) 528.5 3) 532.5 4) 536.5 5) 540.5 1) 4% pa 2) 5% pa 3) 6% pa 4) 7% pa 5) 8% pa
101. At what rate per annum compound interest will ` 20
86. 1156 × 42.25 = ? lakh amount to ` 25.19424 lakh in three years?
1) 212 2) 215 3) 218 4) 220 5) 221 1) 4% pa 2) 5% pa 3) 6% pa 4) 7% pa 5) 8% pa
102. A, B and C can do a work in 6, 10 and 24 days
171.5 ?
87.  respectively. For this work they get ` 4699. What is
? 56 the share of B in this amount?
1) 84 2) 98 3) 104 4) 116 5) 128 1) ` 1524 2) ` 1648 3) ` 1296 4) ` 1442 5) ` 1718
88. 44 × 6.5 + 2970 ÷ 18 = ? 103. Pipe A can fill an empty tank in 15 hours and Pipe B
1) 451 2) 463 3) 484 4) 497 5) 504 can empty it in 10 hours. If both the pipes are opened
89. and after 4 hours Pipe B is closed, how much time will
4624  90.25  ?
A take to fill the remaining tank?
1) 632 2) 635 3) 642 4) 646 5) 651 1) 3 hours 2) 5 hours 3) 9 hours
3 7 5 4) 10 hours 5) 12 hours
90. of of of 6864 = ? 104. Find the number of permutations of the letters of the
8 11 13
1) 540 2) 590 3) 630 4) 680 5) 720 word CALCULATOR.
91. 81.6 ÷ 0.048 = ? × 25 1) 362880 2) 453600 3) 181440
1) 64 2) 68 3) 72 4) 76 5) 84 4) 226800 5) None of these
92. 18% of 325 + 28% of 445 = ? 105. If two dice are thrown simultaneously, then what is
1) 181.4 2) 183.1 3) 185.6 4) 187.5 5) 189.2 the probability of getting a total of 7?

K
 3  1 5 1
93.  7 of 675   45  ? 1) 2) 3)
 5  6 36 5
1) 112 2) 114 3) 116 4) 118 5) 120 1
4) 5) None of these
2 4 12
94. of of 45% of 18270 = ?
3 7 106. Seven-elevenths of three-fourths of a number is 567.
1) 2484 2) 2742 3) 2928 4) 3132 5) 3345 What is 55% of that number?
1) 312.8 2) 442.4 3) 508.6 4) 653.4 5) 678.2
95. (65% of 480) ÷ 12 = ? 107. The ages of two persons A and B are in the ratio of
1) 484 2) 576 3) 676 4) 784 5) None of these 8 : 5. After 12 years the ratio will become 10 : 7. What is

KUNDAN
96. In an election between two candidates Shyam and the present age of A?
Manohar, Shyam got 55% of the total valid votes. 12% 1) 32 years 2) 40 years 3) 48 years
of total votes are declared invalid. If the total number 4) 56 years 5) None of these
of votes is 2.4 lakh then how many valid votes did 108. The cost of 4 pens and 7 pencils is ` 97. The cost of 3
Manohar get? pens and 8 pencils is ` 92. What is the cost of 2 pens
1) 108000 2) 95040 3) 116160 and 2 pencils?
4) 132000 5) None of these 1) ` 36 2) ` 38 3) ` 40 4) ` 42 5) ` 44
97. If 12 men can reap 48 hectares in 40 days, how many 109. If the length of a rectangle is increased by 24% and
hectares can 50 men reap in 15 days? the width is decreased by 15%, what percentage
1) 60 hectares 2) 72 hectares 3) 75 hectares change will occur in the area?
4) 80 hectares 5) 84 hectares 1) 5.4% increase 2) 5.4% decrease 3) 9% increase
98. If a man sells his cycle for ` 1054, he loses 15%. At 4) 9% decrease 5) None of these
what price should he sell it to gain 20%? 110. A student secured 48% marks in a test and failed by 6
1) ` 1164 2) ` 1196 3) ` 1240 4) ` 1372 5) ` 1488 marks. Another student secured 56% marks and got 8
marks more than the minimum pass marks. What is the first time in which of the following cities in India
maximum marks? recently?
1) 125 2) 150 3) 175 4) 200 5) 225 1) Lucknow 2) Bhopal 3) Hyderabad
Directions (Q. 111-115): What will come in place of 4) Pune 5) None of these
question mark (?) in the following number series? 124. Examine the following with respect to the proposals
111. 860 739 658 609 584 ? of the Union Budget for FY 2014-15 and identify the
1) 475 2) 585 3) 575 4) 593 5) 420 one which is NOT correct?
112. 4 7 20 79 394 2363 ? 1) New Watershed programme with outlay of ` 2000 cr

K
1) 16540 2) 15682 3) 16583 4) 14582 5) 14213 2) A ` 10,000-cr venture capital fund for start-ups
113. 7 10.5 21 ? 157.5 551.25 3) ` 3600 cr for national rural drinking water programme
1) 42.5 2) 52 3) 63 4) 52.5 5) 62.5 4) National Centre of Humanities would be set up in
114. 93 95.8 99.8 105 ? 119 Uttar Pradesh
1) 110.4 2) 111.4 3) 110.8 4) 112.4 5) 111.8 5) None of these
115. 34 36 41 51 ? 94 131 125. We often hear about the term KYC in financial
1) 58 2) 67 3) 68 4) 69 5) 55 newspapers. Under the KYC norms, which of the
Directions (Q. 116-120): In each of these questions, following documents will NOT be accepted by banks
an equation is given with question (?) in place of the correct as proof of your correct residential address?
symbol. Based on the value on the right hand side and the 1) Telephone bill

KUNDAN
left hand side of the question mark, you have to decide 2) Bank account statement
which of the following symbols will come in place of the 3) Letter from any recognised public authority
question mark. 4) Letter from employer (subject to satisfaction of the
Give Answer: If in place of question mark (?) the bank)
following will come: 5) None of the above
1) > (greater than) 126. Which of the following real-estate firms in India has
2) < (less than) become country’s first real-estate developer to get ISO
3)  (either less than or equal to) 9001:2008 quality certification for development of
4)  (either greater than or equal to) housing and commercial projects recently?
5) = (equal to) 1) DLF 2) Parsvnath 3) Indiabulls Real
116. [(63  7)  (75  8)] ? [(89 – 23)  12] 4) Omaxe 5) None of these
127. The RBI started issuing its credit policy bi-monthly
117. [135 – (92  69)] ? [(17) 2  2 – 595] on the recommendation of which of the following
committees?
118. [{69 – (6) 2 }  3] ? [42  52  (13  4)] 1) Shome Committee
2) Urjit Patel Committee
119.  [ 324 – 49 ] ? [ 121 ]
3) HR Khan Committee
120. [ 1764  25 ] ?  ( 7056  10) 4) KC Chakrabarty Committee
5) None of these
128. Which of the following initiatives conceptualised by
Test-IV: General Awareness Indian govt to enable delivery of public services
(with Special Reference to Banking Industry) electronically has been awarded a United Nations
public service award recently?
121. Delhi is now the world’s second most populous city 1) Mobile Seva 2) e-Govt 3) e-Post
with 25 million inhabitants. Which of the following 4) IMPS 5) None of these
cities is the most populous in the world as of now? 129. The instruments issued by registered foreign
1) New York 2) Shanghai 3) Beijing institutional investors (FIIs) to investors abroad who
4) Tokyo 5) None of these wish to invest in the Indian stock markets but without
122. Who among the following won the Austrian Grand registering themselves with the market regulator SEBI
Prix Formula One championship recently? are called
1) Daniel Ricciardo 2) Lewis Hamilton 1) Preferential share 2) Rights issue
3) Nico Hulkenberg 4) Nico Rosberg 3) Commercial paper 4) P-Notes
5) None of these 5) None of these
123. A one-stop crisis centre named ‘Gauravi’ for women 130. The committee which has recommended that Indian
who are victims of violence has been set up for the money and currency markets move their benchmarks
towards transaction-based from poll-based in an
attempt to weed out manipulation, recently, was 1) Twitter 2) LinkedIn 3) Yahoo
headed by who among the following? 4) Facebook 5) None of these
1) P Vijaya Bhaskar 2) G Padmanabhan 139. The World Refugee Day is observed across the globe
3) B Mahapatra 4) DK Mohanty on which of the following dates?
5) G Gopalakrishna 1) Jun 14 2) Jun 16 3) Jun 18
131. The state govts have been given three months to 4) Jun 20 5) None of these
implement the Food Security Act. The deadline for 140. The Prime Minister Narendra Modi inaugurated the
implementing the National Food Security Act was on 240-Megawatt Uri-II Hydro Electric Project (HEP)
______. located in which of the following states recently?
1) Jul 1 2) Jul 4 3) Jul 14 1) Uttarakhand 2) Himachal Pradesh
4) Jul 31 5) None of these 3) Jammu & Kashmir 4) Gujarat
132. Who among the following has been formally 5) None of these
appointed the first chairman of the ICC after its 52- 141. What effect is seen on the credit creation capacity of
member council approved a controversial revamp of the market when the RBI lowers the cash reserve ratio
the body’s administrative structure recently? (CRR)?
1) Shashank Manohar 2) Jagmohan Dalmiya 1) It decreases
3) Sunil Gavaskar 4) N Srinivasan 2) It increases
5) None of these 3) First increases then decreases
133. According to a study on world insurance in 2013 by 4) Remains unaffected
the Global re-insurer Swiss Re, India stood at 5) None of these
_____position in the world in terms of premium 142. Which of the following regulates housing finance
volume. companies in India?
1) 20th 2) 18th 3) 16th 4) 15th 5) None of these 1) Reserve Bank of India 2) IRDA
134. Who among the following won IBSF 6-Red World 3) National Housing Bank 4) State govts
Snooker title in Egypt recently and created history by 5) SEBI
becoming the first player in the world to win world 143. Which of the following countries has adopted a
titles in the long and shorter formats of both billiards comprehensive anti-terror law recently, giving security
and snooker? forces unbridled powers, including shoot-at-sight
1) Kacper Flilpiak 2) Pankaj Advani suspects? It will remain in force for two years.

K
3) Shaun Murphy 4) Aditya Mehta 1) Afghanistan 2) Pakistan 3) Iraq
5) None of these 4) Syria 5) None of these
135. The international forum to discuss and implement 144. Who among the following has topped the Forbes list
supervisory norms for banks across the globe is of the world’s 100 most powerful women released
1) WEF recently?
2) IDA 1) Dilma Rousseff 2) Melinda Gates
3) Public Accounts Committee 3) Janet Yellen 4) Angela Merkel
4) Basel Committee 5) None of these
5) None of these 145. Reuven Rivlin has been elected tenth President of
136. The trend of non-cash transactions is picking up pace which of the following countries recently?
day by day. Which of the following is NOT a product 1) Canada 2) Israel 3) Spain

KUNDAN
aimed at facilitating non-cash transactions? 4) Sweden 5) None of these
1) Cheque 2) Credit card 3) Gift vouchers 146. Who among the following won the men’s singles title
4) Travel card 5) None of these of the Wimbledon tennis tournament recently?
137. The Great Himalayan National Park Conservation Area 1) Stan Wawrinka 2) Roger Federer
has been approved for inscription on the UNESCO 3) Rafael Nadal 4) Novak Djokovic
World Heritage list recently. It is located in 5) None of these
1) Kullu, Himachal Pradesh 147. Name the Indian scientist who has been elected to UN
2) Shimla, Himachal Pradesh entity, the Commission on the Limits of Continental
3) Samba, Jammu and Kashmir Shelf (CLCS), recently.
4) Garhwal, Uttarakhand 1) Rasik Ravindra
5) None of these 2) Rajan Sivaramakrishnan
138. Sheryl Sandberg, who visited India recently, is present 3) Jayant Narlikar
Chief Operating Officer of which of the following social 4) Anil Kakodkar
media firms? 5) None of these
148. The panel headed by who among the following run by the EPFO recently?
presented a new formula to estimate poverty in India 1) ` 1,000 2) ` 1,500 3) ` 2,000
recently? As per the panel estimates, three out of 10 in 4) ` 2,500 5) None of these
India are poor. 158. Who among the following won the Afghanistan’s
1) Suresh Tendulkar 2) C Rangarajan presidential election as per the preliminary results
3) KC Chakrabarty 4) Bimal Jalan released recently?
5) None of these 1) Abdul Qayum Karzai 2) Daud Sultanzoy
149. Despite accepting deposits and giving loans, the 3) Abdullah Abdullah 4) Ashraf Ghani

K
operation of which of the following entities do not 5) None of these
come under the purview of the RBI as of now? 159. As per the recent notification issued by the telecom
1) Regional Rural Banks sector regulator Trai, the minimum broadband speed
2) Cooperative credit societies has been raised from the earlier 256 kbps to
3) Payment banks 1) 512 kbps 2) 768 kbps 3) 1024 kbps
4) Non-Banking Finance Companies 4) 1280 kbps 5) None of these
5) Development banks 160. In a relief to the depositors, the Finance Minister
150. Under deficit financing, a way of funding of spending announced in the Union Budget that the PPF (Public
by borrowing, the govt borrows from which of the Provident Fund) deposit ceiling will be raised from the
following sources? existing ` 1 lakh to

KUNDAN
1) World Bank 2) IMF 3) RBI 1) ` 1.5 lakh 2) ` 2 lakh 3) ` 2.5 lakh
4) Gold loan providers 5) None of these 4) ` 3 lakh 5) None of these
151. As per the recent Economic Survey, India has the
second fastest growing services sector. Which of the Test-V: Computer Knowledge
following countries has the fastest growing services
sector? 161. Suppose you could see the world through the eyes of
1) Malaysia 2) Philippines 3) Thailand an ant. This simulation would be an example of
4) China 5) None of these 1) virtual reality 2) fuzzy logic
152. Who among the following Finance Ministers of India 3) knowledge system 4) expert system
had/has presented the Union Budget ten times, the 5) None of these
most by any Finance Minister so far? 162. Another name for a knowledge-based system is
1) Morarji Desai 2) Manmohan Singh 1) a perception system
3) Pranab Mukherjee 4) Yashwant Sinha 2) a conventional system
5) None of these 3) an expert system
153. The FIFA World Cup 2018 will be hosted by which of 4) a database management system
the following countries? 5) None of these
1) Russia 2) Germany 3) Italy 163. Which of the following software products is most
4) Argentina 5) None of these appropriate for designing interesting and attractive
154. With respect to loans, a time period during the loan Web pages?
term when the borrower is not required to make any 1) Microsoft Project
repayment, is termed as 2) Macromedia Dreamweaver
1) provisioning 2) moratorium 3) sanction 3) Windows Media Player
4) limitation 5) probation 4) QuarkXPress
155. What is the initial minimum deposit required for 5) None of these
opening a Basic Savings Bank Deposit Account 164. Forms that provide the basic structure for a particular
(BSBDA) with a bank? kind of document are known as
1) ` 200 2) ` 500 3) ` 700 1) wizards 2) templates 3) macros
4) ` 1000 5) No initial deposit required 4) patches 5) None of these
156. The RBI granted licences to two entities to open new 165. A method of transferring information directly from a
private banks in India recently. As per the RBI norms, mass storage device such as hard disk or from adapter
the new banks are bound to open what per cent of card into memory (or vice versa) without information
their branches in unbanked rural areas? passing through processor is commonly known as
1) 5 per cent 2) 10 per cent 3) 15 per cent 1) Direct memory access
4) 20 per cent 5) 25 per cent 2) Casting
157. What minimum pension has been approved by the 3) Standard memory access
govt under Employees Pension Scheme 1995 (EPS-95) 4) Directional memory access
5) None of these 4) Virtual Pact Network
166. ________ is the portion of the data block (oracle 5) None of these
block) that contains information about the table having 178. Which is the shortcut key used in PowerPoint to justify
rows in this block. a paragraph?
1) Table directory 2) Table row 3) Table column 1) CTRL + Z 2) CTRL + X 3) CTRL + J
4) Header 5) Footer 4) CTRL + E 5) None of these
167. The system unit of a personal computer typically 179. The time it takes a device to locate data and
contains all of the following except instructions and make them available to the CPU is
1) Microprocessor 2) Disk controller known as
3) Serial interface 4) Modem 1) clock speed 2) a processing cycle
5) None of these 3) CPU speed 4) access time 5) None of these
168. EPROM can be used for 180. ________ controls the way in which the computer
1) Erasing the contents of ROM system functions and provides a means by which we
2) Reconstructing the contents of ROM can interact with the computer.
3) Erasing and reconstructing the contents of ROM 1) The platform 2) The operating system
4) Duplicating ROM 3) Application software 4) The motherboard
5) None of these 5) None of these
169. With reference to a computer, an assembler is a 181. ________ is a technique for transmitting a large
1) symbol 2) language 3) program amount of information, including voice, data and video,
4) grammar 5) person who assembles the parts over long distances.
170. ________ is the largest storage unit. 1) Multiplexor 2) Bridge
1) Byte 2) Kilobyte 3) Terabyte 3) Broadband network 4) Beta software
4) Gigabyte 5) Megabyte 5) None of these
171. DSL stands for 182. A popular network protocol and cabling scheme with
1) Data Subscriber Line 2) Direct Subscriber Line a transfer rate of 10 megabytes per second is known
3) Digital Subscriber Line 4) Dual Subscriber Line as
5) Divisible Subscriber Line 1) Bandwidth 2) Ethernet 3) Boot
172. In ________ network topology there are bi-directional 4) Auto flow 5) None of these
links between each possible node. 183. Computers manipulate data in many ways, and this

K
1) bus 2) ring 3) star 4) tree 5) mesh manipulation is called
173. A permanent virtual circuit involves 1) utilizing 2) batching 3) upgrading
1) Connection establishment 4) processing 5) None of these
2) Data transfer 184. The name a user assigns to a document is called a(n)
3) Connection release 1) filename 2) program 3) record
4) All of the above 4) data 5) None of these
5) None of these 185. Formulas in Excel start with
174. ________ input/output device is not associated with 1) % 2) = 3) + 4) – 5) ;
personal computers. 186. The ________ key will launch the start button.
1) Colour monitors 2) Dot-matrix printer 1) Esc 2) Shift 3) Windows
3) Punched cards 4) Mouse 4) Ctrl 5) shortcut

KUNDAN
5) Optical scanners 187. Bridge operates at which of the following OSI layers?
175. Workspace means 1) Physical 2) Network 3) Data link
1) Group of worksheets 2) Group of rows 4) Both (1) and (3) 5) Session
3) Group of columns 4) Group of workbooks 188. ________ provides electronic data interchange
5) Group of cells facility.
176. Which of the following graphical models is used to 1) ISDN 2) LAN 3) VAN
define a database? 4) CAN 5) None of these
1) Organisation model 2) Programming model 189. If you do not want to select any option after opening
3) Flow chart 4) E-R diagram a menu, click menu title again or press ________ key
5) None of these to close the menu.
177. VPN stands for 1) Shift 2) Tab 3) Escape
1) Variable Private Network 4) F1 5) None of these
2) Virtual Private Network 190. Which of the following are valid minimum and maximum
3) Virtual Perfect Network zoom sizes in MS Office?
1) 10, 100 2) 20, 250 3) 10, 500 passwords for a match before granting access.
4) 10, 1000 5) None of these 1) website 2) network 3) backup file
191. Information on a computer is stored as 4) database 5) None of these
1) analog data 2) digital data 3) modem data 197. To print a document, press ________, then press
4) watts data 5) None of these ENTER.
192. A program that works like a calculator for keeping track 1) SHIFT + P 2) CTRL + P 3) ALT + P
of money and making budgets: 4) ESC + P 5) None of these
1) calculator 2) scholastic 3) keyboard 198. Multiplexing involves _______ path(s) and ________

K
4) spreadsheet 5) None of these channel(s).
193. Date and Time are available on the desktop at 1) one, one 2) one, multiple 3) multiple, one
1) Taskbar 2) My Computer 3) Recycle Bin 4) multiple, multiple 5) None of these
4) Only (2) and (3) 5) None of these 199. Microsoft Office is a/an
194. Which type of resource has greater probability to 1) shareware
become shared resource in a computer network? 2) public domain software
1) Printers 2) Speakers 3) open-source software
3) Floppy Disc Drivers 4) Keyboards 4) vertical market application
5) None of these 5) application suite
195. If you want to move an icon on your desktop, this is 200. A collection of programs that controls how your

KUNDAN
called computer system runs and processes information is
1) double clicking 2) highlighting 3) dragging called
4) pointing 5) None of these 1) operating system 2) computer
196. A computer checks the ________ of usernames and 3) office 4) compiler 5) interpreter
26. 2 27. 1 28. 5 29. 3 30. 4
Answers 20. 1;
31. 2; Given statements: M < J ... (i)
1. 1; Some cloths are shirts  conversion  JR ... (ii)
Some shirts are cloths. Hence conclusion II R=K ... (iii)
does not follow. There is no negative 21. 2; K<B ... (iv)
statement. Thus the possibility in I exists. Combining all these statements, we get
Hence conclusion I follows.
2. 4; Some hours are minutes (I) + Some
Hence, G is daughter of H.
minutes are not seconds (O) = I + O = No
conclusion. Hence, neither conclusion I nor 22. 2; Option 1)
II follows.
3. 5; No pin is a staple  conversion  No Hence, conclusion II is true.
staple is a pin. Hence, conclusion I follows 32. 5; Given statements:P = W ... (i)
Hence, B is not husband of A.
as an implication. VU ... (ii)
Option 2) W<C ... (iii)
Again, Some pens are staples (I) + No staple
is a pin (E) = I + E = O = Some pens are not C>V ... (iv)
pins. Hence, conclusion II follows. Combining all these statements, we get
4. 2; All letters are numbers (A) + Some Hence, B is husband of A.
numbers are not squares (O) = A + O = No Option 3)
conclusion. Hence conclusion I does not
follow.
Again, C > U or U < C is true.
Again, All letters are numbers  conversion
Hence, B is not husband of A. Hence, both conclusions I and II are true.
 Some numbers are letters. Hence,
Option 4) 33. 1; Given statements: Q  R ... (i)
conclusion II follows.
N > M ... (ii)
5. 5; All hankies are towels (A) + No towel is
NR ... (iii)
cloth (E) = A + E = E = No hanky is cloth.
Combining all these statements, we get
Hence conclusion I follows. Hence, B is not husband of A.
Again, All hankies are towels  conversion 23. 4;
 Some towels are hankies. Hence
conclusion II follows.
(6-10):
Hence, V is aunt of T.
Hence, conclusion I is true. But conclusion
24. 4; II (R  M) is not true.
(34-35):
Given statements: U > X ... (i)
In the expression the gender of P is not given. Y Z ... (ii)

K
Hence P is either sister or brother of J. C=D ... (iii)
6. 3 7. 1 8. 4 9. 2 10. 1 YX ... (iv)
25. 3; Option 1)
(11-15): D>Z ... (v)
Combining all these statements, we get
Candidate (i) (ii)/(a) (iii) (iv) (v)/(b) U>XYZ<D=C
Hence, J is cousin of E.
Payal      34. 1;
Chandan   –   Option 2)

Seema  ()   
Pranav  ×    Hence, J is not wife of E.
Vinod     () Hence, conclusion I is true.
Option 3)
11. 1 12. 3 13. 4 14. 2 15. 5 35. 5;
16. 1; Condition 2) applies

KUNDAN
Hence, J is wife of E.
(26-30):
There are two possibilities: Thus, Z < C or C > Z is true. Hence,
17. 2; Condition 1) applies conclusion I is true.
Possibility (i)
Again, Y < C is true. Hence, conclusion II is
true.
36. 2;
18. 4; Condition 3) applies Possibility (ii)

19. 3; Condition 2) applies There are two possibilities to arrange the


cars according to their prices:  Distance = AB – BE = 20 – 6 = 14m north.
Possibility (i) T > S > Q > P > R > U Hence, he is 14m north from his house.
Possibility (ii) T > Q > S > P > R > U
37. 3; 18  325 28  445 3
92. 2; ? =   r  2519424
100 100 or, 1   =
 100  2000000
= 58.5 + 124.6 = 183.1
3 3
 38  675   r  19683  27 
Similarly, 93. 2; ? =    45 or, 1   = =  
 5   100  15625  25 
= 5130 ÷ 45 = 114
r 27
38. 5; 2  4  45  18270 1+ =
100 25

K
94. 4; ? =  3132
3  7 100
39. 3; BRAG, GARB, GRAB r 2
40. 4; 132465/132465/132465/13246  65  480  or, =
100 25
41. 3 42. 5 43. 1 44. 2 45. 4 95. 3; ? =  100   12  312  12  26
 
46. 2; Replace ‘serve’ with ‘serving’ 200
47. 3; Replace ‘manage’ with ‘managing’  ? = (26)2 = 676 r= = 8% pa
25
48. 1; Replace ‘specially’ with ‘especially’ 88
49. 5 96. 2; Valid votes = 2.4  = 2.112 lakh 1
100 102. 1; A’s 1 day’s work =
50. 4; Replace ‘a’ with ‘an’ 6
(51-55): BDAEC Manohar got 45% of valid votes.
51. 4 52. 1 53. 5 54. 2 55. 3 45 1
56. 4 57. 2 58. 1 59. 5 60. 3  Manohar got 211200 = 95040 B’s 1 day’s work =
100 10
61. 1; restaurants
valid votes

KUNDAN
62. 4; hue (the correct phrase is ‘hue and 1
97. 3; 12 men can reap 48 hectares in 40 C’s 1 day’s work =
cry’) 24
63. 1; harass days.
64. 5 48 1 1 1
A’s share : B’s share : C’s share = : :
65. 2; led  1 man can reap in 1 day = hectares 6 10 24
12  40
66. 2 67. 3 68. 5 69. 4 70. 1 = 20 : 12 : 5
 50 men can reap in 15 days
71. 5 72. 1 73. 3 74. 2 75. 4
76. 1 77. 3 78. 5 79. 2 80. 4 4699
48  B’s share =  12 = ` 1524
=  50  15 = 75 hectares 37
18.5 124 12  40
81. 2; ? =  22.94
100 98. 5; Let the cost price be ` x. 1
103. 2; In one hour A fills of the tank
85 15
82. 1; ? = ( 1444 ÷ 5) × 3.25 Then, x × = 1054
100 1
= (38 ÷ 5) × 3.25 = 7.6 × 3.25 = 24.7
or, x = ` 1240 and in one hour B fills of the tank.
10
3  89.6
83. 3; ? = 17.8  = 17.8 + 38.4 120
7 Now, SP = 1240 × = ` 1488 1 1 3 2
100 Then, (A + B) fill in 1 hour =  
= 56.2 10 15 30
84. 3; ? = 373.816 + 274.102 – 199.573 – 99. 1; Let the cost price be ` x.
108.108 = 340.237 1
132x = of the tanks.
So, marked price = ` 6
20 100
85. 4;  ?  14.8  7.25  107.3
100 132 x 75 4 2
On 25% discount, SP =  So in 4 hours (A + B) will fill = of the
100 100 6 3
10730
?= = 536.5 132x  75 ta nk .
20
Now, = 891
100 100 2 1
86. 5; ? = 1156  42.25  Remaining part = 1 – =
3 3
= 34 × 6.5 = 221 891 100 100
x= = ` 900
87. 2; (?)2 = 171.5 × 56 = 9604 132  75 1
A fills in 1 hour
 ? = 98 100. 4; Let the rate of interest be r% pa 15
2970 11500  r  2 1  1
88. 1; ? = 44 × 6.5 + = 286 + 165 Then, SIA = = 230r  A fills in 15   = 5 hours
18 100 3  3
= 451
8500  r  3 104. 2; There are 10 letters and C, L and A
89. 4; ? = 68 × 9.5 = 646 Now, SIB = = 255r
100 appear twice.
3  7  5  6864  Number of arrangements
90. 3; ? = = 630 230r + 255r = 3395
8 11 13 10!`
3395 =  453600
81.6 r= = 7% 2!2!2!
485
91. 2; 25 × ? = = 1700 105. 1; n(S) = 6 × 6 = 36
0.048 101. 5; Let the interest rate be r% pa.
 n(E) = 6 ie
1700 3 (1, 6), (2, 5), (3, 4), (4, 3), (5, 2), (6, 1)
or, ? = = 68  r 
25 Then, 2519424 = 2000000 1  
 100 
117. 2; (135 – 161) ? [(289 × 2) – 595]
n (E ) 6 1
 P(E) = n (S) = = or, –26 ? (578 – 595)
36 6 or, –26 ? –17
106. 4; Let the number be x. or, –26 < –17
7 3 118. 1; [(69 – 36) × 3] ? [16 + 25 + 52]
Then,   x  567 or, (33 × 3) ? (41 + 52)
11 4
or, (99) ? (94)
 x = 1188
or, 99 > 94
Now, 55% of 1188 = 653.4
107. 3; Let the present age of A and B be x 119. 3;  [18 – 7]? [11]
years and y years respectively. or,  (11) ? (11)
x 8 or,  11  11
Then, y  5
120. 4; [ 1764 ÷ 25 ] ?  [84  10]
or, 5x = 8y ... (i) or, [42  5] ? [8.4]
x  12 10 or, 8.4   8.4
After 12 years, y  12  17
121. 4 122. 4 123. 2 124. 4 125. 5
or, 7x + 84 = 10y + 120 126. 1 127. 2 128. 1 129. 4 130. 1
or, 7x – 10y = 36 ... (ii) 131. 2 132. 4 133. 4 134. 2 135. 4
Solving eqn (i) and (ii), 136. 5 137. 1 138. 4 139. 4 140. 3
x = 48 years, y = 30 years 141. 2 142. 3 143. 2 144. 4 145. 2
108. 2; Let the cost of a pen be ` x and that 146. 4 147. 1 148. 2 149. 2 150. 3
of a pencil be ` y. 151. 4 152. 1 153. 1 154. 2 155. 5
Then, 4x + 7y = 97 ... (i) 156. 5 157. 1 158. 4 159. 1 160. 1
And, 3x + 8y = 92 ... (ii)
161. 1 162. 3 163. 2 164. 2 165. 1
Solving eqn (i) and (ii), we get
x = 12, y = 7 166. 1 167. 4 168. 3 169. 3 170. 3
 2x + 2y = 24 + 14 = ` 38 171. 3 172. 5 173. 2 174. 3 175. 4
109. 1; Let the length and breadth be 100 176. 4 177. 2 178. 3 179. 4 180. 2
cm and 100 cm respectively. 181. 3 182. 2 183. 4 184. 1 185. 2
 Area = 10000 sq cm 186. 3
After change, length = 124 cm and width 187. 3
= 85 cm 188. 3; Value Added Network (VAN) service
 Area = 10540 sq cm carries electronic mail and access to
10540 – 10000 54000 commercial databases.

K
 % change =  100 = 189. 3 190. 3 191. 2 192. 4 193. 1
10000 10000
194. 1 195. 3 196. 4 197. 2 198. 2
= 5.4% increase
199. 5 200. 1
110. 3; Let the maximum marks be x.
56x 48x
Then, – = 8 + 6 = 14
100 100

8x
or,  14
100
1400
x= = 175
8
111. 3; The series is –112, –9 2, –7 2, –52 , –32 ,

KUNDAN
...
112. 1; The series is 4 × 2 – 1 = 7, 7 × 3 – 1
= 20, 20 × 4 – 1 = 79, 79 × 5 – 1 = 394, 394
× 6 – 1 = 2363, 2363 × 7 – 1 = 16540, ...
113. 4; The series is ×1.5, ×2, ×2.5, ×3, ×3.5,
...
114. 2; The series is 93 + 2.8 = 95.8, 95.8 +
4 = 99.8, 99.8 + 5.2 = 105, 105 + 6.4 =
111.4, 111.4 + 7.6 = 119, ...
115. 3; The series is +12 + 1, +22 + 1, +32 + 1,
+42 + 1, +52 + 1, +62 + 1, ...
116. 1;
[(63 ÷ 7) + (75 + 8)] ? [89 – 23] ÷ 12]
or, (9 + 83) ? (66 ÷ 12)
or, 92 ? 5.5
 92 > 5.5
Clerk Mock Test 10
9. What is the position of A with respect to H?
Test-I: Reasoning Ability 1) Second to the right 2) Third to the left
3) Second to the left 4) Fifth to the right
1. M, T, B, D and R are five friends each of them having 5) None of these
a different weight. R is heavier than M and T but lighter 10. Who sits third to the left of E?
than only B. M is heavier than only T. Who among the 1) A 2) G 3) H 4) C 5) None of these
following is at the third position according to their Directions (Q. 11-15): The following questions are
weights? based on the diagram given below:
1) R 2) M 3) D 4) T 5) None of these
2. How many pairs of letters are there in the word
MEDITATION each of which has as many letters
between them in the word as in the English alphabetical
series?
1) None 2) One 3) Two

K
4) Three 5) More than three
3. Meera is eleventh from the left end in a row of 35 girls (i) Circle represents Doctors
and Seema is twentyseventh from the right end in the (ii) Square represents Politicians
same row. How many girls are there between them in (iii) Triangle represents Actors
the row? 11. Who among the following represents Politician and
1) None 2) One 3) Two Doctor but not Actor?
4) Three 5) None of these 1) F, I 2) J, H 3) H 4) G 5) None of these
4. Four of the following five are alike in a certain way and 12. Who among the following represents Actors and
so form a group. Which is the one that does not Politicians?
belong to that group? 1) E, B, D 2) A, B, E 3) F, G, C
1) Garlic 2) Sesame 3) Mustard 4) G, H, E 5) None of these

KUNDAN
4) Olive 5) Corn 13. Who among the following is neither Actor nor
5. In a certain code language JANUARY is written as Politician?
ZSBTOBK. How is OCTOBER written in that code 1) J, I 2) D, I, J 3) F, G, J
language? 4) C, I, J 5) None of these
1) SFCPUDP 2) SFCNUDP 3) SCFNDUP 14. Who among the following is neither Doctor nor Actor?
4) FSCNUDP 5) None of these 1) D, I 2) A, G 3) B, F
Directions (Q. 6-10): Study the following information 4) E, G 5) None of these
carefully and answer the given questions: 15. C represents which of the following professions?
A, B, C, D, E, F, G, H and I are sitting around a circular 1) Doctor 2) Politician 3) Actor
table facing the centre. B is fourth to the left of G, who is 4) Doctor and Actor 5) Actor and Politician
second to the right of C. F is fourth to the right of C and is Directions (Q. 16-20): In each question below are
second to the left of I. D is not an immediate neighbour of given two/three statements followed by two conclusions
either I or B. A is not the neighbour of F. H is third to the numbered I and II. You have to take the given statements
right of E. to be true even if they seem to be at variance with
6. Who among the following is second to the right of I? commonly known facts. Read all the conclusions and then
1) C 2) E 3) H 4) F 5) None of these decide which of the given conclusions logically follows
7. Who is sitting third to the right of the one who is from the given statements, disregarding commonly
sitting second to the right of A? known facts. Give answer
1) D 2) C 3) I 4) H 5) None of these 1) if only conclusion I follows.
8. How many persons are sitting between H and D, if 2) if only conclusion II follows.
counted ACW from D? 3) if either conclusion I or II follows.
1) One 2) Two 3) Three 4) if neither conclusion I nor II follows.
4) Four 5) None of these 5) if both conclusions I and II follow.
16. Statements: All rings are clutches. 1) if only conclusion I is true.
All clutches are bangles. 2) if only conclusion II is true.
Conclusions: I. All bangles being rings is a 3) if either conclusion I or II is true.
possibility. 4) if neither conclusion I nor II is true.
II. Some clutches are rings. 5) if both conclusions I and II are true.
17. Statements: Some poles are wires. 26. Statements: A > B  C < D, C = E > G
No wire is a phone. Conclusions: I. D > E II. B > G
Conclusions: I. Some poles are wires. 27. Statements: P  Q > M  N, Q = S
II. No pole is a phone. Conclusions: I. S > P II. N < S
18. Statements: No pin is a tube. 28. Statement: S> M=Z> T< Q>V
No toy is a pin. Conclusions: I. V = S II. Q > M
Conclusions: I. No toy is a tube. 29. Statement: T<U=VS>PQ
II. No pin is a toy. Conclusions: I. S > T II. V  Q
19. Statements: All discs are pens. 30. Statements: M  N > R > W, E = J > L  W
No pen is a pencil. Conclusions: I. E > W II. M > L
Conclusions: I. All discs are definitely not pencils. Directions (Q. 31-35): Study the following information
II. Some pencils are not pens. carefully and answer the given questions:
20. Statements: Some knives are drums. A, B, C, D, E, F and G are seven friends. They play
All drums are swords. three types of games, viz Hockey, Football and Cricket.
Conclusions: I. Some knives are swords. Each game is played by at least two players. Each one of

K
II. All swords being knives is a them has a favourite colour, viz Pink, Blue, White, Green,
possibility. Yellow, Red and Black, but not necessarily in the same
Directions (Q. 21-25): These questions are based on order.
the following arrangement of numbers, letters and B likes Yellow and does not play Cricket. The one who
symbols. Study them carefully and answer the questions likes Black plays the same game as E. C likes Blue and
given below: plays the same game as G. D plays Football only with the
1 8 QW 3  Z 5 A 2 J @ UK 9 PI % R 4 $ M E 7T #A6 F one who likes Pink. G plays neither Football nor Cricket. F
21. Which of the following should come in place of does not like Black. G likes neither Green nor White. D
question mark (?) in the following series based on the does not like Green. E does not like Pink.
elements in the above arrangement? 31. Which of the following groups plays Hockey?
3WZ 2A@ 9KI ? 1) B, G 2) A, B, C 3) B, C, G

KUNDAN
1) %I4 2) 4RM 3) 4%$ 4) $4E 5) None of these 4) D, G, B 5) None of these
22. How many such letters are there in the above 32. Who likes White Colour?
arrangement each of which is not immediately preceded 1) A 2) G 3) F 4) D 5) None of these
by a symbol? 33. Which of the following colours does A like?
1) Five 2) Six 3) Seven 1) White 2) Black 3) Pink
4) Eight 5) More than eight 4) Either Black or Red 5) None of these
23. Four of the following five are alike in a certain way 34. Which of the following combinations is true?
based on their positions in the above arrangement 1) A – Black – Cricket 2) G – Pink – Cricket
and hence form a group. Find out the one that does 3) B – Yellow – Football 4) D – White – Hockey
not belong to that group. 5) None is true
1) W1 2) A 3) @9 4) IK 5) 4I 35. Who likes Pink?
24. Which of the following is fifth to the right of the 1) G 2) A 3) E 4) C 5) F
fifteenth from the right end? Directions (Q. 36-40): Each of the questions below
1) 4 2) 9 3) $ 4) P 5) None of these consists of a question and two statements numbered I and
25. How many such numbers are there in the above II given below it. You have to decide whether the data
arrangement each of which is immediately followed provided in the statements are sufficient to answer the
by a symbol and also immediately preceded by a letter? question. Read both the statements and give answer
1) One 2) Two 3) Three 1) if the data in statement I alone are sufficient to
4) Four 5) None of these answer the question, while the data in statement II
Directions (Q. 26-30): In these questions, a alone are not sufficient to answer the question.
relationship between different elements is shown in the 2) if the data in statement II alone are sufficient to
statement(s). The statements are followed by two answer the question, while the data in statement I
conclusions. Give answer alone are not sufficient to answer the question.
3) if the data either in statement I alone or in statement cause, adopting measures for cooperative banks and
II alone are sufficient to answer the question. regional rural banks, enabling kisan credit cards and
4) if the data in both the statements I and II together spreading financial literacy. In addition, a number of private
are not sufficient to answer the question. sector institutions have also been operating in the area.
5) if the data in both the statements I and II together Despite all this, nearly half the population/ households do
are necessary to answer the question. not even have a bank account.
36. How is R related to N? It is in this context that Finance Minister Arun Jaitley,
I. R is son of M, whose daughter is mother of N. in his maiden budget speech, mentioned the Financial
II. M is father of R and S is mother of N. Inclusion Mission (FIM) to provide banking services to all
37. Who is the tallest among A, B, C, D and E? households in India. The FIM, focused on empowering
I. A is taller than only D. C is taller than E but as tall the weaker sections of society, was launched on August
as B. 15, with two bank accounts eligible for credit per family.
II. E is taller than only A and D. C is as tall as E. The scheme reportedly conceives six pillars to achieve
38. In which direction was P facing after he stopped comprehensive or sampoorn financial inclusion. These are
walking? — universal access to banking facilities, financial literacy,
I. P walked 12m towards west, took a left turn and basic bank accounts and availability of micro credit, micro
walked 10m. He again took a left turn and stopped insurance, and pension scheme. The vision is to have a
after 15 metres. banking account for each family and beneficiary by March
II. P walked 15m towards east, took a right turn and 2016. To encourage people to start using banking facilities,
walked 12m. Then he took a left turn and stopped the government is considering providing an overdraft of

K
after walking 8m. `5,000 through a debit card to every basic banking account
39. P, Q, R, S, T and U are sitting in a circle. Who sits on holder. The strategy is to strengthen the existing business
the immediate left of P? correspondent model (BCM) to make it operationally flexible
I. P sits second to the right of Q, who is third to the and viable with the expansion of banking services.
left of R. 41. What is the main agenda behind extension of financial
II. T sits second to the right of P and opposite S. inclusion policy?
40. What does ‘@’ stand for in a code language? 1) To offer tenors to the local population that would
I. ‘95@’ means ‘flower is red’ and ‘$@9’ means ‘red convince them to invest their savings in financial
colour flower’ in a code language. institutions
II. ‘7@8’ means ‘red blood cell’ and ‘$@6’ means ‘its 2) To make banking transaction easy and accessible
red colour’ in a code language. to the farmers and astisans only

KUNDAN
3) To make monitoring and supervision possible by
Test-II: English Language the Ministry of Finance
4) To change the mindset of the financal institutions.
Directions (Q. 41-50): Read the passage carefully 5) To ensure that the unbanked population avail of all
and answer the questions given below it. Certain words/ kinds of banking services, including opening of
phrases are given in bold to help you locate them while bank accounts and debit card system
answering some of the questions. 42. What is financial inclusion all about?
Financial inclusion is a process of ensuring the (A) It is about inclusion of all financial institutions,
availability of financial services to all sections of society, regional rural banks and rural cooperatives into
at an affordable cost. A key objective of financial inclusion the rubric of Finance Commission of India.
is to help the unbanked population with institutional (B) It is about ensuring access to financial services
finance, to enable them to become self-employed and for all sections of society.
ensure a stable income. (C) It is about ensuring access to institutional finance
The government has made a concerted effort to extend under KYC norms.
financial inclusion by nationalising banks and establishing 1) Only (A) 2) Only (B)
a network of rural cooperatives and regional rural banks. 3) Only (C) 4) Only (A) and (B)
The RBI too has initiatives like priority sector lending since 5) Only (B)and (C)
the early 1970s, opening of no-frills accounts, establishing 43. What effort(s) has/have been made to extend financial
business correspondents (BCs) and easing of know-your- inclusion?
customer (KYC) norms. Its latest efforts include the 1) Nationalisation of banks
preparation of guidelines for licensing payments banks 2) Establishment of a network of rural cooperative
and small banks. The National Bank for Agriculture and banks and RRBs
Rural Development (NABARD) has also supported the 3) Opening of zero-balance accounts
4) Easing of KYC norms 51. Addressing his first campaign rally in Jammu, Modi
5) All the above 1) 2)
44. Which of the following is not one of the functions of had reterated that the BJP wanted a discussion on
NABARD mentioned in the passage? 3)
1) Enabling kisan credit cards Article 370 since it concerned the people of India.
2) Spreading financial literacy 4)
3) Allowing financial assistance for establishing All correct
medium-and small-scale industries 5)
4) Helping RRBs and cooperative banks in performing 52. Making fighting corruption and transparency his
certain functions clarion call, he held out the promise of cleansing our
5) None of these 1) 2)
45. What are the goals to be achieved by the Financial political system of its shady machinations. All correct
Inclusion Mission of India? 3) 4) 5)
(A) Empowerment of the weaker sections of society 53. The enthusiastic acceptance of the invitation by
(B) To make the unbanked population eligible for bank SAARC leaders stands testimoney to the burgeoning
credit 1) 2)
(C) To provide universal access to financial literacy popularity of and changing perception about Modi in
and availability of micro credit, micro insurance the region, especially in Pakistan. All correct
and pension scheme 3) 4) 5)
1) Only (A) and (B) 2) Only (B) and (C) 54. The stunning success of the Islamic State of Iraq and

K
3) Only (A) and (C) 4) All (A), (B) and (C) 1)
5) Only (A) al-Sham (ISIS), is seizing control of almost a
Directions (Q. 46-48): Choose the word/group of 2)
words which is MOST SIMILAR in meaning to the word/ third of Iraq in less than a weak came as a shock to
group of words printed in bold as used in the passage. 3) 4)
46. Maiden Washington. All correct
1) final 2) concluding 3) first 5)
4) latest 5) interesting 55. Vocational training, entrepreneur mentoring and the
47. Conceives 1) 2)
1) avoids 2) accepts 3) disregards interdisciplinery interactions will help students go
4) dislikes 5) knows 3)

KUNDAN
48. Strategy from being just job seekers to job creators. All correct
1) honesty 2) openness 3) structure 4) 5)
4) working 5) plan Directions (Q. 56-60): Read each sentence to find out
Directions (Q. 49-50): Choose the word/group of whether there is any grammatical or idiomatic error in it.
words which is MOST OPPOSITE in meaning of the word/ The error, if any, will be in one part of the sentence. The
group of words printed in bold as used in the passage. number of that part is the answer. If there is ‘No error’,
49. Stable the answer is 5). (Ignore errors of punctuation, if any.)
1) secure 2) uncertain 3) steady 56. 1) My memories of Delhi University start / 2) when I
4) balanced 5) lasting had been a child / 3) and visited the university / 4) and
50. Concerted its surrounding ridge. / 5) No error
1) united 2) joint 3) planned 57. 1) The largest residential university in Asia, / 2) BHU
4) disorganised 5) mutual was established in 1916 as an exemplar of academic
Directions (Q. 51-55): In each question below is excellence / 3) and has become synonymous with a
given a sentence with four words printed in bold type. life / 4) teeming with enriching experiences. / 5) No
These are numbered as 1), 2), 3) and 4). One of these error
four words printed in bold may be either wrongly spelt 58. 1) Narendra Modi must help his party return to power
or inappropriate in the context of the sentence. Find out in Maharashtra / 2) later this year and in UP and Bihar,
the word which is wrongly spelt or inappropriate, if any. where / 3) the electorate have voted in his favour but
The number of that word is your answer. If all the words / 4) the state government is run by an adversary. / 5)
printed in bold are correctly spelt and also appropriate No error
in the context of the sentence, mark 5), ie ‘All correct’, 59. 1) A majority of weather forecasting models indicates
as your answer. that / 2) an El Nino weather phenomenon may develop
/ 3) around the middle of the year / 4) but it is too early
to assess its likely strength. / 5) No error (E) The contract, if it is well settled, can be inferred
60. 1) Two sharpshooters of the gang have arrested / 2) in from the conduct of the parties.
connection with the sensational road rage / 3) incident (F) Its constituents enjoy the overdraft facility in
which claimed two lives / 4) in Bawana more than a order to develop their business.
month ago. / 5) No error 66. Which of the following will be the SIXTH (LAST)
Directions (Q. 61-65): Each question below has two sentence after rearrangement?
blanks, each blank indicating that something has been 1) B 2) C 3) D 4) E 5) A
omitted. Choose the set of words for each blank that best 67. Which of the following will be the FOURTH sentence
fits the meaning of the sentence as a whole. after rearrangement?
61. The main parties in Jharkhand are led by ________ 1) A 2) D 3) E 4) C 5) B
tribal leaders, but Adivasis are demanding more than 68. Which of the following will be the THIRD sentence
what has been ________ to them so far. after rearrangement?
1) magnetic, conveyed 2) appealing, sent 1) C 2) A 3) B 4) D 5) E
3) poised, mailed 4) charismatic, delivered 69. Which of the following will be the FIRST sentence
5) strong, dispatched after rearrangement?
62. In 2013, generic pharmaceutical company Ranbaxy 1) E 2) F 3) A 4) B 5) D
________ guilty to seven criminal ________ for drug 70. Which of the following will be the SECOND sentence
manufacturing fraud. after rearrangement?
1) pleaded, felonies 2) appealed, wrongdoing 1) D 2) B 3) F 4) A 5) C
3) asked, assons 4) beseeched, cases Directions (Q. 71-80): In the following passage, some

K
5) prayed, offenses of the words have been left out, each of which is indicated
63. Moody’s Analytics said that ________ in credit by a number. Find the suitable word from the options given
conditions was already being felt in India, where against each number and fill up the blanks with
slower economic growth and raising interest rates had appropriate words to make the paragraph meaningfully
made it ________ for borrowers to repay debt. complete.
1) degradation, rigid 2) dislocation, stiff The (71) of clean drinking water is a duty that has
3) downturn, resilient 4) depreciation, hard been (72) in the Constitution of India. Article 47 (73) this
5) deterioration, tougher duty, along with (74) public health standards, on the State.
64. State Bank of India launched digital banking ________ But due to the (75) in the quality of both surface and
at the customer’s doorstep using tab banking – one groundwater sources, (76) to good-quality water continues
for customers________ Savings Bank accounts and to (77) a challenge.

KUNDAN
another for Housing Loan applicants. The Planning Commission has said that in the 12th
1) addresses, beginning 2) propensity, launching Five-Year Plan, ending in 2017, an investment of $26.5 billion
3) facilities, opening 4) options, serving is required to (78) safe water to all Indians. But the growing
5) turn, starting (79) of pollutants – iron, arsenic, fluoride – is (80) the
65. Your whole body needs to be ________ time to time problem difficult to solve.
in order to work _________ during working hours. 71. 1) furnishing 2) stock 3) provision
1) rejuvenated, accurately 2) recharged, properly 4) plan 5) storage
3) revived, fitly 4) energised, rightly 72. 1) enshrined 2) cherished 3) preserved
5) restored, perfectly 4) exalted 5) dedicated
Directions (Q. 66-70): Rearrange the following six 73. 1) talks 2) discusses 3) converses
sentences (A), (B), (C), (D), (E) and (F) in the proper 4) confers 5) argues
sequence to form a meaningful paragraph and then answer 74. 1) elaborating 2) improving 3) developing
the questions given below. 4) correcting 5) fixing
(A) Therefore, both are deeply interested in such an 75. 1) slope 2) lapse 3) cut
arrangement. 4) plunge 5) fall
(B) The enjoyment of overdraft facility for a period of 76. 1) connection 2) entrance 3) access
four years unfailingly points to the conduct of 4) approach 5) passage
the bank. 77. 1) remain 2) endure 3) prevail
(C) Such an arrangement – euphemistically called by 4) survive 5) dwell
Mr Chhatrapati as a facility – is nothing but a 78. 1) contribute 2) afford 3) add
contract. 4) provide 5) equip
(D) The bank grants overdraft facility in order to earn 79. 1) height 2) layer 3) stratum
interest.
4) altitude 5) levels value.)
80. 1) executing 2) making 3) building 96. 165% of 1349 + 16.5% of 1519 = ?
4) composing 5) producing 1) 2480 2) 2475 3) 2380 4) 2580 5) 2575
97. 3 12165.5 + 240% of 628.9 – 24% of 1639.89 = ?
Test-III: Quantitative Aptitude 1) 1531 2) 1230 3) 1141 4) 1030 5) 1461
Directions (Q. 81-95): What value should come in 98. {(7.88)2 × 17.98} ÷ 82 = ?
place of question mark (?) in the following questions? 1) 120 2) 124 3) 132 4) 140 5) None of these
81. 7.2 × 8.5 × 3.5 = ? 99. 118.003 + 13.987 + 26.04 × 12.93 = ?
1) 142.2 2) 241.2 3) 214.2 4) 154.4 5) 162.4 1) 2090 2) 1890 3) 2000 4) 1990 5) 1975
2 1 2 1 100. (38.898)2 + (41.23)2 – (39.568)2 = ?
82. 3  8 – 5  2  ? 1) 1660 2) 1665 3) 1615 4) 1645 5) 1655
7 7 7 14
101. A and B can complete a piece of work in 80 days and
3 5 1 5 1
1) 8 2) 8 3) 7 4) 6 5) 8 120 days respectively. They started working together
14 7 7 14 7 but A left after 20 days. After another 12 days C joined
83. 158% of 705 + 60.5% of 320 = ? B and they completed the work in 28 more days. In
1) 1405 2) 1035.5 3) 1307.5 how many days can C alone complete the work?
4) 1305 5) None of these 1) 110 days 2) 112 days 3) 114 days
84. (2 × 2 × 2 × 2 × 2 × 2)2 × (4 × 4)3 ÷ (2 × 2 × 2 × 2)3 = (4)? 4) 120 days 5) None of these
1) 4 2) 5 3) 3 4) 6 5) None of these 102. 2 men and 3 boys can do a piece of work in 10 days

K
85. 17% of 290 + ? = 32% of 160 while 3 men and 2 boys can do the same piece of work
1) 19 2) 29 3) 2.9 4) 1.7 5) 1.9 in 8 days. In how many days can 2 men and 1 boy do
86. the same piece of work?
? – 17 = 1369
1
1) 2916 2) 54 3) (54)2 1) 12 days 2) 24 days 3) 32 days
2
4) 2516 5) None of these 1
87. 36 ÷ 0.6 – 0.6 = ? 4) 11 days 5) None of these
4
1) 59.4 2) 49.4 3) 16 4) 6 5) None of these 103. A man sets out on cycle from Delhi to Faridabad, and
88. 7 × ? = (1728 ÷ 3) × 49 at the same time another man starts from Faridabad on
1) 4132 2) 4032 3) 3432 4) 5432 5) 5032 cycle for Delhi. After passing each other they complete

KUNDAN
89. 12632.65 + 632.35 + 421.5 – 11734.5 = ? 6 3
1) 1852 2) 1754 3) 1952 4) 2052 5) None of these their journeys in 2 and 5 hours respectively. At
7 5
90. 7832 + 6898 = ? × 49.1 what rate does the second man cycle if the first man
1) 300 2) 310 3) 410 4) 200 5) None of these cycles at 14 kmph?
91. 3
46656 ÷ 0.3 = (?)2 – 108 1) 10 kmph 2) 5 kmph 3) 7 kmph
4) 8 kmph 5) None of these
1) 228 2)
228 3) 18 104. The inner circumference of a circular race track 14 m
4) 15 5) None of these wide is 440m. Find the radius of the outer circle.
1) 21m 2) 12m 3) 84m 4) 20m 5) None of these
92. 5184  ? = 6192
105. The ratio of the radii of the bases of two cylinders A
1) 7396 2) 7744 3) 7569 4) 7496 5) 6724
and B is 3 : 2 and the ratio of their heights is n : 1. If the
4 3 42 volume of cylinder A is one-and-a-half times the
93. of of of 2275  39  ?
7 5 56 volume of cylinder B, then what will be the value of n?
1) 15 2) 17 3) 18 4) 25 5) 45 4 2 3 3
94. 63 × 42 + 26 × 19 – 51 × 23 = ? + 209 1) 2) 3) 4) 5) None of these
3 3 4 2
1) 2831 2) 2931 3) 3991
106. The volumes of a right circular cylinder and a sphere
4) 4031 5) None of these
are equal. Their radii are also equal. If h is the height of
95. 76% of 750 + 24% of 1875 – ?% of 650 = 773
the cylinder and d is the diameter of the sphere, then
1) 48 2) 28 3) 39 4) 38 5) 58
find the correct relation from the given options.
Directions (Q. 96-100): What approximate value
1) h = d 2) 2h = d 3) 2h = 3d
should come in place of question mark (?) in the following
4) 3h = 2d 5) None of these
questions? (Note: You are expected to calculate the exact
107. If 10% discount is allowed on the marked price then
the profit is 20%. If the discount is increased to 20% when the same wire is bent in the form of a circle?
then what will be the profit percentage?
22
2 (Take  = )
7
1) 6 % 2) 5% 3) 8%
3 1) 462 sq cm 2) 539 sq cm 3) 616 sq cm
4) 693 sq cm 5) None of these
1 1
4) 5 % 5) 12 % 6
3 2
117. A boy on being asked what of a certain fraction
7
3
108. A, B and C jointly started a business. A got part of 6
5
was, made the mistake of dividing the fraction by
the whole profit. The remaining profit was distributed 7
equally between B and C. If C got `400 less than A and so got an answer which exceeded the correct
then find the total amount of profit.
13
1) `1600 2) `1200 3) `1000 answer by . Find the fraction.
4) `800 5) None of these 70
109. Two numbers are in the ratio of 1 : 2. If 7 is added to 2 3 4 7
both the numbers their ratio changes to 3 : 5. The 1) 2) 3) 4) 5) None of these
3 5 5 9
greater number is
118. The successive discounts of 40% and 30% are
1) 24 2) 36 3) 28 4) 32 5) None of these
equivalent to a single discount of
110. Eight litres are drawn from a cask full of wine and the

K
1) 30% 2) 15% 3) 58% 4) 25% 5) None of these
cask is filled with water. This operation is performed
119. A, B and C enter into a partnership with a total capital
three more times. The ratio of the quantity of wine
of `8200. A’s capital is `1000 more than B’s and `2000
now left in cask to that of water is 16 : 65. How much
less than C’s. At the end of the year what is B’s share
wine did the cask hold originally?
in the profit of `2460?
1) 18 litres 2) 24 litres 3) 32 litres
1) `420 2) `480 3) `540
4) 42 litres 5) None of these
4) `620 5) None of these
111. 3 years ago, the average age of a family of 5 members
120. A hall, whose length is 16 metres and breadth twice its
was 17 years. A baby having been born, the average
height, takes 168 metres of paper, which is 2 metres
age of the family is the same today. The present age of
wide, for its four walls. Find the area of the floor.
the baby is
1) 192 sq metres 2) 194 sq metres 3) 189 sq metres

KUNDAN
1 4) 196 sq metres 5) None of these
1) 1 year 2) 1 years 3) 2 years
2
4) 3 years 5) None of these Test-IV: General Awareness
112. The ratio of the number of boys to that of girls of a (with Special Reference to Banking Industry)
school with 504 students is 13 : 11. What will be the
new ratio if 12 more girl students are admitted? 121. The CBDT has constituted a six-member committee to
1) 91 : 81 2) 81 : 91 3) 9 : 10 recommend steps to reduce litigations. What is the
4) 10 : 9 5) None of these full form of CBDT?
113. Two numbers are in the ratio of 5 : 7. On diminishing 1) Central Bureau of Direct Taxes
each of them by 40, their ratio becomes 17 : 27. The 2) Central Bureau of Distinct Taxes
difference of the numbers is 3) Central Board of Distinct Taxes
1) 18 2) 52 3) 137 4) 50 5) None of these 4) Central Board of Direct Taxes
114. Out of three numbers, the first is twice the second and 5) None of these
half of the third. If the average of the three numbers is 122. Philipp Lahm, who retired from international football,
56, then the difference of the first and the third number recently, was erstwhile captain of which of the
is following teams?
1) 12 2) 20 3) 24 4) 48 5) None of these 1) Argentina 2) Brazil 3) Germany
115. In how many different ways can the letters of the word 4) France 5) None of these
APTITUDE be arranged? 123. What is the (approximate) current installed power
1) 2520 2) 2160 3) 20160 generation capacity of the country?
4) 5400 5) None of these 1) 2,25,000 MW 2) 2,50,000 MW 3) 2,70,000 MW
116. A wire when bent in the form of a square encloses an 4) 2,95,000 MW 5) None of these
area of 484 sq cm. What will be the enclosed area
124. An asset would be classified as doubtful if it has 134. The ‘No-frills’ accounts offered by banks earlier are
remained in the substandard category for a period of now converted into which of the following types of
1) 6 months 2) 9 months 3) 12 months accounts in persuasion of the RBI guidelines?
4) 18 months 5) None of these 1) Primary Savings Bank Deposit Account
125. What does the letter ‘A’ denote in the term CRAR? 2) Basic Savings Bank Deposit Account
1) Adequacy 2) Adjustment 3) Zero Savings Bank Deposit Account
3) Assets 4) Accretion 4) Micro Savings Bank Deposit Account
5) None of these 5) None of these
126. Who among the following is the president of the 135. Loans to individuals for educational purposes
Eleventh Conference of Parties (CoP-11) to the including vocational courses up to ______for studies
Convention on Biological Diversity (CBD)? in India and ______for studies abroad are included
1) Nobuteru Ishihara 2) Prakash Javadekar under priority sector.
3) Ahmed Yusuf Nuristani 4) Yoon Seong-kyu 1) `5 lakh, `10 lakh 2) `8 lakh, `15 lakh
5) None of these 3) `10 lakh, `20 lakh 4) `15 lakh, `25 lakh
127. Who among the following has been appointed a 5) None of these
United Nations Goodwill Ambassador by UN Women 136. The Cabinet cleared the Securities Laws (Amendment)
recently? Bill recently to empower the market regulator Sebi to
1) Madhuri Dixit deal with which of the following, among other things?
2) Aishwarya Rai Bachchan 1) Ponzi schemes
3) Sandra Bullock 2) Microfinance institutions

K
4) Emma Watson 3) Pension funds
5) None of these 4) Urban Cooperative Banks
128. What is govt’s disinvestment target for the financial 5) None of these
year 2014-15? 137. The Central Govt has notified rules under the Lokpal
1) `25,500 cr 2) `35,625 cr 3) `36,925 cr Act recently. It provides for the establishment of a
4) `58,425 cr 5) None of these Lokpal for the Union and ________for the states.
129. Which of the following is NOT a word related to 1) Lokayuktas 2) Loksewaka
banking/finance? 3) Lok Prahari 4) Lok Pradhan
1) Leverage 2) Ammunition 3) CASA 5) None of these
4) Restructuring 5) None of these 138. India is on course to complete the world’s highest
130. The cash parked by the banks in their specified current railway bridge in Jammu & Kashmir by 2016. The bridge

KUNDAN
account maintained with the Reserve Bank of India is being built over which of the following rivers?
(RBI) is termed as which of the following? 1) Chenab 2) Jhelum 3) Indus
1) Statutory Liquidity Ratio (SLR) 4) Ravi 5) None of these
2) Excess reserve 139. An enterprise engaged in the manufacture/production,
3) Cash Reserve Ratio (CRR) processing/preservation of goods is specified as a
4) Liquid asset micro enterprise if its investment in plant and
5) None of these machinery does not exceed
131. The book titled A Tale of Two Drops has been authored 1) `15 lakh 2) `20 lakh 3) `25 lakh
by who among the following? 4) `30 lakh 5) `50 lakh
1) Dr APJ Abdul Kalam 2) Arundhati Roy 140. Coins of Rupee one and above are called
3) Ghulam Nabi Azad 4) Dr Harsh Vardhan 1) Small Coins 2) Currency Coins
5) None of these 3) Rupiah Coins 4) Rupee Coins
132. India has put on hold all defence procurements from 5) None of these
the Finmeccanica company and its affiliate firms in the 141. Apurvi Chandela and Rahi Sarnobot clinched a gold
wake of an alleged chopper scam. This company is medal each in which of the following games for India
based in at the 20th Commonwealth Games?
1) France 2) Italy 3) Israel 1) Weightlifting 2) Archery 3) Shooting
4) Sweden 5) None of these 4) Wrestling 5) None of these
133. The Govt has approved the sale of its residual stakes 142. Which of the following companies recently announced
in which of the following PSUs recently? to acquire all the three hydroelectric power plants of
1) BALCO 2) Hindustan Zinc Jaiprakash Associates Ltd?
3) BPCL 4) All the above 1) Tata Power 2) Reliance Power 3) Essar Power
5) Both 1) and 2) 4) Adani Power 5) None of these
143. Which of the following countries is the Chair of the 3) Mario Gotze 4) Manuel Neuer
ASEAN group in 2014? 5) None of these
1) Bangladesh 2) Myanmar 3) Malaysia 153. People across the world can now view images from 76
4) Indonesia 5) None of these heritage sites located in India. With the support of the
144. A Non-Banking Financial Company (NBFC) is a Archaeological Survey of India (ASI), which of the
company whose principal business includes following technology firms has made this possible?
1) insurance business 1) Microsoft 2) IBM 3) Google
2) industrial activity 4) Yahoo 5) None of these
3) agricultural activity 154. Which of the following are the short-term (with original
4) sale/purchase/construction of immovable property maturities of less than one year) debt instruments
5) Any of the above issued by the Govt of India?
145. A Commercial Paper (CP) is an unsecured money 1) Dated Govt Securities
market instrument issued in the form of a promissory 2) Development loans
note and can be issued in denominations of 3) Treasury bills
1) `2 lakh or multiples 4) National Savings Certificates
2) `5 lakh or multiples 5) None of these
3) `10 lakh or multiples 155. The minimum amount to be remitted through RTGS is
4) `50 lakh or multiples `2 lakh. What is the upper ceiling for RTGS
5) None of these transactions?
146. The Govt has revised investment target for Dedicated 1) No upper limit 2) `5 lakh 3) `10 lakh

K
Freight Corridor (DFC) of the Indian Railways from 4) `50 lakh 5) `2 cr
`27,000 cr to 156. The provisional results of the Sixth Economic Census
1) `60,000 cr 2) `80,000 cr 3) `100,000 cr of India were released recently. When was the first
4) `120,000 cr 5) None of these Economic Census conducted?
147. Who among the following won the Hungarian Grand 1) 1977 2) 1980 3) 1990
Prix 2014 recently? 4) 2004 5) None of these
1) Fernando Alonso 2) Lewis Hamilton 157. The third India-Africa Forum Summit (IAFS-III) in 2014
3) Nico Rosberg 4) Daniel Ricciardo will be hosted by which of the following cities?
5) None of these 1) Addis Ababa 2) New Delhi 3) Nairobi
148. Who among the following is the present Chairman of 4) Windhoek 5) None of these
the Railway Board? 158. Due to India’s opposition, the World Trade

KUNDAN
1) VK Gupta 2) Arunendra Kumar Organization (WTO) failed to reach a consensus on
3) Alok Johri 4) Kul Bhushan the Trade Facilitation Agreement (TFA) recently. Who
5) None of these among the following is the current President of the
149. The deposits kept by a person in different branches WTO?
of a bank are aggregated for the purpose of insurance 1) Guy Ryder 2) Roberto Azevedo
cover by the DICGC and in case of claim it pays a 3) Pascal Lamy 4) Gita Wirjawan
maximum amount up to 5) None of these
1) `1 lakh 2) `2 lakh 3) `5 lakh 159. Aadi Perukku festival, which is thanksgiving festival
4) `10 lakh 5) half of the aggregate deposit by farmers to River Cauvery, is celebrated in which of
150. Which are the banks covered under the Banking the following states?
Ombudsman Scheme, 2006? 1) Kerala 2) Tamil Nadu
1) Public Sector Banks 3) Karnataka 4) Andhra Pradesh
2) Private Banks 5) None of these
3) Regional Rural Banks 160. Which of the following Union Territories/States
4) Scheduled Primary Co-operative Banks celebrated its 61st Liberation Day on 2nd Aug?
5) All the above 1) Dadra and Nagar Haveli 2) Goa
151. The World Hepatitis Day is observed on which of the 3) Puducherry 4) Andaman and Nicobar
following dates? 5) None of these
1) 22 Jul 2) 26 Jul 3) 28 Jul
4) 29 Jul 5) None of these Test-V: Computer Knowledge
152. Who among the following won the Golden Glove
award at 2014 FIFA World Cup? 161. Memory, also called random access memory, or RAM,
1) James Rodriguez 2) Lionel Messi
1) contains the electronic circuits that cause 1) Search 2) Cut 3) Document
processing to occur. 4) Undo 5) None of these
2) makes the information resulting from processing 173. Most World Wide Web pages contain commands in
available for use. the ________ language.
3) allots data, programs, commands, and user 1) NIH 2) URL 3) HTML
responses to be entered into a computer. 4) IRC 5) FTP
4) consists of electronic components that store data. 174. ________ are graphical objects used to represent
5) None of these commonly used application.
162. Correcting errors in a program is referred to as 1) GUI 2) Drivers 3) Windows
1) debugging 2) bugging 3) rectifying 4) Application 5) Icons
4) modifying 5) None of these 175. In Windows, by default the files when deleted are sent
163. Computer programs are written in a high-level to
programming language; however, the human-readable 1) Dust Bin 2) Recycle Bin 3) Waste Bin
version of a program is called 4) Recycle Waste Bin 5) None of these
1) cache 2) instruction set 3) source code 176. Which of the following is an Operating System?
4) word size 5) None of these 1) MS Windows 2) Mac 3) MS DOS
164. In word processing, an efficient way to move the 3rd 4) Windows NT 5) All of the above
paragraph to place it after the 5th paragraph is 177. Developing sets of instructions for the computer to
1) copy and paste 2) copy, cut and paste follow and to do the task the same way as many times
3) cut, copy and paste 4) cut and paste as needed is called

K
5) None of these 1) listing 2) sequencing 3) programming
165. A series of instructions that tells a computer what to 4) directing 5) None of these
do and how to do is called a 178. The term used to define all input and output devices
1) program 2) command 3) user response in a computer system is
4) processor 5) None of these 1) Monitor 2) Software
166. The communications device that allows the computer 3) Shared resources 4) Hardware
to access a network is called a ________ card. 5) None of these
1) modem 2) video 3) sound 179. If you receive an e-mail from someone you don’t know,
4) network 5) None of these what should you do?
167. A 32-bit-word computer can access ________ GiB of 1) Forward it to the police immediately
byte-addressable memory. 2) Delete it without opening it

KUNDAN
1) 4 2) 8 3) 16 3) Open it and respond to them saying you don’t know
4) 32 5) 30 them
168. A megabyte is actually equal to ________ kilobytes. 4) Reply and ask them for their personal information
1) 100 2) 1000 3) 1024 5) Reply and tell them you want to keep in touch with
4) 1024 × 1024 5) None of these them
169. The capability of the operating system to enable two 180. Which of the following can handle most system
or more than two instructions to execute functions that aren’t handled directly by the operating
simultaneously in a single computer system by using system?
multiple CPUs is called 1) Vertical market applications
1) Multitasking 2) Multiprogramming 2) Utilities
3) Multiprocessing 4) Multiexecution 3) Algorithms
5) None of these 4) Integrated software
170. All of the following storage media have Read and Write 5) Compilers
capabilities except 181. DOS and Windows 3.x support file names up to
1) flash memory cards 2) CD-ROMs ________ characters in length.
3) hard disk drives 4) floppy disks 1) Two 2) Four 3) Six
5) None of these 4) Eight 5) None of these
171. If you type a word that is not in Word’s dictionary, a 182. ________ is a collection of webpages and ________
wavy ________ underline appears below the word. is the very first page that we see on opening of a web-
1) red 2) green 3) blue site.
4) black 5) None of these 1) Homepage, Webpage 2) Website, Homepage
172. The ________ button on the Quick Access Toolbar 3) Webpage, Homepage 4) Webpage, Website
allows you to cancel your recent commands or actions. 5) None of these
183. A(n) ________ is a program that makes the computer 3) Mouse, Keyboard, Plotter
easier to use. 4) Mouse, Keyboard, Scanner
1) utility 2) application 5) None of these
3) operating system 4) network 192. A menu contains a list of
5) None of these 1) commands 2) data 3) objects
184. The ________ tells the computer how to use its 4) reports 5) None of these
components. 193. Any component of the computer you can see and
1) utility 2) network touch is called
3) application program 4) Operating system 1) Software 2) Peripheral 3) Storage
5) None of these 4) CPU 5) Hardware
185. A scanner scans 194. What is the value of the binary number 101?
1) Pictures 1) 3 2) 5 3) 6 4) 101 5) 9
2) Text 195. If your computer keeps rebooting itself, it is likely that
3) Both Pictures and Text 1) it has a virus
4) Neither Pictures nor Text 2) it does not have enough memory
5) None of these 3) there is no printer
186. A directory within a directory is called 4) there has been a power surge
1) Mini Directory 2) Junior Directory 5) it needs a CD-ROM
3) Part Directory 4) Sub Directory 196. What does RAM stand for?
5) None of these 1) Read Access Memory

K
187. What is the permanent memory built into your 2) Read Anywhere Memory
computer called? 3) Random Anything Memory
1) RAM 2) ROM 4) Random Access Module
3) CPU 4) CD-ROM 5) Random Access Memory
5) None of these 197. The secret code that restricts entry to some programs
188. Various applications and documents are represented is called
on the Windows desktop by 1) password 2) passport 3) entry code
1) Symbols 2) Labels 3) Graphs 4) access code 5) None of these
4) Icons 5) None of these 198. The basic goal of a computer process is to convert
189. A computer cannot perform which of the following data into
functions? 1) files 2) tables 3) information

KUNDAN
1) Addition 2) Subtraction 3) Bake a cake 4) graphs 5) None of these
4) Division 5) None of these 199. Plotter accuracy is measured in terms of repeatability
190. Which of the following has the smallest storage and
capacity? 1) vertical dimension 2) intelligence
1) zip disk 2) hard disk 3) floppy disk 3) resolution 4) pixels
4) data cartridge 5) CD 5) buffer size
191. Which of the following groups consists of only input 200. The speed at which the monitor accepts data is called
devices? 1) Bandwidth 2) Interlacing 3) Response time
1) Mouse, Keyboard, Monitor 4) Scanning 5) Maximum speed
2) Mouse, Keyboard, Printer
24. 1; Fifth to the right of fifteenth from 38. 3;
Answers the right end = (15 – 5 =) 10th from the From I.
1. 3; B > R > D > M > T right, ie 4.
Hence, D is third heaviest among them. 25. 3; W3, Z5A, R4$
26. 5; Given statements:
2. 5; A > B  C < D ... (i)
3. 2; Meera is 11th from the left. C=E>G ... (ii)
Hence P is facing east.
Seema is 27th from the right. Combining both the statements, we get
A>BC=E>G From II.
So, Seema is (35 – 27 + 1 =) 9th from left
Hence, there is only one girl between them. And, A > B  C = E < D
4. 1; All others are grown above the ground Thus, B > G is true. Hence II is true.
while garlic grows under the ground. Again, D > E is true. Hence I is also true.
27. 2; Given statements: Hence, P is facing west.
5. 2; P  Q > M  N ... (i) Hence either I or II is sufficient to answer
Q=S ... (ii) the question.
Combining both the statements, we get 39. 2; From I.
PQ=S>MN
Similarly, Thus, P  S or S  P is true. Hence conclusion
I (S > P) is not definitely true.
Again, S > N or N < S is true. Hence II is true.
From II.
28. 4; S > M = Z > T < Q > V
(6-10): Thus, we can’t compare V and S. Hence
conclusion I is not true. And also we can’t
compare Q and M. Hence II is not true. Hence S is on the immediate left of P.
29. 1; T < U = V  S > P  Q 40. 2; From II. 7 @ 8  red blood cell ... (i)

K
Thus, T < S or S > T is true. Hence I is true. $ @ 6  its red colour ... (ii)
But we can’t compare V and Q. Hence From (i) and (ii), @  red
conclusion II is not true. II is sufficient to answer the question.
6. 3 7. 3 8. 4 9. 1 10. 2 30. 1; Given statements: 41. 5 42. 2 43. 5 44. 3 45. 4
11. 3 12. 2 13. 1 14. 5; F, G MN>R>W ... (i) 46. 3 47. 2 48. 5 49. 2 50. 4
15. 3 E=J>LW ... (ii) 51. 3; The correct spelling is ‘reiterated’
16. 5; There is no negative statement. Thus Combining both the statements, we get 52. 5
the possibility in I exists. Hence conclusion MN>R>WL<J=E 53. 1; The correct spelling is ‘testimony’
I follows. Again, All rings are clutches  Thus, W < E or E > W is true. Hence 54. 3; The appropriate word is ‘week’
conversion  Some clutches are rings. Hence conclusion I is true. We can’t compare M 55. 3; The correct spelling is
conclusion II follows. and L. Hence II is not true. ‘interdisciplinary’
17. 4; It is a restatement; hence conclusion I (31-35): 56. 2; Replace ‘had been’ with ‘was’
does not follow. Again, Some poles are wires 57. 5

KUNDAN
(I) + No wire is a phone (E) = I + E = O = 58. 3; Replace ‘have’ with ‘has’
Friend Game Colour
Some poles are not phones. Hence conclusion 59. 1; Replace ‘indicates’ with ‘indicate’
II does not follow. A Cricket Black 60. 1; Add ‘been’ after ‘have’
18. 2; No toy is a pin (E) + No pin is a tube B Hockey Yellow 61. 4 62. 1 63. 5 64. 3 65. 2
(E) = E + E = No conclusion. Hence C Hockey Blue (66-70): DFACEB
conclusion I does not follow. Again, No toy D Football White 66. 1 67. 4 68. 2 69. 5 70. 3
is a pin (E)  conversion  No pin is a toy 71. 3 72. 1 73. 4 74. 2 75. 5
(E). Hence, conclusion II follows. E Cricket Green
76. 3 77. 1 78. 4 79. 5 80. 2
19. 5; All discs are pens (A) + No pen is a F Football Pink 81. 3; ? = 7.2 × 8.5 × 3.5 = 214.2
pencil (E) = A + E = E = No disc is a pencil. G Hockey Red 2 1 2 1
Hence conclusion I follows. Again, No pen is 82. 1; ? = 3  8 – 5  2
31. 3 32. 4 33. 2 34. 1 35. 5 7 7 7 14
a pencil  conversion  No pencil is a pen.
Hence, conclusion II follows as an implication 36. 1; From I. 2 1 1 2
= (3 + 8 + 2 – 5) +    – 
of this.  7 7 14 7 
20. 5; Some knives are drums (I) + All drums
are swords (A) = I + A = I = Some knives are  4  2 1 – 4  3 3
= 8  = 8 = 8
swords. Hence conclusion I follows. There is  14  14 14
no negative statement. Hence the possibility
158  705 60.5  320
in II exists. Hence conclusion II follows. Hence R is maternal uncle of N. 83. 3; ? = 
21. 2; 100 100
From II. = 1113.9 + 193.6 = 1307.5
84. 4; 4? = 43 × 2 × 42 × 3 ÷ 42 × 3
or, 4?= 46 + 6 – 6 = 46 ?=6
Hence statement II is not sufficient. 85. 5; ? = 32% of 160 – 17% of 290
37. 3; From I. B = C > E > A > D
22. 5; 8Q, 2J, 9P, 7T, 6F, QW, UK, ME,PI 32  160 17  290
Both B and C are the tallest. Hence statement = – = 51.2 – 49.3 = 1.9
Hence, there are nine letters in the given 100 100
I is sufficient to answer the question.
arrangement. 86. 3; ? – 17 = 1369 = 37
From II. B > C = E > A, D
23. 3; B is the tallest. Hence statements II is or, ? = 37 + 17 = 54
sufficient to answer the question.
? = 54 × 54 = 2916
87. 1; ? = 36 ÷ 0.6 – 0.6 = 60 – 0.6 = 59.4 boy’s 1 day’s work be y. 3
88. 2; ? × 7 = (1728 ÷ 3) × 49 Then, the share of A in the profit = ` x
1 5
576 49 Then, 2x + 3y = ... (i)
10 3x 2x
or, ? = = 4032 Remaining profit = x – =
7 1 5 5
89. 3; ? = 12632.65 + 632.35 + 421.5 – and 3x + 2y = ... (ii)
8 x
11734.5 = 13686.5 – 11734.5 = 1952 Solving (i) and (ii), we get Share of B = Share of C =
90. 1; ? × 49.1 = 7832 + 6898 = 14730 5
7 1 3x x
14730 x= ,y= Now, –  400
?= = 300 200 100 5 5
49.1 (2 men + 1 boy)’s 1 day’s work
91. 2; 3 46656  0.3  (?)2 – 108 2x
 7 1  16 2 or, = 200
36 = 2  1  =  5
 108 = 120 + 108 = 228  200 100  200 25
or, (?)2 = or, x = `1000
0.3 So, 2 men and 1 boy together can finish the
109. 3; Let the numbers be x and 2x.
 ? = 228 25 1
work in  12 days x7 3
92. 1; 5184  ?  6192 2 2 Then, =
103. 2; By theorem, 2x  7 5
or, 72 × ? = 6192 or, 5x + 35 = 6x + 21
A’s rate : B’s rate = b: a or, x = 35 – 21 = 14
6192
 ? = = 86 3  Greater number = 14 × 2 = 2x = 28
72 5 110. 2; Let the quantity of wine in the cask
? = 86 × 86 = 7396 1st man's rate 5
originally be x litres.
4 3 42 Now, = 6
2nd man' s rate 2 Then, the quantity of wine left in the cask
93. 1;    2275 = 39 × ?
7 5 56 7   4
8 
9 2275 28 7 7  
or, 39 × ? = = 585 =   after 4 operations =  x1 – x   litres
   

K
35 5 20 5
585 5
? = = 15 2nd man’s rate =  14 = 10 kmph
39 7 4
 8
94. 5; ? + 209 = 63 × 42 + 26 × 19 – 51 × 23 104. 3; Let the inner radius be r metres. x 1 – 
or, ? = 2646 + 494 – 1173 – 209 = 3140 – Then 2r = 440  x 16

1382 = 1758 Now,   8  4  65
7
76  750 24  1875 ? 650  r = 440 × = 70m x 1 – 1 –  
 – 773 = 44   x  
95. 4;
100 100 100  Radius of outer circle = 70 + 14 = 84m
4 4
? 650 Volume of cylinder A  8  8
or, = 570 + 450 – 773 = 247 105. 2; Volume of cylinder B or, 65 × 1 –  = 16 – 16 × 1 – 
100  x  x
247 100 4
? = = 38 (3) 2  n 3 9n 3  8
650 = = or,  or, (65 + 16) × 1 –  = 16
(2) 2  1 2 4 2  x

KUNDAN
165  1349 16.5  1519
96. 1; ? =  4
100 100 3 4 12 2  8 16
 165 × 13.5 + 16.5 × 15.20  2227.5 + n= =  or, 1 –  =
92 18 3  x 81
250.8  2478.3  2480 4 4
106. 4; Volume of right circular cylinder
97. 3;  8  2
= Volume of sphere or, 1 –    
3 240  628.9 24  1639.89
2 3
 x  3
? = 12165.6  – d 4 d
100 100 Then,    h    8 2
 23 + 2.40 × 628.9 – 24 × 16.40 2
  3  2 or, 1 – =
x 3
 23 + 2.40 × 630 – 24 × 16.40
d2 4 d3 h d
8 2
 23 + 1512 – 393.6  1535 – 394  1141 or, h   or, 
4 3 8 4 6 or, 1–
2
98. 2; ? = {(7.88) × 17.98} ÷ x 3
82 or, 3h = 2d  x = 24 litres
 (62.09 × 17.98) ÷ 9 107. 1; In first condition the cost price 111. 3; 3 years ago the total age of 5 members
 62 × 18 ÷ 9 = 124 = `100. = (17 × 5) years = 85 years
99. 4; ? = 118 × 14 + 26 × 13 = 1652 + 338  Selling price = `120 Total age of 5 members now = (85 + 3 × 5)
= 1990 100 400 years = 100 years
 Marked price =  120 = `
100. 5; ? = (38.898)2 + (41.23)2 – (39.568)2 90 3 Total age of 6 members now = (17 × 6) years
 (39)2 + (41.2)2 – (39.57) 2 In second condition, = 102 years
 1521 + 1697.44 – 1565.78 20 400 80  Age of the baby = (102 – 100) years
 3218.44 – 1565.78 = 1652.66  1655 Discount allowed =  
100 3 3 = 2 years
101. 2; A did the work for 20 days, B did the 112. 1; Number of boys in the school
400 80 320
work for (20 + 12 + 28) = 60 days, and C did  Selling price = – =` 13
the work for 28 days. 3 3 3 = 504 = 273
24
Let C alone complete the work in x days. 320
– 100 And the number of girls = 504 – 273 = 231
20 60 28 3 20 2
Then,   = 1  Profit % = 100  %6 % When 12 more girl students are admitted,
80 120 x 100 3 3 the new ratio = 273 : 231 + 12
or, x = 28 × 4 = 112 days 108. 3; Let the profit be `x. 273 91
102. 1; Let 1 man’s 1 day’s work be x and 1 = = = 91 : 81
243 81
113. 4; Let the numbers be 5x and 7x.
7x 6x 13
or, – =
5x – 40 17 6 7 70
Then, =
7 x – 40 27 49 x – 36x 13
or, 135x – 1080 + 680 – 119x = 0 or, =
42 70
or, 16x = 400
 x = 25 13x 13
or, =
 Difference = 175 – 125 = 50 42 70
Y
114. 4; Second number = 13  42 3
2 x= =
first number = Y and third number = 2Y 70 13 5
118. 3; Single discount
Y
Y  2Y xy 40 30
Then, 2  56 x+y– = 40 + 30 – = 58%
3 100 100
119. 1; A = B + 1000 = C – 2000
2Y  Y  4 Y
or, = 56 × 3 C = B + 3000
2 Now, A + B + C = (B + 1000) + B + (B +
56  3  2 3000) = 8200
or, Y = = 48
7 or, 3B + 4000 = 8200
 Third number = 2 × 48 = 96  B = `1400
First number = 48 1400
 difference = 96 – 48 = 48  B’s share in the profit =  2460
8200
115. 3; There are eight letters in the given = `420
word, out of which T appears twice. 120. 1; Let the breadth be 2h metres.

K
8! Then, height = h metres
Number of ways =
2!  Area of the walls = 2(16 + 2h)h sq metres
= 3 × 4 × 5 × 6 × 7 × 8 = 20160  Area of the paper = 168 × 2 sq metres
116. 3; Area of the square = (side)2 Now, 2(16 + 2h)h = 168 × 2
Then, 484 sq cm = (side) 2 or, (8 + h)h = 84
 Side = or, (8 + h)h = 14 × 6
484 = 22 cm
Now, we find that only 14 and 6 are such two
 Perimeter of the square = 4 × side numbers whose products is 84 and the
= 4 × 22 = 88 cm difference is 8.
According to the question,  h = 6 and breadth = 12
The circle is made of the same wire.  Area of the floor = 16 × 12
Therefore, the perimeter of the square = = 192 sq metres
Circumference of the circle = 2r = 88 cm 121. 4 122. 3 123. 2 124. 3 125. 3

KUNDAN
22 126. 2 127. 4 128. 4 129. 2 130. 3
Now, 88 = 2  r
131. 4 132. 2 133. 5 134. 2 135. 3
7
 r = 14 cm 136. 1 137. 1 138. 1 139. 3 140. 4
141. 3 142. 2 143. 2 144. 1 145. 2
22
 Area of the circle = r2 = 14  14 146. 2 147. 4 148. 2 149. 1 150. 5
7 151. 3 152. 4 153. 3 154. 3 155. 1
= 616 sq cm 156. 1 157. 2 158. 2 159. 2 160. 1
117. 2; Let the fraction be x. 161. 4 162. 1 163. 3 164. 4 165. 1
According to the question, 166. 4 167. 1 168. 3 169. 3 170. 2
171. 5 172. 4 173. 3 174. 1 175. 2
6 x 13
of x = 6 – 176. 1 177. 3 178. 4 179. 2 180. 2
7 70 181. 4 182. 2 183. 1 184. 4 185. 3
7 186. 4 187. 2 188. 4 189. 3 190. 3
6x 7 x 13 191. 4 192. 1 193. 5 194. 2 195. 1
or,  – 196. 5 197. 1 198. 3 199. 3 200. 1
7 6 70

S-ar putea să vă placă și